Exam 3

Pataasin ang iyong marka sa homework at exams ngayon gamit ang Quizwiz!

To monitor for complications in a patient with type 2 diabetes, which tests will the nurse in the diabetic clinic schedule at least annually (select all that apply)? a. Chest x-ray b. Blood pressure c. Serum creatinine d. Urine for microalbuminuria e. Complete blood count (CBC) f. Monofilament testing of the foot

B C D F b. Blood pressure c. Serum creatinine d. Urine for microalbuminuria f. Monofilament testing of the foot

A 29-year-old patient in the outpatient clinic will be scheduled for blood cortisol testing. Which instruction will the nurse provide? A. "Avoid adding any salt to your foods for 24 hours before the test." B. "You will need to lie down for 30 minutes before the blood is drawn." C. "Come to the laboratory to have the blood drawn early in the morning." D. "Do not have anything to eat or drink before the blood test is obtained."

C. "Come to the laboratory to have the blood drawn early in the morning." - Cortisol levels are usually drawn in the morning, when levels are highest. The other instructions would be given to patients who were having other endocrine testing.

How should the nurse explain the importance of follow-up cystoscopies?

Follow-up cystoscopies on a regular basis are essential to evaluate the effectiveness of the treatment and detect any new tumors while they are in a superficial stage. Two thirds of patients have tumor recurrence within 5 years and nearly 95% have recurrence by 15 years.

A patient sustains a soft tissue injury to the ankle, and a large amount of edema develops. What nursing intervention will relieve edema associated with this injury? 1 Elevating the extremity 2 Massaging the ankle every 1-2 hours 3 Applying a warm compress to the ankle 4 Applying warm saline soaks to the extremity

1 Elevation promotes good venous return, allowing extracellular fluid to flow more readily away from the edematous area. In addition, ice or a cold compress helps ease edema and pain. Warm saline soaks, massage of the extremity, and warm compresses will increase venous circulation and congestion, thereby worsening the edema, as well as the pain. Text Reference - p. 293

It is especially important for the nurse to assess for which clinical manifestation(s) in a patient with primary hypoparathyroidism? Select all that apply. 1 Anorexia 2 Easy fatigability 3 Depressed reflexes 4 Circumoral numbness 5 Positive Trousseau's sign

2, 4, 5 Primary hypoparathyroidism can result in a lack of parathyroid hormone, leading to hypocalcemia. Manifestations of low serum calcium levels include easy fatigability, depression, anxiety, confusion, numbness and tingling in extremities and the region around the mouth, hyperreflexia, muscle cramps, positive Chvostek's and Trousseau's signs, and others. Anorexia and depressed reflexes are manifestations of hypercalcemia. Text Reference - p. 300

The nurse is caring for a patient admitted with dehydration because of nausea and vomiting. The nurse anticipates which acid-base imbalance based on this information? 1 Respiratory acidosis 2 Respiratory alkalosis 3 Metabolic acidosis 4 Metabolic alkalosis

4 The nurse is caring for a patient admitted with dehydration because of nausea and vomiting. The nurse anticipates which acid-base imbalance based on this information? 1 Respiratory acidosis 2 Respiratory alkalosis 3 Metabolic acidosis 4 Metabolic alkalosis

A

A patient receives 3% NaCl solution for correction of hyponatremia. Which assessment is most important for the nurse to monitor for while the patient is receiving this infusion? a. Lung sounds b. Urinary output c. Peripheral pulses d. Peripheral edema

A patient is to receive methylprednisolone (Solu-Medrol) 100 mg. The label on the medication states: methylprednisolone 125 mg in 2 mL. How many milliliters will the nurse administer?

ANS: 1.6 A concentration of 125 mg in 2 mL will result in 100 mg in 1.6 mL.

25. A 34-year-old female patient with a new ileostomy asks how much drainage to expect. The nurse explains that after the bowel adjusts to the ileostomy, the usual drainage will be about _____ cups. a. 2 b. 3 c. 4 d. 5

ANS: A After the proximal small bowel adapts to reabsorb more fluid, the average amount of ileostomy drainage is about 500 mL daily. One cup is about 240 mL.

15. A 51-year-old male patient has a new diagnosis of Crohn's disease after having frequent diarrhea and a weight loss of 10 pounds (4.5 kg) over 2 months. The nurse will plan to teach about a. medication use. b. fluid restriction. c. enteral nutrition. d. activity restrictions.

ANS: A Medications are used to induce and maintain remission in patients with inflammatory bowel disease (IBD). Decreased activity level is indicated only if the patient has severe fatigue and weakness. Fluids are needed to prevent dehydration. There is no advantage to enteral feedings.

Which prescribed medication should the nurse administer first to a 60-year-old patient admitted to the emergency department in thyroid storm? a. Propranolol (Inderal) b. Propylthiouracil (PTU) c. Methimazole (Tapazole) d. Iodine (Lugol's solution)

ANS: A b-Adrenergic blockers work rapidly to decrease the cardiovascular manifestations of thyroid storm. The other medications take days to weeks to have an impact on thyroid function

24. When assessing a patient with a urinary tract infection, indicate on the accompanying figure where the nurse will percuss to assess for possible pyelonephritis. a. 1 b. 2 c. 3 d. 4

ANS: B Costovertebral angle (CVA) tenderness with percussion suggests pyelonephritis or polycystic kidney disease.

The home health nurse notes that an elderly patient has a low serum protein level. The nurse will plan to assess for a. pallor. b. edema. c. confusion. d. restlessness.

ANS: B edema. Low serum protein levels cause a decrease in plasma oncotic pressure and allow fluid to remain in interstitial tissues, causing edema. Confusion, restlessness, and pallor are not associated with low serum protein levels.

A patient with bladder cancer is scheduled for intravesical chemotherapy. In preparation for the treatment the nurse will teach the patient about a. premedicating to prevent nausea. b. where to obtain wigs and scarves. c. the importance of oral care during treatment. d. the need to empty the bladder before treatment.

ANS: D The patient will be asked to empty the bladder before instillation of the chemotherapy. Systemic side effects are not experienced with intravesical chemotherapy. DIF: Cognitive Level: Application REF: 1146

29. A nurse reminds a patient with type I diabetes to rotate the insulin injection sites to prevent _____.

ANS: lipohypertrophy Using the same area for insulin injections causes swollen lumpy areas that interfere with the absorption of insulin.

Which hormone regulates fluid volume in the renal tubules?

Antidiuretic hormone (ADH)

D

The nurse explains to the patient with Vincent's infection that treatment will include: A. smallpox vaccinations B. viscous lidocaine rinses C. amphoterin B suspension D. topical application of antibiotics

A

The nurse is caring for a patient with a massive burn injury and possible hypovolemia. Which assessment data will be of most concern to the nurse? a. Blood pressure is 90/40 mm Hg. b. Urine output is 30 mL over the last hour. c. Oral fluid intake is 100 mL for the last 8 hours. d. There is prolonged skin tenting over the sternum.

C

The nurse is involved in health promotion related to oral cancer. Teaching young adults about behaviors that put them at risk for oral cancer includes: A. discouraging use of chewing gum B. avoiding the use of perfumed lip gloss C. avoiding use of smokeless tobacco D. discouraging drinking of carbonated beverages

The patient asks the nurse why she needs to have surgery for a femoral, strangulated hernia. What is the best explanation the nurse can give the patient?

The surgery is needed to allow intestinal flow and prevent necrosis

B, C

The teaching plan for a patient being discharged following an acute episode of upper GI bleeding will include information concerning the importance of (select all that apply): A. only taking aspirin with milk or bread products B. avoiding taking aspirin and drugs containing aspirin C. taking only drugs prescribed by the healthcare provider D. taking all drugs 1 hour before mealtime to prevent further bleeding E. reading all OTC drug labels to avoid taking those containing stearic acid and calcium

When evaluating the patient's understanding about the care of the ileostomy, what statement by the patient indicates the patient needs more teaching? a. "I will be able to regulate when I have stools." b. "I will be able to wear the pouch until it leaks." c. "Dried fruit and popcorn must be chewed very well." d. "The drainage from my stoma can damage my skin."

a. "I will be able to regulate when I have stools." The ileostomy is in the ileum and drains liquid stool frequently, unlike the colostomy which has more formed stool the further distal the ostomy is in the colon. The ileostomy pouch is usually worn 4-7 days or until it leaks. It must be changed immediately if it leaks because the drainage is very irritating to the skin. To avoid obstruction, popcorn, dried fruit, coconut, mushrooms, olives, stringy vegetables, food with skin, and meats with casings must be chewed extremely well before swallowing because of the narrow diameter of the ileostomy lumen.

The nurse is evaluating a 45-year-old patient diagnosed with type 2 diabetes mellitus. Which symptom reported by the patient is considered one of the classic clinical manifestations of diabetes? a. Excessive thirst b. Gradual weight gain c. Overwhelming fatigue d. Recurrent blurred vision

a. Excessive thirst The classic symptoms of diabetes are polydipsia (excessive thirst), polyuria, (excessive urine output), and polyphagia (increased hunger). Weight gain, fatigue, and blurred vision may all occur with type 2 diabetes, but are not classic manifestations.

A 45-year-old male patient with suspected acromegaly is seen at the clinic. To assist in making the diagnosis, which question should the nurse ask? a. "Have you had a recent head injury?" b. "Do you have to wear larger shoes now?" c. "Is there a family history of acromegaly?" d. "Are you experiencing tremors or anxiety?"

b. "Do you have to wear larger shoes now?" Acromegaly causes an enlargement of the hands and feet. Head injury and family history are not risk factors for acromegaly. Tremors and anxiety are not clinical manifestations of acromegaly

The nurse is planning care for a 68-year-old patient with an abdominal mass and suspected bowel obstruction. Which factor in the patient's history increases the patient's risk for colorectal cancer? a. Osteoarthritis b. History of colorectal polyps c. History of lactose intolerance d. Use of herbs as dietary supplements

b. History of colorectal polyps A history of colorectal polyps places this patient at risk for colorectal cancer. This tissue can degenerate over time and become malignant. Osteoarthritis, lactose intolerance, and the use of herbs do not pose additional risk to the patient.

Which laboratory value reported to the nurse by the unlicensed assistive personnel (UAP) indicates the most urgent need for the nurse's assessment of the patient? a. Bedtime glucose of 140 mg/dL b. Noon blood glucose of 52 mg/dL c. Fasting blood glucose of 130 mg/dL d. 2-hr postprandial glucose of 220 mg/dL

b. Noon blood glucose of 52 mg/dL

The female patient with a UTI also has renal calculi. The nurse knows that these are most likely which type of stone? a. Cystine b. Struvite c. Uric acid d. Calcium phosphate

b. Struvite calculi are most common in women and always occur with UTIs. They are also usually large staghorn type.

With which diagnosis will the patient benefit from being taught to do self-catheterization? a. Renal trauma b. Urethral stricture c. Renal artery stenosis d. Accelerated nephrosclerosis

b. The patient with urethral stricture will benefit from being taught to dilate the urethra by self-catheterization every few days. Renal trauma is treated related to the severity of the injury with bed rest, fluids, and analgesia. Renal artery stenosis includes control of hypertension with possible surgical revascularization. Accelerated nephrosclerosis is associated with malignant hypertension that must be aggressively treated as well as monitoring kidney function.

The nurse is interviewing a new patient with diabetes who receives rosiglitazone (Avandia) through a restricted access medication program. What is most important for the nurse to report immediately to the health care provider? a. The patient's blood pressure is 154/92. b. The patient has a history of emphysema. c. The patient's blood glucose is 86 mg/dL. d. The patient has chest pressure when walking.

d. The patient has chest pressure when walking.

A patient is diagnosed with celiac disease following a workup for iron deficiency anemia and decreased bone density. The nurse identifies that additional teaching about disease management is needed when the patient makes which statement?

"I don't need to restrict gluten intake because I don't have diarrhea or bowel symptoms."

The nurse identifies a need for additional teaching when a patient with acute infectious diarrhea makes which statement?

"I may use over the counter Imodium or Parepectolin when I need to control my diarrhea."

The primary health care provider concludes that a patient has metabolic alkalosis. Which signs in the patient help the primary health care provider to reach this conclusion? Select all that apply. 1 Tremors 2 Vomiting 3 Tachycardia 4 Epigastric pain 5 Numbness of limbs

1, 2, 3 Tremors, vomiting, and tachycardia are signs of metabolic alkalosis. Epigastric pain and numbness of limbs are signs of respiratory alkalosis. Text Reference - p. 305

A nurse is caring for a patient with malignant lung cancer who experiences weakness, lethargy, depressed reflexes, and bone pain. The nurse suspects the patient may have hypercalcemia. Which changes in the electrocardiogram indicate hypercalcemia? Select all that apply. 1 Shortened ST segment 2 Elongation of ST segment 3 Shortened QT interval 4 Prolonged QT interval 5 Flattened or inverted T wave

1, 3 Hypercalcemia may result from malignancies. Bone destruction due to tumor invasion may cause a release of calcium, leading to high levels of calcium in the blood. This causes altered transmembrane potentials affecting conduction time, and is manifested as a shortened ST segment and QT interval. An elongated ST segment and a prolonged QT interval are manifestations of hypocalcemia. A flattened or inverted T wave is a manifestation of hypokalemia. Text Reference - p. 299

The nurse reviews a patient's glycosylated hemoglobin (Hb A1C) results to evaluate A. Fasting preprandial glucose levels. B. Glucose levels 2 hours after a meal. C. Glucose control over the past 90 days. D. Hypoglycemic episodes in the past 3 months.

C. Glucose control over the past 90 days. - Glycosylated hemoglobin testing measures glucose control over the last 3 months. Glucose testing before/after a meal or random testing may reveal impaired glucose tolerance and indicate prediabetes, but it is not done on patients who already have a diagnosis of diabetes. There is no test to evaluate for hypoglycemic episodes in the past.

The nurse is caring for a patient admitted with heart failure. The morning laboratory results reveal a serum potassium level of 2.9 mEq/L. Which classification of medications should be withheld until consulting with the health care provider? 1 Antibiotics 2 Loop diuretics 3 Bronchodilators 4 Antihypertensives

2 Loop diuretics are contraindicated during episodes of hypokalemia because these medications cause the kidneys to excrete sodium and potassium. Thus, administration of this type of medication at this time would worsen the hypokalemia, putting the patient at risk for dysrhythmias. The prescribing health care provider should be consulted for potassium replacement therapy, and the drug should be withheld until the potassium has returned to normal range. Text Reference - p. 296

While documenting the arterial blood gas values of a group of patients, the nurse suspects a patient to have respiratory alkalosis. Which patient's findings support the nurse's suspicion? 1 Patient A 2 Patient B 3 Patient C 4 Patient D

2 Respiratory alkalosis is characterized by an increased pH and decreased carbon dioxide concentration (PaCO2) in blood. The normal values of blood pH, partial pressure of carbon dioxide (PaCO2), and bicarbonate ion (HCO3-) are between 7.35 and 7.45, 35 and 45 mm Hg, and 22 and 26 mEq/L, respectively. The increased pH and decreased PaCO2 in patient B are indicators of respiratory alkalosis. Test-Taking Tip: Respiratory alkalosis is caused by hyperventilation. Text Reference - p. 304

The surgical-unit nurse notes that a patient has an intravenous (IV) prescription for 0.9% normal saline (NS) with 20 mEq KCl/L at 100 mL/hr. The nurse regulates the IV flow rate at how many drops (gtt) per minute, noting that the tubing has a drop factor of 10 drops/mL? 1 15 gtt/minute 2 17 gtt/minute 3 19 gtt/minute 4 21 gtt/minute

2 Use the following formula to calculate the rate of IV solutions: Volume × drop factor, divided by time (in minutes). Multiply 100 by 10 to yield 1000 and divide this by 60 to yield 16.6 or 17 gtt/minute (because the nurse cannot count a fraction of a drop). Text Reference - p. 298

While performing patient teaching regarding hypercalcemia, which statements are appropriate? Select all that apply. 1 Have the patient restrict fluid intake to less than 2000 mL/day. 2 Renal calculi may occur as a complication of hypercalcemia. 3 Weight-bearing exercises can help keep calcium in the bones. 4 The patient should increase daily fluid intake to 3000 to 4000 mL. 5 Treatment of heartburn can be managed best with Tums as needed.

2, 3, 4 A daily fluid intake of 3000 to 4000 mL is necessary to enhance calcium excretion and prevent the formation of renal calculi, a potential complication of hypercalcemia. Weight-bearing exercise does enhance bone mineralization. Tums are a calcium-based antacid that should not be used in patients with hypercalcemia. Text Reference - p. 299

After reviewing the laboratory reports of four patients, the primary health care provider orders the nurse to prepare one of the patients for mechanical ventilation. Which patient's reports indicate the need for this intervention? 1 Patient A 2 Patient B 3 Patient C 4 Patient D

3 A need for mechanical ventilation arises when the patient is not able to breathe properly. This is manifested by decreased oxygen and increased carbon dioxide in blood. The normal partial pressure of carbon dioxide (PaCO2) value lies between 35 and 45 mm Hg, and the normal range of blood pH is 7.35 to 7.45. Patient C has an increased concentration of carbon dioxide in the blood and a low pH, which indicate that the patient has difficulty breathing and requires mechanical ventilation. Text Reference - p. 304

A patient asks why the primary health care provider prescribed a b-type natriuretic peptide (BNP). Which response by the nurse is accurate? 1 It is a diagnostic procedure to rule out urine retention. 2 It is a blood test that is elevated in patients with hyponatremia. 3 It is a blood test that shows if there is excess fluid in the heart. 4 It is an x-ray that helps determine the presence of stomach ulcers

3 BNP is a hormone that is produced when the atrial pressure increases. This blood test is used to diagnose the severity and treatment outcomes of congestive heart failure (CHF). The atrial pressure increases because of increased venous return and hypernatremia. The test gives no information to rule out urine retention or the presence of stomach ulcers. A serum sodium level is needed to determine hyponatremia. Text Reference - p. 290

A patient is admitted with alcohol abuse. Laboratory data reveals a phosphate level of 1.8 mg/dL. Which assessment finding is consistent with this data? 1 Seizure activity 2 Diarrhea 3 Weakness 4 Tetany

3 Signs of hypophosphatemia include weakness, confusion, coma, and diminished reflexes. Seizure activity, diarrhea, and tetany are not associated with this electrolyte imbalance. Text Reference - p. 301

An 82-year-old patient is admitted with pneumonia. The nurse monitors the patient's intake and output carefully, knowing that the patient is at risk for fluid and electrolyte imbalances primarily because 1 Older adults are at an increased risk of impaired renal function. 2 Older adults have an impaired level of consciousness and need to be reminded to drink fluids. 3 Older adults are more likely than younger adults to lose extracellular fluid during severe illnesses. 4 Small losses of fluid are more significant because body water accounts for only about 50% of body weight in older adults.

4 Older adults, with less muscle mass and more fat content, have less body water than younger adults. In the older adult, body water content averages 45% to 55% of body weight, leaving them at a higher risk for fluid-related problems than young adults. Text Reference - p. 291

What is the normal range of blood pH? 1 7.05 to 7.15 2 7.15 to 7.25 3 7.25 to 7.35 4 7.35 to 7.45

4 The normal range of blood pH is 7.35 to 7.45. A pH less than 7.35 indicates acidosis. Text Reference - p. 302

The nurse needs to add potassium chloride 10 mEq to an intravenous (IV) of lactated Ringer's solution. Available are multidose vials containing 5 mEq KCl in 10 mL of solution. How many milliliters of KCl should be added to the IV bag? 1. 2 mL 2. 5 mL 3. 10 mL 4. 20 mL

4 Using ratio and proportion, multiply 5 by x and multiply 10 × 10 to yield 5x = 100. Divide 100 by 5 to yield 20 mL. Text Reference - p. 298

Which finding indicates to the nurse that the current therapies are effective for a patient with acute adrenal insufficiency? a. Increasing serum sodium levels b. Decreasing blood glucose levels c. Decreasing serum chloride levels d. Increasing serum potassium levels

A Clinical manifestations of Addison's disease include hyponatremia and an increase in sodium level indicates improvement. The other values indicate that treatment has not been effective.

Which finding indicates to the nurse that the current therapies are effective for a patient with acute adrenal insufficiency? a. Increasing serum sodium levels b. Decreasing blood glucose levels c. Decreasing serum chloride levels d. Increasing serum potassium levels

A Clinical manifestations of Addison's disease include hyponatremia and an increase in sodium level indicates improvement. The other values indicate that treatment has not been effective.

A patient complains of nausea. When administering a dose of metoclopramide (Reglan), the nurse should teach the patient to report which potential adverse effect? a) Tremors b) Constipation c) Double vision d) Numbness in fingers and toes

A Extrapyramidal side effects, including tremors and tardive dyskinesias, may occur as a result of metoclopramide (Reglan) administration. Constipation, double vision, and numbness in fingers and toes are not adverse effects of metoclopramide.

The nurse providing care for a patient who has an adrenocortical adenoma causing hyperaldosteronism should a. monitor the blood pressure every 4 hours. b. elevate the patient's legs to relieve edema. c. monitor blood glucose level every 4 hours. d. order the patient a potassium-restricted diet.

A Hypertension caused by sodium retention is a common complication of hyperaldosteronism. Hyperaldosteronism does not cause an elevation in blood glucose. The patient will be hypokalemic and require potassium supplementation before surgery. Edema does not usually occur with hyperaldosteronism.

The nurse determines that additional instruction is needed for a patient with chronic syndrome of inappropriate antidiuretic hormone (SIADH) when the patient makes which statement? a. "I need to shop for foods low in sodium and avoid adding salt to food." b. "I should weigh myself daily and report any sudden weight loss or gain." c. "I need to limit my fluid intake to no more than 1 quart of liquids a day." d. "I should eat foods high in potassium because diuretics cause potassium loss."

A Patients with SIADH are at risk for hyponatremia, and a sodium supplement may be prescribed. The other patient statements are correct and indicate successful teaching has occurred.

The nurse is caring for a patient admitted with diabetes insipidus (DI). Which information is most important to report to the health care provider? a. The patient is confused and lethargic. b. The patient reports a recent head injury. c. The patient has a urine output of 400 mL/hr. d. The patient's urine specific gravity is 1.003.

A The patient's confusion and lethargy may indicate hypernatremia and should be addressed quickly. In addition, patients with DI compensate for fluid losses by drinking copious amounts of fluids, but a patient who is lethargic will be unable to drink enough fluids and will become hypovolemic. A high urine output, low urine specific gravity, and history of a recent head injury are consistent with diabetes insipidus, but they do not require immediate nursing action to avoid life-threatening complications.

On examining a patient 8 hours after having surgery to create a colostomy, what should the nurse expect to find?

A brick- red, puffy stoma that oozes blood.

A

A newly admitted patient is diagnosed with hyponatremia. When making room assignments, the charge nurse should take which action? a. Assign the patient to a room near the nurse's station. b. Place the patient in a room nearest to the water fountain. c. Place the patient on telemetry to monitor for peaked T waves. d. Assign the patient to a semi-private room and place an order for a low-salt diet.

A

A nurse in the outpatient clinic is caring for a patient who has a magnesium level of 1.3 mg/dL. Which assessment would be most important for the nurse to make? a. Daily alcohol intake b. Intake of dietary protein c. Multivitamin/mineral use d. Use of over-the-counter (OTC) laxatives

C

A nurse is assessing a newly admitted patient with chronic heart failure who forgot to take prescribed medications and seems confused. The patient complains of "just blowing up" and has peripheral edema and shortness of breath. Which assessment should the nurse complete first? a. Skin turgor b. Heart sounds c. Mental status d. Capillary refill

B

A patient comes to the clinic complaining of frequent, watery stools for the last 2 days. Which action should the nurse take first? a. Obtain the baseline weight. b. Check the patient's blood pressure. c. Draw blood for serum electrolyte levels. d. Ask about any extremity numbness or tingling.

C

A patient has a parenteral nutrition infusion of 25% dextrose. A student nurse asks the nurse why a peripherally inserted central catheter was inserted. Which response by the nurse is most appropriate? a. "There is a decreased risk for infection when 25% dextrose is infused through a central line." b. "The prescribed infusion can be given much more rapidly when the patient has a central line." c. "The 25% dextrose is hypertonic and will be more rapidly diluted when given through a central line." d. "The required blood glucose monitoring is more accurate when samples are obtained from a central line."

B

A patient is admitted to the emergency department with severe fatigue and confusion. Laboratory studies are done. Which laboratory value will require the most immediate action by the nurse? a. Arterial blood pH is 7.32. b. Serum calcium is 18 mg/dL. c. Serum potassium is 5.1 mEq/L. d. Arterial oxygen saturation is 91%.

D

A patient is receiving a 3% saline continuous IV infusion for hyponatremia. Which assessment data will require the most rapid response by the nurse? a. The patient's radial pulse is 105 beats/minute. b. There is sediment and blood in the patient's urine. c. The blood pressure increases from 120/80 to 142/94. d. There are crackles audible throughout both lung fields.

B

A patient who had a transverse colectomy for diverticulosis 18 hours ago has nasogastric suction and is complaining of anxiety and incisional pain. The patient's respiratory rate is 32 breaths/minute and the arterial blood gases (ABGs) indicate respiratory alkalosis. Which action should the nurse take first? a. Discontinue the nasogastric suction. b. Give the patient the PRN IV morphine sulfate 4 mg. c. Notify the health care provider about the ABG results. d. Teach the patient how to take slow, deep breaths when anxious.

C

A patient who has a small cell carcinoma of the lung develops syndrome of inappropriate antidiuretic hormone (SIADH). The nurse should notify the health care provider about which assessment finding? a. Reported weight gain b. Serum hematocrit of 42% c. Serum sodium level of 120 mg/dL d. Total urinary output of 280 mL during past 8 hours

D

A patient who is taking a potassium-wasting diuretic for treatment of hypertension complains of generalized weakness. It is most appropriate for the nurse to take which action? a. Assess for facial muscle spasms. b. Ask the patient about loose stools. c. Suggest that the patient avoid orange juice with meals. d. Ask the health care provider to order a basic metabolic panel.

D

A patient who was involved in a motor vehicle crash has had a tracheostomy placed to allow for continued mechanical ventilation. How should the nurse interpret the following arterial blood gas results: pH 7.48, PaO2 85 mm Hg, PaCO2 32 mm Hg, and HCO3 25 mEq/L? a. Metabolic acidosis b. Metabolic alkalosis c. Respiratory acidosis d. Respiratory alkalosis

A

A patient with renal failure has been taking aluminum hydroxide/magnesium hydroxide suspension (Maalox) at home for indigestion. The patient arrives for outpatient hemodialysis and is unresponsive to questions and has decreased deep tendon reflexes. Which action should the dialysis nurse take first? a. Notify the patient's health care provider. b. Obtain an order to draw a potassium level. c. Review the magnesium level on the patient's chart. d. Teach the patient about the risk of magnesium-containing antacids

A

A postoperative patient who had surgery for a perforated gastric ulcer has been receiving nasogastric suction for 3 days. The patient now has a serum sodium level of 127 mEq/L (127 mmol/L). Which prescribed therapy should the nurse question? a. Infuse 5% dextrose in water at 125 mL/hr. b. Administer IV morphine sulfate 4 mg every 2 hours PRN. c. Give IV metoclopramide (Reglan) 10 mg every 6 hours PRN for nausea. d. Administer 3% saline if serum sodium decreases to less than 128 mEq/L.

Which statements will the nurse include when teaching a patient who is scheduled for oral glucose tolerance testing in the outpatient clinic (select all that apply)? A. "You will need to avoid smoking before the test." B. "Exercise should be avoided until the testing is complete." C. "Several blood samples will be obtained during the testing." D. "You should follow a low-calorie diet the day before the test." E. "The test requires that you fast for at least 8 hours before testing."

A, C, E A. "You will need to avoid smoking before the test." C. "Several blood samples will be obtained during the testing." E. "The test requires that you fast for at least 8 hours before testing." - Smoking may affect the results of oral glucose tolerance tests. Blood samples are obtained at baseline and at 30, 60, and 120 minutes. Accuracy requires that the patient be fasting before the test. The patient should consume at least 1500 calories/day for 3 days before the test. The patient should be ambulatory and active for accurate test results.

The nurse is caring for a 63-year-old with a possible pituitary tumor who is scheduled for a computed tomography (CT) scan with contrast. Which information about the patient is most important to discuss with the health care provider before the test? A. History of renal insufficiency B. Complains of chronic headache C. Recent bilateral visual field loss D, Blood glucose level of 134 mg/dL

A. History of renal insufficiency - Because contrast media may cause acute kidney injury in patients with poor renal function, the health care provider will need to prescribe therapies such as IV fluids to prevent this complication. The other findings are consistent with the patient's diagnosis of a pituitary tumor.

51. Which menu choice by the patient with diverticulosis is best for preventing diverticulitis? a. Navy bean soup and vegetable salad b. Whole grain pasta with tomato sauce c. Baked potato with low-fat sour cream d. Roast beef sandwich on whole wheat bread

ANS: A A diet high in fiber and low in fats and red meat is recommended to prevent diverticulitis. Although all of the choices have some fiber, the bean soup and salad will be the highest in fiber and the lowest in fat.

Which menu choice by the patient with diverticulosis is best for preventing diverticulitis? a. Navy bean soup and vegetable salad b. Whole grain pasta with tomato sauce c. Baked potato with low-fat sour cream d. Roast beef sandwich on whole wheat bread

ANS: A A diet high in fiber and low in fats and red meat is recommended to prevent diverticulitis. Although all of the choices have some fiber, the bean soup and salad will be the highest in fiber and the lowest in fat.

A 50-year-old female patient calls the clinic to report a new onset of severe diarrhea. The nurse anticipates that the patient will need to a. collect a stool specimen. b. prepare for colonoscopy. c. schedule a barium enema. d. have blood cultures drawn.

ANS: A Acute diarrhea is usually caused by an infectious process, and stool specimens are obtained for culture and examined for parasites or white blood cells. There is no indication that the patient needs a colonoscopy, blood cultures, or a barium enema.

When admitting a patient with acute glomerulonephritis, it is most important that the nurse ask the patient about a. recent sore throat and fever. b. history of high blood pressure. c. frequency of bladder infections. d. family history of kidney stones.

ANS: A Acute glomerulonephritis frequently occurs after a streptococcal infection such as strep throat. It is not caused by hypertension, urinary tract infection (UTI), or kidney stones. DIF: Cognitive Level: Application REF: 1131-1132

A 34-year-old female patient with a new ileostomy asks how much drainage to expect. The nurse explains that after the bowel adjusts to the ileostomy, the usual drainage will be about _____ cups. a. 2 b. 3 c. 4 d. 5

ANS: A After the proximal small bowel adapts to reabsorb more fluid, the average amount of ileostomy drainage is about 500 mL daily. One cup is about 240 mL.

8. Which nursing action will be included in the plan of care for a 27-year-old male patient with bowel irregularity and a new diagnosis of irritable bowel syndrome (IBS)? a. Encourage the patient to express concerns and ask questions about IBS. b. Suggest that the patient increase the intake of milk and other dairy products. c. Educate the patient about the use of alosetron (Lotronex) to reduce symptoms. d. Teach the patient to avoid using nonsteroidal antiinflammatory drugs (NSAIDs).

ANS: A Because psychologic and emotional factors can affect the symptoms for IBS, encouraging the patient to discuss emotions and ask questions is an important intervention. Alosetron has serious side effects, and is used only for female patients who have not responded to other therapies. Although yogurt may be beneficial, milk is avoided because lactose intolerance can contribute to symptoms in some patients. NSAIDs can be used by patients with IBS.

20. What should a nurse include when drawing up a patient's diabetes teaching plan? a. Develop an exercise plan because regular exercise helps control blood glucose levels. b. Monitor blood sugar levels only if not feeling well to ensure that the fingertips are not pricked too much. c. If nervousness, palpitations, or hunger is experienced, take a small dose (1 to 2 U) of regular insulin and call the physician. d. Use over-the-counter measures for any foot blisters, calluses, or wounds before seeking medical help.

ANS: A Exercise is an integral part of the patient's ability to take charge of his or her diabetes and needs to be included in the teaching plan.

21. The health care provider orders intravenous (IV) ranitidine (Zantac) for a patient with gastrointestinal (GI) bleeding caused by peptic ulcer disease. When teaching the patient about the effect of the medication, which information will the nurse include? a. "Ranitidine decreases secretion of gastric acid." b. "Ranitidine neutralizes the acid in the stomach." c. "Ranitidine constricts the blood vessels in the stomach and decreases bleeding." d. "Ranitidine covers the ulcer with a protective material that promotes healing."

ANS: A Ranitidine is a histamine-2 (H2) receptor blocker, which decreases the secretion of gastric acid. The response beginning, "Ranitidine constricts the blood vessels" describes the effect of vasopressin. The response beginning "Ranitidine neutralizes the acid" describes the effect of antacids. And the response beginning "Ranitidine covers the ulcer" describes the action of sucralfate (Carafate).

Which nursing action will be most helpful in decreasing the risk for hospital-acquired infection (HAI) of the urinary tract in patients admitted to the hospital? a. Avoid unnecessary catheterizations. b. Encourage adequate oral fluid intake. c. Test urine with a dipstick daily for nitrites. d. Provide thorough perineal hygiene to patients.

ANS: A Since catheterization bypasses many of the protective mechanisms that prevent urinary tract infection (UTI), avoidance of catheterization is the most effective means of reducing HAI. The other actions will also be helpful, but are not as useful as decreasing urinary catheter use. DIF: Cognitive Level: Application REF: 1125-1127

32. When counseling a patient with a family history of stomach cancer about ways to decrease risk for developing stomach cancer, the nurse will teach the patient to avoid a. smoked foods such as bacon and ham. b. foods that cause abdominal distention. c. chronic use of H2 blocking medications. d. emotionally or physically stressful situations.

ANS: A Smoked foods such as bacon, ham, and smoked sausage increase the risk for stomach cancer. Use of H2 blockers, stressful situations, and abdominal distention are not associated with an increased incidence of stomach cancer.

9. What glomerular filtration rate (GFR) would the nurse estimate for a 30-year-old patient with a creatinine clearance result of 60 mL/min? a. 60 mL/min b. 90 mL/min c. 120 mL/min d. 180 mL/min

ANS: A The creatinine clearance approximates the GFR. The other responses are not accurate.

A patient who is diagnosed with nephrotic syndrome has 3+ ankle and leg edema and ascites. Which nursing diagnosis is a priority for the patient? a. Excess fluid volume related to low serum protein levels b. Activity intolerance related to increased weight and fatigue c. Disturbed body image related to peripheral edema and ascites d. Altered nutrition: less than required related to protein restriction

ANS: A The patient has massive edema, so the priority problem at this time is the excess fluid volume. The other nursing diagnoses also are appropriate, but the focus of nursing care should be resolution of the edema and ascites. DIF: Cognitive Level: Application REF: 1133-1135

27. A patient recovering from a gastrojejunostomy (Billroth II) for treatment of a duodenal ulcer develops dizziness, weakness, and palpitations about 20 minutes after eating. To avoid recurrence of these symptoms, the nurse teaches the patient to a. lie down for about 30 minutes after eating. b. choose foods that are high in carbohydrates. c. increase the amount of fluid intake with meals. d. drink sugared fluids or eat candy after each meal.

ANS: A The patient is experiencing symptoms of dumping syndrome, which may be reduced by lying down after eating. Increasing fluid intake and choosing high carbohydrate foods will increase the risk for dumping syndrome. Having a sweet drink or hard candy will correct the hypoglycemia that is associated with dumping syndrome but will not prevent dumping syndrome.

A patient who has been receiving diuretic therapy is admitted to the emergency department with a serum potassium level of 3.1 mEq/L. Of the following medications that the patient has been taking at home, the nurse will be most concerned about a. oral digoxin (Lanoxin) 0.25 mg daily. b. ibuprofen (Motrin) 400 mg every 6 hours. c. metoprolol (Lopressor) 12.5 mg orally daily. d. lantus insulin 24 U subcutaneously every evening.

ANS: A oral digoxin (Lanoxin) 0.25 mg daily Hypokalemia increases the risk for digoxin toxicity, which can cause serious dysrhythmias. The nurse also will need to do more assessment regarding the other medications, but there is not as much concern with the potassium level.

28. How is the Whipple triad described? (Select all that apply.) a. Symptoms of hypoglycemia are present. b. Low blood glucose levels are documented when symptoms are present. c. Symptoms can be reproduced with an injection of regular insulin, 10 units. d. Muscular activity does not have any effect on blood glucose. e. Symptoms improved when the blood glucose level rises.

ANS: A, B, E Whipple triad is the presence of the symptoms of hypoglycemia (e.g., diaphoresis, pallor, tachycardia), the documentation of low blood glucose levels when symptoms are present, and the improvement of symptoms as the blood glucose level rises.

34. Which information about a patient who has just been admitted to the hospital with nausea and vomiting will require the most rapid intervention by the nurse? a. The patient has taken only sips of water. b. The patient is lethargic and difficult to arouse. c. The patient's chart indicates a recent resection of the small intestine. d. The patient has been vomiting several times a day for the last 4 days.

ANS: B A lethargic patient is at risk for aspiration, and the nurse will need to position the patient to decrease aspiration risk. The other information also is important to collect, but it does not require as quick action as the risk for aspiration.

46. A 33-year-old male patient with a gunshot wound to the abdomen undergoes surgery, and a colostomy is formed as shown in the accompanying figure. Which information will be included in patient teaching? a. Stool will be expelled from both stomas. b. This type of colostomy is usually temporary. c. Soft, formed stool can be expected as drainage. d. Irrigations can regulate drainage from the stomas.

ANS: B A loop, or double-barrel stoma, is usually temporary. Stool will be expelled from the proximal stoma only. The stool from the transverse colon will be liquid and regulation through irrigations will not be possible.

53. The nurse is admitting a 67-year-old patient with new-onset steatorrhea. Which question is most important for the nurse to ask? a. "How much milk do you usually drink?" b. "Have you noticed a recent weight loss?" c. "What time of day do your bowels move?" d. "Do you eat meat or other animal products?"

ANS: B Although all of the questions provide useful information, it is most important to determine if the patient has an imbalance in nutrition because of the steatorrhea.

18. The nurse preparing for the annual physical exam of a 50-year-old man will plan to teach the patient about a. endoscopy. b. colonoscopy. c. computerized tomography screening. d. carcinoembryonic antigen (CEA) testing.

ANS: B At age 50, individuals with an average risk for colorectal cancer (CRC) should begin screening for CRC. Colonoscopy is the gold standard for CRC screening. The other diagnostic tests are not recommended as part of a routine annual physical exam at age 50.

7. When a patient with type 2 diabetes says, "Why in the world are they looking at my hemoglobin? I thought my problem was with my blood sugar." What should the nurse explain about the level of hemoglobin A1c? a. Shows how a high level of glucose can cause a significant drop in the hemoglobin level b. Shows what the glucose level has done during the past 3 months c. Indicates a true picture of the patient's nutritional state d. Reflects the effect of a high level of glucose on the ability to produce red blood cells (RBCs)

ANS: B By analyzing the amount of glucose bound to the hemoglobin, the level of blood glucose can be evaluated for the past 3 months because the glucose stays bound to the hemoglobin for the life of the RBC.

38. Which of these assessment findings in a patient with a hiatal hernia who returned from a laparoscopic Nissen fundoplication 4 hours ago is most important for the nurse to address immediately? a. The patient is experiencing intermittent waves of nausea. b. The patient has absent breath sounds throughout the left lung. c. The patient has decreased bowel sounds in all four quadrants. d. The patient complains of 6/10 (0 to 10 scale) abdominal pain.

ANS: B Decreased breath sounds on one side may indicate a pneumothorax, which requires rapid diagnosis and treatment. The abdominal pain and nausea also should be addressed but they are not as high priority as the patient's respiratory status. The patient's decreased bowel sounds are expected after surgery and require ongoing monitoring but no other action.

33. A 54-year-old critically ill patient with sepsis is frequently incontinent of watery stools. What action by the nurse will prevent complications associated with ongoing incontinence? a. Apply incontinence briefs. b. Use a fecal management system c. Insert a rectal tube with a drainage bag. d. Assist the patient to a commode frequently.

ANS: B Fecal management systems are designed to contain loose stools and can be in place for as long as 4 weeks without causing damage to the rectum or anal sphincters. Although incontinence briefs may be helpful, unless they are changed frequently, they are likely to increase the risk for skin breakdown. Rectal tubes are avoided because of possible damage to the anal sphincter and ulceration of the rectal mucosa. A critically ill patient will not be able to tolerate getting up frequently to use the commode or bathroom.

After receiving change-of-shift report about the following four patients, which patient should the nurse assess first? a. A 31-year-old female with Cushing syndrome and a blood glucose level of 244 mg/dL b. A 70-year-old female taking levothyroxine (Synthroid) who has an irregular pulse of 134 c. A 53-year-old male who has Addison's disease and is due for a scheduled dose of hydrocortisone (Solu-Cortef). d. A 22-year-old male admitted with syndrome of inappropriate antidiuretic hormone (SIADH) who has a serum sodium level of 130 mEq/L

ANS: B Initiation of thyroid replacement in older adults may cause angina and cardiac dysrhythmias. The patient's high pulse rate needs rapid investigation by the nurse to assess for and intervene with any cardiac problems. The other patients also require nursing assessment and/or actions but are not at risk for life-threatening complications

21. Which information will the nurse include when teaching a patient with peptic ulcer disease about the effect of ranitidine (Zantac)? a. "Ranitidine absorbs the gastric acid." b. "Ranitidine decreases gastric acid secretion." c. "Ranitidine constricts the blood vessels near the ulcer." d. "Ranitidine covers the ulcer with a protective material."

ANS: B Ranitidine is a histamine-2 (H2) receptor blocker, which decreases the secretion of gastric acid. The response beginning, "Ranitidine constricts the blood vessels" describes the effect of vasopressin. The response "Ranitidine absorbs the gastric acid" describes the effect of antacids. The response beginning "Ranitidine covers the ulcer" describes the action of sucralfate (Carafate).

6. A 58-year-old man with blunt abdominal trauma from a motor vehicle crash undergoes peritoneal lavage. If the lavage returns brown fecal drainage, which action will the nurse plan to take next? a. Auscultate the bowel sounds. b. Prepare the patient for surgery. c. Check the patient's oral temperature. d. Obtain information about the accident.

ANS: B Return of brown drainage and fecal material suggests perforation of the bowel and the need for immediate surgery. Auscultation of bowel sounds, checking the temperature, and obtaining information about the accident are appropriate actions, but the priority is to prepare to send the patient for emergency surgery.

Which patient statement indicates that the nurse's teaching about sulfasalazine (Azulfidine) for ulcerative colitis has been effective? a. "The medication will be tapered if I need surgery." b. "I will need to use a sunscreen when I am outdoors." c. "I will need to avoid contact with people who are sick." d. "The medication will prevent infections that cause the diarrhea."

ANS: B Sulfasalazine may cause photosensitivity in some patients. It is not used to treat infections. Sulfasalazine does not reduce immune function. Unlike corticosteroids, tapering of sulfasalazine is not needed.

A 63-year-old patient with primary hyperparathyroidism has a serum phosphorus level of 1.7 mg/dL (0.55 mmol/L) and calcium of 14 mg/dL (3.5 mmol/L). Which nursing action should be included in the plan of care? a. Restrict the patient to bed rest. b. Encourage 4000 mL of fluids daily. c. Institute routine seizure precautions. d. Assess for positive Chvostek's sign.

ANS: B The patient with hypercalcemia is at risk for kidney stones, which may be prevented by a high fluid intake. Seizure precautions and monitoring for Chvostek's or Trousseau's sign are appropriate for hypocalcemic patients. The patient should engage in weight-bearing exercise to decrease calcium loss from bone

6. A 79-year-old man has been admitted with benign prostatic hyperplasia. What is most appropriate to include in the nursing plan of care? a. Limit fluid intake to no more than 1000 mL/day. b. Leave a light on in the bathroom during the night. c. Ask the patient to use a urinal so that urine can be measured. d. Pad the patient's bed to accommodate overflow incontinence.

ANS: B The patient's age and diagnosis indicate a likelihood of nocturia, so leaving the light on in the bathroom is appropriate. Fluids should be encouraged because dehydration is more common in older patients. The information in the question does not indicate that measurement of the patient's output is necessary or that the patient has overflow incontinence.

Following a thyroidectomy, a patient complains of "a tingling feeling around my mouth." The nurse will immediately check for a. an elevated serum potassium level. b. the presence of Chvostek's sign. c. a decreased thyroid hormone level. d. bleeding on the patient's dressing.

ANS: B the presence of Chvostek's sign. The patient's symptoms indicate possible hypocalcemia, which can occur secondary to parathyroid injury/removal during thyroidectomy. There is no indication of a need to check the potassium level, the thyroid hormone level, or for bleeding.

14. The nurse will plan to teach the patient with newly diagnosed achalasia that a. a liquid or blenderized diet will be necessary. b. drinking fluids with meals should be avoided. c. endoscopic procedures may be used for treatment. d. lying down and resting after meals is recommended.

ANS: C Endoscopic and laparoscopic procedures are the most effective therapy for improving symptoms caused by achalasia. Patients are advised to drink fluid with meals. Keeping the head elevated after eating will improve esophageal emptying. A semisoft diet is recommended to improve esophageal emptying.

11. A home health care nurse is assessing a patient with type 1 diabetes who has been controlled for 6 months. The nurse is surprised and concerned about a blood glucose reading of 52 mg/dL. What action by this patient most likely caused this episode of hypoglycemia? a. Taking a new form of birth control pill this morning b. Using large amounts of sugar substitute in her tea this morning c. A 2-hour long exercise class at the spa this morning d. Administering an insufficient dose of insulin this morning

ANS: C Excessive exercise used up the glucose that was made available by the insulin taken by the patient. The patient now has too much insulin for the available glucose and has become hypoglycemic.

43. An 80-year-old who is hospitalized with peptic ulcer disease develops new-onset auditory hallucinations. Which prescribed medication will the nurse discuss with the health care provider before administration? a. Sucralfate (Carafate) b. Omeprazole (Prilosec) c. Metoclopramide (Reglan) d. Aluminum hydroxide (Amphojel)

ANS: C Metoclopramide can cause central nervous system (CNS) side effects ranging from anxiety to hallucinations. Hallucinations are not a side effect of proton-pump inhibitors, mucosal protectants, or antacids

2. When a patient's urine dipstick test indicates a small amount of protein, the nurse's next action should be to a. send a urine specimen to the laboratory to test for ketones. b. obtain a clean-catch urine for culture and sensitivity testing. c. inquire about which medications the patient is currently taking. d. ask the patient about any family history of chronic renal failure.

ANS: C Normally the urinalysis will show zero to trace amounts of protein, but some medications may give false-positive readings. The other actions by the nurse may be appropriate, but checking for medications that may affect the dipstick accuracy should be done first.

A patient's renal calculus is analyzed as being very high in uric acid. To prevent recurrence of stones, the nurse teaches the patient to avoid eating a. milk and dairy products. b. legumes and dried fruits. c. organ meats and sardines. d. spinach, chocolate, and tea.

ANS: C Organ meats and fish such as sardines increase purine levels and uric acid. Spinach, chocolate, and tomatoes should be avoided in patients who have oxalate stones. Milk, dairy products, legumes, and dried fruits may increase the incidence of calcium-containing stones. DIF: Cognitive Level: Application REF: 1139

Which information will the nurse include when teaching the patient with a urinary tract infection (UTI) about the use of phenazopyridine (Pyridium)? a. Take the medication for at least 7 days. b. Use sunscreen while taking the Pyridium. c. The urine may turn a reddish-orange color. d. Use the Pyridium before sexual intercourse.

ANS: C Patients should be taught that Pyridium will color the urine deep orange. Urinary analgesics should only be needed for a few days until the prescribed antibiotics decrease the bacterial count. Taking Pyridium before intercourse will not be helpful in reducing the risk for UTI. Pyridium does not cause photosensitivity.

A 37-year-old patient is being admitted with a diagnosis of Cushing syndrome. Which findings will the nurse expect during the assessment? a. Chronically low blood pressure b. Bronzed appearance of the skin c. Purplish streaks on the abdomen d. Decreased axillary and pubic hair

ANS: C Purplish-red striae on the abdomen are a common clinical manifestation of Cushing syndrome. Hypotension and bronzed-appearing skin are manifestations of Addison's disease. Decreased axillary and pubic hair occur with androgen deficiency

38. Four hours after a bowel resection, a 74-year-old male patient with a nasogastric tube to suction complains of nausea and abdominal distention. The first action by the nurse should be to a. auscultate for hypotonic bowel sounds. b. notify the patient's health care provider. c. reposition the tube and check for placement. d. remove the tube and replace it with a new one.

ANS: C Repositioning the tube will frequently facilitate drainage. Because this is a common occurrence, it is not appropriate to notify the health care provider unless other interventions do not resolve the problem. Information about the presence or absence of bowel sounds will not be helpful in improving drainage. Removing the tube and replacing it are unnecessarily traumatic to the patient, so that would only be done if the tube was completely occluded.

Which assessment finding for a patient who has had a cystectomy with an ileal conduit the previous day is most important for the nurse to communicate to the physician? a. Cloudy appearing urine b. Hypotonic bowel sounds c. Heart rate 102 beats/minute d. Continuous drainage from stoma

ANS: C Tachycardia may indicate infection, hemorrhage, or hypovolemia, which are all serious complications of this surgery. The urine from an ileal conduit normally contains mucus and is cloudy. Hypotonic bowel sounds are expected after bowel surgery. Continuous drainage of urine from the stoma is normal. DIF: Cognitive Level: Application REF: 1157 | 1159-1160 | 1158-1159

A 26-year-old patient with a history of polycystic kidney disease is admitted to the surgical unit after having knee surgery. Which of the routine postoperative orders is most important for the nurse to discuss with the health care provider? a. Infuse 5% dextrose in normal saline at 75 mL/hr. b. Order regular diet after patient is awake and alert. c. Give ketorolac (Toradol) 10 mg PO PRN for pain. d. Obtain blood urea nitrogen (BUN), creatinine, and electrolytes in 2 hours.

ANS: C The NSAIDs should be avoided in patients with decreased renal function because nephrotoxicity is a potential adverse effect. The other orders do not need any clarification or change. DIF: Cognitive Level: Application REF: 1142-1143

39. A 19-year-old female is brought to the emergency department with a knife handle protruding from the abdomen. During the initial assessment of the patient, the nurse should a. remove the knife and assess the wound. b. determine the presence of Rovsing sign. c. check for circulation and tissue perfusion. d. insert a urinary catheter and assess for hematuria.

ANS: C The initial assessment is focused on determining whether the patient has hypovolemic shock. The knife should not be removed until the patient is in surgery, where bleeding can be controlled. Rovsing sign is assessed in the patient with suspected appendicitis. A patient with a knife in place will be taken to surgery and assessed for bladder trauma there.

A 19-year-old female is brought to the emergency department with a knife handle protruding from the abdomen. During the initial assessment of the patient, the nurse should a. remove the knife and assess the wound. b. determine the presence of Rovsing sign. c. check for circulation and tissue perfusion. d. insert a urinary catheter and assess for hematuria.

ANS: C The initial assessment is focused on determining whether the patient has hypovolemic shock. The knife should not be removed until the patient is in surgery, where bleeding can be controlled. Rovsing sign is assessed in the patient with suspected appendicitis. A patient with a knife in place will be taken to surgery and assessed for bladder trauma there.

Which assessment finding for a patient who has just been admitted with acute pyelonephritis is most important for the nurse to report to the health care provider? a. Foul-smelling urine b. Complaint of flank pain c. Blood pressure 88/45 mm Hg d. Temperature 100.1° F (57.8° C)

ANS: C The low blood pressure indicates that urosepsis and septic shock may be occurring and should be immediately reported. The other findings are typical of pyelonephritis. DIF: Cognitive Level: Application REF: 1126

27. At his first postoperative checkup appointment after a gastrojejunostomy (Billroth II), a patient reports that dizziness, weakness, and palpitations occur about 20 minutes after each meal. The nurse will teach the patient to a. increase the amount of fluid with meals. b. eat foods that are higher in carbohydrates. c. lie down for about 30 minutes after eating. d. drink sugared fluids or eat candy after meals.

ANS: C The patient is experiencing symptoms of dumping syndrome, which may be reduced by lying down after eating. Increasing fluid intake and choosing high carbohydrate foods will increase the risk for dumping syndrome. Having a sweet drink or hard candy will correct the hypoglycemia that is associated with dumping syndrome but will not prevent dumping syndrome.

37. After receiving change-of-shift report, which patient should the nurse assess first? a. A patient who was admitted yesterday with gastrointestinal (GI) bleeding and has melena b. A patient who is crying after receiving a diagnosis of esophageal cancer c. A patient with esophageal varices who has a blood pressure of 96/54 mm Hg d. A patient with nausea who has a dose of metoclopramide (Reglan) scheduled

ANS: C The patient's history and blood pressure indicate possible hemodynamic instability caused by GI bleeding. The data about the other patients do not indicate acutely life-threatening complications.

The long-term care nurse is evaluating the effectiveness of protein supplements on a patient who has low serum total protein level. Which of these data indicate that the patient's condition has improved? a. Hematocrit 28% b. Good skin turgor c. Absence of peripheral edema d. Blood pressure 110/72 mm Hg

ANS: C Absence of peripheral edema Edema is caused by low oncotic pressure in individuals with low serum protein levels; the absence of edema indicates an improvement in the patient's protein status. Good skin turgor is an indicator of fluid balance, not protein status. A low hematocrit could be caused by poor protein intake. Blood pressure does not provide a useful clinical tool for monitoring protein status.

17. Which nursing action is essential for a patient immediately after a renal biopsy? a. Check blood glucose to assess for hyperglycemia or hypoglycemia. b. Insert a urinary catheter and test urine for gross or microscopic hematuria. c. Monitor the blood urea nitrogen (BUN) and creatinine to assess renal function. d. Apply a pressure dressing and keep the patient on the affected side for 30 minutes.

ANS: D A pressure dressing is applied and the patient is kept on the affected side for 30 to 60 minutes to put pressure on the biopsy side and decrease the risk for bleeding. The blood glucose and BUN/creatinine will not be affected by the biopsy. Although monitoring for hematuria is needed, there is no need for catheterization.

40. All of the following orders are received for a patient who has vomited 1500 mL of bright red blood. Which order will the nurse implement first? a. Insert a nasogastric (NG) tube and connect to suction. b. Administer intravenous (IV) famotidine (Pepcid) 40 mg. c. Draw blood for typing and crossmatching. d. Infuse 1000 mL of lactated Ringer's solution.

ANS: D Because the patient has vomited a large amount of blood, correction of hypovolemia and prevention of hypovolemic shock are the priorities. The other actions also are important to implement quickly but are not the highest priorities.

A 72-year-old who has benign prostatic hyperplasia is admitted to the hospital with chills, fever, and vomiting. Which finding by the nurse will be most helpful in determining whether the patient has an upper urinary tract infection (UTI)? a. Suprapubic pain b. Bladder distention c. Foul-smelling urine d. Costovertebral tenderness

ANS: D Costovertebral tenderness is characteristic of pyelonephritis. The other symptoms are characteristic of lower UTI and are likely to be present if the patient also has an upper UTI. DIF: Cognitive Level: Application REF: 1128

9. After the nurse teaches a patient with gastroesophageal reflux disease (GERD) about recommended dietary modifications, which diet choice for a snack 2 hours before bedtime indicates that the teaching has been effective? a. Chocolate pudding b. Glass of low-fat milk c. Peanut butter sandwich d. Cherry gelatin and fruit

ANS: D Gelatin and fruit are low fat and will not decrease lower esophageal sphincter (LES) pressure. Foods like chocolate are avoided because they lower LES pressure. Milk products increase gastric acid secretion. High-fat foods such as peanut butter decrease both gastric emptying and LES pressure.

17. A patient with type 1 diabetes has an insulin order for NPH insulin, 35 U, to be given at 0700. The patient has also been instructed not to take anything by mouth (NPO) in preparation for laboratory work that will not be drawn until 1000. What action should the nurse implement? a. Give the insulin as ordered. b. Give the insulin with a small snack. c. Inform the charge nurse. d. Hold the insulin until after the blood draw.

ANS: D Holding the insulin to adhere to the NPO order is appropriate. The patient will not be getting food until after the laboratory work; consequently, the insulin will not be needed until then. Giving the insulin as ordered will create a possibility of hypoglycemia before the blood is drawn. Giving a snack to a patient who is NPO is inappropriate.

22. How long does it take for Humulin R 20 units to peak? a. 15 minutes b. 30 minutes c. 1 hour d. 2 hours

ANS: D Humulin R has its onset in approximately 30 minutes, but its peak is in 2 hours.

A 78-year-old who has been admitted to the hospital with dehydration is confused and incontinent of urine. Which nursing action will be best to include in the plan of care? a. Apply absorbent incontinent pads. b. Restrict fluids after the evening meal. c. Insert an indwelling catheter until the symptoms have resolved. d. Assist the patient to the bathroom every 2 hours during the day.

ANS: D In older or confused patients, incontinence may be avoided by using scheduled toileting times. Indwelling catheters increase the risk for urinary tract infection (UTI). Incontinent pads increase the risk for skin breakdown. Restricting fluids is not appropriate in a patient with dehydration. DIF: Cognitive Level: Application REF: 1151-1152

Which finding by the nurse when assessing a patient with a large pituitary adenoma is most important to report to the health care provider? a. Changes in visual field b. Milk leaking from breasts c. Blood glucose 150 mg/dL d. Nausea and projectile vomiting

ANS: D Nausea and projectile vomiting may indicate increased intracranial pressure, which will require rapid actions for diagnosis and treatment. Changes in the visual field, elevated blood glucose, and galactorrhea are common with pituitary adenoma, but these do not require rapid action to prevent life-threatening complications

When assessing the patient who has a lower urinary tract infection (UTI), the nurse will initially ask about a. nausea. b. flank pain. c. poor urine output. d. pain with urination.

ANS: D Pain with urination is a common symptom of a lower UTI. Urine output does not decrease, but frequency may be experienced. Flank pain and nausea are associated with an upper UTI. DIF: Cognitive Level: Application REF: 1123-1124

29. The health care provider prescribes antacids and sucralfate (Carafate) for treatment of a patient's peptic ulcer. The nurse will teach the patient to take a. sucralfate at bedtime and antacids before each meal. b. sucralfate and antacids together 30 minutes before meals. c. antacids 30 minutes before each dose of sucralfate is taken. d. antacids after meals and sucralfate 30 minutes before meals.

ANS: D Sucralfate is most effective when the pH is low and should not be given with or soon after antacids. Antacids are most effective when taken after eating. Administration of sucralfate 30 minutes before eating and antacids just after eating will ensure that both drugs can be most effective. The other regimens will decrease the effectiveness of the medications.

The nurse observes nursing assistive personnel (NAP) taking the following actions when caring for a patient with a retention catheter. Which action requires that the nurse intervene? a. Taping the catheter to the skin on the patient's upper inner thigh b. Cleaning around the patient's urinary meatus with soap and water c. Using an alcohol-based hand cleaner before performing catheter care d. Disconnecting the catheter from the drainage tube to obtain a specimen

ANS: D The catheter should not be disconnected from the drainage tube because this increases the risk for urinary tract infection (UTI). The other actions are appropriate and do not require any intervention. DIF: Cognitive Level: Application REF: 1152-1154

A 44-year-old female patient with Cushing syndrome is admitted for adrenalectomy. Which intervention by the nurse will be most helpful for a nursing diagnosis of disturbed body image related to changes in appearance? a. Reassure the patient that the physical changes are very common in patients with Cushing syndrome. b. Discuss the use of diet and exercise in controlling the weight gain associated with Cushing syndrome. c. Teach the patient that the metabolic impact of Cushing syndrome is of more importance than appearance. d. Remind the patient that most of the physical changes caused by Cushing syndrome will resolve after surgery.

ANS: D The most reassuring communication to the patient is that the physical and emotional changes caused by the Cushing syndrome will resolve after hormone levels return to normal postoperatively. Reassurance that the physical changes are expected or that there are more serious physiologic problems associated with Cushing syndrome are not therapeutic responses. The patient's physiological changes are caused by the high hormone levels, not by the patient's diet or exercise choices

A patient is taking a potassium-wasting diuretic for treatment of hypertension. The nurse will teach the patient to report symptoms of adverse effects such as a. personality changes. b. frequent loose stools. c. facial muscle spasms. d.generalized weakness.

ANS: D generalized weakness. Generalized weakness progressing to flaccidity is a manifestation of hypokalemia. Facial muscle spasms might occur with hypocalcemia. Loose stools are associated with hyperkalemia. Personality changes are not associated with electrolyte disturbances, although changes in mental status are common manifestations with sodium excess or deficit.

A patient who has required prolonged mechanical ventilation has the following arterial blood gas results: pH 7.48, PaO2 85 mm Hg, PaCO2 32 mm Hg, and HCO3 25 mEq/L. The nurse interprets these results as a. metabolic acidosis. b. metabolic alkalosis. c. respiratory acidosis. d. respiratory alkalosis.

ANS: D respiratory alkalosis. The pH indicates that the patient has alkalosis and the low PaCO2 indicates a respiratory cause. The other responses are incorrect based on the pH and the normal HCO3.

Which hormone is released in response to stress?

Adrenaline

Which method is preferred for immediate treatment of an acute episode of constipation?

An emema

B

An older adult patient who is malnourished presents to the emergency department with a serum protein level of 5.2 g/dL. The nurse would expect which clinical manifestation? a. Pallor b. Edema c. Confusion d. Restlessness

C

An older patient receiving iso-osmolar continuous tube feedings develops restlessness, agitation, and weakness. Which laboratory result should the nurse report to the health care provider immediately? a. K+ 3.4 mEq/L (3.4 mmol/L) b. Ca+2 7.8 mg/dL (1.95 mmol/L) c. Na+ 154 mEq/L (154 mmol/L) d. PO4-3 4.8 mg/dL (1.55 mmol/L)

D

An optimal teaching plan for an outpatient with stomach cancer receiving radiation therapy should include information about: A. cancer support groups, alopecia and stomatitis B. avitaminosis, ostomy care, and community resources C. prosthetic devices, skin conductance, and grief counseling D. wound and skin are, nutrition, drugs and community resources

The hypothalamus secretes releasing hormones and inhibiting hormones. What is the target tissue of these releasing hormones and inhibiting hormones?

Anterior pituitary

The patient is receiving the following medications. Which one is prescribed to relieve symptoms rather than treat a disease?

Antidiarrheal agents

A patient is admitted with dehydration and polyuria. The nurse knows that the patient might be experiencing an alteration in which hormone?

Antidiuretic hormone (ADH)

A patient's blood test reveals elevated levels of growth hormone (GH) and adrenocorticotropic hormone (ACTH). In discussing these results with the patient, the nurse explains that both of these hormones have which feature?

Are secreted by the anterior pituitary

In instituting a bowel training program for a patient with fecal incontinence, what should the nurse first plan to do?

Assist the patient to the bathroom at the time of the patient's normal defecation.

How is the most common form of malabsorption syndrome treated?

Avoidance of milk and milk products.

A patient is seeking emergency care after choking on a piece of steak. The nursing assessment reveals a history of alcoholism, cigarette smoking, and hemoptysis. Which diagnostic study is most likely to be performed on this patient?' a) Barium swallow b) Endoscopic biopsy c) Capsule endoscopy d) Endoscopic ultrasonography

B Because of this patient's history of excessive alcohol intake, smoking, hemoptysis, and the current choking episode, cancer may be present. A biopsy is necessary to make a definitive diagnosis of carcinoma, so an endoscope will be used to obtain a biopsy and observe other abnormalities as well. A barium swallow may show narrowing of the esophagus, but it is more diagnostic for achalasia. An endoscopic ultrasonography may be used to stage esophageal cancer. Capsule endoscopy can show alterations in the esophagus but is more often used for small intestine problems. A barium swallow, capsule endoscopy, and endoscopic ultrasonography cannot provide a definitive diagnosis for cancer when it is suspected.

A 40-yr-old patient with suspected acromegaly is seen at the clinic. To assist in making the diagnosis, which question should the nurse ask? a. "Have you had a recent head injury?" b. "Do you have to wear larger shoes now?" c. "Is there a family history of acromegaly?" d. "Are you experiencing tremors or anxiety?"

B Acromegaly causes an enlargement of the hands and feet. Head injury and family history are not risk factors for acromegaly. Tremors and anxiety are not clinical manifestations of acromegaly.

A 29-yr-old woman with systemic lupus erythematosus has been prescribed 2 weeks of high-dose prednisone therapy. Which information about the prednisone is most important for the nurse to include? a. "Weigh yourself daily to monitor for weight gain." b. "The prednisone dose should be decreased gradually." c. "A weight-bearing exercise program will help minimize risk for osteoporosis." d. "Call the health care provider if you have mood changes with the prednisone."

B Acute adrenal insufficiency may occur if exogenous corticosteroids are suddenly stopped. Mood alterations and weight gain are possible adverse effects of corticosteroid use, but these are not life- threatening effects. Osteoporosis occurs when patients take corticosteroids for longer periods.

The nurse is caring for a patient treated with IV fluid therapy for severe vomiting. As the patient recovers and begins to tolerate oral intake, which food choice does the nurse understand would be most appropriate? a) Iced tea b) Dry toast c) Hot coffee d) Plain hamburger

B Dry toast or crackers may alleviate the feeling of nausea and prevent further vomiting. Water is the initial fluid of choice. Extremely hot or cold liquids and fatty foods are generally not well tolerated.

The cardiac telemetry unit charge nurse receives status reports from other nursing units about four patients who need cardiac monitoring. Which patient should be transferred to the cardiac unit first? a. Patient with Hashimoto's thyroiditis and a heart rate of 102 b. Patient with tetany who has a new order for IV calcium chloride c. Patient with Cushing syndrome and a blood glucose of 140 mg/dL d. Patient with Addison's disease who takes hydrocortisone twice daily

B Emergency treatment of tetany requires IV administration of calcium; electrocardiographic monitoring will be required because cardiac arrest may occur if high calcium levels result from too-rapid administration. The information about the other patients indicates that they are more stable than the patient with tetany.

Which question will the nurse in the endocrine clinic ask to help determine a patient's risk factors for goiter? a. "How much milk do you drink?" b. "What medications are you taking?" c. "Are your immunizations up to date?" d. "Have you had any recent neck injuries?"

B Medications that contain thyroid-inhibiting substances can cause goiter. Milk intake, neck injury, and immunization history are not risk factors for goiter.

A patient who had a subtotal thyroidectomy earlier today develops laryngeal stridor and a cramp in the right hand upon returning to the surgical nursing unit. Which collaborative action will the nurse anticipate next? a. Suction the patient's airway. b. Administer IV calcium gluconate. c. Plan for emergency tracheostomy. d. Prepare for endotracheal intubation.

B The patient's clinical manifestations of stridor and cramping are consistent with tetany caused by hypocalcemia resulting from damage to the parathyroid glands during surgery. Endotracheal intubation or tracheostomy may be needed if the calcium does not resolve the stridor. Suctioning will not correct the stridor.

A patient who had a subtotal thyroidectomy earlier today develops laryngeal stridor and a cramp in the right hand upon returning to the surgical nursing unit. Which collaborative action will the nurse anticipate next? a. Suction the patient's airway. b. Administer IV calcium gluconate. c. Plan for emergency tracheostomy. d. Prepare for endotracheal intubation.

B The patient's clinical manifestations of stridor and cramping are consistent with tetany caused by hypocalcemia resulting from damage to the parathyroid glands during surgery. Endotracheal intubation or tracheostomy may be needed if the calcium does not resolve the stridor. Suctioning will not correct the stridor.

A patient has been diagnosed with urinary tract calculi that are high in uric acid. Which foods will the nurse teach the patient to avoid (select all that apply)? a. Milk b. Liver c. Spinach d. Chicken e. Cabbage f. Chocolate

B & D (Liver & chicken) Meats contain purines, which are metabolized to uric acid. The other foods might be restricted in patients who have calcium or oxalate stones.

A 35-year-old female patient with a possible pituitary adenoma is scheduled for a computed tomography (CT) scan with contrast media. Which patient information is most important for the nurse to communicate to the health care provider before the test? A. Bilateral poor peripheral vision B. Allergies to iodine and shellfish C. Recent weight loss of 20 pounds D. Complaint of ongoing headaches

B. Allergies to iodine and shellfish - Because the usual contrast media is iodine-based, the health care provider will need to know about the allergy before the CT scan. The other findings are common with any mass in the brain such as a pituitary adenoma.

When planning teaching for a 59-year-old male patient with benign nephrosclerosis the nurse should include instructions regarding a. preventing bleeding with anticoagulants. b. monitoring and recording blood pressure. c. obtaining and documenting daily weights. d. measuring daily intake and output volumes.

B. monitoring and recording blood pressure Hypertension is the major symptom of nephrosclerosis. Measurements of intake and output and daily weights are not necessary unless the patient develops renal insufficiency. Anticoagulants are not used to treat nephrosclerosis.

After a patient with a pituitary adenoma has had a hypophysectomy, the nurse will teach about the need for a. sodium restriction to prevent fluid retention. b. insulin to maintain normal blood glucose levels. c. oral corticosteroids to replace endogenous cortisol. d. chemotherapy to prevent malignant tumor recurrence.

C Antidiuretic hormone (ADH), cortisol, and thyroid hormone replacement will be needed for life after hypophysectomy. Without the effects of adrenocorticotropic hormone (ACTH) and cortisol, the blood glucose and serum sodium will be low unless cortisol is replaced. An adenoma is a benign tumor, and chemotherapy will not be needed.

Which information will the nurse teach a patient who has been newly diagnosed with Graves' disease? a. Exercise is contraindicated to avoid increasing metabolic rate. b. Restriction of iodine intake is needed to reduce thyroid activity. c. Antithyroid medications may take several months for full effect. d. Surgery will eventually be required to remove the thyroid gland.

C Medications used to block the synthesis of thyroid hormones may take 2 to 3 months before the full effect is seen. Large doses of iodine are used to inhibit the synthesis of thyroid hormones. Exercise using large muscle groups is encouraged to decrease the irritability and hyperactivity associated with high levels of thyroid hormones. Radioactive iodine is the most common treatment for Graves' disease, although surgery may be used.

A patient is being admitted with a diagnosis of Cushing syndrome. Which findings will the nurse expect during the assessment? a. Chronically low blood pressure b. Bronzed appearance of the skin c. Purplish streaks on the abdomen d. Decreased axillary and pubic hair

C Purplish-red striae on the abdomen are a common clinical manifestation of Cushing syndrome. Hypotension and bronzed-appearing skin are manifestations of Addison's disease. Decreased axillary and pubic hair occur with androgen deficiency.

Which information obtained by the nurse in the endocrine clinic about a patient who has been taking prednisone 40 mg daily for 3 weeks is most important to report to the health care provider? a. Patient's blood pressure is 148/94 mm Hg. b. Patient has bilateral 2+ pitting ankle edema. c. Patient stopped taking the medication 2 days ago. d. Patient has not been taking the prescribed vitamin D.

C Sudden cessation of corticosteroids after taking the medication for a week or more can lead to adrenal insufficiency, with problems such as severe hypotension and hypoglycemia. The patient will need immediate evaluation by the health care provider to prevent or treat adrenal insufficiency. The other information will also be reported but does not require rapid treatment.

After obtaining the information shown in the accompanying figure regarding a patient with Addison's disease, which prescribed action will the nurse take first? a. Give 4 oz of fruit juice orally. b. Recheck the blood glucose level. c. Infuse 5% dextrose and 0.9% saline. d. Administer O2 therapy as needed.

C The patient's poor skin turgor, hypotension, and hyponatremia indicate an Addisonian crisis. Immediate correction of the hypovolemia and hyponatremia is needed. The other actions may also be needed but are not the initial action for the patient.

After obtaining the information shown in the accompanying figure regarding a patient with Addison's disease, which prescribed action will the nurse take first? a. Give 4 oz of fruit juice orally. b. Recheck the blood glucose level. c. Infuse 5% dextrose and 0.9% saline. d. Administer O2 therapy as needed.

C The patient's poor skin turgor, hypotension, and hyponatremia indicate an Addisonian crisis. Immediate correction of the hypovolemia and hyponatremia is needed. The other actions may also be needed but are not the initial action for the patient.

A 62-yr-old patient with hyperthyroidism is to be treated with radioactive iodine (RAI). The nurse instructs the patient a. about radioactive precautions to take with all body secretions. b. that symptoms of hyperthyroidism should be relieved in about a week. c. that symptoms of hypothyroidism may occur as the RAI therapy takes effect. d. to discontinue the antithyroid medications taken before the radioactive therapy.

C There is a high incidence of postradiation hypothyroidism after RAI, and the patient should be monitored for symptoms of hypothyroidism. RAI has a delayed response, with the maximum effect not seen for 2 to 3 months, and the patient will continue to take antithyroid medications during this time. The therapeutic dose of radioactive iodine is low enough that no radiation safety precautions are needed.

A patient is scheduled for transsphenoidal hypophysectomy to treat a pituitary adenoma. During preoperative teaching, the nurse instructs the patient about the need to a. cough and deep breathe every 2 hours postoperatively. b. remain on bed rest for the first 48 hours after the surgery. c. avoid brushing teeth for at least 10 days after the surgery. d. be positioned flat with sandbags at the head postoperatively.

C To avoid disruption of the suture line, the patient should avoid brushing the teeth for 10 days after surgery. It is not necessary to remain on bed rest after this surgery. Coughing is discouraged because it may cause leakage of cerebrospinal fluid (CSF) from the suture line. The head of the bed should be elevated 30 degrees to reduce pressure on the sella turcica and decrease the risk for headaches.

An 82-yr-old patient in a long-term care facility is newly diagnosed with hypothyroidism. The nurse will need to consult with the health care provider before administering the prescribed a. docusate (Colace). b. ibuprofen (Motrin). c. diazepam (Valium). d. cefoxitin (Mefoxin).

C Worsening of mental status and myxedema coma can be precipitated by the use of sedatives, especially in older adults. The nurse should discuss the use of diazepam with the health care provider before administration. The other medications may be given safely to the patient.

An 82-yr-old patient in a long-term care facility is newly diagnosed with hypothyroidism. The nurse will need to consult with the health care provider before administering the prescribed a. docusate (Colace). b. ibuprofen (Motrin). c. diazepam (Valium). d. cefoxitin (Mefoxin).

C Worsening of mental status and myxedema coma can be precipitated by the use of sedatives, especially in older adults. The nurse should discuss the use of diazepam with the health care provider before administration. The other medications may be given safely to the patient.

What change in the aging process helps the nurse determine that a geriatric patient is at an increased risk for diabetes mellitus?

Increased fibrosis and fatty acid deposits in the pancreas

The nurse will teach a patient to plan to minimize physical and emotional stress while the patient is undergoing A. A water deprivation test. B. Testing for serum T3 and T4 levels. C. A 24-hour urine test for free cortisol. D. A radioactive iodine (I-131) uptake test.

C. A 24-hour urine test for free cortisol. - Physical and emotional stress can affect the results of the free cortisol test. The other tests are not impacted by stress.

Which hormones act on a patient's mammary glands? Select all that apply.

Oxytocin Prolactin

The nurse is caring for an elderly woman who has a decline in estrogen concentration. Which complications should the nurse monitor in the patient? Select all that apply.

Osteoporosis Atherosclerosis

Which hormone stimulates milk production and uterine contractility?

Oxytocin

Which hormone stimulates milk secretion in a postpartum patient?

Oxytocin

A patient with IBD has a nursing diagnosis of imbalanced nutrition: less than body requirements related to decreased nutritional intake and decreased intestinal absorption. Which assessment data support this nursing diagnosis?

Pallor and hair loss

Which gland has both endocrine and exocrine functions?

Pancreas

What information and instructions would the nurse provide about intravesical therapy?

The drug will be instilled into the empty bladder via a urethral catheter at weekly intervals for 6 to 12 weeks. The drug needs to be retained for about 2 hours with patient's position changed about every 15 minutes to ensure that the drug comes into maximum contact with all areas of the bladder. He may have irritative symptoms, such as frequency, urgency, and bladder spasms, in addition to hematuria during the weeks of treatment. Bacille Calmette-Guérin (BCG) therapy may cause flu-like symptoms or systemic infection because BCG stimulates the immune system rather than directly destroying cancer cells. The usual side effects of cancer chemotherapy are not experienced with BCG therapy or with intravesical chemotherapy.

The nurse provides information to a group of nursing students about the positive feedback mechanism. Which information is appropriate for the nurse to include in the education?

The female ovarian hormone estradiol operates by this mechanism.

B

The nurse is caring for a patient who has a central venous access device (CVAD). Which action by the nurse is appropriate? a. Avoid using friction when cleaning around the CVAD insertion site. b. Use the push-pause method to flush the CVAD after giving medications. c. Obtain an order from the health care provider to change CVAD dressing. d. Position the patient's face toward the CVAD during injection cap changes.

D

The nurse is teaching the patient and family about possible causative factors for peptic ulcers. The nurse explains that ulcer formation is: A. caused by stressful lifestyle and other acid-producing factors such as H. pylori B. inherited within families and reinforced by bacterial spread of staph aureus in childhood C. promoted by factors that tend to cause oversecretion of acid, such as excess dietary fats, smoking, and H pylori D. promoted by a combination of possible factors that may result in erosion of the gastric mucosa, including certain drugs and alcohol

A

The nurse notes a serum calcium level of 7.9 mg/dL for a patient who has chronic malnutrition. Which action should the nurse take next? a. Monitor ionized calcium level. b. Give oral calcium citrate tablets. c. Check parathyroid hormone level. d. Administer vitamin D supplements.

C

The nurse notes that a patient who was admitted with diabetic ketoacidosis has rapid, deep respirations. Which action should the nurse take? a. Give the prescribed PRN lorazepam (Ativan). b. Start the prescribed PRN oxygen at 2 to 4 L/min. c. Administer the prescribed normal saline bolus and insulin. d. Encourage the patient to take deep, slow breaths with guided imagery.

What does staging of bladder tumors indicate?

The staging of bladder cancer is determined by the depth of invasion of the bladder wall and surrounding tissue. Stage II indicates that the tumor has grown into the muscle layer of the bladder but not passed completely through it. The TNM grading system indicates the characteristics of the tumor (T), the nodal involvement (N), and the presence of distant metastasis (M). (See Chapter 16 for TNM classification.)

The patient comes to the emergency department with intermittent crampy abdominal pain, nausea, projectile vomiting, and dehydration. The nurse suspects a GI obstruction. Based on the manifestations, what area of the bowel should the nurse suspect is obstructed?

Upper small intestine.

The patient calls the clinic and describes a bump at the site of a previous incision that disappears when he lies down. The nurse suspects that this is which type of hernia?

Ventral Reducible

What is the most important thing the nurse should do when caring for a patient who has contracted C-diff?

Wear gloves and wash hands with soap and water

A 20 year old patient with a history of Crohn's disease comes to the clinic with persistent diarrhea. What are characteristics of Crohn's disease?

Weight loss Abdominal pain Has segmented distribution Involves the entire thickness of the bowel wall

B

When assessing a pregnant patient with eclampsia who is receiving IV magnesium sulfate, which finding should the nurse report to the health care provider immediately? a. The bibasilar breath sounds are decreased. b. The patellar and triceps reflexes are absent. c. The patient has been sleeping most of the day. d. The patient reports feeling "sick to my stomach."

The nurse determines that additional instruction is needed for a 60-year-old patient with chronic syndrome of inappropriate antidiuretic hormone (SIADH) when the patient says which of the following? a. "I need to shop for foods low in sodium and avoid adding salt to food." b. "I should weigh myself daily and report any sudden weight loss or gain." c. "I need to limit my fluid intake to no more than 1 quart of liquids a day." d. "I will eat foods high in potassium because diuretics cause potassium loss."

a. "I need to shop for foods low in sodium and avoid adding salt to food." Patients with SIADH are at risk for hyponatremia, and a sodium supplement may be prescribed. The other patient statements are correct and indicate successful teaching has occurred

Number in sequence the following ascending pathologic changes that occur in the urinary tract in the presence of a bladder outlet obstruction. a. Hydronephrosis b. Reflux of urine into ureter c. Bladder detrusor muscle hypertrophy d. Ureteral dilation e. Renal atrophy f. Vesicoureteral reflux g. Large residual urine in bladder h. Chronic pyelonephritis

a. 6; b. 3; c. 1; d. 4; e. 8; f. 5; g. 2; h. 7

Which information will the nurse include in teaching a female patient who has peripheral arterial disease, type 2 diabetes, and sensory neuropathy of the feet and legs? a. Choose flat-soled leather shoes. b. Set heating pads on a low temperature. c. Use callus remover for corns or calluses. d. Soak feet in warm water for an hour each day.

a. Choose flat-soled leather shoes.

The patient with type 2 diabetes has a second UTI within one month of being treated for a previous UTI. Which medication should the nurse expect to teach the patient about taking for this infection? a. Ciprofloxacin (Cipro) b. Fosfomycin (Monurol) c. Nitrofurantoin (Macrodantin) d. Trimethoprim/ sulfamethoxazole (Bactrim)

a. Ciprofloxacin (Cipro) This UTI is a complicated UTI because the patient has type 2 diabetes and the UTI is recurrent. Ciprofloxacin (Cipro) would be used for a complicated UTI. Fosfomycin (Monurol), nitrofurantoin (Macrodantin), and trimethoprim/sulfamethoxazole (Bactrim) should be used for uncomplicated UTIs.

A 54-year-old patient is admitted with diabetic ketoacidosis. Which admission order should the nurse implement first? a. Infuse 1 liter of normal saline per hour. b. Give sodium bicarbonate 50 mEq IV push. c. Administer regular insulin 10 U by IV push. d. Start a regular insulin infusion at 0.1 units/kg/hr.

a. Infuse 1 liter of normal saline per hour.

After change-of-shift report, which patient should the nurse assess first? a. Patient with a urethral stricture who has not voided for 12 hours b. Patient who has cloudy urine after orthotopic bladder reconstruction c. Patient with polycystic kidney disease whose blood pressure is 186/98 mm Hg d. Patient who voided bright red urine immediately after returning from lithotripsy

a. Patient with a urethral stricture who has not voided for 12 hours The patient information suggests acute urinary retention, a medical emergency. The nurse will need to assess the patient and consider whether to insert a retention catheter. The other patients will also be assessed, but their findings are consistent with their diagnoses and do not require immediate assessment or possible intervention.

A 38-year-old patient who has type 1 diabetes plans to swim laps daily at 1:00 PM. The clinic nurse will plan to teach the patient to a. check glucose level before, during, and after swimming. b. delay eating the noon meal until after the swimming class. c. increase the morning dose of neutral protamine Hagedorn (NPH) insulin. d. time the morning insulin injection so that the peak occurs while swimming.

a. check glucose level before, during, and after swimming.

It is important for the nurse to assess for which clinical manifestation (s) in a patient who has just undergone a total thyroidectomy (select all that apply) a. confusion b. weight gain c. depressed reflexes d. circumoral numbness e. positive Chvostek's sign

a. confusion d. circumoral numbness e. positive Chvostek's sign

The lungs act as an acid-base buffer by a. increasing respiratory rate and depth when CO2 levels in the blood are high, reducing acid load b. increasing respiratory rate depth when CO2 levels in the blood are low, reducing base load c. decreasing respiratory rate and depth when CO2 levels in the blood are high, reducing acid load d. decreasing respiratory rate and depth when CO2 levels in the blood are low, increasing acid load

a. increasing respiratory rate and depth when CO2 levels in the blood are high, reducing acid load

During administration of a hypertonic IV solution, the mechanism involved in equalizing the fluid concentration between ECF and the cells is a. osmosis b. diffusion c. active transport d. facilitated diffusion

a. osmosis

Assessment findings suggestive of peritonitis include a. rebound abdominal pain b. a soft, distended abdomen c. dull, continuous abdominal pain d. observing that the patient is restless

a. rebound abdominal pain Rationale: With peritoneal irritation, the abdomen is hard, like a board, and the patient has severe abdominal pain that is worse with any sudden movement. The patient lies very still. Palpating the abdomen and releasing the hands suddenly causes sudden movement within the abdomen and severe pain. This is called rebound tenderness.

The nurse has taught a patient admitted with diabetes, cellulitis, and osteomyelitis about the principles of foot care. The nurse evaluates that the patient understands the principles of foot care if the patient makes what statement? a. "I should only walk barefoot in nice dry weather." b. "I should look at the condition of my feet every day." c. "I am lucky my shoes fit so nice and tight because they give me firm support." d. "When I am allowed up out of bed, I should check the shower water with my toes."

b. "I should look at the condition of my feet every day." Patients with diabetes mellitus need to inspect their feet daily for broken areas that are at risk for infection and delayed wound healing. Properly fitted (not tight) shoes should be worn at all times. Water temperature should be tested with the hands first.

During assessment of the patient who has a nephrectomy, what should the nurse expect to find? a. Shallow, slow respirations b. Clear breath sounds in all lung fields c. Decreased breath sounds in the lower left lobe d. Decreased breath sounds in the right and left lower lobes

b. A nephrectomy incision is usually in the flank, just below the diaphragm or in the abdominal area. Although the patient is reluctant to breathe deeply because of incisional pain, the lungs should be clear. Decreased sounds and shallow respirations are abnormal and would require intervention.

A diabetic patient who has reported burning foot pain at night receives a new prescription. Which information should the nurse teach the patient about amitriptyline (Elavil)? a. Amitriptyline decreases the depression caused by your foot pain. b. Amitriptyline helps prevent transmission of pain impulses to the brain. c. Amitriptyline corrects some of the blood vessel changes that cause pain. d. Amitriptyline improves sleep and makes you less aware of nighttime pain.

b. Amitriptyline helps prevent transmission of pain impulses to the brain.

The nurse is preparing to teach a 43-year-old man who is newly diagnosed with type 2 diabetes about home management of the disease. Which action should the nurse take first? a. Ask the patient's family to participate in the diabetes education program. b. Assess the patient's perception of what it means to have diabetes mellitus. c. Demonstrate how to check glucose using capillary blood glucose monitoring. d. Discuss the need for the patient to actively participate in diabetes management.

b. Assess the patient's perception of what it means to have diabetes mellitus.

A teaching plan developed by the nurse for the patient with a new ileal conduit includes instructions to do what? a. Clean the skin around the stoma with alcohol every day. b. Use a wick to keep the skin dry during appliance changes. c. Use sterile supplies and technique during care of the stoma. d. Change the appliance every day and wash it with soap and warm water.

b. Because the stoma continuously drains urine, a wick formed of a rolled-up 4 × 4 gauze or a tampon is held against the stoma to absorb the urine while the skin is cleaned and a new appliance is attached. The skin is cleaned with warm water only because soap and other agents cause drying and irritation and clean, not sterile, technique is used. The appliance should be left in place for as long as possible before it loosens and allows leakage onto the skin, perhaps up to 14 days.

A teaching plan developed by the nurse for the patient with a new ileal conduit includes instructions to do what? a. Clean the skin around the stoma with alcohol every day. b. Use a wick to keep the skin dry during appliance changes. c. Use sterile supplies and technique during care of the stoma. d. Change the appliance every day and wash it with soap and warm water.

b. Because the stoma continuously drains urine, a wick formed of a rolled-up 4 × 4 gauze or a tampon is held against the stoma to absorb the urine while the skin is cleaned and a new appliance is attached. The skin is cleaned with warm water only because soap and other agents cause drying and irritation and clean, not sterile, technique is used. The appliance should be left in place for as long as possible before it loosens and allows leakage onto the skin, perhaps up to 14 days.

A 58-year-old male patient who is diagnosed with nephrotic syndrome has ascites and 4+ leg edema. Which nursing diagnosis is a priority for the patient? a. Activity intolerance related to rapidly increased weight b. Excess fluid volume related to low serum protein levels c. Disturbed body image related to peripheral edema and ascites d. Altered nutrition: less than required related to protein restriction

b. Excess fluid volume related to low serum protein levels The patient has massive edema, so the priority problem at this time is the excess fluid volume. The other nursing diagnoses are also appropriate, but the focus of nursing care should be resolution of the edema and ascites.

Which patient action indicates good understanding of the nurse's teaching about administration of aspart (NovoLog) insulin? a. The patient avoids injecting the insulin into the upper abdominal area. b. The patient cleans the skin with soap and water before insulin administration. c. The patient stores the insulin in the freezer after administering the prescribed dose. d. The patient pushes the plunger down while removing the syringe from the injection site.

b. The patient cleans the skin with soap and water before insulin administration.

With which diagnosis will the patient benefit from being taught to do self-catheterization? a. Renal trauma b. Urethral stricture c. Renal artery stenosis d. Accelerated nephrosclerosis

b. The patient with urethral stricture will benefit from being taught to dilate the urethra by self-catheterization every few days. Renal trauma is treated related to the severity of the injury with bed rest, fluids, and analgesia. Renal artery stenosis includes control of hypertension with possible surgical revascularization. Accelerated nephrosclerosis is associated with malignant hypertension that must be aggressively treated as well as monitoring kidney function.

A 34-year-old has a new diagnosis of type 2 diabetes. The nurse will discuss the need to schedule a dilated eye exam a. every 2 years. b. as soon as possible. c. when the patient is 39 years old. d. within the first year after diagnosis.

b. as soon as possible.

The nurse will plan to teach a 27-year-old female who smokes 2 packs of cigarettes daily about the increased risk for a. kidney stones. b. bladder cancer. c. bladder infection. d. interstitial cystitis.

b. bladder infection Cigarette smoking is a risk factor for bladder cancer. The patient's risk for developing interstitial cystitis, urinary tract infection (UTI), or kidney stones will not be reduced by quitting smoking.

A 56-year-old patient who is disoriented and reports a headache and muscle cramps is hospitalized with possible syndrome of inappropriate antidiuretic hormone (SIADH). The nurse would expect the initial laboratory results to include a(n) a. elevated hematocrit. b. decreased serum sodium. c. low urine specific gravity. d. increased serum chloride.

b. decreased serum sodium. When water is retained, the serum sodium level will drop below normal, causing the clinical manifestations reported by the patient. The hematocrit will decrease because of the dilution caused by water retention. Urine will be more concentrated with a higher specific gravity. The serum chloride level will usually decrease along with the sodium level

*8. The nurse identifies a risk factor for kidney and bladder cancer in a patient who relates a history of* a.aspirin use b.tobacco use c.chronic alcohol abuse d.use of artificial sweeteners

b.tobacco use

Which clinical manifestations of inflammatory bowel disease are common to both patients with ulcerative colitis (UC) and Crohn's disease (select all that apply)? a. Restricted to rectum b. Strictures are common. c. Bloody, diarrhea stools d. Cramping abdominal pain e. Lesions penetrate intestine.

c & d Clinical manifestations of UC and Crohn's disease include bloody diarrhea, cramping abdominal pain, and nutritional disorders. Intestinal lesions associated with UC are usually restricted to the rectum before moving into the colon. Lesions that penetrate the intestine or cause strictures are characteristic of Crohn's disease.

What is the priority action for the nurse to take if the patient with type 2 diabetes complains of blurred vision and irritability? a) Call the physician b) Administer insulin as ordered c) Check the patient's blood glucose level d) Assess for other neurologic symptoms

c) Check the patient's blood glucose level Blood glucose testing should be performed whenever hypoglycemia is suspected so that immediate action can be taken if necessary.

A diabetic patient has a serum glucose level of 824 mg/dL (45.7 mmol/L) and is unresponsive. After assessing the patient, the nurse suspects diabetic ketoacidosis rather than hyperosmolar hyperglycemic syndrome based on the finding of: a) polyuria b) severe dehydration c) rapid, deep respirations d) decreased serum potassium

c) rapid, deep respirations Signs and symptoms of DKA include manifestations of dehydration, such as poor skin turgor, dry mucous membranes, tachycardia, and orthostatic hypotension. Early symptoms may include lethargy and weakness. As the patient becomes severely dehydrated, the skin becomes dry and loose, and the eyeballs become soft and sunken. Abdominal pain is another symptom of DKA that may be accompanied by anorexia and vomiting. Kussmaul respirations (i.e., rapid, deep breathing associated with dyspnea) are the body's attempt to reverse metabolic acidosis through the exhalation of excess carbon dioxide. Acetone is identified on the breath as a sweet, fruity odor. Laboratory findings include a blood glucose level greater than 250 mg/dL, arterial blood pH less than 7.30, serum bicarbonate level less than 15 mEq/L, and moderate to high ketone levels in the urine or blood.

Which drugs are used to treat overflow incontinence (select all that apply)? a. Baclofen (Lioresal) b. Anticholinergic drugs c. α-Adrenergic blockers d. 5α-reductase inhibitors e. Bethanechol (Urecholine)

c, d, e. α-Adrenergic blockers block the stimulation of the smooth muscle of the bladder, 5α-reductase inhibitors decrease outlet resistance, and bethanechol enhances bladder contractions. Baclofen or diazepam is used to relax the external sphincter for reflex incontinence. Anticholinergics are used to relax bladder tone and increase sphincter tone with urge incontinence.

Which drugs are used to treat overflow incontinence (select all that apply)? a. Baclofen (Lioresal) b. Anticholinergic drugs c. α-Adrenergic blockers d. 5α-reductase inhibitors e. Bethanechol (Urecholine)

c, d, e. α-Adrenergic blockers block the stimulation of the smooth muscle of the bladder, 5α-reductase inhibitors decrease outlet resistance, and bethanechol enhances bladder contractions. Baclofen or diazepam is used to relax the external sphincter for reflex incontinence. Anticholinergics are used to relax bladder tone and increase sphincter tone with urge incontinence.

The nurse is assessing a 22-year-old patient experiencing the onset of symptoms of type 1 diabetes. Which question is most appropriate for the nurse to ask? a. "Are you anorexic?" b. "Is your urine dark colored?" c. "Have you lost weight lately?" d. "Do you crave sugary drinks?"

c. "Have you lost weight lately?"

The nurse determines a need for additional instruction when the patient with newly diagnosed type 1 diabetes says which of the following? a. "I can have an occasional alcoholic drink if I include it in my meal plan." b. "I will need a bedtime snack because I take an evening dose of NPH insulin." c. "I can choose any foods, as long as I use enough insulin to cover the calories." d. "I will eat something at meal times to prevent hypoglycemia, even if I am not hungry."

c. "I can choose any foods, as long as I use enough insulin to cover the calories."

When a patient with type 2 diabetes is admitted for a cholecystectomy, which nursing action can the nurse delegate to a licensed practical/vocational nurse (LPN/LVN)? a. Communicate the blood glucose level and insulin dose to the circulating nurse in surgery. b. Discuss the reason for the use of insulin therapy during the immediate postoperative period. c. Administer the prescribed lispro (Humalog) insulin before transporting the patient to surgery. d. Plan strategies to minimize the risk for hypoglycemia or hyperglycemia during the postoperative period.

c. Administer the prescribed lispro (Humalog) insulin before transporting the patient to surgery.

Which information about a patient with Goodpasture syndrome requires the most rapid action by the nurse? a. Blood urea nitrogen level is 70 mg/dL. b. Urine output over the last 2 hours is 30 mL. c. Audible crackles bilaterally over the posterior chest to the midscapular level. d. Elevated level of antiglomerular basement membrane (anti-GBM) antibodies.

c. Audible crackles bilaterally over the posterior chest to the midscapular level. Crackles heard to a high level indicate a need for rapid actions such as assessment of oxygen saturation, reporting the findings to the health care provider, initiating oxygen therapy, and dialysis. The other findings will also be reported, but are typical of Goodpasture syndrome and do not require immediate nursing action.

A few weeks after an 82-year-old with a new diagnosis of type 2 diabetes has been placed on metformin (Glucophage) therapy and taught about appropriate diet and exercise, the home health nurse makes a visit. Which finding by the nurse is most important to discuss with the health care provider? a. Hemoglobin A1C level is 7.9%. b. Last eye exam was 18 months ago. c. Glomerular filtration rate is decreased. d. Patient has questions about the prescribed diet.

c. Glomerular filtration rate is decreased.

The newly diagnosed patient with type 2 diabetes has been prescribed metformin (Glucophage). What should the nurse tell the patient to best explain how this medication works? a. Increases insulin production from the pancreas. b. Slows the absorption of carbohydrate in the small intestine. c. Reduces glucose production by the liver and enhances insulin sensitivity. d. Increases insulin release from the pancreas, inhibits glucagon secretion, and decreases gastric emptying.

c. Reduces glucose production by the liver and enhances insulin sensitivity. Metformin is a biguanide that reduces glucose production by the liver and enhances the tissue's insulin sensitivity. Sulfonylureas and meglitinides increase insulin production from the pancreas. α-glucosidase inhibitors slow the absorption of carbohydrate in the intestine. Glucagon-like peptide receptor agonists increase insulin synthesis and release from the pancreas, inhibit glucagon secretion, and decrease gastric emptying.

A 26-year-old patient with diabetes rides a bicycle to and from work every day. Which site should the nurse teach the patient to administer the morning insulin? a. thigh. b. buttock. c. abdomen. d. upper arm.

c. abdomen.

The nurse should be alert for which manifestation in a patient receiving a loop diuretic? a. restlessness and agitation b. paresthesia and irritability c. weak irregular pulse and poor muscle tone d. increased BP and muscle spasms

c. weak irregular pulse and poor muscle tone

The nurse is teaching a patient with type 2 diabetes mellitus about exercise to help control his blood glucose. The nurse knows the patient understands when the patient elicits which exercise plan? a. "I want to go fishing for 30 minutes each day; I will drink fluids and wear sunscreen." b. "I will go running each day when my blood sugar is too high to bring it back to normal." c. "I will plan to keep my job as a teacher because I get a lot of exercise every school day." d. "I will take a brisk 30-minute walk 5 days per week and do resistance training 3 times a week."

d. "I will take a brisk 30-minute walk 5 days per week and do resistance training 3 times a week." The best exercise plan for the person with type 2 diabetes is for 30 minutes of moderate activity 5 days per week and resistance training 3 times a week. Brisk walking is moderate activity. Fishing and teaching are light activity, and running is considered vigorous activity.

Which finding by the nurse will be most helpful in determining whether a 67-year-old patient with benign prostatic hyperplasia has an upper urinary tract infection (UTI)? a. Bladder distention b. Foul-smelling urine c. Suprapubic discomfort d. Costovertebral tenderness

d. Costovertebral tenderness Costovertebral tenderness is characteristic of pyelonephritis. Bladder distention, foul-smelling urine, and suprapubic discomfort are characteristic of lower UTI and are likely to be present if the patient also has an upper UTI.

The nurse asks a 68-year-old patient scheduled for colectomy to sign the operative permit as directed in the physician's preoperative orders. The patient states that the physician has not really explained very well what is involved in the surgical procedure. What is the most appropriate action by the nurse? a. Ask family members whether they have discussed the surgical procedure with the physician. b. Have the patient sign the form and state the physician will visit to explain the procedure before surgery. c. Explain the planned surgical procedure as well as possible and have the patient sign the consent form. d. Delay the patient's signature on the consent and notify the physician about the conversation with the patient.

d. Delay the patient's signature on the consent and notify the physician about the conversation with the patient. The patient should not be asked to sign a consent form unless the procedure has been explained to the satisfaction of the patient. The nurse should notify the physician, who has the responsibility for obtaining consent.

The nurse is preparing to administer a dose of bisacodyl (Dulcolax). In explaining the medication to the patient, the nurse would explain that it acts in what way? a. Increases bulk in the stool b. Lubricates the intestinal tract to soften feces c. Increases fluid retention in the intestinal tract d. Increases peristalsis by stimulating nerves in the colon wall

d. Increases peristalsis by stimulating nerves in the colon wall Bisacodyl is a stimulant laxative that aids in producing a bowel movement by irritating the colon wall and stimulating enteric nerves. It is available in oral and suppository forms. Fiber and bulk forming drugs increase bulk in the stool; water and stool softeners soften feces, and saline and osmotic solutions cause fluid retention in the intestinal tract.

Which disease causes connective tissue changes that cause glomerulonephritis? a. Gout b. Amyloidosis c. Diabetes mellitus d. Systemic lupus erythematosus

d. Systemic lupus erythematosus causes connective tissue damage that affects the glomerulus. Gout deposits uric acid crystals in the kidney. Amyloidosis deposits hyaline bodies in the kidney. Diabetes mellitus causes microvascular damage affecting the kidney.

What should the nurse instruct the patient to do to best enhance the effectiveness of a daily dose of docusate sodium (Colace)? a. Take a dose of mineral oil at the same time. b. Add extra salt to food on at least one meal tray. c. Ensure dietary intake of 10 g of fiber each day. d. Take each dose with a full glass of water or other liquid.

d. Take each dose with a full glass of water or other liquid. Docusate lowers the surface tension of stool, permitting water and fats to penetrate and soften the stool for easier passage. The patient should take the dose with a full glass of water and should increase overall fluid intake, if able, to enhance effectiveness of the medication. Dietary fiber intake should be a minimum of 20 g daily to prevent constipation. Mineral oil and extra salt are not recommended.

Which action by a patient indicates that the home health nurse's teaching about glargine and regular insulin has been successful? a. The patient administers the glargine 30 minutes before each meal. b. The patient's family prefills the syringes with the mix of insulins weekly. c. The patient draws up the regular insulin and then the glargine in the same syringe. d. The patient disposes of the open vials of glargine and regular insulin after 4 weeks.

d. The patient disposes of the open vials of glargine and regular insulin after 4 weeks.

A patient with bladder cancer undergoes cystectomy with formation of an ileal conduit. During the patient's first postoperative day, what should the nurse plan to do? a. Measure and fit the stoma for a permanent appliance. b. Encourage high oral intake to flush mucus from the conduit. c. Teach the patient to self-catheterize the stoma every 4 to 6 hours. d. Empty the drainage bag every 2 to 3 hours and measure the urinary output.

d. Urine drains continuously from an ileal conduit and the drainage bag must be emptied every 2 to 3 hours and measured to ensure adequate urinary output. Fitting for a permanent appliance is not done until the stoma shrinks to its normal size in a few weeks. With an ileal conduit, mucus is present in the urine because it is secreted by the ileal segment as a result of the irritating effect of the urine but the surgery causes paralytic ileus and the patient will be NPO for several days postoperatively. Self-catheterization is performed when patients have formation of a continent Kock pouch.

*1. In teaching a patient with pyelonephritis about the disorder, the nurse informs the paitent that the organisms that cause pyelonephritis most commonly reach the kidneys through* a. the bloodstream b.the lymphatic system c. a descending infection d. an ascending infection

d. an ascending infection

Which cells of the islets of Langerhans produce and secrete insulin and amylin?

β cells

The laboratory reports of a patient reveal reduced parafollicular cells of the thyroid gland. Which hormonal deficiency does the nurse recognize through this finding?

Calcitonin

Which hormone participates in catabolism during the fasting state?

Catecholamines

A patient returns to the surgical unit with a NG tube to low intermittent suction, IV fluids, and a Jackson- Pratt drain at the surgical site following an exploratory laparotomy and repair of a bowel perforation. Four hours after administration, the patient experiences nausea and vomiting. What is a priority nursing intervention for the patient?

Check the amount and character of gastric drainage and the patency of the NG tube

The immunologic mechanisms involved in acute poststreptococcal glomerulonephritis include a. tubular blocking by precipitates of bacteria and antibody reactions. b. deposition of immune complexes and complement along the GBM. c. thickening of the GBM from autoimmune microangiopathic changes. d. destruction of glomeruli by proteolytic enzymes contained in the GBM.

Correct answer: b Rationale: All forms of immune complex disease are characterized by an accumulation of antigen, antibody, and complement in the glomeruli, which can result in tissue injury. The immune complexes activate complement. Complement activation results in the release of chemotactic factors that attract polymorphonuclear leukocytes, histamine, and other inflammatory mediators. The result of these processes is glomerular injury.

The nurse recommends genetic counseling for the children of a patient with a. nephrotic syndrome. b. chronic pyelonephritis. c. malignant nephrosclerosis. d. adult-onset polycystic kidney disease.

Correct answer: d Rationale: The adult form of polycystic kidney disease (PKD) is an autosomal dominant disorder. If one parent has the disease, the child has a 50% chance of developing PKD. Many patients who have adult PKD have had children by the time the disease is diagnosed. Patients need appropriate counseling regarding plans for having more children, and genetic counseling resources should be provided for the children.

A patient is admitted to the hospital with severe burns. Which hormone level does the nurse expect to be elevated in the patient's laboratory reports?

Cortisol

Which assessment finding of a 42-yr-old patient who had a bilateral adrenalectomy requires the most rapid action by the nurse? a. The blood glucose is 192 mg/dL. b. The lungs have bibasilar crackles. c. The patient reports 6/10 incisional pain. d. The blood pressure (BP) is 88/50 mm Hg.

D The decreased BP indicates possible adrenal insufficiency. The nurse should immediately notify the health care provider so that corticosteroid medications can be administered. The nurse should also address the elevated glucose, incisional pain, and crackles with appropriate collaborative or nursing actions, but prevention and treatment of acute adrenal insufficiency are the priorities after adrenalectomy.

Which assessment finding of a 42-yr-old patient who had a bilateral adrenalectomy requires the most rapid action by the nurse? a. The blood glucose is 192 mg/dL. b. The lungs have bibasilar crackles. c. The patient reports 6/10 incisional pain. d. The blood pressure (BP) is 88/50 mm Hg.

D The decreased BP indicates possible adrenal insufficiency. The nurse should immediately notify the health care provider so that corticosteroid medications can be administered. The nurse should also address the elevated glucose, incisional pain, and crackles with appropriate collaborative or nursing actions, but prevention and treatment of acute adrenal insufficiency are the priorities after adrenalectomy.

A 44-yr-old female patient with Cushing syndrome is admitted for adrenalectomy. Which intervention by the nurse will be most helpful for the patient problem of disturbed body image related to changes in appearance? a. Reassure the patient that the physical changes are very common in patients with Cushing syndrome. b. Discuss the use of diet and exercise in controlling the weight gain associated with Cushing syndrome. c. Teach the patient that the metabolic impact of Cushing syndrome is of more importance than appearance. d. Remind the patient that most of the physical changes caused by Cushing syndrome will resolve after surgery.

D The most reassuring and accurate communication to the patient is that the physical and emotional changes caused by the Cushing syndrome will resolve after hormone levels return to normal postoperatively. Reassurance that the physical changes are expected or that there are more serious physiologic problems associated with Cushing syndrome are not therapeutic responses. The patient's physiological changes are caused by the high hormone levels, not by the patient's diet or exercise choices.

Which prescribed medication should the nurse expect will have rapid effects on a patient admitted to the emergency department in thyroid storm? a. Iodine c. Propylthiouracil b. Methimazole d. Propranolol (Inderal)

D b-Adrenergic blockers work rapidly to decrease the cardiovascular manifestations of thyroid storm. The other medications take days to weeks to have an impact on thyroid function.

Which prescribed medication should the nurse expect will have rapid effects on a patient admitted to the emergency department in thyroid storm? a. Iodine c. Propylthiouracil b. Methimazole d. Propranolol (Inderal)

D b-Adrenergic blockers work rapidly to decrease the cardiovascular manifestations of thyroid storm. The other medications take days to weeks to have an impact on thyroid function.

A 68-year-old female patient admitted to the hospital with dehydration is confused and incontinent of urine. Which nursing action will be best to include in the plan of care? a. Restrict fluids between meals and after the evening meal. b. Apply absorbent incontinent pads liberally over the bed linens. c. Insert an indwelling catheter until the symptoms have resolved. d. Assist the patient to the bathroom every 2 hours during the day.

D. Assist the patient to the bathroom every 2 hours during the day In older or confused patients, incontinence may be avoided by using scheduled toileting times. Indwelling catheters increase the risk for urinary tract infection (UTI). Incontinent pads increase the risk for skin breakdown. Restricting fluids is not appropriate in a patient with dehydration.

The nurse teaches a 64-year-old woman to prevent the recurrence of renal calculi by a. using a filter to strain all urine. b. avoiding dietary sources of calcium. c. choosing diuretic fluids such as coffee. d. drinking 2000 to 3000 mL of fluid a day.

D. drinking 2000 to 3000 mL of fluid a day A fluid intake of 2000 to 3000 mL daily is recommended to help flush out minerals before stones can form. Avoidance of calcium is not usually recommended for patients with renal calculi. Coffee tends to increase stone recurrence. There is no need for a patient to strain all urine routinely after a stone has passed, and this will not prevent stones.

Which change in the endocrine system places the elderly population at an increased risk of hypertension?

Decreased β-adrenergic response to norepinephrine

Which hormone is referred to as the "fight or flight" hormone?

Epinephrine

What extraintestinal manifestations are seen in both ulcerative colitis and Crohn's disease?

Erythema nodosum and osteoporosis

A male patient who has undergone an anterior- posterior repair is worried about his sexuality. What is an appropriate nursing intervention for this patient?

Explain that physical and emotional factors can affect sexual function but not necessarily the patient's sexuality

The nurse is educating a patient newly diagnosed with diabetes about the role of insulin in controlling blood glucose levels. Arrange the following processes in the order in which they occur after eating a meal in the nondiabetic patient.

First, blood glucose levels become elevated, which stimulates the secretion of insulin from the pancreas. Because blood glucose levels decrease as a result of the insulin, the stimulus for insulin secretion also decreases. The pattern of insulin secretion is a physiologic example of a negative feedback between glucose and insulin.

Which hormone stimulates the process of gluconeogenesis and glycogenolysis?

Glucagon

Which patient is most likely to be diagnosed with short bowel syndrome?

Had extensive resection of the ileum

B

IV potassium chloride (KCl) 60 mEq is prescribed for treatment of a patient with severe hypokalemia. Which action should the nurse take? a. Administer the KCl as a rapid IV bolus. b. Infuse the KCl at a rate of 10 mEq/hour. c. Only give the KCl through a central venous line. d. Discontinue cardiac monitoring during the infusion.

The patient with a new ileostomy needs discharge teaching. What should the nurse plan to include in this teaching?

If leakage occurs, promptly remove the pouch, clean the skin, and apply a new pouch

A patient with ulcerative colitis has a total proctocolectomy with formation of a terminal ileum stoma. What is the most important nursing intervention for this patient post op

Measure the ileostomy output to determine the status of the patient's fluid balance.

A patient is admitted to the emergency department with acute abdominal pain. What nursing intervention should the nurse implement first?

Measurement of vital signs

During a routine health check-up, a patient reports a recent increase in the pigmentation on the facial skin. Which hormone is responsible for the condition caused by its increased activity?

Melanocyte-stimulating hormone (MSH)

The patient has persistent and continuous pain at McBurney's point. The nursing assessment reveals rebound tenderness and muscle guarding with the patient preferring to lie still with the right leg flexed. What should the nursing interventions for this patient include?

NPO status in preparation for possible appendectomy

Which value of blood pH indicates acidosis? 1. 7.25 2. 7.35 3. 7.45 4. 7.55

1 A pH below 7.35 indicates acidosis; thus a pH of 7.25 a sign of acidosis. Normal blood pH lies between 7.35 and 7.45. A pH of 7.55 indicates alkalosis. Text Reference - p. 302

While caring for a patient who had a pituitary tumor removed, which finding should be reported immediately to the primary health care provider? 1 Excessive thirst 2 Calcium level of 8.6 mg/dL 3 Potassium level of 3.5 mEq/L 4 Urine output of 300 mL in eight hours

1 A patient who has had surgery on the pituitary gland is at risk for diabetes insipidus. Excessive thirst is an indicator of inadequate antidiuretic hormone (ADH) synthesis or release. The nurse should monitor the urine output closely and notify the primary health care provider of excessive thirst. One would expect large volumes of urine in the absences of ADH production. A urine output of 300 mL in eight hours would not be alarming. The calcium and potassium levels are at the low end of normal. The nurse should continue to monitor these electrolytes. Text Reference - p. 294

A patient is diagnosed with Cushing syndrome. What manifestation does the nurse anticipate while assessing this patient? 1 Dyspnea 2 Dry mouth 3 Weight loss 4 Restlessness

1 Excess extracellular volume may result from fluid retention during Cushing syndrome. This shift of fluid into the interstitial spaces leads to blockage of air spaces (pulmonary edema) resulting in dyspnea, crackles, and peripheral edema. Dry mouth, weight loss, and restlessness are the common manifestations resulting from extracellular fluid depletion. Test-Taking Tip: Identifying content and what is being asked about that content is critical to your choosing the correct response. Be alert for words in the stem of the item that are the same or similar in nature to those in one or two of the options. Text Reference - p. 292

Which is a normal value of the anion gap? 1 14 mmol/L 2 15 mmol/L 3 16 mmol/L 4 17 mmol/L

1 The normal range of the anion gap is 10 to14 mmol/L. Anion gap values of 15, 16, and 17 mmol/L are abnormal. Text Reference - p. 305

While taking a patient's blood pressure, a nurse notices that a carpal spasm occurs. To diagnose a possible cause for the carpal spasm, the nurse should expect the primary health care provider to prescribe which level? 1 Calcium 2 Sodium 3 Potassium 4 Magnesium

1 Trousseau's sign (carpal spasm when blood pressure cuff is inflated for a few minutes) is indicative of hypocalcemia. It does not occur with changes in sodium, potassium, or magnesium levels. The nurse should expect the primary health care provider to prescribe a calcium level be drawn. Text Reference - p. 300

A patient is to receive methylprednisolone (Solu-Medrol) 100 mg. The label on the medication states: methylprednisolone 125 mg in 2 mL. How many milliliters will the nurse administer?

1.6 A concentration of 125 mg in 2 mL will result in 100 mg in 1.6 mL.

A patient is to receive methylprednisolone (Solu-Medrol) 100 mg. The label on the medication states: methylprednisolone 125 mg in 2 mL. How many milliliters will the nurse administer?

1.6 A concentration of 125 mg in 2 mL will result in 100 mg in 1.6 mL.

What does the nurse infer from a patient who reports abdominal cramping and diarrhea and has a serum potassium level of 6.8 mEq/L? Select all that apply. 1 The patient is on insulin therapy. 2 The patient is on a potassium-sparing diuretic. 3 The patient is on β-adrenergic agonist therapy. 4 The patient is on an aldosterone receptor blocker. 5 The patient is on an angiotensin-converting enzyme inhibitor.

2, 3, 5 Potassium levels greater than 5.0 mEq/L indicate hyperkalemia. This can manifest as abdominal cramping and diarrhea. Potassium-sparing diuretics, β-adrenergic agonist therapy, and angiotensin-converting enzyme inhibitors increase the potassium levels in the body. This results in hyperkalemia. Insulin and aldosterone receptor blockers decrease potassium levels. Text Reference - p. 296

A nurse is assessing a patient's weight in order to evaluate fluid volume status. The patient's weight on the day of admission was 60 kg. On day 2, the weight is 62 kg. What is the quantity of fluid retention in the patient? Record your answer using a whole number and no punctuation. _______ mL

2000 An increase in 1 kg is equal to 1000 mL of fluid retention. The patient has gained 2 kg, which is equal to 2000 mL of fluid retention. Text Reference - p. 292

A 71-year-old patient is admitted with nausea and vomiting. Which manifestations would the nurse assess to check for the presence of dehydration? Select all that apply. 1 Hypertension 2 Bradypnea 3 Urine output 10 mL/hr 4 Tachycardia 5 Sunken eyes

3, 4, 5 Decreased urine output below 30 mL/hour, tachycardia, and sunken eyes are all signs of dehydration. Dehydration will cause hypotension and would increase, not decrease, respiratory rate. Text Reference - p. 292

Which statement is true regarding magnesium? 1 A decrease in magnesium levels decreases blood pressure. 2 Magnesium is the most abundant intracellular cation present in the body. 3 The majority of the magnesium in the human body is present in extracellular fluid. 4 Alterations in serum magnesium levels profoundly affect neuromuscular excitability and contractility.

4 Alterations in serum magnesium levels profoundly affect neuromuscular excitability and contractility because magnesium directly acts on the myoneural junction. A decrease in blood magnesium levels increases the blood pressure. Magnesium is the second most abundant intracellular cation. The majority of the body's magnesium is present in the bones. Test-Taking Tip: Identify option components as correct or incorrect. This may help you identify a wrong answer. Text Reference - p. 301

The nurse preparing to administer a dose of potassium phosphate would hold the medication after noting which laboratory value? 1 Sodium 133 mEq/L 2 Magnesium 1.8 mEq/L 3 Potassium 5.2 mEq/L 4 Calcium 6.4 mg/dL

4 Phosphorus and calcium have inverse or reciprocal relationships, meaning that when calcium levels are high, phosphorus levels tend to be low. Therefore, administration of phosphorus will reduce a patient's already abnormally low calcium level, which can result in life-threatening complications. Potassium phosphate will not have any effect on sodium, magnesium, or potassium levels. Text Reference - p. 301

A

A patient has a serum calcium level of 7.0 mEq/L. Which assessment finding is most important for the nurse to report to the health care provider? a. The patient is experiencing laryngeal stridor. b. The patient complains of generalized fatigue. c. The patient's bowels have not moved for 4 days. d. The patient has numbness and tingling of the lips.

B

A patient is admitted for hypovolemia associated with multiple draining wounds. Which assessment would be the most accurate way for the nurse to evaluate fluid balance? a. Skin turgor b. Daily weight c. Presence of edema d. Hourly urine output

A

A patient who is lethargic and exhibits deep, rapid respirations has the following arterial blood gas (ABG) results: pH 7.32, PaO2 88 mm Hg, PaCO2 37 mm Hg, and HCO3 16 mEq/L. How should the nurse interpret these results? a. Metabolic acidosis b. Metabolic alkalosis c. Respiratory acidosis d. Respiratory alkalosis

16. A nurse is formulating a teaching plan for a 22-year-old woman taking rosiglitazone (Avandia). What should the nurse include information about in this plan to caution this patient? a. Decreased effectiveness of her birth control pills b. Excessive exposure to the sun c. Sudden drop in blood pressure with dizziness d. Possible severe diarrhea

ANS: A Avandia causes some birth control pills to be less effective.

27. A teaching plan for a patient with diabetes is focused on smoking cessation and the control of hypertension for the avoidance of microvascular complications. What are examples of microvascular complications? (Select all that apply.) a. Macular degeneration b. End-stage renal disease (ESRD) c. Coronary artery disease (CAD) d. Peripheral vascular disease (PVD) e. Cerebrovascular accident (CVA)

ANS: A, B Macular degeneration and ESRD are both microvascular complications. CAD, PVD, and CVA are all macrovascular complications.

10. A patient who recently has been experiencing frequent heartburn is seen in the clinic. The nurse will anticipate teaching the patient about a. barium swallow. b. radionuclide tests. c. endoscopy procedures. d. proton pump inhibitors.

ANS: D Because diagnostic testing for heartburn that is probably caused by gastroesophageal reflux disease (GERD) is expensive and uncomfortable, proton pump inhibitors are frequently used for a short period as the first step in the diagnosis of GERD. The other tests may be used but are not usually the first step in diagnosis.

34. Which question from the nurse would help determine if a patient's abdominal pain might indicate irritable bowel syndrome? a. "Have you been passing a lot of gas?" b. "What foods affect your bowel patterns?" c. "Do you have any abdominal distention?" d. "How long have you had abdominal pain?"

ANS: D One criterion for the diagnosis of irritable bowel syndrome (IBS) is the presence of abdominal discomfort or pain for at least 3 months. Abdominal distention, flatulence, and food intolerance are also associated with IBS, but are not diagnostic criteria.

39. A patient who is vomiting bright red blood is admitted to the emergency department. Which assessment should the nurse perform first? a. Checking the level of consciousness b. Measuring the quantity of any emesis c. Auscultating the chest for breath sounds d. Taking the blood pressure (BP) and pulse

ANS: D The nurse is concerned about blood loss and possible hypovolemic shock in a patient with acute gastrointestinal (GI) bleeding; BP and pulse are the best indicators of these complications. The other information also is important to obtain, but BP and pulse rate are the best indicators for hypoperfusion.

Which hormone is essential for the maintenance of fluid and electrolyte balance?

Aldosterone

What information should be included when the nurse teaches a patient about colostomy irrigation?

Allow 30 to 45 minutes for the solution and feces to be expelled.

A postop patient has a nursing diagnosis of pain related to effects of medication and decreased GI motility as evidenced by abdominal pain and distension and inability to pass flatus. Which nursing intervention is most appropriate for this patient?

Ambulate the patient more frequently

Which nursing assessment of a 70-yr-old patient is most important to make during initiation of thyroid replacement with levothyroxine (Synthroid)? a. Fluid balance b. Apical pulse rate c. Nutritional intake d. Orientation and alertness

B In older patients, initiation of levothyroxine therapy can increase myocardial oxygen demand and cause angina or dysrhythmias. The medication also is expected to improve mental status and fluid balance and will increase metabolic rate and nutritional needs, but these changes will not result in potentially life-threatening complications.

Which nursing assessment of a 70-yr-old patient is most important to make during initiation of thyroid replacement with levothyroxine (Synthroid)? a. Fluid balance b. Apical pulse rate c. Nutritional intake d. Orientation and alertness

B In older patients, initiation of levothyroxine therapy can increase myocardial oxygen demand and cause angina or dysrhythmias. The medication also is expected to improve mental status and fluid balance and will increase metabolic rate and nutritional needs, but these changes will not result in potentially life-threatening complications.

A 40-year-old male patient has been newly diagnosed with type 2 diabetes mellitus. Which information about the patient will be most useful to the nurse who is helping the patient develop strategies for successful adaptation to this disease? A. Ideal weight B. Value system C. Activity level D. Visual changes

B. Value system - When dealing with a patient with a chronic condition such as diabetes, identification of the patient's values and beliefs can assist the health care team in choosing strategies for successful lifestyle change. The other information also will be useful, but is not as important in developing an individualized plan for the necessary lifestyle changes.

The nurse is caring for a patient following an adrenalectomy. The highest priority in the immediate postoperative period is to a. protect the patient's skin. b. monitor for signs of infection. c. balance fluids and electrolytes. d. prevent emotional disturbances.

C After adrenalectomy, the patient is at risk for circulatory instability caused by fluctuating hormone levels, and the focus of care is to assess and maintain fluid and electrolyte status through the use of IV fluids and corticosteroids. The other goals are also important for the patient but are not as immediately life threatening as the circulatory collapse that can occur with fluid and electrolyte disturbances.

The nurse is caring for a patient following an adrenalectomy. The highest priority in the immediate postoperative period is to a. protect the patient's skin. b. monitor for signs of infection. c. balance fluids and electrolytes. d. prevent emotional disturbances.

C After adrenalectomy, the patient is at risk for circulatory instability caused by fluctuating hormone levels, and the focus of care is to assess and maintain fluid and electrolyte status through the use of IV fluids and corticosteroids. The other goals are also important for the patient but are not as immediately life threatening as the circulatory collapse that can occur with fluid and electrolyte disturbances.

What lab findings are expected in ulcerative colitis as a result of diarrhea and vomiting?

Decreased NA+, K+, Mg+, Cl-, and HCO3-

When caring for a patient with IBS, what is the most important for the nurse to do?

Develop a trusting relationship with the patient to provide support and symptomatic care.

When obtaining a nursing history from the patient with colorectal cancer, the nurse should specifically ask the patient about

Dietary intake

Which hormones are released by the adrenal medulla? Select all that apply.

Dopamine Epinephrine Norepinephrine

The nurse should teach the patient with chronic constipation that which food has the highest dietary fiber?

Dried beans

C

During the admission process, the nurse obtains information about a patient through the physical assessment and diagnostic testing. Based on the data shown in the accompanying figure, which nursing diagnosis is appropriate? a. Deficient fluid volume b. Impaired gas exchange c. Risk for injury: Seizures d. Risk for impaired skin integrity

Which cells of the islets of Langerhans produce a hormone that influences a patient's metabolism of absorbed nutrients?

F cells

A

Following a thyroidectomy, a patient complains of "a tingling feeling around my mouth." Which assessment should the nurse complete immediately? a. Presence of the Chvostek's sign b. Abnormal serum potassium level c. Decreased thyroid hormone level d. Bleeding on the patient's dressing

A 60 year old African American patient is afraid she might have anal cancer. What assessment finding puts her at high risk for anal cancer?

Human papillomavirus (HPV)

The patient has peritonitis, which is a major complication of appendicitis. What treatment will the nurse plan to include?

IV fluid replacement.

The nurse knows that the hormone produced by the parathyroid glands targets which organ?

Kidneys

What should the nurse teach the patient with diverticulosis to do?

Maintain a high- fiber diet and use bulk laxatives to increase fecal volume

The nurse reviews the laboratory reports of four patients. Which patient may have impaired functioning of the sympathetic nervous system?

Patient D

How can patient help to prevent future bladder tumors from occurring?

Patient should stop smoking; it is the only significant risk factor in his history. Maintenance therapy after the initial induction regimen may be beneficial, as there is a high rate of disease recurrence.

When providing care to a patient with a tumor located on the anterior pituitary gland, which of the patient's hormones will the nurse expect will be affected? Select all that apply.

Prolactin Thyrotropin Somatotropin

An important nursing intervention for a patient with a small intestinal obstruction who has an NG tube is to

Provide mouth care every 1 to 2 hours.

What is the function of parathormone?

Regulation of calcium and phosphorus blood levels

In report, the nurse learns that the patient has a transverse colostomy. What should the nurse expect when providing care for this patient?

Semiformed stools in a pouch with the need to monitor fluid balance

B

Several patient are seen at an urgent care center with symptoms of nausea, vomiting, and diarrhea that began 2 hours ago while attending a large family reunion potluck dinner. You question the patients specifically about foods they ingested containing: A. beef B. meat and milk C. poultry and eggs D. home preserved vegetables

A physician just told a patient that she has a volvulus. When the patient asks the nurse what this is, what is the best description for the nurse to give her?

Twisting of the bowel on itself

When a 35-year-old female patient is admitted to the emergency department with acute abdominal pain, which possible diagnosis should you consider that may be the cause of her pain (select all that apply)? a. Gastroenteritis b. Ectopic pregnancy c. GI bleeding d. Irritable bowel syndrome e. Inflammatory bowel disease

a, b, c, d & e Rationale: All these conditions could cause acute abdominal pain.

A patient who is given a bisacodyl (Dulcolax) suppository asks the nurse how long it will take to work. The nurse replies that the patient will probably need to use the bedpan or commode within which time frame after administration? a. 2-5 minutes b. 15-60 minutes c. 2-4 hours d. 6-8 hours

b. 15-60 minutes Bisacodyl suppositories usually are effective within 15 to 60 minutes of administration, so the nurse should plan accordingly to assist the patient to use the bedpan or commode.

A nurse is admitting a patient with the diagnosis of advanced renal carcinoma. Based upon this diagnosis, the nurse will expect to find what clinical manifestations as the "classic triad" occurring in patients with renal cancer? a. Fever, chills, flank pain b. Hematuria, flank pain, palpable mass c. Hematuria, proteinuria, palpable mass d. Flank pain, palpable abdominal mass, and proteinuria

b. Hematuria, flank pain, palpable mass There are no characteristic early symptoms of renal carcinoma. The classic manifestations of gross hematuria, flank pain, and a palpable mass are those of advanced disease.

A 48-year-old male patient screened for diabetes at a clinic has a fasting plasma glucose level of 120 mg/dL (6.7 mmol/L). The nurse will plan to teach the patient about a. self-monitoring of blood glucose. b. using low doses of regular insulin. c. lifestyle changes to lower blood glucose. d. effects of oral hypoglycemic medications.

c. lifestyle changes to lower blood glucose.

Which statement by a 50-year-old female patient indicates to the nurse that further assessment of thyroid function may be necessary? A. "I notice my breasts are tender lately." B. "I am so thirsty that I drink all day long." C. "I get up several times at night to urinate." D. "I feel a lump in my throat when I swallow."

D. "I feel a lump in my throat when I swallow." - Difficulty in swallowing can occur with a goiter. Nocturia is associated with diseases such as diabetes mellitus, diabetes insipidus, or chronic kidney disease. Breast tenderness would occur with excessive gonadal hormone levels. Thirst is a sign of disease such as diabetes.

An 18-year-old male patient with a small stature is scheduled for a growth hormone stimulation test. In preparation for the test, the nurse will obtain A. Ice in a basin. B. Glargine insulin. C. A cardiac monitor. D. 50% dextrose solution.

D. 50% dextrose solution. - Hypoglycemia is induced during the growth hormone stimulation test, and the nurse should be ready to administer 50% dextrose immediately. Regular insulin is used to induce hypoglycemia (glargine is never given IV). The patient does not require cardiac monitoring during the test. Although blood samples for some tests must be kept on ice, this is not true for the growth hormone stimulation test.

Which serum potassium results best support the rationale for administering a stat dose of potassium chloride 20 mEq in 250 mL of normal saline over two hours? 1 3.1 mEq/L 2 3.9 mEq/L 3 4.6 mEq/L 4 5.3 mEq/L

1 The normal range for serum potassium is 3.5 to 5.0 mEq/L. This intravenous (IV) prescription provides a substantial amount of potassium. Thus the patient's potassium level must be low. The only low value shown is 3.1 mEq/L; 3.9 mEq/L, 4.6 mEq/L, and 5.3 mEq/L are not low values. Text Reference - p. 296

25. What are functional causes of hypoglycemia? (Select all that apply.) a. Dumping syndrome b. Overdose of insulin c. Addison disease d. Prolonged muscular exercise e. Chronic alcoholism

ANS: A, C, D Dumping syndrome, Addison disease, and prolonged exercise are functional causes of hypoglycemia. Overdose of insulin and chronic alcoholism are exogenous causes.

During routine screening colonoscopy on a 56 year old patient, a rectosigmoidal polyp was identified and removed. The patient asks the nurse if his risk for colon cancer is increased because of the polyp. What is the best response by the nurse?

"All polyps are abnormal and should be removed but the risk for cancer depends on the type and if malignant changes are present."

A patient with blood pressure of 160/90 mm Hg has pedal edema. Which process of transport of molecules would be in action? 1 Osmosis 2 Diffusion 3 Active transport 4 Facilitated diffusion

1 A patient with blood pressure of 160/90 mm Hg has hypertension and develops pedal edema due to excess sodium in the blood. This leads to movement of water down the gradient. Therefore, the water from the blood vessels moves from higher concentrations to lower concentration across the semipermeable membrane with the help of osmotic pressure and leads to accumulation of water in the extracellular spaces. Diffusion, active transport, and facilitated diffusion involve the movement of molecules from lower concentrations to higher concentrations. Text Reference - p. 287

Which phosphate level would the nurse be likely to find in the patient who has alcohol withdrawal symptoms? 1 1.4 mg/dL 2 2.4 mg/dL 3 3.8 mg/dL 4 4.8 mg/dL

1 Alcohol withdrawal can result in hypophosphatemia. Phosphate levels of less than 2.4 mg/dL indicate hypophosphatemia. The nurse would be likely to find the patient's phosphate level at 1.4 mg/dL. Phosphate levels of 2.4, 3.8, and 4.8 mg/dL indicate hyperphosphatemia. The patient with symptoms of alcohol withdrawal does not have hyperphosphatemia. Test-Taking Tip: Read the question carefully before looking at the answers: (1) Determine what the question is really asking; look for key words; (2) Read each answer thoroughly and see if it completely covers the material asked by the question; (3) Narrow the choices by immediately eliminating answers you know are incorrect. Text Reference - p. 301

The nurse anticipates that treatment of the patient with severe hyperphosphatemia includes: 1 Insulin infusion 2 Fluid restriction 3 Calcium supplements 4 Loop diuretic therapy

1 For severe hyperphosphatemia, hemodialysis or an insulin and glucose infusion can decrease levels rapidly. Fluid restriction, calcium supplements, and diuretic therapy are not treatment options for hyperphosphatemia. Text Reference - p. 301

The nurse is caring for a patient with hyponatremia related to excess fluid. Which of these interventions is appropriate initially? 1 Fluid restriction 2 Administration of hypotonic intravenous (IV) fluids 3 Administration of a cation-exchange resin 4 Oral doses of sodium chloride three times a day

1 In hyponatremia that is caused by water excess, fluid restriction often is all that is needed to treat the problem. Small amounts of IV hypertonic saline solution (3%) may be given if severe symptoms, such as seizures, develop. Treatment of hyponatremia associated with abnormal fluid loss includes fluid replacement with sodium-containing solutions. Administration of oral sodium chloride is not appropriate. Text Reference - p. 296

A patient sustains multiple injuries in a motor vehicle accident and is hypovolemic due to hemorrhage. Blood transfusions are given to replace the lost blood. The nurse finds that the patient has now developed laryngeal stridor, dysphagia, and numbness and tingling around the mouth. What could be the reason for these new manifestations? 1 The patient has developed hypocalcemia 2 The patient has developed fluid overload 3 The patient has developed anemia 4 The patient has developed a hemolytic reaction

1 Laryngeal stridor, dysphagia, and numbness and tingling around the mouth after multiple blood transfusions can be attributed to hypocalcemia. Blood and blood products have citrate in them, which can bind with calcium in the body and make it unavailable. Multiple blood transfusions have thus caused hypocalcemia. This usually manifests as laryngeal stridor, dysphagia, and numbness and tingling around the mouth. Such symptoms are not caused by fluid overload, which manifests as edema. Anemia can be the result of hemorrhage but does not present with laryngeal stridor and dysphagia. There are chances of hemolytic reactions, because the patient is receiving multiple transfusions. However, a hemolytic reaction manifests as severe anaphylaxis, so the patient is not having a hemolytic reaction. Text Reference - p. 300

A nurse is providing postoperative care for a patient. The primary health care provider assesses the patient and advises the nurse to stop all intravenous fluids except for continuous maintenance fluids. Which fluid will the nurse administer as a maintenance fluid? 1 0.45% NaCl 2 Lactated Ringer's solution 3 5% dextrose in 0.9% saline 4 25% albumin solution

1 Maintenance fluids are hypotonic solutions because the body's daily losses are hypotonic. The nurse should therefore administer 0.45% NaCl, which is hypotonic. Lactated Ringer's solution is an isotonic solution and is not used as a maintenance fluid, because it tends to expand the extracellular compartment. Five percent dextrose in 0.9% saline is a hypertonic solution that expands intravascular volume and replaces extracellular fluid losses; 25% albumin solution is a plasma expander and not used as a maintenance fluid. Text Reference - p. 308

The arterial blood gas analysis shows a partial pressure of carbon dioxide (PaCO2) of 43 mm Hg and pH of 5.1 in a patient with diabetes. What would be the patient's diagnosis? 1 Metabolic acidosis 2 Metabolic alkalosis 3 Respiratory acidosis 4 Respiratory alkalosis

1 Metabolic acidosis is characterized by increased levels of acid in the blood. As a result, pH of the blood decreases. The normal range of pH of blood is 7.35 to 7.45, and the normal value of partial pressure of carbon dioxide (PaCO2) lies between 35 and 45 mm Hg. The patient's numbers indicate metabolic acidosis. Metabolic alkalosis is manifested by an increased pH. A decreased pH and elevated PaCO2 indicate respiratory acidosis. Respiratory alkalosis is manifested by increased plasma pH and decreased PaCO2. Text Reference - p. 304

A patient has been treated successfully for dehydration. The nurse would expect: 1 Oral intake balances output. 2 Oral intake is less than output. 3 Oral intake is greater than output. 4 No significant difference in fluid balance.

1 Oral intake should equal output if fluid balance has been restored and dehydration has been corrected. Less intake than output would result in dehydration. Greater intake than output may indicate decreased renal function or impaired ability to excrete urine. Text Reference - p. 292

Which treatment could be a possible reason for a cancer patient to have a serum phosphate level of 5.4 mg/dL? 1 Chemotherapy 2 Insulin therapy 3 Total parenteral nutrition 4 Phosphate-binding antacids

1 Phosphate levels greater than 4.4 mg/dL indicate hyperphosphatemia. Chemotherapy drugs increase the patient's phosphate levels. Insulin therapy decreases the phosphate levels to less than 2.4 mg/dL. Patients with total parenteral nutrition have decreased phosphate levels. Phosphate-binding antacids remove phosphates from the body, resulting in hypophosphatemia. Text Reference - p. 301

While caring for a patient with metastatic bone cancer, which clinical manifestations would alert the nurse to the possibility of hypercalcemia in this patient? 1 Weakness 2 Paresthesia 3 Facial spasms 4 Muscle tremors

1 Signs of hypercalcemia are lethargy, headache, weakness, muscle flaccidity, heart block, anorexia, nausea, and vomiting. Paresthesia, facial spasms, and muscle tremors are symptoms of hypocalcemia. Text Reference - p. 299

What is the pH of urine excreted by the kidneys as a compensatory response? 1. 8 2. 9 3. 10 4. 11

1 The average pH of urine excreted by the kidneys is 6. As a compensatory response, the kidneys excrete urine with a pH ranging from 4 to 8. Text Reference - p. 303

When assessing the patient with a multilumen central line, the nurse notices that the cap is off one of the lines. On assessment, the patient is in respiratory distress and the vital signs show hypotension and tachycardia. What is the nurse's priority action? 1 Administer oxygen 2 Notify the health care provider 3 Rapidly administer more intravenous (IV) fluid 4 Reposition the patient to the right side

1 The cap off the central line could allow entry of air into the circulation. For an air embolus, the priority is to administer oxygen; next, the catheter is clamped and the patient is positioned on the left side with the head down. Then the health care provider is notified. Test-Taking Tip: Being emotionally prepared for an examination is key to your success. Proper use of this text over an extended period of time ensures your understanding of the mechanics of the examination and increases your confidence about your nursing knowledge. Your lifelong dream of becoming a nurse is now within your reach! You are excited, yet anxious. This feeling is normal. A little anxiety can be good because it increases awareness of reality; but excessive anxiety has the opposite effect, acting as a barrier and keeping you from reaching your goal. Your attitude about yourself and your goals will help keep you focused, adding to your strength and inner conviction to achieve success. Text Reference - p. 311

The nurse receives a health care provider's prescription to change a patient's intravenous (IV) from D5½ normal saline (NS) with 40 mEq KCl/L to D5NS with 20 mEq KCl/L. Which serum laboratory value on this same patient best supports the rationale for this IV prescription change? 1 Sodium 136 mEq/L, potassium 4.5 mEq/L 2 Sodium 145 mEq/L, potassium 4.8 mEq/L 3 Sodium 135 mEq/L, potassium 3.6 mEq/L 4 Sodium 144 mEq/L, potassium 3.7 mEq/L

1 The normal range for serum sodium is 135 to 145 mEq/L, and the normal range for potassium is 3.5 to 5.0 mEq/L. The change in the IV prescription decreases the amount of potassium and increases the amount of sodium. For this prescription to be appropriate, the potassium level must be near the high end and the sodium level near the low end of their respective ranges. Test-Taking Tip: Avoid looking for an answer pattern or code. There may be times when four or five consecutive questions have the same letter or number for the correct answer. Text Reference - p. 296

While caring for a patient with encephalitis, the nurse suspects that the patient has developed respiratory alkalosis. Which finding in the patient supports the nurse's suspicion? 1 Bicarbonate ion concentration, 18 mEq/L, partial pressure of carbon dioxide, 30 mm Hg 2 Bicarbonate ion concentration, 22 mEq/L, partial pressure of carbon dioxide, 35 mm Hg 3 Bicarbonate ion concentration, 24 mEq/L, partial pressure of carbon dioxide, 43 mm Hg 4 Bicarbonate ion concentration, 26 mEq/L, partial pressure of carbon dioxide, 45 mm Hg

1 The normal range of bicarbonate (HCO3-) ion concentration in blood is 22 to 26 mEq/L and the normal range of partial pressure of carbon dioxide (PaCO2) is 35 to 45 mm Hg. When the respiratory center is stimulated, patients with encephalitis will hyperventilate. This condition causes a decrease in partial pressure of carbon dioxide, resulting in decreased carbonic acid concentration. Because the laboratory reports show a decreased partial pressure of carbon dioxide and bicarbonate ion concentration, the nurse suspects respiratory alkalosis. Test-Taking Tip: Encephalitis is an inflammatory condition of brain, wherein the brain's key functions may get altered. Use this tip in answering the question. Text Reference - p. 304

A diabetic patient fasting before surgery reports feeling dizzy and deep rapid breathing. A nurse observes that the patient has developed Kussmaul respirations. What condition is the patient most likely experiencing? 1 Metabolic acidosis 2 Metabolic alkalosis 3 Respiratory acidosis 4 Respiratory alkalosis

1 The patient has been fasting and complains of dizziness. The patient has likely developed diabetic ketoacidosis, a type of metabolic acidosis. Kussmaul respiration is deep, rapid breathing that develops in response to metabolic acidosis. This type of breathing is a compensatory mechanism to excrete excess carbon dioxide from the lungs. Metabolic alkalosis occurs when there is a loss of acid or a gain in bicarbonate. It is not associated with Kussmaul respiration. Respiratory acidosis results when the person hypoventilates and carbonic acid accumulates in the blood. Respiratory alkalosis occurs when the person hyperventilates. Text Reference - p. 303

Which electrolyte changes can be identified by Trousseau's sign? 1 Hypocalcemia 2 Hypercalcemia 3 Hypermagnesemia 4 Hyperphosphatemia

1 Trousseau's sign refers to carpal spasms induced by inflating a blood pressure cuff on the arm. Hypocalcemia can be identified by Trousseau's sign. Hypercalcemia, hypermagnesemia, and hyperphosphatemia cannot be identified by Trousseau's sign. Text Reference - p. 300

A nurse is caring for a patient who reports diarrhea and vomiting for the past five days. As a result, the patient has developed severe hypokalemia. The primary health care provider prescribes IV potassium chloride (KCl) treatment. How can the nurse ensure the safety of the patient when administering IV KCl? Select all that apply. 1 Continuously monitor cardiac function 2 Check hourly for the presence of phlebitis at the IV site 3 Monitor the urine output 4 Assess for signs of tetany 5 Assess for laryngeal spasms

1, 2, 3 IV potassium chloride (KCl) is administered to treat hypokalemia. IV administration of KCl may cause rapid changes in potassium levels, which may adversely affect the heart. Therefore, the patient should be under continuous cardiac monitoring. KCl is an irritant and may cause phlebitis and infiltration, leading to necrosis and sloughing. So the nurse should frequently check the IV site for phlebitis and infiltration. KCl is administered when the urine output is at least 0.5 mL/kg of body weight per hour. The urine output should hence be monitored to check for its adequacy. Tetany and laryngeal spasms occur when there are low levels of calcium. They are not related to potassium levels. Text Reference - p. 298

A nurse is teaching the nursing assistants the importance of performing accurate weighings. Which content should be included? Select all that apply. 1 Calibrate the scale before using 2 Weigh patient with same garments each day 3 Weigh the patient at the same time each day 4 Empty the Foley catheter after weight obtained 5 Teach the patient the importance of daily weighings 6 If bed scale used, weigh with same number of linens on the bed

1, 2, 3, 6 In many settings the nursing assistant weighs the patient. Accurate weighings help determine medical treatment. It is essential to teach these health care providers the importance of calibrating the scale before using, weighing the patients with the same garments and at the same time each day, and if using a bed scale, documenting and weighing with the same number of linens and pillows on the bed. The drains, including the Foley catheter, should be drained before weighing. If the patient does not have an indwelling catheter, the weight should be obtained after the patient voids. Teaching the patient the importance of weighing daily is a nurse function and should not be delegated to the nursing assistant. Text Reference - p. 292

The dehydrated patient is receiving a hypertonic solution. What assessments must be done to avoid risk factors of these solutions? Select all that apply. 1 Lung sounds 2 Bowel sounds 3 Blood pressure (BP) 4 Serum sodium level 5 Serum potassium level

1, 3, 4 BP, lung sounds, and serum sodium levels must be monitored frequently because of the risk for excess intravascular volume with hypertonic solutions. Bowel sounds and serum potassium level do not need to be monitored frequently. Text Reference - p. 309

A nurse has been caring for a patient with a high fever and diarrhea for the past five days. Which clinical manifestations suggest extracellular fluid volume deficit? Select all that apply. 1 Restlessness 2 Peripheral edema 3 Concentrated urine 4 Dry mucous membranes 5 Jugular venous distention

1, 3, 4 Hypovolemia or extracellular fluid deficit occurs when there is excess loss of body fluid, as can often occur after prolonged diarrhea. Its clinical manifestations include restlessness, concentrated urine, and dry mucous membranes. Dehydration may make the patient restless. Urine may be concentrated due to low urine output. The mucous membranes get dry because of fluid deficit. Peripheral edema and jugular venous distention are signs of extracellular volume excess caused by abnormal retention of fluids. Text Reference - p. 292

When the nurse is caring for a patient with a central venous access device, which nursing interventions are important to maintain a safe, functioning device? Select all that apply. 1 Change the catheter dressing regularly. 2 Monitor the heart rate and blood pressure. 3 Change the injection caps at regular intervals. 4 Cleanse around the catheter insertion site. 5 Measure and record oral intake and output

1, 3, 4 Nursing management of central venous access devices is important in keeping the devices safe and functioning and in reducing risk of infection. The catheter dressing and the injection caps should be regularly changed, and the catheter site should be regularly cleansed; these steps keep the site free from infection. Flushing is an important intervention to maintain the patency of the catheter and prevent occlusion. Monitoring vital parameters and assessing intake and output are general measures that are not specific to the care of central venous access devices. Text Reference - p. 311

The nurse is reviewing the mechanisms of acid-base buffers in the body. The kidneys act as an acid-base buffer by which of these mechanisms? Select all that apply. 1 Eliminating excess H+ 2 Excreting excess water 3 Eliminating excess CO2 4 Reabsorbing additional HCO3- 5 Reabsorbing additional sodium ions

1, 4 As a compensatory mechanism, the pH of the urine can decrease to 4 or increase to 8. To compensate for acidosis, the kidneys can reabsorb additional HCO3- and eliminate excess H+. Thus, the pH of the blood increases and the pH of the urine decreases. Text Reference - p. 303

On assessment, the nurse finds that a patient has a headache, increased blood pressure, peripheral edema, dyspnea, and jugular venous distention. The symptoms indicate excess fluid volume. Which causes of excess fluid volume might the nurse find in the patient? Select all that apply. 1 Heart failure 2 Hemorrhage 3 Diabetic insipidus 4 Long-term use of corticosteroids 5 Syndrome of inappropriate antidiuretic hormone (SIADH

1, 4, 5 Excess volume of fluid can accumulate in illnesses such as heart failure and SIADH, or due to long-term use of corticosteroids. In heart failure, the heart is unable to pump adequate blood to the body, resulting in pooling of blood in the periphery. In SIADH, abnormal levels of ADH cause reabsorption of water from the kidneys, leading to water retention in the body. Long-term use of corticosteroids causes altered homeostatic regulation of sodium and water, resulting in excess fluid volume. Hemorrhage and diabetic insipidus cause a deficit in fluid volume. Text Reference - p. 292

The nurse is caring for a patient admitted with a diagnosis of chronic obstructive pulmonary disease (COPD) who has the following arterial blood gas results: pH 7.33, PaO2 47 mm Hg, PaCO2 60 mm Hg, HCO3 32 mEq/L, and O2 saturation of 92%. What is the correct interpretation of these results? 1 Fully compensated respiratory alkalosis 2 Partially compensated respiratory acidosis 3 Normal acid-base balance with hypoxemia 4 Normal acid-base balance with hypercapnia

2 A low pH (normal 7.35-7.45) indicates acidosis. In the patient with a respiratory disease such as COPD, the patient retains carbon dioxide (normal 35-45 mm Hg), which acts as an acid in the body. For this reason, the patient has respiratory acidosis. The elevated HCO3 indicates a partial compensation for the elevated CO2. Text Reference - p. 306

Which condition is an example of a mixed acid-base disorder? 1 Hypoxia and respiratory alkalosis 2 Septicemia and metabolic alkalosis 3 Sedative overdose and respiratory acidosis 4 Diabetic ketoacidosis and metabolic acidosis

2 A mixed acid-base disorder is a condition in which two or more disorders that affect the acid-base balance are present at the same time. Septicemia causes respiratory alkalosis, which causes acid-base imbalance. Metabolic alkalosis also affects the acid-base balance. Thus, septicemia and metabolic alkalosis are examples of a mixed acid-base disorder. Hypoxia causes respiratory alkalosis. Overdose of sedatives causes respiratory acidosis. Diabetic ketoacidosis results in metabolic acidosis. Test-Taking Tip: You have at least a 25 percent chance of selecting the correct response in multiple-choice items. If you are uncertain about a question, eliminate the choices that you believe are wrong and then call on your knowledge, skills, and abilities to choose from the remaining responses. Text Reference - p. 305

The patient was admitted for a paracentesis to remove ascites fluid. Five liters of fluid were removed. What intravenous (IV) solution may be used to pull fluid into the intravascular space after the paracentesis? 1 0.9% sodium chloride 2 25% albumin solution 3 Lactated Ringer's solution 4 5% dextrose in 0.45% saline

2 After a paracentesis of 5 L or greater of ascites fluid, 25% albumin solution may be used as a volume expander. Normal saline, lactated Ringer's, and 5% dextrose in 0.45% saline will not be effective for this action. Test-Taking Tip: Read every word of each question and option before responding to the item. Glossing over the questions just to get through the examination quickly can cause you to misread or misinterpret the real intent of the question. Text Reference - p. 309

A 68-year-old patient was admitted with abdominal pain, nausea, and severe diarrhea. Based on this information, the nurse assesses this patient for which primary acid-base imbalance? 1 Metabolic alkalosis 2 Metabolic acidosis 3 Respiratory alkalosis 4 Respiratory acidosis

2 Because gastric secretions are rich in hydrochloric acid, the patient with severe diarrhea will lose significant amounts of bicarbonate and is at increased risk for metabolic acidosis and a fall in pH. Metabolic alkalosis, respiratory alkalosis, and respiratory acidosis will not occur as a result of increased loss of bicarbonate. STUDY TIP: The old standbys of enough sleep and adequate nutritional intake also help keep excessive stress at bay. Although nursing students learn about the body's energy needs in anatomy and physiology classes, somehow they tend to forget that glucose is necessary for brain cells to work. Skipping breakfast or lunch or surviving on junk food puts the brain at a disadvantage. Text Reference - p. 304

What amount of plasma is present in the body of a patient who weighs 55 kg? Record your answer using two decimal places. ____________ L

2 Extracellular fluid accounts to one-third of the body water, and plasma represents 20 percent of the extracellular fluid. Therefore, a patient weighing 55 kg has 30.3 L of total body water and about 10 L of extracellular fluid, and plasma represents 20 percent of the extracellular fluid, which equals approximately 2 L. Test-Taking Tip: Plasma is the component of extracellular fluid. Extracellular fluid accounts to one third of the body fluids and plasma accommodates 20 percent of extracellular fluid. Derive the correct answer by focusing on the details provided. Text Reference - p. 286

A patient reports weight gain, diarrhea, headache, nausea, and vomiting. The patient's blood pressure is 140/90 mm Hg. The patient's previous health records reveal primary hypoaldosteronism. Which medication may be beneficial for the patient? 1 Amiloride 2 Conivaptan 3 Propranolol 4 Sodium polystyrene sulfonate

2 Hyponatremia is decreased sodium levels in the body. Hyponatremia can be manifested by weight gain, diarrhea, headache, nausea, vomiting, and elevated blood pressure. Conivaptan blocks the activity of antidiuretic hormone and results in increased urine output without loss of electrolytes, thereby improving the patient's hyponatremia. Amiloride, propranolol, and sodium polystyrene sulfate treat hypokalemia, but not hyponatremia. Test-Taking Tip: Recall the clinical manifestations of hyponatremia and drugs used to treat hyponatremia to answer this question accurately. Text Reference - p. 296

Which statement made by the student nurse regarding the hypothalamic-pituitary regulation needs correction? 1 "Consumption of water is less during stress." 2 "Dry mouth is the measuring tool for fluid deficit." 3 "Morphine facilitates antidiuretic hormone release." 4 "Osmoreceptors in the hypothalamus stimulate thirst."

2 Hypothalamic-pituitary regulation helps to maintain water balance in the body. A dry mouth is not considered to be a tool to measure fluid deficit in the body. Stress and other social and psychologic factors affect thirst. Medications such as morphine and nicotine stimulate secretion of antidiuretic hormones that prevent urinary elimination of water. Osmoreceptors in the hypothalamus maintain body fluid osmolality, which also stimulates thirst. Text Reference - p. 289

The nurse finds that the patient with renal disease is irritable and has an irregular pulse. ECG changes suggest severe hyperkalemia. What should be the first nursing intervention? 1 Stop all sources of dietary potassium 2 Administer intravenous calcium gluconate 3 Administer ion-exchange resins 4 Administer intravenous insulin with glucose

2 In the case of severe hyperkalemia, manifested by irritation, irregular pulse, and changes in ECG findings, the nurse should act immediately to prevent cardiac arrest. The nurse should administer intravenous calcium gluconate to reverse the membrane potential effects of extracellular fluid (ECF) potassium. Administering ion-exchange resins (to increase elimination of potassium) and intravenous insulin with glucose (to force potassium from ECF to intracellular fluid [ICF]) can be done once the patient is stable. Stopping all sources of dietary potassium is an important measure when hyperkalemia is mild. Text Reference - p. 297

A patient has been admitted for dehydration. What is a priority nursing intervention? 1 Reorient the patient hourly 2 Perform daily weights 3 Provide continuous oxygen saturation monitoring 4 Restrict sodium intake to 2 grams per day

2 Measuring weight is the most reliable means of detecting changes in fluid balance. Weight loss would indicate that the dehydration is worsening, whereas weight gain would indicate restoration of fluid volume. The nurse would recall that a 1-kg weight gain indicates a gain of approximately 1000 mL of body water. This patient is not disoriented, and that is not a common assessment finding in the patient with dehydration. Continuous oxygen saturation monitoring is not indicated. Sodium intake does not need to be restricted. Text Reference - p. 292

The nurse is teaching a caregiver for an older adult patient with dementia about fluid balance maintenance at home. Which statement made by the caregiver indicates that he or she requires further education? 1 "I should increase fluids and lower dietary sodium." 2 "I should provide fluids when the patient feels thirsty." 3 "I should assist the patient when holding utensils and cups." 4 "I should request the patient to pass urine before going to bed."

2 Mental status alterations are a common problem in old age and may lead to decreased ability to express thirst and obtain fluids. Therefore, older patients are always encouraged to drink fluids and also to decrease dietary sodium in the diet. Musculoskeletal changes such as stiffness of the hands and fingers may lead to an inability to hold containers. The patient should make a habit of urinating before bed to decrease the chance of nocturia. Text Reference - p. 291

After performing gastric suctioning on a patient who has ingested pesticides, the nurse suspects that the patient has developed metabolic alkalosis. Which finding supports the nurse's suspicion? 1 pH, 7, partial pressure of carbon dioxide (PaCO2), 34 mm Hg 2 pH, 10, partial pressure of carbon dioxide (PaCO2), 52 mm Hg 3 pH, 8.3, partial pressure of carbon dioxide (PaCO2), 44 mm Hg 4 pH, 7.2, partial pressure of carbon dioxide (PaCO2), 38 mm Hg

2 Metabolic alkalosis is manifested by increased plasma pH and partial pressure of carbon dioxide (PaCO2). Normal pH of the blood ranges from 7.35 to 7.45 and the normal range of PaCO2 in blood ranges from 35 to 45 mm Hg. The laboratory findings of pH 10 and PaCO2 of 52 mm Hg support the nurse's suspicion. A pH of 7 and PaCO2 of 34 mm Hg do not indicate metabolic alkalosis. A pH of blood of 8.3 is higher than normal and indicates alkalinity, and a PaCO2 of 44 mm Hg is normal. A pH value of 7.2 and a PaCO2 value of 38 mm Hg are normal. Test-Taking Tip: Gastric suctioning may lead to acid-base imbalance in the body. Using this tip may help you in selecting the right answer. Text Reference - p. 304

After reviewing the patient's arterial blood gas analysis report, the primary health care provider concludes that the patient has respiratory acidosis. Which findings made the primary health care provider reach this conclusion? 1 pH, 7.4, partial pressure of carbon dioxide (PaCO2), 44, bicarbonate ion (HCO3-), 26 2 pH, 7.2, partial pressure of carbon dioxide (PaCO2), 47, bicarbonate ion (HCO3-), 25 3 pH, 7.36, partial pressure of carbon dioxide (PaCO2), 41, bicarbonate ion (HCO3-), 23 4 pH, 7.42, partial pressure of carbon dioxide (PaCO2), 42, bicarbonate ion (HCO3-), 24

2 Patients with respiratory acidosis have increased concentration of carbon dioxide (PaCO2) in the blood and decreased blood pH, whereas the bicarbonate ion concentration (HCO3-) is normal. The normal value ranges of pH, PaCO2, and HCO3- are 7.37 to 7.45, 35 to 45 mm Hg, and 22 to 26 mEq/L, respectively. Thus, a pH value of 7.2, PaCO2 of 47, and HCO3- concentration of 25 indicate respiratory acidosis in the patient Text Reference - p. 304

Which condition may precipitate respiratory acidosis? 1 Severe vomiting 2 Severe pneumonia 3 Pulmonary embolism 4 Diabetic ketoacidosis

2 Pneumonia is an inflammatory condition that causes hypoventilation, which results in increased concentration of carbon dioxide in blood and precipitates respiratory acidosis. Severe vomiting may cause loss of strong acids from the body, resulting in metabolic alkalosis. A pulmonary embolism causes hyperventilation, resulting in respiratory alkalosis. Diabetic ketoacidosis causes accumulation of ketone bodies in the body, resulting in metabolic acidosis. Text Reference - p. 304

A nurse reviews a patient's blood gas results: pH 7.15, PaO2 40 mm Hg, PaCO2 70 mm Hg, and HCO3 25 mEq/L. The nurse suspects hypoxia and what other condition? 1 Metabolic acidosis 2 Respiratory acidosis 3 Respiratory alkalosis 4 Compensating respiratory acidosis

2 This patient is not breathing effectively and therefore has a buildup of carbon dioxide in the form of carbonic acid. This places the patient in an acidotic state, because the pH is less than 7.35. Metabolic and respiratory alkalosis are therefore eliminated as possibilities. Because the PaCO2 is high at 70 mm Hg (normal range is 35 to 45 mm Hg) and the metabolic measure of HCO3- is normal at 25 mEq/L (normal range is 22 to 28 mEq/L), the patient is in respiratory acidosis. The patient is not compensated, because the HCO3- is still within normal range. If the HCO3- were increased, this would be an indication of compensation. Text Reference - p. 303

When administering intravenous (IV) potassium chloride (KCl) to a patient to correct hypokalemia, which interventions are important? Select all that apply. 1 IV KCl can be added to a hanging IV bag. 2 KCl should never be given via IV push or bolus. 3 KCl must always be diluted before administering. 4 IV site should be regularly assessed for infiltration. 5 IV KCl should be administered at a rate of 50 mEq/h.

2, 3, 4 IV KCl is given to correct hypokalemia. IV KCl should never be given via IV push or as a bolus. IV KCl must never be given in a concentrated form; it should always be diluted before administration. The nurse should check the IV site regularly, because KCl can irritate the veins, causing phlebitis and infiltration. The solution should not be added to a hanging IV bag; following this rule lowers the risk of a bolus dose being given. The rate of IV administration of KCl should not exceed 10 to 20 mEq/h. The solution should be administered by infusion pump so that correct doses are administered at the correct rate. STUDY TIP: Identify your problem areas that need attention. Do not waste time on restudying information you know. Text Reference - p. 298

A patient has been prescribed IV potassium chloride (KCl) for the treatment of hypokalemia. What precautions should the nurse take to prevent severe complications related to KCl administration? Select all that apply. 1 Administer KCl as an IV push or as a bolus 2 Turn IV bags containing KCl upside down and upright several times prior to administration 3 Never add KCl to an existing IV bag 4 Dilute KCl before administering 5 Ascertain an adequate urine output prior to administration

2, 3, 4, 5 Severe hypokalemia can be treated by administering KCl. IV bags containing KCl should be inverted several times to ensure dilution of the drug and prevent accidental bolus delivery. Adding KCl to a hanging IV bag may cause accidental bolus delivery and lead to cardiac complications. It can cause severe cardiac complications and so should not be given undiluted. KCl is given to patients who have a urine output of at least 0.5 mL/kg of body weight per hour. This ensures timely clearance of unused potassium. KCl should never be administered as IV bolus or push because it can affect the cardiac function. Text Reference - p. 298

The nurse finds polyuria, soft flabby muscles, and an irregular pulse in a patient who has a blood glucose level of 165 mg/dL and who is on is on intravenous dextrose therapy. Which changes are observed in the patient's electrocardiogram? Select all that apply. 1 Loss of P wave 2 Prolonged QRS 3 Peaked T waves 4 Presence of U wave 5 ST segment depression

2, 4, 5 When the potassium levels in the blood are low, it is known as hypokalemia. Symptoms include fatigue, nausea, excessive urination, and muscle weakness. Hypokalemia alters the resting membrane potential, resulting in hyperpolarization and changes in electrolytes. Prolonged QRS complex, presence of a U wave, and ST segment depression are caused by hyperpolarization, which is found in hypokalemia. Loss of the P wave and peaked T waves are both seen in hyperkalemia. Test-Taking Tip: Recall the symptoms of hypokalemia and use your knowledge to eliminate the wrong options. Text Reference - p. 296

A person weighing 100 kg has 40 L of intracellular fluid. What amount of intracellular fluid does a 60 kg individual have? Record your answer using one decimal place. ____________ L

24.0 Intracellular fluid constitutes approximately 40 percent of the body weight of an adult. The person weighing 60 kg would have approximately 24 liters of intracellular fluid, because 60 kg x 40/100 x 40/100 = 24.0 L. Text Reference - p. 286

Shortly after having a central IV catheter inserted into the subclavian vein, the patient experiences shortness of breath, anxiety, and restlessness. What is the highest priority for the nurse? 1 Administering a sedative 2 Advising the patient to relax 3 Auscultating the breath sounds 4 Obtaining an arterial blood gas analysis

3 Because this is an acute episode, the nurse should first listen to the patient's lungs to see whether anything has changed. In this situation the probability is high that the patient sustained a pneumothorax during the subclavian IV catheter insertion procedure. The patient will need oxygen, and the care provider should be notified of the findings. Administering a sedative is not appropriate. Advising the patient to relax does provide reassurance, but the anxiety and restlessness are probably due to hypoxia. Obtaining an arterial blood gas analysis would likely be the next nursing action. Text Reference - p. 311

What is the function of a buffer? 1 To excrete weak acids 2 To secrete hydrogen ions 3 To convert strong acids to weak acids 4 To convert ammonia to ammonium ions

3 Buffers convert strong acids to weak acids. Excretion of weak acids, secretion of hydrogen ions into the renal tubule, and conversion of ammonia to ammonium ions takes place in the kidneys. Text Reference - p. 303

The nurse is caring for a patient with fluid imbalance related to a deficit. Which action by the nurse provides comfort to the patient? 1 Providing tube feeding to the patient 2 Supplementing water to the patient with nasogastric suction 3 Helping the patient to apply moisturizer and use moisturizing soaps 4 Weighing the patient regularly while the drainage bags are attached

3 Fluid imbalance leads to abnormal homeostasis. It also leads to breakdown of skin, so a patient with fluid imbalance should practice proper skin care by applying lotion and moisturizing soaps. Tube feeding may be recommended for the patient with severe fluid imbalance. The patient on nasogastric suction should either consume ice cubes, or the nasogastric tube should be irrigated with isotonic saline solution. Consumption of water may increase the loss of electrolytes. The nurse has to remove or detach the drainage bags before weighing a patient. Text Reference - p. 293

The student nurse is educating a patient regarding skin care management. What statement made by the student requires correction? 1 "Avoid extreme temperatures." 2 "Apply moisturizers even at night." 3 "Wash hands and legs frequently with soap." 4 "Change your position regularly while at rest."

3 Good skin care management is essential to prevent fluid loss. The patient should limit the use of soap to prevent the skin from drying. The nurse should advise the patient to either take precautions or to avoid extreme temperatures to avoid dehydration of the skin. Regular skin care by applying moisturizers and changing positions while at rest help to maintain skin hydration and may prevent skin breakdown. Text Reference - p. 293

A patient is diagnosed with Guillain-Barren syndrome. Which complication does the nurse anticipate? 1 Metabolic acidosis 2 Metabolic alkalosis 3 Respiratory acidosis 4 Respiratory alkalosis

3 Guillain-Barren syndrome is a disease of the respiratory system that causes hypoventilation. Hypoventilation increases the concentration of carbonic acid, which results in respiratory acidosis. Metabolic acidosis, metabolic alkalosis, and respiratory alkalosis are not caused by Guillain-Barren syndrome. Text Reference - p. 304

The nurse is caring for a patient diagnosed with heat stroke and with a urine output of 4000 mL per day. What is the most appropriate nursing action? 1 Transfusing blood 2 Applying moisturizer regularly 3 Administrating lactated Ringer's solution 4 Administrating supplementary water in enteric formula

3 Heat stroke and an increased amount of urine output of about 4000 mL leads to a deficit in extracellular fluid volume, causing dehydration. Administering lactated Ringer's solution to maintain fluid and electrolyte balance is beneficial. Blood transfusions are performed only when the fluid loss is due to blood loss. Moisturizers are applied to patients with dry skin to prevent the fluid loss. Tube feeding is preferred in the patient with severe extracellular fluid loss. The patient on tube feeding must be thereby supplemented with water added to the enteric formula. Text Reference - p. 291

Which protein regulates the pH of blood? 1 Albumin 2 Globulin 3 Hemoglobin 4 Alpha acid glycoprotein

3 Hemoglobin shifts chloride ions in and out of red blood cells in exchange for bicarbonate ions, which helps to regulate blood pH. Albumin, globulin, and alpha acid lipoprotein are plasma proteins that help transport of lipids, minerals, and vitamins. Text Reference - p. 303

Which is the clinical manifestation of metabolic acidosis? 1 Seizures 2 Hypoventilation 3 Rapid respirations 4 Ventricular fibrillation

3 Metabolic acidosis is manifested by deep and rapid respirations called Kussmaul's respirations. Seizures, hypoventilation with hypoxia, and ventricular fibrillation are signs of respiratory acidosis. Text Reference - p. 305

A patient has the following arterial blood gas results: pH 7.32; PaCO2 56 mm Hg; HCO3- 24 mEq/L. The nurse determines that these results indicate: 1 Metabolic acidosis 2 Metabolic alkalosis 3 Respiratory acidosis 4 Respiratory alkalosis

3 Respiratory acidosis (carbonic acid excess) occurs whenever a person experiences hypoventilation. Hypoventilation leads to a buildup of CO2, resulting in an accumulation of carbonic acid in the blood. Carbonic acid dissociates, liberating H+, and there is a decrease in pH. The patient is not experiencing metabolic acidosis. These results are not indicative of metabolic alkalosis or respiratory alkalosis (because the pH is high). Text Reference - p. 303

Which electrolyte imbalance is associated with sickle cell anemia? 1 Increased calcium levels 2 Increased potassium levels 3 Increased phosphate levels 4 Increased magnesium levels

3 Sickle cell anemia leads to increased concentration of phosphates in the body, thus causing hyperphosphatemia. Hypercalcemia, or increased calcium levels, is associated with hyperparathyroidism. Hyperkalemia, or increased potassium levels, is associated with tumor-lysis syndrome. Hypermagnesemia, or increased magnesium levels, is associated with diabetic ketoacidosis. Text Reference - p. 301

A patient has the following arterial blood gas results: pH 7.16, PaCO2 80 mm Hg, PaO2 46 mm Hg, HCO3- 24 mEq/L, and SaO2 81%. The nurse recognizes that the results represent: 1 Metabolic acidosis 2 Metabolic alkalosis 3 Respiratory acidosis 4 Respiratory alkalosis

3 The pH is less than 7.35, indicating acidosis. This eliminates metabolic and respiratory alkalosis as possibilities. Because the PaCO2 is high at 80 mm Hg (normal range is 35 to 45 mm Hg) and the metabolic measure of HCO3- is normal (range is 22 to 28 mEq/L), the patient is in respiratory acidosis, not alkalosis. Text Reference - p. 303

A nurse is assessing a patient with hyperkalemia. Which changes in the electrocardiogram might the nurse find? Select all that apply. 1 Flattened T wave 2 Prolonged QRS 3 Loss of P wave 4 Tall, peaked T wave 5 ST segment depression

3, 4, 5 Hyperkalemia causes a decrease in cardiac depolarization, leading to a flattening of the P wave. High potassium levels lead to rapid repolarization, evident as tall T waves and ST segment depression. T waves are flattened and QRS is prolonged in hypokalemia. Text Reference - p. 297

A patient weighing 60 kg with diabetes mellitus is excreting 3500 mL of urine per day. What is the amount of weight lost by the patient in 24 hours? Record your answer using one decimal place. ____________ kg 00:00:27 Question Answer Confidence Buttons

3.5 A sudden change in body weight is an indicator of fluid imbalance. One liter of water weighs one kg. Therefore, a patient of 60 kg with diabetes mellitus who is excreting 3500 mL of urine per day would lose 3.5 kg. This sudden change of weight is due to polyuria, which further leads to fluid imbalance. Text Reference - p. 286

The nurse is caring for an older patient who is receiving intravenous (IV) fluids postoperatively. During the 8 AM assessment of this patient, the nurse notes that the IV solution, which was prescribed to infuse at 125 mL/hr, has infused 950 mL since it was hung at 4 AM. What is the priority nursing intervention? 1 Notify the health care provider and complete an incident report. 2 Slow the rate to keep the vein open until the next bag is due at noon. 3 Obtain a new bag of IV solution to maintain patency of the site. 4 Listen to the patient's lung sounds and assess respiratory status.

4 After four hours of infusion time, 500 mL of IV solution should have infused, not 950 mL. This patient is at risk for fluid volume excess, and the nurse should assess the patient's respiratory status and lung sounds as the priority action and then notify the health care provider for further prescriptions. Test-Taking Tip: Key words or phrases in the stem of the question such as first, primary, early, or best are important. Similarly, words such as only, always, never, and all in the alternatives are frequently evidence of a wrong response. As in life, no real absolutes exist in nursing; however, every rule has its exceptions, so answer with care. Text Reference - p. 292

Which treatment strategy would be beneficial in a patient with a blood sodium level of 170 mEq/L who experiences intense thirst, agitation, and decreased alertness? 1 Intravenous furosemide 2 Intravenous cation-exchange resin 3 Intravenous phosphate-binding agent 4 Intravenous 0.45% sodium chloride saline solution

4 Hypernatremia is a condition in which water shifts out of the cells into the extracellular fluid, resulting in dehydration. Therefore, the patient with hypernatremia would experience intense thirst, agitation, and decreased alertness. To reduce dehydration, fluid should be replaced by administering hypotonic intravenous fluids such as 5% dextrose in water or 0.45% sodium chloride saline solution. Administering intravenous furosemide may help treat hypercalcemia. A cation-exchange resin may be administered to treat hyperkalemia. A phosphate-binding agent may be administered to treat hyperphosphatemia. Text Reference - p. 295

The nurse is preparing to administer intravenous (IV) potassium chloride (KCl) to a patient. Which action should the nurse perform to ensure the patient's safety? 1 Give KCl via IV push. 2 Add KCl to the hanging IV bag. 3 Give IV KCl in concentrated amounts. 4 Invert IV bags containing KCl several times

4 Hypokalemia is characterized by a decreased concentration of potassium in the body. Therefore, KCl should be administered to maintain normal potassium levels. Inverting the IV bags containing KCl several times ensures even distribution of KCl medication in the bag. The nurse should administer KCl through an infusion pump, not by IV push, to ensure that it is administered at an accurate rate. The nurse should not add KCl to the hanging IV bag because this would result in administering a bolus dose. The nurse will give IV KCl in diluted forms, rather than in concentrated amounts, to ensure the patient's safety. Text Reference - p. 298

When planning care for a patient with dehydration related to nausea and vomiting, the nurse would anticipate which fluid shift to occur because of the fluid volume deficit? 1 Fluid movement from the blood vessels into the cells 2 Fluid movement from the interstitial spaces into the cells 3 Fluid movement from the blood vessels into interstitial spaces 4 Fluid movement from the interstitial space into the blood vessels

4 In dehydration, fluid is lost first from the blood vessels. To compensate, fluid moves out of the interstitial spaces into the blood vessels to restore circulating volume in that compartment. As the interstitial spaces then become volume depleted, fluid moves out of the cells into the interstitial spaces. Test-Taking Tip: Avoid taking a wild guess at an answer. However, should you feel insecure about a question, eliminate the alternatives that you believe are definitely incorrect, and reread the information given to make sure you understand the intent of the question. This approach increases your chances of randomly selecting the correct answer or getting a clearer understanding of what is being asked. Although there is no penalty for guessing on the NCLEX examination, the subsequent question will be based, to an extent, on the response you give to the question at hand; that is, if you answer a question incorrectly, the computer will adapt the next question accordingly based on your knowledge and skill performance on the examination up to that point. Text Reference - p. 292

Which is a manifestation of respiratory acidosis? 1 Diarrhea 2 Nausea 3 Abdominal pain 4 Ventricular fibrillation

4 Respiratory acidosis causes compensatory hyperkalemia, which leads to ventricular fibrillation. Diarrhea, nausea, and abdominal pain are manifestations of metabolic acidosis. Test-Taking Tip: Have confidence in your initial response to an item because it more than likely is the correct answer. Text Reference - p. 305

Which condition is manifested by hyperreflexia? 1 Metabolic acidosis 2 Metabolic alkalosis 3 Respiratory acidosis 4 Respiratory alkalosis

4 Respiratory alkalosis is manifested by hyperreflexia. Metabolic acidosis is manifested by abdominal pain and Kussmaul respirations. Tachycardia, anorexia, and muscle cramps are the manifestations of metabolic alkalosis. Headache, seizures, and hypotension are the manifestations of respiratory acidosis. Test-Taking Tip: If you can eliminate any responses as incorrect based on your knowledge, you will not be guessing randomly but will be exercising "informed guessing." Text Reference - p. 305

What concentration of bicarbonate ion in blood is an indicator of a compensatory response in patients with respiratory acidosis? 1 24 mEq/L 2 25 mEq/L 3 26 mEq/L 4 27 mEq/L

4 The bicarbonate ion concentration in blood increases as a compensatory response in patients with respiratory acidosis. The normal range of bicarbonate ion is 22 to 26 mEq/L. Therefore, 27 mEq/L indicates a compensatory response. Test-Taking Tip: You have at least a 25 percent chance of selecting the correct response in multiple-choice items. If you are uncertain about a question, eliminate the choices that you believe are wrong and then call on your knowledge, skills, and abilities to choose from the remaining responses. Text Reference - p. 304

The nurse is caring for a 92-year-old patient who has dehydration. The nurse would instruct the unlicensed assistive personnel (UAP) to report which finding? 1 Ambulation in the hallway without assistance 2 Temperature 97.1o F 3 Frequent use of the urinal 4 Urine output of 350 mL in 24 hours

4 The minimal urine output necessary to maintain kidney function is 30 mLs per hour, or 720 mL per 24 hours. The nurse should be notified of a decrease in urine output so that additional fluid volume-replacement therapy can be instituted. Ambulation is encouraged. The temperature is normal. Frequent use of the urinal would not indicate dehydration. Text Reference - p. 292

The nurse needs to add potassium chloride (KCl) 40 mEq to an intravenous (IV) solution of 0.9% sodium chloride. The multidose vial contains 10 mEq KCl in 5 mL of solution. How many milliliters of KCl should the nurse add to the IV bag? 1. 2 mL 2. 5 mL 3. 10 mL 4. 20 mL

4 Using ratio and proportion, multiply 10 by x and multiply 40 × 5 to yield 10x = 200. Divide 200 by 10 to yield 20 mL. Text Reference - p. 298

The nurse is recording a patient weight of 68 kg in the ward. How much interstitial fluid is estimated to be present there? Record your answer using one decimal place. ____________ L

8.6 About 70 percent of the extracellular fluid is found in the interstitial space and the extracellular fluid comprises about 33 percent of the total body water amount. The total body water amount represents about 55 percent of the body weight. The patient weighs 68 kg and has approximately 37.4 L of total body water and extracellular fluid is 33 percent of that amount, which equals12.3 L. Seventy percent of the extracellular fluid is 12.3 x 70/100 = 8.6 L.(((68 kg x 55/100) x 33/100) x 70/100) = 8.6 L Text Reference - p. 286

A patient with a history of peptic ulcer disease has presented to the emergency department reporting severe abdominal pain and has a rigid, boardlike abdomen that prompts the health care team to suspect a perforated ulcer. What intervention should the nurse anticipate? a) Providing IV fluids and inserting a nasogastric (NG) tube b) Administering oral bicarbonate and testing the patient's gastric pH level c) Performing a fecal occult blood test and administering IV calcium gluconate d) Starting parenteral nutrition and placing the patient in a high-Fowler's position

A A perforated peptic ulcer requires IV replacement of fluid losses and continued gastric aspiration by NG tube. Nothing is given by mouth, and gastric pH testing is not a priority. Calcium gluconate is not a medication directly relevant to the patient's suspected diagnosis, and parenteral nutrition is not a priority in the short term.

Which finding for a patient who has hypothyroidism and hypertension indicates that the nurse should contact the health care provider before administering levothyroxine (Synthroid)? a. Increased thyroxine (T4 ) level b. Blood pressure 112/62 mm Hg c. Distant and difficult to hear heart sounds d. Elevated thyroid stimulating hormone level

A An increased thyroxine level indicates the levothyroxine dose needs to be decreased. The other data are consistent with hypothyroidism and the nurse should administer the levothyroxine.

Which finding for a patient who has hypothyroidism and hypertension indicates that the nurse should contact the health care provider before administering levothyroxine (Synthroid)? a. Increased thyroxine (T4) level b. Blood pressure 112/62 mm Hg c. Distant and difficult to hear heart sounds d. Elevated thyroid stimulating hormone level

A An increased thyroxine level indicates the levothyroxine dose needs to be decreased. The other data are consistent with hypothyroidism and the nurse should administer the levothyroxine.

Which finding by the nurse when assessing a patient with Hashimoto's thyroiditis and a goiter will require the most immediate action? a. New-onset changes in the patient's voice b. Elevation in the patient's T3 and T4 levels c. Resting apical pulse rate 112 beats/minute d. Bruit audible bilaterally over the thyroid gland

A Changes in the patient's voice indicate that the goiter is compressing the laryngeal nerve and may lead to airway compression. The other findings will also be reported but are expected with Hashimoto's thyroiditis and do not require immediate action.

Which finding by the nurse when assessing a patient with Hashimoto's thyroiditis and a goiter will require the most immediate action? a. New-onset changes in the patient's voice b. Elevation in the patient's T3 and T4 levels c. Resting apical pulse rate 112 beats/minute d. Bruit audible bilaterally over the thyroid gland

A Changes in the patient's voice indicate that the goiter is compressing the laryngeal nerve and may lead to airway compression. The other findings will also be reported but are expected with Hashimoto's thyroiditis and do not require immediate action.

The nurse providing care for a patient who has an adrenocortical adenoma causing hyperaldosteronism should a. monitor the blood pressure every 4 hours. b. elevate the patient's legs to relieve edema. c. monitor blood glucose level every 4 hours. d. order the patient a potassium-restricted diet.

A Hypertension caused by sodium retention is a common complication of hyperaldosteronism. Hyperaldosteronism does not cause an elevation in blood glucose. The patient will be hypokalemic and require potassium supplementation before surgery. Edema does not usually occur with hyperaldosteronism.

In which order will the nurse take these steps to prepare NPH 20 units and regular insulin 2 units using the same syringe? (Put a comma and a space between each answer choice [A, B, C, D, E]). a. Rotate NPH vial. b. Withdraw regular insulin. c. Withdraw 20 units of NPH. d. Inject 20 units of air into NPH vial. e. Inject 2 units of air into regular insulin vial.

A D E B C Rotate NPH vial Inject 20 units of air into NPH vial. Inject 2 units of air into regular insulin vial. Withdraw regular insulin. Withdraw 20 units of NPH.

What surgery might be indicated if the chemotherapy is not effective (in eradicating bladder cancer)?

A cystectomy with urinary diversion would be indicated. This could be an incontinent urinary diversion or a continent urinary diversion.

Which statement correctly illustrates the concept of the negative feedback system for control of serum cortisol?

A decreased secretion of adrenocorticotropic hormone (ACTH) when serum cortisol is increased.

A patient with ulcerative colitis undergoes the first phase of total proctocolectomy with ileal pouch and anal anastomosis. On post op assessment of the patient, what should the nurse expect to find?

A loop ileostomy with a plastic rod to hold it in place

A

A patient who has been receiving diuretic therapy is admitted to the emergency department with a serum potassium level of 3.0 mEq/L. The nurse should alert the health care provider immediately that the patient is on which medication? a. Oral digoxin (Lanoxin) 0.25 mg daily b. Ibuprofen (Motrin) 400 mg every 6 hours c. Metoprolol (Lopressor) 12.5 mg orally daily d. Lantus insulin 24 U subcutaneously every evening

C

A patient who has undergone an esophagectomy for esophageal cancer develops increasing pain, fever, and dyspnea when a full liquid diet is started postoperatively. The nurse recognizes that these symptoms are most indicative of: A. an intolerance to the feedings B. extension of the tumor into the aorta C. leakage of food or fluids into the mediastinum D. esophageal perforation with fistula formation into the lungs

When a patient returns to the clinical unit after an abdominal- perineal resection (APR), what should the nurse expect?

A perineal wound, drains, and a stoma.

A 22-year-old patient is being seen in the clinic with increased secretion of the anterior pituitary hormones. The nurse would expect the laboratory results to show A. Increased urinary cortisol. B. Decreased serum thyroxine. C. Elevated serum aldosterone levels. D. Low urinary catecholamines excretion.

A. Increased urinary cortisol. - Increased secretion of adrenocorticotropic hormone (ACTH) by the anterior pituitary gland will lead to an increase in serum and urinary cortisol levels. An increase, rather than a decrease, in thyroxine level would be expected with increased secretion of thyroid stimulating hormone (TSH) by the anterior pituitary. Aldosterone and catecholamine levels are not controlled by the anterior pituitary.

26. The nurse admitting a patient with acute diverticulitis explains that the initial plan of care is to a. administer IV fluids. b. give stool softeners and enemas. c. order a diet high in fiber and fluids. d. prepare the patient for colonoscopy.

ANS: A A patient with acute diverticulitis will be NPO and given parenteral fluids. A diet high in fiber and fluids will be implemented before discharge. Bulk-forming laxatives, rather than stool softeners, are usually given, and these will be implemented later in the hospitalization. The patient with acute diverticulitis will not have enemas or a colonoscopy because of the risk for perforation and peritonitis.

The nurse admitting a patient with acute diverticulitis explains that the initial plan of care is to a. administer IV fluids. b. give stool softeners and enemas. c. order a diet high in fiber and fluids. d. prepare the patient for colonoscopy.

ANS: A A patient with acute diverticulitis will be NPO and given parenteral fluids. A diet high in fiber and fluids will be implemented before discharge. Bulk-forming laxatives, rather than stool softeners, are usually given, and these will be implemented later in the hospitalization. The patient with acute diverticulitis will not have enemas or a colonoscopy because of the risk for perforation and peritonitis.

30. A 50-year-old female patient calls the clinic to report a new onset of severe diarrhea. The nurse anticipates that the patient will need to a. collect a stool specimen. b. prepare for colonoscopy. c. schedule a barium enema. d. have blood cultures drawn.

ANS: A Acute diarrhea is usually caused by an infectious process, and stool specimens are obtained for culture and examined for parasites or white blood cells. There is no indication that the patient needs a colonoscopy, blood cultures, or a barium enema.

5. A patient complains of gas pains and abdominal distention two days after a small bowel resection. Which nursing action is best to take? a. Encourage the patient to ambulate. b. Instill a mineral oil retention enema. c. Administer the ordered IV morphine sulfate. d. Offer the ordered promethazine (Phenergan) suppository.

ANS: A Ambulation will improve peristalsis and help the patient eliminate flatus and reduce gas pain. A mineral oil retention enema is helpful for constipation with hard stool. A return-flow enema might be used to relieve persistent gas pains. Morphine will further reduce peristalsis. Promethazine (Phenergan) is used as an antiemetic rather than to decrease gas pains or distention.

A patient complains of gas pains and abdominal distention two days after a small bowel resection. Which nursing action is best to take? a. Encourage the patient to ambulate. b. Instill a mineral oil retention enema. c. Administer the ordered IV morphine sulfate. d. Offer the ordered promethazine (Phenergan) suppository.

ANS: A Ambulation will improve peristalsis and help the patient eliminate flatus and reduce gas pain. A mineral oil retention enema is helpful for constipation with hard stool. A return-flow enema might be used to relieve persistent gas pains. Morphine will further reduce peristalsis. Promethazine (Phenergan) is used as an antiemetic rather than to decrease gas pains or distention.

Which finding for a patient who has hypothyroidism and hypertension indicates that the nurse should contact the health care provider before administering levothyroxine (Synthroid)? a. Increased thyroxine (T4) level b. Blood pressure 112/62 mm Hg c. Distant and difficult to hear heart sounds d. Elevated thyroid stimulating hormone level

ANS: A An increased thyroxine level indicates the levothyroxine dose needs to be decreased. The other data are consistent with hypothyroidism and the nurse should administer the levothyroxine

Following rectal surgery, a patient voids about 50 mL of urine every 30 to 60 minutes. Which nursing action is most appropriate? a. Use an ultrasound scanner to check the postvoiding residual. b. Monitor the patient's intake and output over the next few hours. c. Have the patient take small amounts of fluid frequently throughout the day. d. Reassure the patient that this is normal after rectal surgery because of anesthesia.

ANS: A An ultrasound scanner can be used to check for residual urine after the patient voids. Because the patient's history and clinical manifestations are consistent with overflow incontinence, it is not appropriate to have the patient drink small amounts. Although overflow incontinence is not unusual after surgery, the nurse should intervene to correct the physiologic problem, not just reassure the patient. The patient may develop reflux into the renal pelvis as well as discomfort from a full bladder if the nurse waits to address the problem for several hours. DIF: Cognitive Level: Application REF: 1146-1147 | 1154

A 32-year-old patient is diagnosed with polycystic kidney disease. Which information is most appropriate for the nurse to include in teaching at this time? a. Importance of genetic counseling b. Complications of renal transplantation c. Methods for treating chronic and severe pain d. Differences between hemodialysis and peritoneal dialysis

ANS: A Because a 32-year-old patient may be considering having children, the nurse should include information about genetic counseling when teaching the patient. The well-managed patient will not need to choose between hemodialysis and peritoneal dialysis or know about the effects of transplantation for many years. There is no indication that the patient has chronic pain. DIF: Cognitive Level: Application REF: 1143

36. The health care provider prescribes the following therapies for a patient who has been admitted with dehydration and hypotension after 3 days of nausea and vomiting. Which order will the nurse implement first? a. Infuse normal saline at 250 mL/hr. b. Administer IV ondansetron (Zofran). c. Provide oral care with moistened swabs. d. Insert a 16-gauge nasogastric (NG) tube.

ANS: A Because the patient has severe dehydration, rehydration with IV fluids is the priority. The other orders should be accomplished as quickly as possible after the IV fluids are initiated.

19. A female patient being admitted with pneumonia has a history of neurogenic bladder as a result of a spinal cord injury. Which action will the nurse plan to take first? a. Ask about the usual urinary pattern and any measures used for bladder control. b. Assist the patient to the toilet at scheduled times to help ensure bladder emptying. c. Check the patient for urinary incontinence every 2 hours to maintain skin integrity. d. Use intermittent catheterization on a regular schedule to avoid the risk of infection.

ANS: A Before planning any interventions, the nurse should complete the assessment and determine the patient's normal bladder pattern and the usual measures used by the patient at home. All the other responses may be appropriate, but until the assessment is complete, an individualized plan for the patient cannot be developed.

Which finding by the nurse when assessing a patient with Hashimoto's thyroiditis and a goiter will require the most immediate action? a. New-onset changes in the patient's voice b. Apical pulse rate at rest 112 beats/minute c. Elevation in the patient's T3 and T4 levels d. Bruit audible bilaterally over the thyroid gland

ANS: A Changes in the patient's voice indicate that the goiter is compressing the laryngeal nerve and may lead to airway compression. The other findings will also be reported but are expected with Hashimoto's thyroiditis and do not require immediate action

Which finding indicates to the nurse that the current therapies are effective for a patient with acute adrenal insufficiency? a. Increasing serum sodium levels b. Decreasing blood glucose levels c. Decreasing serum chloride levels d. Increasing serum potassium levels

ANS: A Clinical manifestations of Addison's disease include hyponatremia and an increase in sodium level indicates improvement. The other values indicate that treatment has not been effective

16. Cobalamin injections have been prescribed for a patient with chronic atrophic gastritis. The nurse determines that teaching regarding the injections has been effective when the patient states, a. "The cobalamin injections will prevent me from becoming anemic." b. "These injections will increase the hydrochloric acid in my stomach." c. "These injections will decrease my risk for developing stomach cancer." d. "The cobalamin injections need to be taken until my inflamed stomach heals."

ANS: A Cobalamin supplementation prevents the development of pernicious anemia. The incidence of stomach cancer is higher in patients with chronic gastritis, but cobalamin does not reduce the risk for stomach cancer. Chronic gastritis may cause achlorhydria, but cobalamin does not correct this. The loss of intrinsic factor secretion with chronic gastritis is permanent, and the patient will need lifelong supplementation with cobalamin.

32. The nurse is assessing a 31-year-old female patient with abdominal pain. Th nurse,who notes that there is ecchymosis around the area of umbilicus, will document this finding as a. Cullen sign. b. Rovsing sign. c. McBurney sign. d. Grey-Turner's signt.

ANS: A Cullen sign is ecchymosis around the umbilicus. Rovsing sign occurs when palpation of the left lower quadrant causes pain in the right lower quadrant. Deep tenderness at McBurney's point (halfway between the umbilicus and the right iliac crest), known as McBurney's sign, is a sign of acute appendicitis.

9. A nurse assigned to care for a patient with diabetic ketoacidosis (DKA) is aware that this is a life-threatening condition. What will DKA result in? a. Disorder of carbohydrates, fats, and proteins metabolism b. Storage of glycogen, resulting in a severe shortage of glucose in the bloodstream c. Dangerously elevated pH and bicarbonate levels in the blood d. Severe hypoglycemia, which can result in coma and convulsions

ANS: A DKA is mainly related to the use of fat as an energy source because of an inability of the body to use glucose. The metabolism of fat produces ketones.

13. A patient has just arrived on the postoperative unit after having a laparoscopic esophagectomy for treatment of esophageal cancer. Which nursing action should be included in the postoperative plan of care? a. Elevate the head of the bed to at least 30 degrees. b. Reposition the nasogastric (NG) tube if drainage stops or decreases. c. Notify the doctor immediately about bloody NG drainage. d. Start oral fluids when the patient has active bowel sounds.

ANS: A Elevation of the head of the bed decreases the risk for reflux and aspiration of gastric secretions. The NG tube should not be repositioned without consulting with the health care provider. Bloody NG drainage is expected for the first 8 to 12 hours. A swallowing study is needed before oral fluids are started.

19. A patient is hospitalized with vomiting of "coffee-ground" emesis. The nurse will anticipate preparing the patient for a. endoscopy. b. angiography. c. gastric analysis testing. d. barium contrast studies.

ANS: A Endoscopy is the primary tool for visualization and diagnosis of upper gastrointestinal (GI) bleeding. Angiography is used only when endoscopy cannot be done because it is more invasive and has more possible complications. Gastric analysis testing may help with determining the cause of gastric irritation, but it is not used for acute GI bleeding. Barium studies are helpful in determining the presence of gastric lesions, but not whether the lesions are actively bleeding.

14. A patient has elevated blood urea nitrogen (BUN) and serum creatinine levels. Which bowel preparation order would the nurse question for this patient who is scheduled for a renal arteriogram? a. Fleet enema b. Tap-water enema c. Senna/docusate (Senokot-S) d. Bisacodyl (Dulcolax) tablets

ANS: A High-phosphate enemas, such as Fleet enemas, should be avoided in patients with elevated BUN and creatinine because phosphate cannot be excreted by patients with renal failure. The other medications for bowel evacuation are more appropriate.

A 56-year-old female patient has an adrenocortical adenoma, causing hyperaldosteronism. The nurse providing care should a. monitor the blood pressure every 4 hours. b. elevate the patient's legs to relieve edema. c. monitor blood glucose level every 4 hours. d. order the patient a potassium-restricted diet.

ANS: A Hypertension caused by sodium retention is a common complication of hyperaldosteronism. Hyperaldosteronism does not cause an elevation in blood glucose. The patient will be hypokalemic and require potassium supplementation before surgery. Edema does not usually occur with hyperaldosteronism

When planning teaching for a patient with benign nephrosclerosis the nurse should include instructions regarding a. monitoring and recording blood pressure. b. obtaining and documenting daily weights. c. measuring daily intake and output amounts. d. preventing bleeding caused by anticoagulants.

ANS: A Hypertension is the major symptom of nephrosclerosis. Measurements of intake and output and daily weights are not necessary unless the patient develops renal insufficiency. Anticoagulants are not used to treat nephrosclerosis. DIF: Cognitive Level: Application REF: 1141-1142

25. A 50-year-old patient who underwent a gastroduodenostomy (Billroth I) earlier today complains of increasing abdominal pain. The patient has no bowel sounds and 200 mL of bright red nasogastric (NG) drainage in the last hour. The highest priority action by the nurse is to a. contact the surgeon. b. irrigate the NG tube. c. monitor the NG drainage. d. administer the prescribed morphine.

ANS: A Increased pain and 200 mL of bright red NG drainage 12 hours after surgery indicate possible postoperative hemorrhage, and immediate actions such as blood transfusion and/or return to surgery are needed. Because the NG is draining, there is no indication that irrigation is needed. Continuing to monitor the NG drainage is not an adequate response. The patient may need morphine, but this is not the highest priority action.

25. Twelve hours after undergoing a gastroduodenostomy (Billroth I), a patient complains of increasing abdominal pain. The patient has absent bowel sounds and 200 mL of bright red nasogastric (NG) drainage in the last hour. The most appropriate action by the nurse at this time is to a. notify the surgeon. b. irrigate the NG tube. c. administer the prescribed morphine. d. continue to monitor the NG drainage.

ANS: A Increased pain and 200 mL of bright red NG drainage 12 hours after surgery indicate possible postoperative hemorrhage, and immediate actions such as blood transfusion and/or return to surgery are needed. Because the NG is draining, there is no indication that irrigation is needed. The patient may need morphine, but this is not the highest priority action. Continuing to monitor the NG drainage is not an adequate response.

A patient in the hospital has a history of functional urinary incontinence. Which nursing action will be included in the plan of care? a. Place a bedside commode near the patient's bed. b. Demonstrate the use of the Credé maneuver to the patient. c. Use an ultrasound scanner to check postvoiding residuals. d. Teach the use of Kegel exercises to strengthen the pelvic floor.

ANS: A Modifications in the environment make it easier to avoid functional incontinence. Checking for residual urine and performing the Credé maneuver are interventions for overflow incontinence. Kegel exercises are useful for stress incontinence. DIF: Cognitive Level: Application REF: 1148

18. A patient who has had several episodes of bloody diarrhea is admitted to the emergency department. Which action should the nurse anticipate taking? a. Obtain a stool specimen for culture. b. Administer antidiarrheal medications. c. Teach about adverse effects of nonsteroidal anti-inflammatory drugs (NSAIDs). d. Provide education about antibiotic therapy.

ANS: A Patients with bloody diarrhea should have a stool culture for E. coli O157:H7. NSAIDs may cause occult blood in the stools, but not diarrhea. Antidiarrheal medications usually are avoided for possible infectious diarrhea to avoid prolonging the infection. Antibiotic therapy in the treatment of infectious diarrhea is controversial because it may precipitate kidney complications.

16. The nurse is caring for a 68-year-old hospitalized patient with a decreased glomerular filtration rate who is scheduled for an intravenous pyelogram (IVP). Which action will be included in the plan of care? a. Monitor the urine output after the procedure. b. Assist with monitored anesthesia care (MAC). c. Give oral contrast solution before the procedure. d. Insert a large size urinary catheter before the IVP.

ANS: A Patients with impaired renal function are at risk for decreased renal function after IVP because the contrast medium used is nephrotoxic, so the nurse should monitor the patient's urine output. MAC sedation and retention catheterization are not required for the procedure. The contrast medium is given IV, not orally.

45. A 51-year-old woman with Crohn's disease who is taking infliximab (Remicade) calls the nurse in the outpatient clinic about new symptoms. Which symptom is most important to communicate to the health care provider? a. Fever b. Nausea c. Joint pain d. Headache

ANS: A Since infliximab suppresses the immune response, rapid treatment of infection is essential. The other patient complaints are common side effects of the medication, but they do not indicate any potentially life-threatening complications.

23. A nurse suspects that a patient with type 1 diabetes may be experiencing the Somogyi phenomenon. What symptom supports this suspicion? a. Headache on awakening and enuresis b. 6 AM blood sugar of 58 mg/dL and nausea c. Abdominal pain and elevated blood pressure d. Drowsiness and disorientation after eating

ANS: A The Somogyi phenomenon occurs because of a rebound hyperglycemia after a period of hypoglycemia during the early morning. The patient wakes with a headache, enuresis, nausea and vomiting, nightmares, and a high level of blood sugar.

30. Which information will be best for the nurse to include when teaching a patient with peptic ulcer disease (PUD) about dietary management of the disease? a. "Avoid foods that cause pain after you eat them." b. "High-protein foods are least likely to cause pain." c. "You will need to remain on a bland diet indefinitely." d. "You should avoid eating many raw fruits and vegetables."

ANS: A The best information is that each individual should choose foods that are not associated with postprandial discomfort. Raw fruits and vegetables may irritate the gastric mucosa, but chewing well seems to decrease this and some patients may tolerate these well. High-protein foods help to neutralize acid, but they also stimulate hydrochloric (HCl) acid secretion and may increase discomfort for some patients. Bland diets may be recommended during an acute exacerbation of PUD, but there is little scientific evidence to support their use.

5. Which medication taken at home by a 47-year-old patient with decreased renal function will be of most concern to the nurse? a. ibuprofen (Motrin) b. warfarin (Coumadin) c. folic acid (vitamin B9) d. penicillin (Bicillin LA)

ANS: A The nonsteroidal antiinflammatory medications (NSAIDs) are nephrotoxic and should be avoided in patients with impaired renal function. The nurse also should ask about reasons the patient is taking the other medications, but the medication of most concern is the ibuprofen.

The nurse is caring for a patient admitted with diabetes insipidus (DI). Which information is most important to report to the health care provider? a. The patient is confused and lethargic. b. The patient reports a recent head injury. c. The patient has a urine output of 400 mL/hr. d. The patient's urine specific gravity is 1.003.

ANS: A The patient's confusion and lethargy may indicate hypernatremia and should be addressed quickly. In addition, patients with DI compensate for fluid losses by drinking copious amounts of fluids, but a patient who is lethargic will be unable to drink enough fluids and will become hypovolemic. A high urine output, low urine specific gravity, and history of a recent head injury are consistent with diabetes insipidus, but they do not require immediate nursing action to avoid life-threatening complications

An 88-year-old with benign prostatic hyperplasia (BPH) has a markedly distended bladder and is agitated and confused. Which of the following interventions prescribed by the health care provider should the nurse implement first? a. Insert a urinary retention catheter. b. Schedule an intravenous pyelogram. c. Administer lorazepam (Ativan) 0.5 mg PO. d. Draw blood for blood urea nitrogen (BUN) and creatinine testing.

ANS: A The patient's history and clinical manifestations are consistent with acute urinary retention, and the priority action is to relieve the retention by catheterization. The BUN and creatinine measurements can be obtained after the catheter is inserted. The patient's agitation may resolve once the bladder distention is corrected, and sedative drugs should be used cautiously in older patients. The IVP is an appropriate test, but does not need to be done urgently. DIF: Cognitive Level: Application REF: 1135-1136

23. A patient with a bleeding duodenal ulcer has a nasogastric (NG) tube in place, and the health care provider orders 30 mL of aluminum hydroxide/magnesium hydroxide (Maalox) to be instilled through the tube every hour. To evaluate the effectiveness of this treatment, the nurse a. periodically aspirates and tests gastric pH. b. monitors arterial blood gas values on a daily basis. c. checks each stool for the presence of occult blood. d. measures the amount of residual stomach contents hourly.

ANS: A The purpose for antacids is to increase gastric pH. Checking gastric pH is the most direct way of evaluating the effectiveness of the medication. Arterial blood gases may change slightly, but this does not directly reflect the effect of antacids on gastric pH. Because the patient has upper gastrointestinal (GI) bleeding, occult blood in the stools will appear even after the acute bleeding has stopped. The amount of residual stomach contents is not a reflection of resolution of bleeding or of gastric pH.

48. A female patient is awaiting surgery for acute peritonitis. Which action will the nurse include in the plan of care? a. Position patient with the knees flexed. b. Avoid use of opioids or sedative drugs. c. Offer frequent small sips of clear liquids. d. Assist patient to breathe deeply and cough.

ANS: A There is less peritoneal irritation with the knees flexed, which will help decrease pain. Opioids and sedatives are typically given to control pain and anxiety. Preoperative patients with peritonitis are given IV fluids for hydration. Deep breathing and coughing will increase the patient's discomfort.

A female patient is awaiting surgery for acute peritonitis. Which action will the nurse include in the plan of care? a. Position patient with the knees flexed. b. Avoid use of opioids or sedative drugs. c. Offer frequent small sips of clear liquids. d. Assist patient to breathe deeply and cough.

ANS: A There is less peritoneal irritation with the knees flexed, which will help decrease pain. Opioids and sedatives are typically given to control pain and anxiety. Preoperative patients with peritonitis are given IV fluids for hydration. Deep breathing and coughing will increase the patient's discomfort.

1. Which information will the nurse include when teaching a patient how to avoid chronic constipation (select all that apply)? a. Many over-the-counter (OTC) medications can cause constipation. b. Stimulant and saline laxatives can be used regularly. c. Bulk-forming laxatives are an excellent source of fiber. d. Walking or cycling frequently will help bowel motility. e. A good time for a bowel movement may be after breakfast.

ANS: A, C, D, E Stimulant and saline laxatives should be used infrequently. Use of bulk-forming laxatives, regular early morning timing of defecation, regular exercise, and avoiding many OTC medications will help the patient avoid constipation.

A patient who was admitted with myxedema coma and diagnosed with hypothyroidism is improving and expected to be discharged in 2 days. Which teaching strategy will be best for the nurse to use? a. Provide written reminders of self-care information. b. Offer multiple options for management of therapies. c. Ensure privacy for teaching by asking visitors to leave. d. Delay teaching until patient discharge date is confirmed.

ANS: A Written instructions will be helpful to the patient because initially the hypothyroid patient may be unable to remember to take medications and other aspects of self-care. Because the treatment regimen is somewhat complex, teaching should be initiated well before discharge. Family members or friends should be included in teaching because the hypothyroid patient is likely to forget some aspects of the treatment plan. A simpler regimen will be easier to understand until the patient is euthyroid

A postoperative patient who has been receiving nasogastric suction for 3 days has a serum sodium level of 125 mEq/L (125 mmol/L). Which of these prescribed therapies that the patient has been receiving should the nurse question? a. Infuse 5% dextrose in water at 125 ml/hr. b. Administer IV morphine sulfate 4 mg every 2 hours PRN. c. Give IV metoclopramide (Reglan) 10 mg every 6 hours PRN for nausea. d. Administer 3% saline if serum sodium drops to less than 128 mEq/L.

ANS: A Infuse 5% dextrose in water at 125 ml/hr. Because the patient's gastric suction has been depleting electrolytes, the IV solution should include electrolyte replacement. Solutions such as lactated Ringer's solution would usually be ordered for this patient. The other orders are appropriate for a postoperative patient with gastric suction.

The nurse notes that a patient who was admitted with diabetic ketoacidosis has rapid, deep respirations. Which action should the nurse take? a. Notify the patient's health care provider. b. Give the prescribed PRN lorazepam (Ativan). c. Start the prescribed PRN oxygen at 2 to 4 L/min. d. Encourage the patient to take deep, slow breaths.

ANS: A Notify the patient's health care provider. The rapid, deep (Kussmaul) respirations indicate a metabolic acidosis and the need for actions such as administration of sodium bicarbonate, which will require a prescription by the health care provider. Oxygen therapy is not indicated because there is no indication that the increased respiratory rate is related to hypoxemia. The respiratory pattern is compensatory, and the patient will not be able to slow the respiratory rate. Ativan administration will slow the respiratory rate and increase the level of acidosis.

When caring for a patient admitted with hyponatremia, which actions will the nurse anticipate taking? a. Restrict patient's oral free water intake. b. Avoid use of electrolyte-containing drinks. c. Infuse a solution of 5% dextrose in 0.45% saline. d. Administer vasopressin (antidiuretic hormone, [ADH]).

ANS: A Restrict patient's oral free water intake. To help improve serum sodium levels, water intake is restricted. Electrolyte-containing beverages will improve the patient's sodium level. Administration of vasopressin or hypotonic IV solutions will decrease the serum sodium level further.

The nurse obtains all of the following assessment data about a patient with deficient fluid volume caused by a massive burn injury. Which of the following assessment data will be of greatest concern? a. The blood pressure is 90/40 mm Hg. b. Urine output is 30 ml over the last hour. c. Oral fluid intake is 100 ml for the last 8 hours. d. There is prolonged skin tenting over the sternum.

ANS: A The blood pressure is 90/40 mm Hg. The blood pressure indicates that the patient may be developing hypovolemic shock as a result of fluid loss. This will require immediate intervention to prevent the complications associated with systemic hypoperfusion. The poor oral intake, decreased urine output, and skin tenting all indicate the need for increasing the patient's fluid intake but not as urgently as the hypotension.

Which assessment finding about a patient who has a serum calcium level of 7.0 mEq/L is most important for the nurse to report to the health care provider? a. The patient is experiencing laryngeal stridor. b. The patient complains of generalized fatigue. c. The patient's bowels have not moved for 4 days. d. The patient has numbness and tingling of the lips.

ANS: A The patient is experiencing laryngeal stridor. Laryngeal stridor may lead to respiratory arrest and requires rapid action to correct the patient's calcium level. The other data also are consistent with hypocalcemia, but do not indicate a need for immediate action.

The nurse in the outpatient clinic who notes that a patient has a decreased magnesium level should ask the patient about a. daily alcohol intake. b. intake of dietary protein. c. multivitamin/mineral use. d. use of over-the-counter (OTC) laxatives.

ANS: A daily alcohol intake. Hypomagnesemia is associated with alcoholism. Protein intake would not have a significant effect on magnesium level. OTC laxatives (such as milk of magnesia) and use of multivitamin/mineral supplements would tend to increase magnesium level.

When teaching a patient with renal failure about a low phosphate diet, the nurse will include information to restrict a. ingestion of dairy products. b. the amount of high-fat foods. c. the quantity of fruits and juices. d. intake of green, leafy vegetables.

ANS: A ingestion of dairy products. Foods high in phosphate include milk and other dairy products, so these are restricted on low-phosphate diets. Green, leafy vegetables; high-fat foods; and fruits/juices are not high in phosphate and are not restricted.

A patient is receiving 3% NaCl solution for correction of hyponatremia. During administration of the solution, the most important assessment for the nurse to monitor is a. lung sounds. b. urinary output. c. peripheral pulses. d. peripheral edema.

ANS: A lung sounds. Hypertonic solutions cause water retention, so the patient should be monitored for symptoms of fluid excess. Crackles in the lungs may indicate the onset of pulmonary edema and are the most serious of the symptoms of fluid excess listed. Bounding peripheral pulses, peripheral edema, or changes in urine output also are important to monitor when administering hypertonic solutions, but they do not indicate acute respiratory or cardiac decompensation.

A patient has the following arterial blood gas (ABG) results: pH 7.32, PaO2 88 mm Hg, PaCO2 37 mm Hg, and HCO3 16 mEq/L. The nurse interprets these results as a. metabolic acidosis. b. metabolic alkalosis. c. respiratory acidosis. d. respiratory alkalosis.

ANS: A metabolic acidosis. The pH and HCO3 indicate that the patient has a metabolic acidosis. The ABGs are inconsistent with the other responses.

Which information will the nurse include when teaching a patient how to avoid chronic constipation (select all that apply)? a. Many over-the-counter (OTC) medications can cause constipation. b. Stimulant and saline laxatives can be used regularly. c. Bulk-forming laxatives are an excellent source of fiber. d. Walking or cycling frequently will help bowel motility. e. A good time for a bowel movement may be after breakfast.

ANS: A, C, D, E Stimulant and saline laxatives should be used infrequently. Use of bulk-forming laxatives, regular early morning timing of defecation, regular exercise, and avoiding many OTC medications will help the patient avoid constipation.

26. What should a teaching plan about foot care include for a patient with diabetes? (Select all that apply.) a. Wash and carefully dry the feet every day. b. Apply lotion between the toes. c. Protect the feet from extreme temperatures. d. Walk barefoot only indoors. e. Buy shoes that are comfortable and supportive.

ANS: A, C, E Washing, inspecting, and drying the feet, especially between the toes, is essential. Protecting the feet from heat and cold and wearing supportive shoes is important to good foot health. Lotion can be applied to the soles and tops of the feet but not between the toes. Walking barefoot is contraindicated for a person with diabetes.

17. Which medications will the nurse teach the patient about whose peptic ulcer disease is associated with Helicobacter pylori? a. Sucralfate (Carafate), nystatin (Mycostatin), and bismuth (Pepto-Bismol) b. Amoxicillin (Amoxil), clarithromycin (Biaxin), and omeprazole (Prilosec) c. Famotidine (Pepcid), magnesium hydroxide (Mylanta), and pantoprazole (Protonix) d. Metoclopramide (Reglan), bethanechol (Urecholine), and promethazine (Phenergan)

ANS: B

A 33-year-old male patient with a gunshot wound to the abdomen undergoes surgery, and a colostomy is formed as shown in the accompanying figure. Which information will be included in patient teaching? a. Stool will be expelled from both stomas. b. This type of colostomy is usually temporary. c. Soft, formed stool can be expected as drainage. d. Irrigations can regulate drainage from the stomas.

ANS: B A loop, or double-barrel stoma, is usually temporary. Stool will be expelled from the proximal stoma only. The stool from the transverse colon will be liquid and regulation through irrigations will not be possible.

Intravenous potassium chloride (KCl) 60 mEq is prescribed for treatment of a patient with severe hypokalemia. Which action should the nurse take? a. Administer the KCl as a rapid IV bolus. b. Infuse the KCl at a rate of 20 mEq/hour. c. Give the KCl only through a central venous line. d. Add no more than 40 mEq/L to a liter of IV fluid.

ANS: B ANS: B Infuse the KCl at a rate of 20 mEq/hour. Intravenous KCl is administered at a maximal rate of 20 mEq/hr. Rapid IV infusion of KCl can cause cardiac arrest. Although the preferred concentration for KCl is no more than 40 mEq/L, concentrations up to 80 mEq/L may be used for some patients. KCl can cause inflammation of peripheral veins, but it can be administered by this route.

A patient with renal calculi is hospitalized with gross hematuria and severe colicky left flank pain. Which nursing action will be of highest priority at this time? a. Encourage oral fluid intake. b. Administer prescribed analgesics. c. Monitor temperature every 4 hours. d. Give antiemetics as needed for nausea.

ANS: B Although all of the nursing actions may be used for patients with renal lithiasis, the patient's presentation indicates that management of pain is the highest priority action. If the patient has urinary obstruction, increasing oral fluids may increase the symptoms. There is no evidence of infection or nausea. DIF: Cognitive Level: Application REF: 1137-1138 | 1139-1141 | 1140

The nurse is admitting a 67-year-old patient with new-onset steatorrhea. Which question is most important for the nurse to ask? a. "How much milk do you usually drink?" b. "Have you noticed a recent weight loss?" c. "What time of day do your bowels move?" d. "Do you eat meat or other animal products?"

ANS: B Although all of the questions provide useful information, it is most important to determine if the patient has an imbalance in nutrition because of the steatorrhea.

9. A patient being admitted with an acute exacerbation of ulcerative colitis reports crampy abdominal pain and passing 15 or more bloody stools a day. The nurse will plan to a. administer IV metoclopramide (Reglan). b. discontinue the patient's oral food intake. c. administer cobalamin (vitamin B12) injections. d. teach the patient about total colectomy surgery.

ANS: B An initial therapy for an acute exacerbation of inflammatory bowel disease (IBD) is to rest the bowel by making the patient NPO. Metoclopramide increases peristalsis and will worsen symptoms. Cobalamin (vitamin B12) is absorbed in the ileum, which is not affected by ulcerative colitis. Although total colectomy is needed for some patients, there is no indication that this patient is a candidate.

A patient being admitted with an acute exacerbation of ulcerative colitis reports crampy abdominal pain and passing 15 or more bloody stools a day. The nurse will plan to a. administer IV metoclopramide (Reglan). b. discontinue the patient's oral food intake. c. administer cobalamin (vitamin B12) injections. d. teach the patient about total colectomy surgery.

ANS: B An initial therapy for an acute exacerbation of inflammatory bowel disease (IBD) is to rest the bowel by making the patient NPO. Metoclopramide increases peristalsis and will worsen symptoms. Cobalamin (vitamin B12) is absorbed in the ileum, which is not affected by ulcerative colitis. Although total colectomy is needed for some patients, there is no indication that this patient is a candidate.

The nurse preparing for the annual physical exam of a 50-year-old man will plan to teach the patient about a. endoscopy. b. colonoscopy. c. computerized tomography screening. d. carcinoembryonic antigen (CEA) testing.

ANS: B At age 50, individuals with an average risk for colorectal cancer (CRC) should begin screening for CRC. Colonoscopy is the gold standard for CRC screening. The other diagnostic tests are not recommended as part of a routine annual physical exam at age 50.

10. Which nursing action will the nurse include in the plan of care for a 35-year-old male patient admitted with an exacerbation of inflammatory bowel disease (IBD)? a. Restrict oral fluid intake. b. Monitor stools for blood. c. Ambulate four times daily. d. Increase dietary fiber intake.

ANS: B Because anemia or hemorrhage may occur with IBD, stools should be assessed for the presence of blood. The other actions would not be appropriate for the patient with IBD. Because dietary fiber may increase gastrointestinal (GI) motility and exacerbate the diarrhea, severe fatigue is common with IBD exacerbations, and dehydration may occur.

Which nursing action will the nurse include in the plan of care for a 35-year-old male patient admitted with an exacerbation of inflammatory bowel disease (IBD)? a. Restrict oral fluid intake. b. Monitor stools for blood. c. Ambulate four times daily. d. Increase dietary fiber intake.

ANS: B Because anemia or hemorrhage may occur with IBD, stools should be assessed for the presence of blood. The other actions would not be appropriate for the patient with IBD. Because dietary fiber may increase gastrointestinal (GI) motility and exacerbate the diarrhea, severe fatigue is common with IBD exacerbations, and dehydration may occur.

35. A patient in the emergency department has just been diagnosed with peritonitis caused by a ruptured diverticulum. Which prescribed intervention will the nurse implement first? a. Insert a urinary catheter to drainage. b. Infuse metronidazole (Flagyl) 500 mg IV. c. Send the patient for a computerized tomography scan. d. Place a nasogastric (NG) tube to intermittent low suction.

ANS: B Because peritonitis can be fatal if treatment is delayed, the initial action should be to start antibiotic therapy (after any ordered cultures are obtained). The other actions can be done after antibiotic therapy is initiated.

A patient in the emergency department has just been diagnosed with peritonitis caused by a ruptured diverticulum. Which prescribed intervention will the nurse implement first? a. Insert a urinary catheter to drainage. b. Infuse metronidazole (Flagyl) 500 mg IV. c. Send the patient for a computerized tomography scan. d. Place a nasogastric (NG) tube to intermittent low suction.

ANS: B Because peritonitis can be fatal if treatment is delayed, the initial action should be to start antibiotic therapy (after any ordered cultures are obtained). The other actions can be done after antibiotic therapy is initiated.

41. Which information obtained by the nurse interviewing a 30-year-old male patient is most important to communicate to the health care provider? a. The patient has a history of constipation. b. The patient has noticed blood in the stools. c. The patient had an appendectomy at age 27. d. The patient smokes a pack/day of cigarettes.

ANS: B Blood in the stools is a possible clinical manifestation of colorectal cancer and requires further assessment by the health care provider. The other patient information will also be communicated to the health care provider, but does not indicate an urgent need for further testing or intervention.

Which information obtained by the nurse interviewing a 30-year-old male patient is most important to communicate to the health care provider? a. The patient has a history of constipation. b. The patient has noticed blood in the stools. c. The patient had an appendectomy at age 27. d. The patient smokes a pack/day of cigarettes.

ANS: B Blood in the stools is a possible clinical manifestation of colorectal cancer and requires further assessment by the health care provider. The other patient information will also be communicated to the health care provider, but does not indicate an urgent need for further testing or intervention.

20. A 74-year-old patient preparing to undergo a colon resection for cancer of the colon asks about the elevated carcinoembryonic antigen (CEA) test result. The nurse explains that the test is used to a. identify any metastasis of the cancer. b. monitor the tumor status after surgery. c. confirm the diagnosis of a specific type of cancer. d. determine the need for postoperative chemotherapy.

ANS: B CEA is used to monitor for cancer recurrence after surgery. CEA levels do not help to determine whether there is metastasis of the cancer. Confirmation of the diagnosis is made on the basis of biopsy. Chemotherapy use is based on factors other than CEA.

A 74-year-old patient preparing to undergo a colon resection for cancer of the colon asks about the elevated carcinoembryonic antigen (CEA) test result. The nurse explains that the test is used to a. identify any metastasis of the cancer. b. monitor the tumor status after surgery. c. confirm the diagnosis of a specific type of cancer. d. determine the need for postoperative chemotherapy.

ANS: B CEA is used to monitor for cancer recurrence after surgery. CEA levels do not help to determine whether there is metastasis of the cancer. Confirmation of the diagnosis is made on the basis of biopsy. Chemotherapy use is based on factors other than CEA.

Which nursing action should the nurse who is caring for a patient who has had an ileal conduit for several years delegate to nursing assistive personnel (NAP)? a. Assess for symptoms of urinary tract infection (UTI). b. Change the ostomy appliance. c. Choose the appropriate ostomy bag. d. Monitor the appearance of the stoma.

ANS: B Changing the ostomy appliance for a stable patient could be done by NAP. Assessments of the site, choosing the appropriate ostomy bag, and assessing for (UTI) symptoms require more education and scope of practice and should be done by the RN. DIF: Cognitive Level: Application REF: 1157 | 1159-1160 | 1158

8. How will the nurse assess for flank tenderness in a 30-year-old female patient with suspected pyelonephritis? a. Palpate along both sides of the lumbar vertebral column. b. Strike a flat hand covering the costovertebral angle (CVA). c. Push fingers upward into the two lowest intercostal spaces. d. Percuss between the iliac crest and ribs along the midaxillary line.

ANS: B Checking for flank pain is best performed by percussion of the CVA and asking about pain. The other techniques would not assess for flank pain.

After obtaining the health history for a 25-year-old who smokes two packs of cigarettes daily, the nurse will plan to do teaching about the increased risk for a. kidney stones. b. bladder cancer. c. bladder infection. d. interstitial cystitis.

ANS: B Cigarette smoking is a risk factor for bladder cancer. The patient's risk for developing interstitial cystitis, urinary tract infection (UTI), or kidney stones will not be reduced by quitting smoking. DIF: Cognitive Level: Application REF: 1145-1146

2. A patient who has been NPO during treatment for nausea and vomiting caused by gastric irritation is to start oral intake. Which of these should the nurse offer to the patient? a. A glass of orange juice b. A dish of lemon gelatin c. A cup of coffee with cream d. A bowl of hot chicken broth

ANS: B Clear liquids are usually the first foods started after a patient has been nauseated. Acidic foods such as orange juice, very hot foods, and coffee are poorly tolerated when patients have been nauseated.

13. Which diet choice by the patient with an acute exacerbation of inflammatory bowel disease (IBD) indicates a need for more teaching? a. Scrambled eggs b. White toast and jam c. Oatmeal with cream d. Pancakes with syrup

ANS: C During acute exacerbations of IBD, the patient should avoid high-fiber foods such as whole grains. High-fat foods also may cause diarrhea in some patients. The other choices are low residue and would be appropriate for this patient.

A patient who has had a transurethral resection with fulguration for bladder cancer 3 days previously calls the nurse at the urology clinic. Which information given by the patient is most important to report to the health care provider? a. The patient is using opioids for pain. b. The patient has noticed clots in the urine. c. The patient is very anxious about the cancer. d. The patient is voiding every 4 hours at night.

ANS: B Clots in the urine are not expected and require further follow-up. Voiding every 4 hours, use of opioids for pain, and anxiety are typical after this procedure. DIF: Cognitive Level: Application REF: 1145-1146

32. The nurse is assessing a 31-year-old female patient with abdominal pain. Th nurse,who notes that there is ecchymosis around the area of umbilicus, will document this finding as a. Cullen sign. b. Rovsing sign. c. McBurney sign. d. Grey-Turner's signt.

ANS: B Cullen sign is ecchymosis around the umbilicus. Rovsing sign occurs when palpation of the left lower quadrant causes pain in the right lower quadrant. Deep tenderness at McBurney's point (halfway between the umbilicus and the right iliac crest), known as McBurney's sign, is a sign of acute appendicitis.

12. Which information will the nurse include for a patient with newly diagnosed gastroesophageal reflux disease (GERD)? a. "Peppermint tea may reduce your symptoms." b. "Keep the head of your bed elevated on blocks." c. "You should avoid eating between meals to reduce acid secretion." d. "Vigorous physical activities may increase the incidence of reflux."

ANS: B Elevating the head of the bed will reduce the incidence of reflux while the patient is sleeping. Peppermint will decrease lower esophageal sphincter (LES) pressure and increase the chance for reflux. Small, frequent meals are recommended to avoid abdominal distention. There is no need to make changes in physical activities because of GERD.

The cardiac telemetry unit charge nurse receives status reports from other nursing units about four patients who need cardiac monitoring. Which patient should be transferred to the cardiac unit first? a. Patient with Hashimoto's thyroiditis and a heart rate of 102 b. Patient with tetany who has a new order for IV calcium chloride c. Patient with Cushing syndrome and a blood glucose of 140 mg/dL d. Patient with Addison's disease who takes hydrocortisone twice daily

ANS: B Emergency treatment of tetany requires IV administration of calcium; ECG monitoring will be required because cardiac arrest may occur if high calcium levels result from too-rapid administration. The information about the other patients indicates that they are more stable than the patient with tetany

14. After a total proctocolectomy and permanent ileostomy, the patient tells the nurse, "I cannot manage all these changes. I don't want to look at the stoma." What is the best action by the nurse? a. Reassure the patient that ileostomy care will become easier. b. Ask the patient about the concerns with stoma management. c. Develop a detailed written list of ostomy care tasks for the patient. d. Postpone any teaching until the patient adjusts to the ileostomy.

ANS: B Encouraging the patient to share concerns assists in helping the patient adjust to the body changes. Acknowledgment of the patient's feelings and concerns is important rather than offering false reassurance. Because the patient indicates that the feelings about the ostomy are the reason for the difficulty with the many changes, development of a detailed ostomy care plan will not improve the patient's ability to manage the ostomy. Although detailed ostomy teaching may be postponed, the nurse should offer teaching about some aspects of living with an ostomy.

After a total proctocolectomy and permanent ileostomy, the patient tells the nurse, "I cannot manage all these changes. I don't want to look at the stoma." What is the best action by the nurse? a. Reassure the patient that ileostomy care will become easier. b. Ask the patient about the concerns with stoma management. c. Develop a detailed written list of ostomy care tasks for the patient. d. Postpone any teaching until the patient adjusts to the ileostomy.

ANS: B Encouraging the patient to share concerns assists in helping the patient adjust to the body changes. Acknowledgment of the patient's feelings and concerns is important rather than offering false reassurance. Because the patient indicates that the feelings about the ostomy are the reason for the difficulty with the many changes, development of a detailed ostomy care plan will not improve the patient's ability to manage the ostomy. Although detailed ostomy teaching may be postponed, the nurse should offer teaching about some aspects of living with an ostomy.

22. A family member of a 28-year-old patient who has suffered massive abdominal trauma in an automobile accident asks the nurse why the patient is receiving famotidine (Pepcid). The nurse will explain that the medication will a. decrease nausea and vomiting. b. inhibit development of stress ulcers. c. lower the risk for H. pylori infection. d. prevent aspiration of gastric contents.

ANS: B Famotidine is administered to prevent the development of physiologic stress ulcers, which are associated with a major physiologic insult such as massive trauma. Famotidine does not decrease nausea or vomiting, prevent aspiration, or prevent H. pylori infection.

22. The family member of a patient who has suffered massive abdominal trauma in an automobile accident asks the nurse why the patient is receiving famotidine (Pepcid). The nurse will explain that the medication will a. prevent aspiration of gastric contents. b. inhibit the development of stress ulcers. c. lower the chance for H. pylori infection. d. decrease the risk for nausea and vomiting.

ANS: B Famotidine is administered to prevent the development of physiologic stress ulcers, which are associated with a major physiologic insult such as massive trauma. Famotidine does not decrease nausea or vomiting, prevent aspiration, or prevent H. pylori infection.

A 54-year-old critically ill patient with sepsis is frequently incontinent of watery stools. What action by the nurse will prevent complications associated with ongoing incontinence? a. Apply incontinence briefs. b. Use a fecal management system c. Insert a rectal tube with a drainage bag. d. Assist the patient to a commode frequently.

ANS: B Fecal management systems are designed to contain loose stools and can be in place for as long as 4 weeks without causing damage to the rectum or anal sphincters. Although incontinence briefs may be helpful, unless they are changed frequently, they are likely to increase the risk for skin breakdown. Rectal tubes are avoided because of possible damage to the anal sphincter and ulceration of the rectal mucosa. A critically ill patient will not be able to tolerate getting up frequently to use the commode or bathroom.

16. A 24-year-old woman with Crohn's disease develops a fever and symptoms of a urinary tract infection (UTI) with tan, fecal-smelling urine. What information will the nurse add to a general teaching plan about UTIs in order to individualize the teaching for this patient? a. Bacteria in the perianal area can enter the urethra. b. Fistulas can form between the bowel and bladder. c. Drink adequate fluids to maintain normal hydration. d. Empty the bladder before and after sexual intercourse.

ANS: B Fistulas between the bowel and bladder occur in Crohn's disease and can lead to UTI. Teaching for UTI prevention in general includes good hygiene, adequate fluid intake, and voiding before and after intercourse.

A patient with nephrotic syndrome develops flank pain. The nurse will anticipate teaching the patient about treatment with a. antibiotics. b. anticoagulants. c. corticosteroids. d. antihypertensives.

ANS: B Flank pain in a patient with nephrosis suggests a renal vein thrombosis, and anticoagulation is needed. Antibiotics are used to treat a patient with flank pain caused by pyelonephritis. Antihypertensives are used if the patient has high blood pressure. Corticosteroids may be used to treat nephrotic syndrome but will not resolve a thrombosis. DIF: Cognitive Level: Application REF: 1133-1134

23. Which information will the nurse include in teaching a patient who had a proctocolectomy and ileostomy for ulcerative colitis? a. Restrict fluid intake to prevent constant liquid drainage from the stoma. b. Use care when eating high-fiber foods to avoid obstruction of the ileum. c. Irrigate the ileostomy daily to avoid having to wear a drainage appliance. d. Change the pouch every day to prevent leakage of contents onto the skin.

ANS: B High-fiber foods are introduced gradually and should be well chewed to avoid obstruction of the ileostomy. Patients with ileostomies lose the absorption of water in the colon and need to take in increased amounts of fluid. The pouch should be drained frequently but is changed every 5 to 7 days. The drainage from an ileostomy is liquid and continuous, so control by irrigation is not possible.

Which information will the nurse include in teaching a patient who had a proctocolectomy and ileostomy for ulcerative colitis? a. Restrict fluid intake to prevent constant liquid drainage from the stoma. b. Use care when eating high-fiber foods to avoid obstruction of the ileum. c. Irrigate the ileostomy daily to avoid having to wear a drainage appliance. d. Change the pouch every day to prevent leakage of contents onto the skin.

ANS: B High-fiber foods are introduced gradually and should be well chewed to avoid obstruction of the ileostomy. Patients with ileostomies lose the absorption of water in the colon and need to take in increased amounts of fluid. The pouch should be drained frequently but is changed every 5 to 7 days. The drainage from an ileostomy is liquid and continuous, so control by irrigation is not possible.

After teaching a patient with interstitial cystitis about management of the condition, the nurse determines that further instruction is needed when the patient says, a. "I will have to stop having coffee and orange juice for breakfast." b. "I should start taking a high potency multiple vitamin every morning." c. "I will buy some calcium glycerophosphate (Prelief) at the pharmacy." d. "I should call the doctor about increased bladder pain or odorous urine."

ANS: B High-potency multiple vitamins may irritate the bladder and increase symptoms. The other patient statements indicate good understanding of the teaching.

24. A patient has been admitted with hyperglycemic hyperosmolar nonketotic syndrome (HHNS). The blood glucose level is very high (880 mg/dL) on admission. The physician believes that the condition is the result of large amounts of glucose solutions administered intravenously (IV) during renal dialysis. What should the nurse anticipate that the patient would exhibit? a. Fruity breath and a high level of ketones in her urine b. Severe dehydration and hypernatremia caused by the hyperglycemia c. Exactly the same symptoms and signs as DKA d. Kussmaul respirations, nausea, and vomiting

ANS: B IV solutions containing glucose bypass the digestive system; consequently, the pancreas is not triggered to release insulin. However, just enough insulin is present to prevent the breakdown of fatty acids and the formation of ketones.

11. A patient gives the nurse health information before a scheduled intravenous pyelogram (IVP). Which item has the most immediate implications for the patient's care? a. The patient has not had food or drink for 8 hours. b. The patient lists allergies to shellfish and penicillin. c. The patient complains of costovertebral angle (CVA) tenderness. d. The patient used a bisacodyl (Dulcolax) tablet the previous night.

ANS: B Iodine-based contrast dye is used during IVP and for many computed tomography (CT) scans. The nurse will need to notify the health care provider before the procedures so that the patient can receive medications such as antihistamines or corticosteroids before the procedures are started. The other information is also important to note and document but does not have immediate implications for the patient's care during the procedures.

54. Which information will the nurse teach a 23-year-old patient with lactose intolerance? a. Ice cream is relatively low in lactose. b. Live-culture yogurt is usually tolerated. c. Heating milk will break down the lactose. d. Nonfat milk is a better choice than whole milk.

ANS: B Lactose-intolerant individuals can usually eat yogurt without experiencing discomfort. Ice cream, nonfat milk, and milk that has been heated are all high in lactose.

Which information will the nurse teach a 23-year-old patient with lactose intolerance? a. Ice cream is relatively low in lactose. b. Live-culture yogurt is usually tolerated. c. Heating milk will break down the lactose. d. Nonfat milk is a better choice than whole milk.

ANS: B Lactose-intolerant individuals can usually eat yogurt without experiencing discomfort. Ice cream, nonfat milk, and milk that has been heated are all high in lactose.

Which question will the nurse in the endocrine clinic ask to help determine a patient's risk factors for goiter? a. "How much milk do you drink?" b. "What medications are you taking?" c. "Are your immunizations up to date?" d. "Have you had any recent neck injuries?"

ANS: B Medications that contain thyroid-inhibiting substances can cause goiter. Milk intake, neck injury, and immunization history are not risk factors for goiter

Which diet choice by the patient with an acute exacerbation of inflammatory bowel disease (IBD) indicates a need for more teaching? a. Scrambled eggs b. White toast and jam c. Oatmeal with cream d. Pancakes with syrup

ANS: C During acute exacerbations of IBD, the patient should avoid high-fiber foods such as whole grains. High-fat foods also may cause diarrhea in some patients. The other choices are low residue and would be appropriate for this patient.

A patient who had radical neck surgery to remove a malignant tumor developed hypoparathyroidism. The nurse should plan to teach the patient about a. bisphosphonates to reduce bone demineralization. b. calcium supplements to normalize serum calcium levels. c. increasing fluid intake to decrease risk for nephrolithiasis. d. including whole grains in the diet to prevent constipation.

ANS: B Oral calcium supplements are used to maintain the serum calcium in normal range and prevent the complications of hypocalcemia. Whole grain foods decrease calcium absorption and will not be recommended. Bisphosphonates will lower serum calcium levels further by preventing calcium from being reabsorbed from bone. Kidney stones are not a complication of hypoparathyroidism and low calcium levels

52. After change-of-shift report, which patient should the nurse assess first? a. 40-year-old male with celiac disease who has frequent frothy diarrhea b. 30-year-old female with a femoral hernia who has abdominal pain and vomiting c. 30-year-old male with ulcerative colitis who has severe perianal skin breakdown d. 40-year-old female with a colostomy bag that is pulling away from the adhesive wafer

ANS: B Pain and vomiting with a femoral hernia suggest possible strangulation, which will necessitate emergency surgery. The other patients have less urgent problems.

After change-of-shift report, which patient should the nurse assess first? a. 40-year-old male with celiac disease who has frequent frothy diarrhea b. 30-year-old female with a femoral hernia who has abdominal pain and vomiting c. 30-year-old male with ulcerative colitis who has severe perianal skin breakdown d. 40-year-old female with a colostomy bag that is pulling away from the adhesive wafer

ANS: B Pain and vomiting with a femoral hernia suggest possible strangulation, which will necessitate emergency surgery. The other patients have less urgent problems.

After the home health nurse teaches a patient with a neurogenic bladder how to use intermittent catheterization for bladder emptying, which patient statement indicates that the teaching has been effective? a. "I will use a sterile catheter and gloves for each time I self-catheterize." b. "I will clean the catheter carefully before and after each catheterization." c. "I will need to buy seven new catheters weekly and use a new one every day." d. "I will need to take prophylactic antibiotics to prevent any urinary tract infections."

ANS: B Patients who are at home can use a clean technique for intermittent self-catheterization and change the catheter every 7 days. There is no need to use a new catheter every day, to use sterile catheters, or to take prophylactic antibiotics. DIF: Cognitive Level: Application REF: 1154

50. A new 19-year-old male patient has familial adenomatous polyposis (FAP). Which action will the nurse in the gastrointestinal clinic include in the plan of care? a. Obtain blood samples for DNA analysis. b. Schedule the patient for yearly colonoscopy. c. Provide preoperative teaching about total colectomy. d. Discuss lifestyle modifications to decrease cancer risk.

ANS: B Patients with FAP should have annual colonoscopy starting at age 16 and usually have total colectomy by age 25 to avoid developing colorectal cancer. DNA analysis is used to make the diagnosis, but is not needed now for this patient. Lifestyle modifications will not decrease cancer risk for this patient.

A new 19-year-old male patient has familial adenomatous polyposis (FAP). Which action will the nurse in the gastrointestinal clinic include in the plan of care? a. Obtain blood samples for DNA analysis. b. Schedule the patient for yearly colonoscopy. c. Provide preoperative teaching about total colectomy. d. Discuss lifestyle modifications to decrease cancer risk.

ANS: B Patients with FAP should have annual colonoscopy starting at age 16 and usually have total colectomy by age 25 to avoid developing colorectal cancer. DNA analysis is used to make the diagnosis, but is not needed now for this patient. Lifestyle modifications will not decrease cancer risk for this patient.

55. Which prescribed intervention for a 61-year-old female patient with chronic short bowel syndrome will the nurse question? a. Ferrous sulfate (Feosol) 325 mg daily b. Senna (Senokot) 1 tablet every day c. Psyllium (Metamucil) 2.1 grams 3 times daily d. Diphenoxylate with atropine (Lomotil) prn loose stools

ANS: B Patients with short bowel syndrome have diarrhea because of decreased nutrient and fluid absorption and would not need stimulant laxatives. Iron supplements are used to prevent iron-deficiency anemia, bulk-forming laxatives help make stools less watery, and opioid antidiarrheal drugs are helpful in slowing intestinal transit time.

Which prescribed intervention for a 61-year-old female patient with chronic short bowel syndrome will the nurse question? a. Ferrous sulfate (Feosol) 325 mg daily b. Senna (Senokot) 1 tablet every day c. Psyllium (Metamucil) 2.1 grams 3 times daily d. Diphenoxylate with atropine (Lomotil) prn loose stools

ANS: B Patients with short bowel syndrome have diarrhea because of decreased nutrient and fluid absorption and would not need stimulant laxatives. Iron supplements are used to prevent iron-deficiency anemia, bulk-forming laxatives help make stools less watery, and opioid antidiarrheal drugs are helpful in slowing intestinal transit time.

13. The nurse caring for a patient after cystoscopy plans that the patient a. learns to request narcotics for pain. b. understands to expect blood-tinged urine. c. restricts activity to bed rest for a 4 to 6 hours. d. remains NPO for 8 hours to prevent vomiting.

ANS: B Pink-tinged urine and urinary frequency are expected after cystoscopy. Burning on urination is common, but pain that requires opioids for relief is not expected. A good fluid intake is encouraged after this procedure. Bed rest is not required following cystoscopy.

A 58-year-old man with blunt abdominal trauma from a motor vehicle crash undergoes peritoneal lavage. If the lavage returns brown fecal drainage, which action will the nurse plan to take next? a. Auscultate the bowel sounds. b. Prepare the patient for surgery. c. Check the patient's oral temperature. d. Obtain information about the accident.

ANS: B Return of brown drainage and fecal material suggests perforation of the bowel and the need for immediate surgery. Auscultation of bowel sounds, checking the temperature, and obtaining information about the accident are appropriate actions, but the priority is to prepare to send the patient for emergency surgery.

21. What has most likely occurred in a patient who has been diagnosed with endogenous hypoglycemia? a. Taken an overdose of hypoglycemic drugs b. Been following a very restricted fasting diet or is malnourished c. Excessive secretion of insulin or an increase in glucose metabolism d. Exercised unwittingly without replenishing needed fluids and nutrients

ANS: C Endogenous refers to within; in this patient, it refers to internal factors, such as an increase of insulin or glucose metabolism. Both conditions would lead to hypoglycemia.

A patient returns to the clinic with recurrent dysuria after being treated with trimethoprim and sulfamethoxazole (Bactrim) for 3 days. Which action will the nurse plan to take? a. Remind the patient about the need to drink 1000 mL of fluids daily. b. Obtain a midstream urine specimen for culture and sensitivity testing. c. Teach the patient to take the prescribed Bactrim for at least 3 more days. d. Suggest that the patient use acetaminophen (Tylenol) to treat the symptoms.

ANS: B Since uncomplicated urinary tract infections (UTIs) are usually successfully treated with 3 days of antibiotic therapy, this patient will need a urine culture and sensitivity to determine appropriate antibiotic therapy. Tylenol would not be as effective as other over-the-counter (OTC) medications such as phenazopyridine (Pyridium) in treating dysuria. The fluid intake should be increased to at least 1800 mL/day. Since the UTI has persisted after treatment with Bactrim, the patient is likely to need a different antibiotic. DIF: Cognitive Level: Application REF: 1123-1125

Following an open loop resection and fulguration of the bladder, a patient is unable to void. Which nursing action should be implemented first? a. Insert a straight catheter and drain the bladder. b. Assist the patient to take a 15-minute sitz bath. c. Encourage the patient to drink several glasses of water. d. Teach the patient how to do isometric perineal exercises.

ANS: B Sitz baths will relax the perineal muscles and promote voiding. Although the patient should be encouraged to drink fluids and Kegel exercises are helpful in the prevention of incontinence, these activities would not be helpful for a patient experiencing retention. Catheter insertion increases the risk for urinary tract infection (UTI) and should be avoided when possible DIF: Cognitive Level: Application REF: 1146

Which intervention will the nurse include in the plan of care for a 52-year-old male patient with syndrome of inappropriate antidiuretic hormone (SIADH)? a. Monitor for peripheral edema. b. Offer patient hard candies to suck on. c. Encourage fluids to 2 to 3 liters per day. d. Keep head of bed elevated to 30 degrees.

ANS: B Sucking on hard candies decreases thirst for a patient on fluid restriction. Patients with SIADH are on fluid restrictions of 800 to 1000 mL/day. Peripheral edema is not seen with SIADH. The head of the bed is elevated no more than 10 degrees to increase left atrial filling pressure and decrease antidiuretic hormone (ADH) release

11. Which patient statement indicates that the nurse's teaching about sulfasalazine (Azulfidine) for ulcerative colitis has been effective? a. "The medication will be tapered if I need surgery." b. "I will need to use a sunscreen when I am outdoors." c. "I will need to avoid contact with people who are sick." d. "The medication will prevent infections that cause the diarrhea."

ANS: B Sulfasalazine may cause photosensitivity in some patients. It is not used to treat infections. Sulfasalazine does not reduce immune function. Unlike corticosteroids, tapering of sulfasalazine is not needed.

A patient who has bladder cancer had a cystectomy with creation of an Indiana pouch. Which topic will be included in patient teaching? a. Application of ostomy appliances b. Catheterization technique and schedule c. Analgesic use before emptying the pouch d. Use of barrier products for skin protection

ANS: B The Indiana pouch enables the patient to self-catheterize every 4 to 6 hours. There is no need for an ostomy device or barrier products. Catheterization of the pouch is not painful. DIF: Cognitive Level: Application REF: 1155-1156

30. Which information about dietary management should the nurse include when teaching a patient with peptic ulcer disease (PUD)? a. "You will need to remain on a bland diet." b. "Avoid foods that cause pain after you eat them." c. "High-protein foods are least likely to cause you pain." d. "You should avoid eating any raw fruits and vegetables."

ANS: B The best information is that each individual should choose foods that are not associated with postprandial discomfort. Raw fruits and vegetables may irritate the gastric mucosa, but chewing well seems to decrease this problem and some patients may tolerate these foods well. High-protein foods help neutralize acid, but they also stimulate hydrochloric (HCl) acid secretion and may increase discomfort for some patients. Bland diets may be recommended during an acute exacerbation of PUD, but there is little scientific evidence to support their use.

The nurse will plan to monitor a patient diagnosed with a pheochromocytoma for a. flushing. b. headache. c. bradycardia. d. hypoglycemia.

ANS: B The classic clinical manifestations of pheochromocytoma are hypertension, tachycardia, severe headache, diaphoresis, and abdominal or chest pain. Elevated blood glucose may also occur because of sympathetic nervous system stimulation. Bradycardia and flushing would not be expected

17. A patient with peptic ulcer disease associated with the presence of Helicobacter pylori is treated with triple drug therapy. The nurse will plan to teach the patient about a. sucralfate (Carafate), nystatin (Mycostatin), and bismuth (Pepto-Bismol). b. amoxicillin (Amoxil), clarithromycin (Biaxin), and omeprazole (Prilosec). c. famotidine (Pepcid), magnesium hydroxide (Mylanta), and pantoprazole (Protonix). d. metoclopramide (Reglan), bethanechol (Urecholine), and promethazine (Phenergan).

ANS: B The drugs used in triple drug therapy include a proton pump inhibitor such as omeprazole and the antibiotics amoxicillin and clarithromycin. The other combinations listed are not included in the protocol for H. pylori infection.

36. A 25-year-old male patient calls the clinic complaining of diarrhea for 24 hours. Which action should the nurse take first? a. Inform the patient that laboratory testing of blood and stools will be necessary. b. Ask the patient to describe the character of the stools and any associated symptoms. c. Suggest that the patient drink clear liquid fluids with electrolytes, such as Gatorade or Pedialyte. d. Advise the patient to use over-the-counter loperamide (Imodium) to slow gastrointestinal (GI) motility.

ANS: B The initial response by the nurse should be further assessment of the patient. The other responses may be appropriate, depending on what is learned in the assessment.

A 25-year-old male patient calls the clinic complaining of diarrhea for 24 hours. Which action should the nurse take first? a. Inform the patient that laboratory testing of blood and stools will be necessary. b. Ask the patient to describe the character of the stools and any associated symptoms. c. Suggest that the patient drink clear liquid fluids with electrolytes, such as Gatorade or Pedialyte. d. Advise the patient to use over-the-counter loperamide (Imodium) to slow gastrointestinal (GI) motility.

ANS: B The initial response by the nurse should be further assessment of the patient. The other responses may be appropriate, depending on what is learned in the assessment.

24. The nurse will determine that teaching a 67-year-old man to irrigate his new colostomy has been effective if the patient a. inserts the irrigation tubing 4 to 6 inches into the stoma. b. hangs the irrigating container 18 inches above the stoma. c. stops the irrigation and removes the irrigating cone if cramping occurs. d. fills the irrigating container with 1000 to 2000 mL of lukewarm tap water.

ANS: B The irrigating container should be hung 18 to 24 inches above the stoma. If cramping occurs, the irrigation should be temporarily stopped and the cone left in place. Five hundred to 1000 mL of water should be used for irrigation. An irrigation cone, rather than tubing, should be inserted into the stoma; 4 to 6 inches would be too far for safe insertion.

The nurse will determine that teaching a 67-year-old man to irrigate his new colostomy has been effective if the patient a. inserts the irrigation tubing 4 to 6 inches into the stoma. b. hangs the irrigating container 18 inches above the stoma. c. stops the irrigation and removes the irrigating cone if cramping occurs. d. fills the irrigating container with 1000 to 2000 mL of lukewarm tap water.

ANS: B The irrigating container should be hung 18 to 24 inches above the stoma. If cramping occurs, the irrigation should be temporarily stopped and the cone left in place. Five hundred to 1000 mL of water should be used for irrigation. An irrigation cone, rather than tubing, should be inserted into the stoma; 4 to 6 inches would be too far for safe insertion.

7. The nurse completing a physical assessment for a newly admitted male patient is unable to feel either kidney on palpation. Which action should the nurse take next? a. Obtain a urine specimen to check for hematuria. b. Document the information on the assessment form. c. Ask the patient about any history of recent sore throat. d. Ask the health care provider about scheduling a renal ultrasound.

ANS: B The kidneys are protected by the abdominal organs, ribs, and muscles of the back, and may not be palpable under normal circumstances, so no action except to document the assessment information is needed. Asking about a recent sore throat, checking for hematuria, or obtaining a renal ultrasound may be appropriate when assessing for renal problems for some patients, but there is nothing in the question stem to indicate that they are appropriate for this patient.

A 37-year-old patient has just arrived in the postanesthesia recovery unit (PACU) after a thyroidectomy. Which information is most important to communicate to the surgeon? a. The patient reports 7/10 incisional pain. b. The patient has increasing neck swelling. c. The patient is sleepy and difficult to arouse. d. The patient's cardiac rate is 112 beats/minute.

ANS: B The neck swelling may lead to respiratory difficulty, and rapid intervention is needed to prevent airway obstruction. The incisional pain should be treated but is not unusual after surgery. A heart rate of 112 is not unusual in a patient who has been hyperthyroid and has just arrived in the PACU from surgery. Sleepiness in the immediate postoperative period is expected.

A 38-year-old male patient is admitted to the hospital in Addisonian crisis. Which patient statement supports a nursing diagnosis of ineffective self-health management related to lack of knowledge about management of Addison's disease? a. "I frequently eat at restaurants, and my food has a lot of added salt." b. "I had the stomach flu earlier this week, so I couldn't take the hydrocortisone." c. "I always double my dose of hydrocortisone on the days that I go for a long run." d. "I take twice as much hydrocortisone in the morning dose as I do in the afternoon."

ANS: B The need for hydrocortisone replacement is increased with stressors such as illness, and the patient needs to be taught to call the health care provider because medication and IV fluids and electrolytes may need to be given. The other patient statements indicate appropriate management of the Addison's disease.

15. A patient has come to the physician's office after finding out that her blood glucose level was 135 mg/dL. She states that she had not eaten before the test and was told to come and see her physician. She asks the nurse if she has diabetes. What is the most accurate nursing response? a. "Having a fasting serum glucose that high certainly indicates diabetes." b. "That test indicates that we need to perform more tests that are specific for diabetes." c. "How do you feel? Do you have any other signs of diabetes?" d. "Do you have a family history of diabetes, stroke, or heart disease? We need to know before making a diagnosis."

ANS: B The nurse needs to answer the patient's question in a way that gives information and is not misleading. Although 135 mg/dL is high, a nonpathologic explanation may be found. More tests should be performed to evaluate the patient.

2. A 71-year-old male patient tells the nurse that growing old causes constipation so he has been using a suppository for constipation every morning. Which action should the nurse take first? a. Encourage the patient to increase oral fluid intake. b. Assess the patient about risk factors for constipation. c. Suggest that the patient increase intake of high-fiber foods. d. Teach the patient that a daily bowel movement is unnecessary.

ANS: B The nurse's initial action should be further assessment of the patient for risk factors for constipation and for his usual bowel pattern. The other actions may be appropriate but will be based on the assessment.

24. A 44-year-old man admitted with a peptic ulcer has a nasogastric (NG) tube in place. When the patient develops sudden, severe upper abdominal pain, diaphoresis, and a firm abdomen, which action should the nurse take? a. Irrigate the NG tube. b. Check the vital signs. c. Give the ordered antacid. d. Elevate the foot of the bed.

ANS: B The patient's symptoms suggest acute perforation, and the nurse should assess for signs of hypovolemic shock. Irrigation of the NG tube, administration of antacids, or both would be contraindicated because any material in the stomach will increase the spillage into the peritoneal cavity. Elevating the foot of the bed may increase abdominal pressure and discomfort, as well as making it more difficult for the patient to breathe.

24. A patient with a peptic ulcer who has a nasogastric (NG) tube develops sudden, severe upper abdominal pain, diaphoresis, and a very firm abdomen. Which action should the nurse take next? a. Irrigate the NG tube. b. Obtain the vital signs. c. Listen for bowel sounds. d. Give the ordered antacid.

ANS: B The patient's symptoms suggest acute perforation, and the nurse should assess for signs of hypovolemic shock. Irrigation of the NG tube, administration of antacids, or both would be contraindicated because any material in the stomach will increase the spillage into the peritoneal cavity. The nurse should assess the bowel sounds, but this is not the first action that should be taken.

Two days after surgery for an ileal conduit, the patient will not look at the stoma or participate in care. The patient insists that no one but the ostomy nurse specialist care for the stoma. The nurse identifies a nursing diagnosis of a. anxiety related to effects of procedure on lifestyle. b. disturbed body image related to change in body function. c. readiness for enhanced coping related to need for information. d. self-care deficit, toileting, related to denial of altered body function.

ANS: B The patient's unwillingness to look at the stoma or participate in care indicates that disturbed body image is the best diagnosis. No data suggest that the impact on lifestyle is a concern for the patient. The patient does not appear to be ready for enhanced coping. The patient's insistence that only the ostomy nurse care for the stoma indicates that denial is not present. DIF: Cognitive Level: Application REF: 1157 | 1159-1160 | 1158-1159

10. The nurse assessing the urinary system of a 45-year-old female would use auscultation to a. determine kidney position. b. identify renal artery bruits. c. check for ureteral peristalsis. d. assess for bladder distention.

ANS: B The presence of a bruit may indicate problems such as renal artery tortuosity or abdominal aortic aneurysm. Auscultation would not be helpful in assessing for the other listed urinary tract information.

23. A 68-year-old patient with a bleeding duodenal ulcer has a nasogastric (NG) tube in place, and the health care provider orders 30 mL of aluminum hydroxide/magnesium hydroxide (Maalox) to be instilled through the tube every hour. To evaluate the effectiveness of this treatment, the nurse a. monitors arterial blood gas values daily. b. periodically aspirates and tests gastric pH. c. checks each stool for the presence of occult blood. d. measures the volume of residual stomach contents.

ANS: B The purpose for antacids is to increase gastric pH. Checking gastric pH is the most direct way of evaluating the effectiveness of the medication. Arterial blood gases may change slightly, but this does not directly reflect the effect of antacids on gastric pH. Because the patient has upper gastrointestinal (GI) bleeding, occult blood in the stools will appear even after the acute bleeding has stopped. The amount of residual stomach contents is not a reflection of resolution of bleeding or of gastric pH.

15. A female patient with a suspected urinary tract infection (UTI) is to provide a clean-catch urine specimen for culture and sensitivity testing. To obtain the specimen, the nurse will a. have the patient empty the bladder completely, then obtain the next urine specimen that the patient is able to void. b. teach the patient to clean the urethral area, void a small amount into the toilet, and then void into a sterile specimen cup. c. insert a short sterile "mini" catheter attached to a collecting container into the urethra and bladder to obtain the specimen. d. clean the area around the meatus with a povidone-iodine (Betadine) swab, and then have the patient void into a sterile container.

ANS: B This answer describes the technique for obtaining a clean-catch specimen. The answer beginning, "insert a short, small, 'mini' catheter attached to a collecting container" describes a technique that would result in a sterile specimen, but a health care provider's order for a catheterized specimen would be required. Using Betadine before obtaining the specimen is not necessary, and might result in suppressing the growth of some bacteria. The technique described in the answer beginning "have the patient empty the bladder completely" would not result in a sterile specimen.

14. A patient has come into the emergency department accompanied by a friend who states that the patient had been acting very strangely and seems confused. The friend states that the patient has diabetes and takes insulin. Which signs of hypoglycemia might the nurse assess? a. Slow pulse rate and low blood pressure b. Irritability, anxiety, confusion, and dizziness c. Flushing, anger, and forgetfulness d. Sleepiness, edema, and sluggishness

ANS: B When blood sugar levels fall, hormones are activated to increase serum glucose. One of the hormones is epinephrine, which causes these symptoms.

A patient who has been hospitalized for 2 days has been receiving normal saline IV at 100 ml/hr, has a nasogastric tube to low suction, and is NPO. Which assessment finding by the nurse is the priority to report to the health care provider? a. Serum sodium level of 138 mEq/L (138 mmol/L) b. Gradually decreasing level of consciousness (LOC) c. Oral temperature of 100.1° F with bibasilar lung crackles d. Weight gain of 2 pounds (1 kg) above the admission weight

ANS: B Gradually decreasing level of consciousness (LOC) The patient's history and change in LOC could be indicative of several fluid and electrolyte disturbances: extracellular fluid (ECF) excess, ECF deficit, hyponatremia, hypernatremia, hypokalemia, or metabolic alkalosis. Further diagnostic information will be ordered by the health care provider to determine the cause of the change in LOC and the appropriate interventions. The weight gain, elevated temperature, crackles, and serum sodium level also will be reported, but do not indicate a need for rapid action to avoid complications.

Which of these actions can the nurse who is caring for a critically ill patient with multiple intravenous (IV) lines delegate to an experienced LPN? a. Administer IV antibiotics through the implantable port. b. Monitor the IV sites for redness, swelling, or tenderness. c. Remove the patient's nontunneled subclavian central venous catheter. d. Adjust the flow rate of the 0.9% normal saline in the peripheral IV line.

ANS: B Monitor the IV sites for redness, swelling, or tenderness. An experienced LPN has the education, experience, and scope of practice to monitor IV sites for signs of infection. Administration of medications, adjustment of infusion rates, and removal of central catheters in critically ill patients require RN level education and scope of practice.

A patient with advanced lung cancer is admitted to the emergency department with urinary retention caused by renal calculi. Which of these laboratory values will require the most immediate action by the nurse? a. Arterial blood pH is 7.32. b. Serum calcium is 18 mEq/L. c. Serum potassium is 5.1 mEq/L. d. Arterial oxygen saturation is 91%.

ANS: B Serum calcium is 18 mEq/L. The serum calcium is well above the normal level and puts the patient at risk for cardiac dysrhythmias. The nurse should initiate cardiac monitoring and notify the health care provider. The potassium, oxygen saturation, and pH also are abnormal, and the nurse should notify the health care provider about these values as well, but they are not immediately life-threatening.

Which information noted by the nurse when caring for a patient with a bladder infection is most important to report to the health care provider? a. Dysuria b. Hematuria c. Left-sided flank pain d. Temperature 100.1° F

ANS: C Flank pain indicates that the patient may have developed pyelonephritis as a complication of the bladder infection. The other clinical manifestations are consistent with a lower urinary tract infection (UTI). DIF: Cognitive Level: Application REF: 1132-1133

The following data are obtained by the nurse when assessing a pregnant patient with eclampsia who is receiving IV magnesium sulfate. Which finding is most important to report to the health care provider immediately? a. The bibasilar breath sounds are decreased. b. The patellar and triceps reflexes are absent. c. The patient has been sleeping most of the day. d. The patient reports feeling "sick to my stomach."

ANS: B The patellar and triceps reflexes are absent. The loss of the deep tendon reflexes indicates that the patient's magnesium level may be reaching toxic levels. Nausea and lethargy also are side effects associated with magnesium elevation and should be reported, but they are not as significant as the loss of deep tendon reflexes. The decreased breath sounds suggest that the patient needs to cough and deep breathe to prevent atelectasis.

Which action will the nurse include in the plan of care for a patient who has a central venous access device (CVAD)? a. Avoid using friction when cleaning around the CVAD insertion site. b. Use the push-pause method to flush the CVAD after giving medications. c. Obtain an order from the health care provider to change CVAD dressing. d. Have the patient turn the head toward the CAVD during injection cap changes.

ANS: B Use the push-pause method to flush the CVAD after giving medications. The push-pause enhances the removal of debris from the CVAD lumen and decreases the risk for clotting. To decrease infection risk, friction should be used when cleaning the CVAD insertion site. The dressing should be changed whenever it becomes damp, loose, or visibly soiled and the patient should turn away from the CVAD during cap changes.

When the nurse is evaluating the fluid balance for a patient admitted for hypovolemia associated with multiple draining wounds, the most accurate assessment to include is a. skin turgor. b. daily weight. c. presence of edema. d. hourly urine output.

ANS: B daily weight. Daily weight is the most easily obtained and accurate means of assessing volume status. Skin turgor varies considerably with age. Considerable excess fluid volume may be present before fluid moves into the interstitial space and causes edema. Hourly urine outputs do not take account of fluid intake or of fluid loss through insensible loss, sweating, or loss from the gastrointestinal tract or wounds.

When caring for an alert and oriented elderly patient with a history of dehydration, the home health nurse will teach the patient to increase fluid intake a. in the late evening hours. b. if the oral mucosa feels dry. c. when the patient feels thirsty. d. as soon as changes in level of consciousness (LOC) occur.

ANS: B if the oral mucosa feels dry. An alert, elderly patient will be able to self-assess for signs of oral dryness such as thick oral secretions or dry-appearing mucosa. The thirst mechanism decreases with age and is not an accurate indicator of volume depletion. Many older patients prefer to restrict fluids slightly in the evening to improve sleep quality. The patient will not be likely to notice and act appropriately when changes in LOC occur.

30. A nurse instructs a patient about how insulin affects blood glucose. (Arrange the events in sequence. Separate letters by a comma and space as follows: A, B, C, D.) A. Beta cells are stimulated to release insulin. B. Glucose enters the bloodstream. C. Glycogen is converted to glucose by alpha cells (glycogenesis). D. Glycogen is stored in the liver. E. Insulin transports glucose to muscle cells.

ANS: B, A, E, D, C Insulin transports the glucose to muscle cells or converts it to glycogen, which is stored in the liver to be accessed when hypoglycemia occurs.

19. A patient with type 1 diabetes asks why his 0700 insulin has been changed from NPH insulin to 70/30 premixed insulin. What is the best explanation by the nurse that explains about 70/30 insulin mixture? a. It is absorbed more rapidly into the bloodstream. b. It has no peak action time and lasts all day. c. It makes insulin administration easier and safer. d. It provides a bolus of rapid-acting insulin to prevent hyperglycemia after breakfast.

ANS: C 70/30 insulin is 30% rapid-acting insulin and 70% intermediate-acting insulin. The rapid action of the 0700 premixed insulin prevents hyperglycemia after the morning meal and the mixed drug reduces the risk of error in drawing up two insulins.

19. The nurse is providing preoperative teaching for a 61-year-old man scheduled for an abdominal-perineal resection. Which information will the nurse include? a. Another surgery in 8 to 12 weeks will be used to create an ileal-anal reservoir. b. The patient will begin sitting in a chair at the bedside on the first postoperative day. c. The patient will drink polyethylene glycol lavage solution (GoLYTELY) preoperatively. d. IV antibiotics will be started at least 24 hours before surgery to reduce the bowel bacteria.

ANS: C A bowel-cleansing agent is used to empty the bowel before surgery to reduce the risk for infection. A permanent colostomy is created with this surgery. Sitting is contraindicated after an abdominal-perineal resection. Oral antibiotics (rather than IV antibiotics) are given to reduce colonic and rectal bacteria.

The nurse is providing preoperative teaching for a 61-year-old man scheduled for an abdominal-perineal resection. Which information will the nurse include? a. Another surgery in 8 to 12 weeks will be used to create an ileal-anal reservoir. b. The patient will begin sitting in a chair at the bedside on the first postoperative day. c. The patient will drink polyethylene glycol lavage solution (GoLYTELY) preoperatively. d. IV antibiotics will be started at least 24 hours before surgery to reduce the bowel bacteria.

ANS: C A bowel-cleansing agent is used to empty the bowel before surgery to reduce the risk for infection. A permanent colostomy is created with this surgery. Sitting is contraindicated after an abdominal-perineal resection. Oral antibiotics (rather than IV antibiotics) are given to reduce colonic and rectal bacteria.

4. A 26-year-old woman is being evaluated for vomiting and abdominal pain. Which question from the nurse will be most useful in determining the cause of the patient's symptoms? a. "What type of foods do you eat?" b. "Is it possible that you are pregnant?" c. "Can you tell me more about the pain?" d. "What is your usual elimination pattern?"

ANS: C A complete description of the pain provides clues about the cause of the problem. Although the nurse should ask whether the patient is pregnant to determine whether the patient might have an ectopic pregnancy and before any radiology studies are done, this information is not the most useful in determining the cause of the pain. The usual diet and elimination patterns are less helpful in determining the reason for the patient's symptoms.

A 26-year-old woman is being evaluated for vomiting and abdominal pain. Which question from the nurse will be most useful in determining the cause of the patient's symptoms? a. "What type of foods do you eat?" b. "Is it possible that you are pregnant?" c. "Can you tell me more about the pain?" d. "What is your usual elimination pattern?"

ANS: C A complete description of the pain provides clues about the cause of the problem. Although the nurse should ask whether the patient is pregnant to determine whether the patient might have an ectopic pregnancy and before any radiology studies are done, this information is not the most useful in determining the cause of the pain. The usual diet and elimination patterns are less helpful in determining the reason for the patient's symptoms.

18. A male patient in the clinic provides a urine sample that is red-orange in color. Which action should the nurse take first? a. Notify the patient's health care provider. b. Teach correct midstream urine collection. c. Ask the patient about current medications. d. Question the patient about urinary tract infection (UTI) risk factors.

ANS: C A red-orange color in the urine is normal with some over-the-counter (OTC) medications such as phenazopyridine (Pyridium). The color would not be expected with urinary tract infection, is not a sign that poor technique was used in obtaining the specimen, and does not need to be communicated to the health care provider until further assessment is done.

A 29-year-old woman with systemic lupus erythematosus has been prescribed 2 weeks of high-dose prednisone therapy. Which information about the prednisone is most important for the nurse to include? a. "Weigh yourself daily to monitor for weight gain caused by increased appetite." b. "A weight-bearing exercise program will help minimize the risk for osteoporosis." c. "The prednisone dose should be decreased gradually rather than stopped suddenly." d. "Call the health care provider if you experience mood alterations with the prednisone."

ANS: C Acute adrenal insufficiency may occur if exogenous corticosteroids are suddenly stopped. Mood alterations and weight gain are possible adverse effects of corticosteroid use, but these are not life-threatening effects. Osteoporosis occurs when patients take corticosteroids for longer periods.

The nurse is caring for a patient following an adrenalectomy. The highest priority in the immediate postoperative period is to a. protect the patient's skin. b. monitor for signs of infection. c. balance fluids and electrolytes. d. prevent emotional disturbances.

ANS: C After adrenalectomy, the patient is at risk for circulatory instability caused by fluctuating hormone levels, and the focus of care is to assess and maintain fluid and electrolyte status through the use of IV fluids and corticosteroids. The other goals are also important for the patient but are not as immediately life threatening as the circulatory collapse that can occur with fluid and electrolyte disturbances

A patient has just arrived on the unit after a thyroidectomy. Which action should the nurse take first? a. Observe the dressing for bleeding. b. Check the blood pressure and pulse. c. Assess the patient's respiratory effort. d. Support the patient's head with pillows.

ANS: C Airway obstruction is a possible complication after thyroidectomy because of swelling or bleeding at the site or tetany. The priority nursing action is to assess the airway. The other actions are also part of the standard nursing care postthyroidectomy but are not as high of a priority

After a 22-year-old female patient with a pituitary adenoma has had a hypophysectomy, the nurse will teach about the need for a. sodium restriction to prevent fluid retention. b. insulin to maintain normal blood glucose levels. c. oral corticosteroids to replace endogenous cortisol. d. chemotherapy to prevent malignant tumor recurrence.

ANS: C Antidiuretic hormone (ADH), cortisol, and thyroid hormone replacement will be needed for life after hypophysectomy. Without the effects of adrenocorticotropic hormone (ACTH) and cortisol, the blood glucose and serum sodium will be low unless cortisol is replaced. An adenoma is a benign tumor, and chemotherapy will not be needed

1. Which action will the nurse include in the plan of care for a 42-year-old patient who is being admitted with Clostridium difficile? a. Educate the patient about proper food storage. b. Order a diet with no dairy products for the patient. c. Place the patient in a private room on contact isolation. d. Teach the patient about why antibiotics will not be used.

ANS: C Because C. difficile is highly contagious, the patient should be placed in a private room and contact precautions should be used. There is no need to restrict dairy products for this type of diarrhea. Metronidazole (Flagyl) is frequently used to treat C. difficile. Improper food handling and storage do not cause C. difficile.

7. A 68-year-old male patient with a stroke is unconscious and unresponsive to stimuli. After learning that the patient has a history of gastroesophageal reflux disease (GERD), the nurse will plan to do frequent assessments of the patient's a. apical pulse. b. bowel sounds. c. breath sounds. d. abdominal girth.

ANS: C Because GERD may cause aspiration, the unconscious patient is at risk for developing aspiration pneumonia. Bowel sounds, abdominal girth, and apical pulse will not be affected by the patient's stroke or GERD and do not require more frequent monitoring than the routine.

7. When admitting a patient with a stroke who is unconscious and unresponsive to stimuli, the nurse learns from the patient's family that the patient has a history of gastroesophageal reflux disease (GERD). The nurse will plan to do frequent assessments of the patient's a. apical pulse. b. bowel sounds. c. breath sounds. d. abdominal girth.

ANS: C Because GERD may cause aspiration, the unconscious patient is at risk for developing aspiration pneumonia. Bowel sounds, abdominal girth, and apical pulse will not be affected by the patient's stroke or GERD and do not require more frequent monitoring than the routine.

3. A hospitalized patient with possible renal insufficiency after coronary artery bypass surgery is scheduled for a creatinine clearance test. Which equipment will the nurse need to obtain? a. Urinary catheter b. Cleaning towelettes c. Large container for urine d. Sterile urine specimen cup

ANS: C Because creatinine clearance testing involves a 24-hour urine specimen, the nurse should obtain a large container for the urine collection. Catheterization, cleaning of the perineum with antiseptic towelettes, and a sterile specimen cup are not needed for this test.

1. A patient with deep partial-thickness burns experiences severe pain associated with nausea during dressing changes. Which action will be most useful in decreasing the patient's nausea? a. The patient NPO for 2 hours before and after dressing changes. b. Avoid performing dressing changes close to the patient's mealtimes. c. Administer the prescribed morphine sulfate before dressing changes. d. Give the ordered prochlorperazine (Compazine) before dressing changes.

ANS: C Because the patient's nausea is associated with severe pain, it is likely that it is precipitated by stress and pain. The best treatment will be to provide adequate pain medication before dressing changes. The nurse should avoid doing painful procedures close to mealtimes, but nausea/vomiting that occur at other times also should be addressed. Keeping the patient NPO does not address the reason for the nausea and vomiting and will have an adverse effect on the patient's nutrition. Administration of antiemetics is not the best choice for a patient with nausea caused by pain.

21. A 71-year-old patient had an abdominal-perineal resection for colon cancer. Which nursing action is most important to include in the plan of care for the day after surgery? a. Teach about a low-residue diet. b. Monitor output from the stoma. c. Assess the perineal drainage and incision. d. Encourage acceptance of the colostomy stoma.

ANS: C Because the perineal wound is at high risk for infection, the initial care is focused on assessment and care of this wound. Teaching about diet is best done closer to discharge from the hospital. There will be very little drainage into the colostomy until peristalsis returns. The patient will be encouraged to assist with the colostomy, but this is not the highest priority in the immediate postoperative period.

A 71-year-old patient had an abdominal-perineal resection for colon cancer. Which nursing action is most important to include in the plan of care for the day after surgery? a. Teach about a low-residue diet. b. Monitor output from the stoma. c. Assess the perineal drainage and incision. d. Encourage acceptance of the colostomy stoma.

ANS: C Because the perineal wound is at high risk for infection, the initial care is focused on assessment and care of this wound. Teaching about diet is best done closer to discharge from the hospital. There will be very little drainage into the colostomy until peristalsis returns. The patient will be encouraged to assist with the colostomy, but this is not the highest priority in the immediate postoperative period.

A patient undergoes a nephrectomy after having massive trauma to the kidney. Which assessment finding obtained postoperatively is most important to communicate to the surgeon? a. Blood pressure is 102/58. b. Incisional pain level is 8/10. c. Urine output is 20 mL/hr for 2 hours. d. Crackles are heard at both lung bases.

ANS: C Because the urine output should be at least 0.5 mL/kg/hr, a 40 mL output for 2 hours indicates that the patient may have decreased renal perfusion because of bleeding, inadequate fluid intake, or obstruction at the suture site. The blood pressure requires ongoing monitoring but does not indicate inadequate perfusion at this time. The patient should cough and deep breathe, but the crackles do not indicate a need for an immediate change in therapy. The incisional pain should be addressed, but this is not as potentially life threatening as decreased renal perfusion. In addition, the nurse can medicate the patient for pain. DIF: Cognitive Level: Application REF: 1154-1155

A 62-year-old patient has had a hemorrhoidectomy at an outpatient surgical center. Which instructions will the nurse include in discharge teaching? a. Maintain a low-residue diet until the surgical area is healed. b. Use ice packs on the perianal area to relieve pain and swelling. c. Take prescribed pain medications before a bowel movement is expected. d. Delay having a bowel movement for several days until healing has occurred.

ANS: C Bowel movements may be very painful, and patients may avoid defecation unless pain medication is taken before the bowel movement. A high-residue diet will increase stool bulk and prevent constipation. Delay of bowel movements is likely to lead to constipation. Warm sitz baths rather than ice packs are used to relieve pain and keep the surgical area clean

29. A 62-year-old patient has had a hemorrhoidectomy at an outpatient surgical center. Which instructions will the nurse include in discharge teaching? a. Maintain a low-residue diet until the surgical area is healed. b. Use ice packs on the perianal area to relieve pain and swelling. c. Take prescribed pain medications before a bowel movement is expected. d. Delay having a bowel movement for several days until healing has occurred.

ANS: C Bowel movements may be very painful, and patients may avoid defecation unless pain medication is taken before the bowel movement. A high-residue diet will increase stool bulk and prevent constipation. Delay of bowel movements is likely to lead to constipation. Warm sitz baths rather than ice packs are used to relieve pain and keep the surgical area clean.

26. The nurse implements discharge teaching for a patient following a gastroduodenostomy for treatment of a peptic ulcer. Which patient statement indicates that the teaching has been effective? a. "Persistent heartburn is expected after surgery." b. "I will try to drink liquids along with my meals." c. "Vitamin supplements may be needed to prevent problems with anemia." d. "I will need to choose foods that are low in fat and high in carbohydrate."

ANS: C Cobalamin deficiency may occur after partial gastrectomy, and the patient may need to receive cobalamin via injections or nasal spray. Foods that have moderate fat and low carbohydrate should be chosen to prevent dumping syndrome. Ingestion of liquids with meals is avoided to prevent dumping syndrome. Although peptic ulcer disease may recur, persistent heartburn is not expected after surgery and the patient should call the health care provider if this occurs.

40. Which activity in the care of a 48-year-old female patient with a new colostomy could the nurse delegate to unlicensed assistive personnel (UAP)? a. Document the appearance of the stoma. b. Place a pouching system over the ostomy. c. Drain and measure the output from the ostomy. d. Check the skin around the stoma for breakdown.

ANS: C Draining and measuring the output from the ostomy is included in UAP education and scope of practice. The other actions should be implemented by LPNs or RNs.

Which activity in the care of a 48-year-old female patient with a new colostomy could the nurse delegate to unlicensed assistive personnel (UAP)? a. Document the appearance of the stoma. b. Place a pouching system over the ostomy. c. Drain and measure the output from the ostomy. d. Check the skin around the stoma for breakdown.

ANS: C Draining and measuring the output from the ostomy is included in UAP education and scope of practice. The other actions should be implemented by LPNs or RNs.

6. A 46-year-old female with gastroesophageal reflux disease (GERD) is experiencing increasing discomfort. Which patient statement indicates that additional teaching about GERD is needed? a. "I take antacids between meals and at bedtime each night." b. "I sleep with the head of the bed elevated on 4-inch blocks." c. "I eat small meals during the day and have a bedtime snack." d. "I quit smoking several years ago, but I still chew a lot of gum."

ANS: C GERD is exacerbated by eating late at night, and the nurse should plan to teach the patient to avoid eating at bedtime. The other patient actions are appropriate to control symptoms of GERD.

9. Which patient choice for a snack 2 hours before bedtime indicates that the nurse's teaching about gastroesophageal reflux disease (GERD) has been effective? a. Chocolate pudding b. Glass of low-fat milk c. Cherry gelatin with fruit d. Peanut butter and jelly sandwich

ANS: C Gelatin and fruit are low fat and will not decrease lower esophageal sphincter (LES) pressure. Foods such as chocolate are avoided because they lower LES pressure. Milk products increase gastric acid secretion. High-fat foods such as peanut butter decrease both gastric emptying and LES pressure.

12. As part of a teaching plan in preparation for discharge, a patient with type 1 diabetes needs guidelines for exercise. Which guideline should be included? a. Plan exercise so that it coincides with the peak action of insulin. b. Insulin should be injected into the lower extremity before exercise because that site provides the greatest absorption. c. Exercise should be performed daily at the same time of day and at the same intensity. d. Keep exercise at a minimum to conserve your energy.

ANS: C If the body is using more glucose than available, the body will draw on fatty acids, which will give off ketones.

12. A patient passing bloody urine is scheduled for a cystoscopy with cystogram. Which description of the procedure by the nurse is accurate? a. "Your doctor will place a catheter into an artery in your groin and inject a dye that will visualize the blood supply to the kidneys." b. "Your doctor will insert a lighted tube into the bladder, and little catheters will be inserted through the tube into your kidney." c. "Your doctor will insert a lighted tube into the bladder through your urethra, inspect the bladder, and instill a dye that will outline your bladder on x-ray." d. "Your doctor will inject a radioactive solution into a vein in your arm and the distribution of the isotope in your kidneys and bladder will be checked."

ANS: C In a cystoscope and cystogram procedure, a cystoscope is inserted into the bladder for direct visualization, and then contrast solution is injected through the scope so that x-rays can be taken. The response beginning, "Your doctor will place a catheter" describes a renal arteriogram procedure. The response beginning, "Your doctor will inject a radioactive solution" describes a nuclear scan. The response beginning, "Your doctor will insert a lighted tube into the bladder, and little catheters will be inserted" describes a retrograde pyelogram.

Four hours after a bowel resection, a 74-year-old male patient with a nasogastric tube to suction complains of nausea and abdominal distention. The first action by the nurse should be to a. auscultate for hypotonic bowel sounds. b. notify the patient's health care provider. c. reposition the tube and check for placement. d. remove the tube and replace it with a new one.

ANS: C Repositioning the tube will frequently facilitate drainage. Because this is a common occurrence, it is not appropriate to notify the health care provider unless other interventions do not resolve the problem. Information about the presence or absence of bowel sounds will not be helpful in improving drainage. Removing the tube and replacing it are unnecessarily traumatic to the patient, so that would only be done if the tube was completely occluded.

Which finding by the nurse for a patient admitted with glomerulonephritis indicates that treatment has been effective? a. The patient denies pain with voiding. b. The urine dipstick is negative for nitrites. c. Peripheral and periorbital edema is resolved. d. The antistreptolysin-O (ASO) titer is decreased.

ANS: C Since edema is a common clinical manifestation of glomerulonephritis, resolution of the edema indicates that the prescribed therapies have been effective. Antibodies to streptococcus will persist after a streptococcal infection. Nitrites will be negative and the patient will not experience dysuria since the patient does not have a urinary tract infection. DIF: Cognitive Level: Application REF: 1131-1133

29. The health care provider prescribes antacids and sucralfate (Carafate) for treatment of a patient's peptic ulcer. The nurse will teach the patient to take a. antacids 30 minutes before the sucralfate. b. sucralfate at bedtime and antacids before meals. c. antacids after eating and sucralfate 30 minutes before eating. d. sucralfate and antacids together 30 minutes before each meal.

ANS: C Sucralfate is most effective when the pH is low and should not be given with or soon after antacid. Antacids are most effective when taken after eating. Administration of sucralfate 30 minutes before eating and antacids just after eating will ensure that both drugs can be most effective. The other regimens will decrease the effectiveness of the medications.

Which information obtained by the nurse in the endocrine clinic about a patient who has been taking prednisone (Deltasone) 40 mg daily for 3 weeks is most important to report to the health care provider? a. Patient's blood pressure is 148/94 mm Hg. b. Patient has bilateral 2+ pitting ankle edema. c. Patient stopped taking the medication 2 days ago. d. Patient has not been taking the prescribed vitamin D.

ANS: C Sudden cessation of corticosteroids after taking the medication for a week or more can lead to adrenal insufficiency, with problems such as severe hypotension and hypoglycemia. The patient will need immediate evaluation by the health care provider to prevent and/or treat adrenal insufficiency. The other information will also be reported, but does not require rapid treatment

3. A patient tells a nurse that she eats "huge" amounts of food but stays hungry most of the time. What should the nurse explain as the cause of hunger experienced by persons with type 1 diabetes? a. Excess amount of glucose b. Need for additional calories to correct the increased metabolism c. Fact that the cells cannot use the blood glucose d. Need for exercise to stimulate insulin secretion

ANS: C The cells cannot use the glucose without insulin, so the patient with diabetes still feels hungry event though abundant glucose is circulating in the blood.

Which assessment finding of a 42-year-old patient who had a bilateral adrenalectomy requires the most rapid action by the nurse? a. The blood glucose is 176 mg/dL. b. The lungs have bibasilar crackles. c. The blood pressure (BP) is 88/50 mm Hg. d. The patient reports 5/10 incisional pain.

ANS: C The decreased BP indicates possible adrenal insufficiency. The nurse should immediately notify the health care provider so that corticosteroid medications can be administered. The nurse should also address the elevated glucose, incisional pain, and crackles with appropriate collaborative or nursing actions, but prevention and treatment of acute adrenal insufficiency is the priority after adrenalectomy.

5. The self-care goal of a patient with diabetes is to keep the blood sugar within normal limits. What causes hyperglycemia to occur? a. Blood glucose levels rise, stimulating the production of insulin. b. Insulin conversion of glycogen to glucose is inhibited. c. The body responds to glucose-starved tissues by changing stored glycogen into glucose. d. Glycogen is unable to be stored in the liver and muscles.

ANS: C The hypothalamus is receiving a message that the cells need glucose, so it responds by adding more glucose to the already overburdened blood.

4. What does the lack of insulin in patients with type 1 diabetes cause that increases the risk for cardiovascular disorders? a. High glucose levels that irritate and shrink the vessels b. Inadequate metabolism of proteins, which causes ketosis c. Increased fatty acid levels d. Increased metabolism of ketones, which causes hypertension

ANS: C The increase in fatty acid levels causes an increase in the level of triglycerides and an attendant rise in low-density lipoprotein levels.

20. Which information from a patient's urinalysis requires that the nurse notify the health care provider? a. pH 6.2 b. Trace protein c. WBC 20 to 26/hpf d. Specific gravity 1.021

ANS: C The increased number of white blood cells (WBCs) indicates the presence of urinary tract infection or inflammation. The other findings are normal.

22. Which assessment of a 62-year-old patient who has just had an intravenous pyelogram (IVP) requires immediate action by the nurse? a. The heart rate is 58 beats/minute. b. The patient complains of a dry mouth. c. The respiratory rate is 38 breaths/minute. d. The urine output is 400 mL after 2 hours.

ANS: C The increased respiratory rate indicates that the patient may be experiencing an allergic reaction to the contrast medium used during the procedure. The nurse should immediately assess the patient's oxygen saturation and breath sounds. The other data are not unusual findings following an IVP.

47. A 76-year-old patient with obstipation has a fecal impaction and is incontinent of liquid stool. Which action should the nurse take first? a. Administer bulk-forming laxatives. b. Assist the patient to sit on the toilet. c. Manually remove the impacted stool. d. Increase the patient's oral fluid intake.

ANS: C The initial action with a fecal impaction is manual disimpaction. The other actions will be used to prevent future constipation and impactions.

A 76-year-old patient with obstipation has a fecal impaction and is incontinent of liquid stool. Which action should the nurse take first? a. Administer bulk-forming laxatives. b. Assist the patient to sit on the toilet. c. Manually remove the impacted stool. d. Increase the patient's oral fluid intake.

ANS: C The initial action with a fecal impaction is manual disimpaction. The other actions will be used to prevent future constipation and impactions.

8. A patient with type 2 diabetes shows a blood sugar reading of 68 at 6 AM. What action should the nurse implement based on the reading of 72 mg/dL? a. Notify the charge nurse of the reading. b. Give regular insulin per a sliding scale. c. Give him 8 oz of skim milk. d. Administer the oral glucose tablet.

ANS: C The patient is hypoglycemic and needs an immediate source of glucose, such as milk or orange juice. The oral hypoglycemic agent will not work quickly enough. The charge nurse can be notified later. Giving insulin per a sliding scale would lower the blood sugar level.

A patient is admitted to the hospital with new onset nephrotic syndrome. Which assessment data will the nurse expect to find related to this illness? a. Poor skin turgor b. High urine ketones c. Recent weight gain d. Low blood pressure

ANS: C The patient with a nephrotic syndrome will have weight gain associated with edema. Hypertension is a clinical manifestation of nephrotic syndrome. Skin turgor is normal because of the edema. Urine protein is high. DIF: Cognitive Level: Comprehension REF: 1132-1134

When assessing a 30-year-old man who complains of a feeling of incomplete bladder emptying and a split, spraying urine stream, the nurse asks about a history of a. bladder infection. b. recent kidney trauma. c. gonococcal urethritis. d. benign prostatic hyperplasia.

ANS: C The patient's clinical manifestations are consistent with urethral strictures, a possible complication of gonococcal urethritis. These symptoms are not consistent with benign prostatic hyperplasia, kidney trauma, or bladder infection. DIF: Cognitive Level: Application REF: 1141

41. A patient with acute gastrointestinal (GI) bleeding is receiving normal saline IV at a rate of 500 mL/hr. Which assessment finding obtained by the nurse is most important to communicate immediately to the health care provider? a. The patient's blood pressure (BP) has increased to 142/94 mm Hg. b. The nasogastric (NG) suction is returning coffee-ground material. c. The patient's lungs have crackles audible to the midline. d. The bowel sounds are very hyperactive in all four quadrants.

ANS: C The patient's lung sounds indicate that pulmonary edema may be developing as a result of the rapid infusion of IV fluid and that the fluid infusion rate should be slowed. The return of coffee-ground material in an NG tube is expected for a patient with upper GI bleeding. The BP is slightly elevated but would not be an indication to contact the health care provider immediately. Hyperactive bowel sounds are common when a patient has GI bleeding.

A patient develops carpopedal spasms and tingling of the lips following a parathyroidectomy. Which action should the nurse take first? a. Administer the ordered muscle relaxant. b. Give the ordered oral calcium supplement. c. Have the patient rebreathe from a paper bag. d. Start the PRN oxygen at 2 L/min per cannula.

ANS: C The patient's symptoms suggest mild hypocalcemia. The symptoms of hypocalcemia will be temporarily reduced by having the patient breathe into a paper bag, which will raise the PaCO2 and create a more acidic pH. The muscle relaxant will have no impact on the ionized calcium level. Although severe hypocalcemia can cause laryngeal stridor, there is no indication that this patient is experiencing laryngeal stridor or needs oxygen. Calcium supplements will be given to normalize calcium levels quickly, but oral supplements will take time to be absorbed

Which assessment finding for a 33-year-old female patient admitted with Graves' disease requires the most rapid intervention by the nurse? a. Bilateral exophthalmos b. Heart rate 136 beats/minute c. Temperature 103.8° F (40.4° C) d. Blood pressure 166/100 mm Hg

ANS: C The patient's temperature indicates that the patient may have thyrotoxic crisis and that interventions to lower the temperature are needed immediately. The other findings also require intervention but do not indicate potentially life-threatening complications

49. A 72-year-old male patient with dehydration caused by an exacerbation of ulcerative colitis is receiving 5% dextrose in normal saline at 125 mL/hour. Which assessment finding by the nurse is most important to report to the health care provider? a. Patient has not voided for the last 4 hours. b. Skin is dry with poor turgor on all extremities. c. Crackles are heard halfway up the posterior chest. d. Patient has had 5 loose stools over the last 6 hours.

ANS: C The presence of crackles in an older patient receiving IV fluids at a high rate suggests volume overload and a need to reduce the rate of the IV infusion. The other data will also be reported, but are consistent with the patient's age and diagnosis and do not require a change in the prescribed treatment.

A 72-year-old male patient with dehydration caused by an exacerbation of ulcerative colitis is receiving 5% dextrose in normal saline at 125 mL/hour. Which assessment finding by the nurse is most important to report to the health care provider? a. Patient has not voided for the last 4 hours. b. Skin is dry with poor turgor on all extremities. c. Crackles are heard halfway up the posterior chest. d. Patient has had 5 loose stools over the last 6 hours.

ANS: C The presence of crackles in an older patient receiving IV fluids at a high rate suggests volume overload and a need to reduce the rate of the IV infusion. The other data will also be reported, but are consistent with the patient's age and diagnosis and do not require a change in the prescribed treatment.

11. A 62-year-old patient who has been diagnosed with esophageal cancer tells the nurse, "I know that my chances are not very good, but I do not feel ready to die yet." Which response by the nurse is most appropriate? a. "You may have quite a few years still left to live." b. "Thinking about dying will only make you feel worse." c. "Having this new diagnosis must be very hard for you." d. "It is important that you be realistic about your prognosis."

ANS: C This response is open-ended and will encourage the patient to further discuss feelings of anxiety or sadness about the diagnosis. Patients with esophageal cancer have only a low survival rate, so the response "You may have quite a few years still left to live" is misleading. The response beginning, "Thinking about dying" indicates that the nurse is not open to discussing the patient's fears of dying. And the response beginning, "It is important that you be realistic," discourages the patient from feeling hopeful, which is important to patients with any life-threatening diagnosis.

1. A nurse explains that type 1 diabetes mellitus is a disease in which the body does not produce enough insulin. What is the reason that the blood glucose is elevated? a. Prolonged elevation of stress hormone (cortisol, epinephrine, glucagon, growth hormone) levels b. Malfunction of the glycogen-storing capabilities of the liver c. Destruction of the beta cells in the pancreas d. Insulin resistance of the receptor cells in the muscle tissue

ANS: C Type 1 diabetes mellitus is a disease in which the pancreas does not produce adequate insulin because of the destruction of beta cells.

42. Which care activity for a patient with a paralytic ileus is appropriate for the registered nurse (RN) to delegate to unlicensed assistive personnel (UAP)? a. Auscultation for bowel sounds b. Nasogastric (NG) tube irrigation c. Applying petroleum jelly to the lips d. Assessment of the nares for irritation

ANS: C UAP education and scope of practice include patient hygiene such as oral care. The other actions require education and scope of practice appropriate to the RN.

Which care activity for a patient with a paralytic ileus is appropriate for the registered nurse (RN) to delegate to unlicensed assistive personnel (UAP)? a. Auscultation for bowel sounds b. Nasogastric (NG) tube irrigation c. Applying petroleum jelly to the lips d. Assessment of the nares for irritation

ANS: C UAP education and scope of practice include patient hygiene such as oral care. The other actions require education and scope of practice appropriate to the RN.

12. A 22-year-old female patient with an exacerbation of ulcerative colitis is having 15 to 20 stools daily and has excoriated perianal skin. Which patient behavior indicates that teaching regarding maintenance of skin integrity has been effective? a. The patient uses incontinence briefs to contain loose stools. b. The patient asks for antidiarrheal medication after each stool. c. The patient uses witch hazel compresses to decrease irritation. d. The patient cleans the perianal area with soap after each stool.

ANS: C Witch hazel compresses are suggested to reduce anal irritation and discomfort. Incontinence briefs may trap diarrhea and increase the incidence of skin breakdown. Antidiarrheal medications are not given 15 to 20 times a day. The perianal area should be washed with plain water after each stool.

A 22-year-old female patient with an exacerbation of ulcerative colitis is having 15 to 20 stools daily and has excoriated perianal skin. Which patient behavior indicates that teaching regarding maintenance of skin integrity has been effective? a. The patient uses incontinence briefs to contain loose stools. b. The patient asks for antidiarrheal medication after each stool. c. The patient uses witch hazel compresses to decrease irritation. d. The patient cleans the perianal area with soap after each stool.

ANS: C Witch hazel compresses are suggested to reduce anal irritation and discomfort. Incontinence briefs may trap diarrhea and increase the incidence of skin breakdown. Antidiarrheal medications are not given 15 to 20 times a day. The perianal area should be washed with plain water after each stool.

An 82-year-old patient in a long-term care facility has several medications prescribed. After the patient is newly diagnosed with hypothyroidism, the nurse will need to consult with the health care provider before administering a. docusate (Colace). b. ibuprofen (Motrin). c. diazepam (Valium). d. cefoxitin (Mefoxin).

ANS: C Worsening of mental status and myxedema coma can be precipitated by the use of sedatives, especially in older adults. The nurse should discuss the use of diazepam with the health care provider before administration. The other medications may be given safely to the patient

A patient receiving isoosmolar continuous tube feedings develops restlessness, agitation, and weakness. Which laboratory result is most important to report to the health care provider? a. K+ 3.4 mEq/L (3.4 mmol/L) b. Ca+2 7.8 mg/dl (1.95 mmol/L) c. Na+ 154 mEq/L (154 mmol/L) d. PO4-3 4.8 mg/dl (1.55 mmol/L)

ANS: C Na+ 154 mEq/L (154 mmol/L) The elevated serum sodium level is consistent with the patient's neurologic symptoms and indicates a need for immediate action to prevent further serious complications such as seizures. The potassium and calcium levels vary slightly from the normal but do not require any immediate action by the nurse. The phosphate level is within the normal parameters.

A patient with renal failure who has been taking aluminum hydroxide/magnesium hydroxide suspension (Maalox) at home for indigestion is somnolent and has decreased deep tendon reflexes. Which action should the nurse take first? a. Notify the patient's health care provider. b. Withhold the next scheduled dose of Maalox. c. Review the magnesium level on the patient's chart. d. Check the chart for the most recent potassium level.

ANS: C Review the magnesium level on the patient's chart. The patient has a history and symptoms consistent with hypermagnesemia; the nurse should check the chart for a recent serum magnesium level. Notification of the health care provider will be done after the nurse knows the magnesium level. The Maalox should be held, but more immediate action is needed to correct the patient's decreased deep tendon reflexes (DTRs) and somnolence. Monitoring of potassium levels also is important for patients with renal failure, but the patient's current symptoms are not consistent with hyperkalemia.

A recently admitted patient has a small cell carcinoma of the lung, which is causing the syndrome of inappropriate antidiuretic hormone (SIADH). The nurse will monitor carefully for a. increased total urinary output. b. elevation of serum hematocrit. c. decreased serum sodium level. d. rapid and unexpected weight loss.

ANS: C decreased serum sodium level. SIADH causes water retention and a decrease in serum sodium level. Weight loss, increased urine output, and elevated serum hematocrit may be associated with excessive loss of water, but not with SIADH and water retention.

When assessing a patient with increased extracellular fluid (ECF) osmolality, the priority assessment for the nurse to obtain is a. skin turgor. b. heart sounds. c. mental status. d. capillary refill.

ANS: C mental status. Changes in ECF osmolality lead to swelling or shrinking of cells in the central nervous system, initially causing confusion, which may progress to coma or seizures. Although skin turgor, capillary refill, and heart sounds also may be affected by ECF osmolality changes and resultant fluid shifts, these are signs that occur later and do not have as immediate an impact on patient outcomes.

A patient who has an infusion of 50% dextrose prescribed asks the nurse why a peripherally inserted central catheter must be inserted. Which explanation by the nurse is correct? a. The prescribed infusion can be given much more rapidly when the patient has a central line. b. There is a decreased risk for infection when 50% dextrose is infused through a central line. c. The 50% dextrose is hypertonic and will be more rapidly diluted when given through a central line. d. The required blood glucose monitoring is more accurate when samples are obtained from a central line.

ANS: C. The 50% dextrose is hypertonic and will be more rapidly diluted when given through a central line. Shrinkage of red blood cells can occur when solutions with dextrose concentrations greater than 10% are administered intravenously. Blood glucose testing is not more accurate when samples are obtained from a central line. The infection risk is higher with a central catheter than with peripheral IV lines. Hypertonic or concentrated IV solutions are not given rapidly.

To prevent the recurrence of renal calculi, the nurse teaches the patient to: a. use a filter to strain all urine. b. avoid dietary sources of calcium. c. drink diuretic fluids such as coffee. d. have 2000 to 3000 mL of fluid a day.

ANS: D A fluid intake of 2000 to 3000 mL daily is recommended to help flush out minerals before stones can form. Avoidance of calcium is not usually recommended for patients with renal calculi. Coffee tends to increase stone recurrence. There is no need for a patient to strain all urine routinely after a stone has passed, and this will not prevent stones.

3. A 64-year-old woman who has chronic constipation asks the nurse about the use of psyllium (Metamucil). Which information will the nurse include in the response? a. Absorption of fat-soluble vitamins may be reduced by fiber-containing laxatives. b. Dietary sources of fiber should be eliminated to prevent excessive gas formation. c. Use of this type of laxative to prevent constipation does not cause adverse effects. d. Large amounts of fluid should be taken to prevent impaction or bowel obstruction.

ANS: D A high fluid intake is needed when patients are using bulk-forming laxatives to avoid worsening constipation. Although bulk-forming laxatives are generally safe, the nurse should emphasize the possibility of constipation or obstipation if inadequate fluid intake occurs. Although increased gas formation is likely to occur with increased dietary fiber, the patient should gradually increase dietary fiber and eventually may not need the psyllium. Fat-soluble vitamin absorption is blocked by stool softeners and lubricants, not by bulk-forming laxatives.

27. A 42-year-old male patient has had a herniorrhaphy to repair an incarcerated inguinal hernia. Which patient teaching will the nurse provide before discharge? a. Soak in sitz baths several times each day. b. Cough 5 times each hour for the next 48 hours. c. Avoid use of acetaminophen (Tylenol) for pain. d. Apply a scrotal support and ice to reduce swelling.

ANS: D A scrotal support and ice are used to reduce edema and pain. Coughing will increase pressure on the incision. Sitz baths will not relieve pain and would not be of use after this surgery. Acetaminophen can be used for postoperative pain.

A 42-year-old male patient has had a herniorrhaphy to repair an incarcerated inguinal hernia. Which patient teaching will the nurse provide before discharge? a. Soak in sitz baths several times each day. b. Cough 5 times each hour for the next 48 hours. c. Avoid use of acetaminophen (Tylenol) for pain. d. Apply a scrotal support and ice to reduce swelling.

ANS: D A scrotal support and ice are used to reduce edema and pain. Coughing will increase pressure on the incision. Sitz baths will not relieve pain and would not be of use after this surgery. Acetaminophen can be used for postoperative pain.

Which information is most important for the nurse to communicate rapidly to the health care provider about a patient admitted with possible syndrome of inappropriate antidiuretic hormone (SIADH)? a. The patient has a recent weight gain of 9 lb. b. The patient complains of dyspnea with activity. c. The patient has a urine specific gravity of 1.025. d. The patient has a serum sodium level of 118 mEq/L.

ANS: D A serum sodium of less than 120 mEq/L increases the risk for complications such as seizures and needs rapid correction. The other data are not unusual for a patient with SIADH and do not indicate the need for rapid action

17. A 73-year-old patient with diverticulosis has a large bowel obstruction. The nurse will monitor for a. referred back pain. b. metabolic alkalosis. c. projectile vomiting. d. abdominal distention.

ANS: D Abdominal distention is seen in lower intestinal obstruction. Referred back pain is not a common clinical manifestation of intestinal obstruction. Metabolic alkalosis is common in high intestinal obstruction because of the loss of HCl acid from vomiting. Projectile vomiting is associated with higher intestinal obstruction.

A 73-year-old patient with diverticulosis has a large bowel obstruction. The nurse will monitor for a. referred back pain. b. metabolic alkalosis. c. projectile vomiting. d. abdominal distention.

ANS: D Abdominal distention is seen in lower intestinal obstruction. Referred back pain is not a common clinical manifestation of intestinal obstruction. Metabolic alkalosis is common in high intestinal obstruction because of the loss of HCl acid from vomiting. Projectile vomiting is associated with higher intestinal obstruction.

A 23-year-old patient is admitted with diabetes insipidus. Which action will be most appropriate for the registered nurse (RN) to delegate to an experienced licensed practical/vocational nurse (LPN/LVN)? a. Titrate the infusion of 5% dextrose in water. b. Teach the patient how to use desmopressin (DDAVP) nasal spray. c. Assess the patient's hydration status every 8 hours. d. Administer subcutaneous DDAVP.

ANS: D Administration of medications is included in LPN/LVN education and scope of practice. Assessments, patient teaching, and titrating fluid infusions are more complex skills and should be done by the RN.

33. Which assessment finding in a patient who had a total gastrectomy 12 hours previously is most important to report to the health care provider? a. Absent bowel sounds b. Scant nasogastric (NG) tube drainage c. Complaints of incisional pain d. Temperature 102.1° F (38.9° C)

ANS: D An elevation in temperature may indicate leakage at the anastomosis, which may require return to surgery or keeping the patient NPO. The other findings are expected in the immediate postoperative period for patients who have this surgery.

28. Which breakfast choice indicates a patient's good understanding of information about a diet for celiac disease? a. Oatmeal with nonfat milk b. Whole wheat toast with butter c. Bagel with low-fat cream cheese d. Corn tortilla with scrambled eggs

ANS: D Avoidance of gluten-containing foods is the only treatment for celiac disease. Corn does not contain gluten, while oatmeal and wheat do.

10. The nurse will anticipate teaching a patient experiencing frequent heartburn about a. a barium swallow. b. radionuclide tests. c. endoscopy procedures. d. proton pump inhibitors.

ANS: D Because diagnostic testing for heartburn that is probably caused by gastroesophageal reflux disease (GERD) is expensive and uncomfortable, proton pump inhibitors are frequently used for a short period as the first step in the diagnosis of GERD. The other tests may be used but are not usually the first step in diagnosis.

When the nurse is caring for a patient who has had left-sided extracorporeal shock wave lithotripsy, which assessment finding is most important to report to the health care provider? a. Blood in urine b. Left flank pain c. Left flank bruising d. Drop in urine output

ANS: D Because lithotripsy breaks the stone into small sand, which could cause obstruction, it is important to report a drop in urine output. Left flank pain, bruising, and hematuria are common after lithotripsy. DIF: Cognitive Level: Application REF: 1138-1139

31. The nurse will plan to teach a patient with Crohn's disease who has megaloblastic anemia about the need for a. oral ferrous sulfate tablets. b. regular blood transfusions. c. iron dextran (Imferon) infusions. d. cobalamin (B12) spray or injections.

ANS: D Crohn's disease frequently affects the ileum, where absorption of cobalamin occurs. Cobalamin must be administered regularly by nasal spray or IM to correct the anemia. Iron deficiency does not cause megaloblastic anemia. The patient may need occasional transfusions but not regularly scheduled transfusions.

10. A patient has been admitted to the hospital with the diagnosis of DKA. What vital signs should a nurse anticipate that the patient will exhibit? a. Temperature, 99° F; pulse, 62 beats/min; respirations, 16 breaths/min and shallow b. Temperature, 98.6° F; pulse, 76 beats/min; respirations, 16 breaths/min and deep c. Temperature, 98° F; pulse, 84 beats/min; respirations, 18 breaths/min and shallow d. Temperature, 97.4° F; pulse, 110 beats/min; respirations, 26 breaths/min and deep

ANS: D DKA is caused by the attempt of the body to metabolize fat for energy, which results in an acidotic state. The classic signs of DKA are hypothermia, tachycardia, and Kussmaul respirations (rapid and deep) to blow off the acid ions via respirations. The respirations will have a fruity odor.

1. To assess whether there is any improvement in a patient's dysuria, which question will the nurse ask? a. "Do you have to urinate at night?" b. "Do you have blood in your urine?" c. "Do you have to urinate frequently?" d. "Do you have pain when you urinate?"

ANS: D Dysuria is painful urination. The alternate responses are used to assess other urinary tract symptoms: hematuria, nocturia, and frequency.

12. Which information will the nurse include when teaching a patient with newly diagnosed gastroesophageal reflux disease (GERD)? a. "Peppermint tea may be helpful in reducing your symptoms." b. "You should avoid eating between meals to reduce acid secretion." c. "Vigorous physical activities may increase the incidence of reflux." d. "It will be helpful to keep the head of your bed elevated on blocks."

ANS: D Elevating the head of the bed will reduce the incidence of reflux while the patient is sleeping. Peppermint will lower LES pressure and increase the chance for reflux. Small, frequent meals are recommended to avoid abdominal distention. There is no need to make changes in physical activities because of GERD.

A 62-year-old asks the nurse for a perineal pad, stating that laughing or coughing causes leakage of urine. Which intervention is most appropriate to include in the care plan? a. Assist the patient to the bathroom q3hr. b. Place a commode at the patient's bedside. c. Demonstrate how to perform the Credé maneuver. d. Teach the patient how to perform Kegel exercises.

ANS: D Exercises to strengthen the pelvic floor muscles will help reduce stress incontinence. The Credé maneuver is used to help empty the bladder for patients with overflow incontinence. Placing the commode close to the bedside and assisting the patient to the bathroom are helpful for functional incontinence. DIF: Cognitive Level: Application REF: 1148

4. A 32-year-old patient who is employed as a hairdresser and has a 15 pack-year history of cigarette smoking is scheduled for an annual physical examination. The nurse will plan to teach the patient about the increased risk for a. renal failure. b. kidney stones. c. pyelonephritis. d. bladder cancer.

ANS: D Exposure to the chemicals involved with working as a hairdresser and in smoking both increase the risk of bladder cancer, and the nurse should assess whether the patient understands this risk. The patient is not at increased risk for renal failure, pyelonephritis, or kidney stones.

6. The nurse is assessing a patient with gastroesophageal reflux disease (GERD) who is experiencing increasing discomfort. Which patient statement indicates that additional patient education about GERD is needed? a. "I take antacids between meals and at bedtime each night." b. "I sleep with the head of the bed elevated on 4-inch blocks." c. "I quit smoking several years ago, but I still chew a lot of gum." d. "I eat small meals throughout the day and have a bedtime snack."

ANS: D GERD is exacerbated by eating late at night, and the nurse should plan to teach the patient to avoid eating at bedtime. The other patient actions are appropriate to control symptoms of GERD.

6. A young patient complains that diabetes is causing her to "have no life at all. It's too hard." What is the most helpful response by the nurse? a. "Yes, you must make some sacrifices." b. "It's hard, but with significant alterations in your lifestyle, you can live a long life." c. "What's hard about exercise, diet, and medicine?" d. "Let's talk about what makes it so hard."

ANS: D Involving the patient in decisions about how she will cope with her diabetes will make the goals more realistic and personal, which will give her a greater chance of success in meeting them.

23. When working in the urology/nephrology clinic, which patient could the nurse delegate to an experienced licensed practical/vocational nurse (LPN/LVN)? a. Patient who is scheduled for a renal biopsy after a recent kidney transplant b. Patient who will need monitoring for several hours after a renal arteriogram c. Patient who requires teaching about possible post-cystoscopy complications d. Patient who will have catheterization to check for residual urine after voiding

ANS: D LPN/LVN education includes common procedures such as catheterization of stable patients. The other patients require more complex assessments and/or patient teaching that are included in registered nurse (RN) education and scope of practice.

28. A patient who requires daily use of a nonsteroidal anti-inflammatory drug (NSAID) for management of severe rheumatoid arthritis has recently developed melena. The nurse will anticipate teaching the patient about a. substitution of acetaminophen (Tylenol) for the NSAID. b. use of enteric-coated NSAIDs to reduce gastric irritation. c. reasons for using corticosteroids to treat the rheumatoid arthritis. d. the benefits of misoprostol (Cytotec) in protecting the gastrointestinal (GI) mucosa.

ANS: D Misoprostol, a prostaglandin analog, reduces acid secretion and incidence of upper GI bleeding associated with NSAID use. Enteric coating of NSAIDs does not reduce the risk for GI bleeding. Corticosteroids increase the risk for ulcer development and will not be substituted for NSAIDs for this patient. Acetaminophen will not be effective in treating the patient's rheumatoid arthritis.

Which question from the nurse would help determine if a patient's abdominal pain might indicate irritable bowel syndrome? a. "Have you been passing a lot of gas?" b. "What foods affect your bowel patterns?" c. "Do you have any abdominal distention?" d. "How long have you had abdominal pain?"

ANS: D One criterion for the diagnosis of irritable bowel syndrome (IBS) is the presence of abdominal discomfort or pain for at least 3 months. Abdominal distention, flatulence, and food intolerance are also associated with IBS, but are not diagnostic criteria.

Which action will the nurse include in the plan of care for a patient who has had a ureterolithotomy and has a left ureteral catheter and a urethral catheter in place? a. Provide education about home care for both catheters. b. Apply continuous steady tension to the ureteral catheter. c. Clamp the ureteral catheter unless output from the urethral catheter stops. d. Call the health care provider if the ureteral catheter output drops suddenly.

ANS: D The health care provider should be notified if the ureteral catheter output decreases since obstruction of this catheter may result in an increase in pressure in the renal pelvis. Tension on the ureteral catheter should be avoided in order to prevent catheter displacement. To avoid pressure in the renal pelvis, the catheter is not clamped. Since the patient is not usually discharged with a ureteral catheter in place, patient teaching about both catheters is not needed. DIF: Cognitive Level: Application REF: 1153-1154

31. A patient with a recent 20-pound unintended weight loss is diagnosed with stomach cancer. Which nursing action will be included in the plan of care? a. Refer the patient for hospice services. b. Infuse IV fluids through a central line. c. Teach the patient about antiemetic therapy. d. Offer supplemental feedings between meals.

ANS: D The patient data indicate a poor nutritional state and improvement in nutrition will be helpful in improving response to therapies such as surgery, chemotherapy, or radiation. Nausea and vomiting are not common clinical manifestations of stomach cancer. There is no indication that the patient requires hospice or IV fluid infusions.

44. Which patient should the nurse assess first after receiving change-of-shift report? a. 60-year-old patient whose new ileostomy has drained 800 mL over the previous 8 hours b. 50-year-old patient with familial adenomatous polyposis who has occult blood in the stool c. 40-year-old patient with ulcerative colitis who has had six liquid stools in the previous 4 hours d. 30-year-old patient who has abdominal distention and an apical heart rate of 136 beats/minute

ANS: D The patient's abdominal distention and tachycardia suggest hypovolemic shock caused by problems such as peritonitis or intestinal obstruction, which will require rapid intervention. The other patients should also be assessed as quickly as possible, but the data do not indicate any life-threatening complications associated with their diagnoses.

Which patient should the nurse assess first after receiving change-of-shift report? a. 60-year-old patient whose new ileostomy has drained 800 mL over the previous 8 hours b. 50-year-old patient with familial adenomatous polyposis who has occult blood in the stool c. 40-year-old patient with ulcerative colitis who has had six liquid stools in the previous 4 hours d. 30-year-old patient who has abdominal distention and an apical heart rate of 136 beats/minute

ANS: D The patient's abdominal distention and tachycardia suggest hypovolemic shock caused by problems such as peritonitis or intestinal obstruction, which will require rapid intervention. The other patients should also be assessed as quickly as possible, but the data do not indicate any life-threatening complications associated with their diagnoses.

7. A 27-year-old female patient is admitted to the hospital for evaluation of right lower quadrant abdominal pain with nausea and vomiting. Which action should the nurse take? a. Encourage the patient to sip clear liquids. b. Assess the abdomen for rebound tenderness. c. Assist the patient to cough and deep breathe. d. Apply an ice pack to the right lower quadrant.

ANS: D The patient's clinical manifestations are consistent with appendicitis, and application of an ice pack will decrease inflammation at the area. Checking for rebound tenderness frequently is unnecessary and uncomfortable for the patient. The patient should be NPO in case immediate surgery is needed. The patient will need to know how to cough and deep breathe postoperatively, but coughing will increase pain at this time.

21. Which statement by a patient who had a cystoscopy the previous day should be reported immediately to the health care provider? a. "My urine looks pink." b. "My IV site is bruised." c. "My sleep was restless." d. "My temperature is 101."

ANS: D The patient's elevated temperature may indicate a bladder infection, a possible complication of cystoscopy. The health care provider should be notified so that antibiotic therapy can be started. Pink-tinged urine is expected after a cystoscopy. The insomnia and bruising should be discussed further with the patient but do not indicate a need to notify the health care provider.

43. After several days of antibiotic therapy, an older hospitalized patient develops watery diarrhea. Which action should the nurse take first? a. Notify the health care provider. b. Obtain a stool specimen for analysis. c. Teach the patient about handwashing. d. Place the patient on contact precautions.

ANS: D The patient's history and new onset diarrhea suggest a C. difficile infection, which requires implementation of contact precautions to prevent spread of the infection to other patients. The other actions are also appropriate but can be accomplished after contact precautions are implemented.

37. A 45-year-old patient is admitted to the emergency department with severe abdominal pain and rebound tenderness. Vital signs include temperature 102° F (38.3° C), pulse 120, respirations 32, and blood pressure (BP) 82/54. Which prescribed intervention should the nurse implement first? a. Administer IV ketorolac (Toradol) 15 mg. b. Draw blood for a complete blood count (CBC). c. Obtain a computed tomography (CT) scan of the abdomen. d. Infuse 1 liter of lactated Ringer's solution over 30 minutes.

ANS: D The priority for this patient is to treat the patient's hypovolemic shock with fluid infusion. The other actions should be implemented after starting the fluid infusion.

A 45-year-old patient is admitted to the emergency department with severe abdominal pain and rebound tenderness. Vital signs include temperature 102° F (38.3° C), pulse 120, respirations 32, and blood pressure (BP) 82/54. Which prescribed intervention should the nurse implement first? a. Administer IV ketorolac (Toradol) 15 mg. b. Draw blood for a complete blood count (CBC). c. Obtain a computed tomography (CT) scan of the abdomen. d. Infuse 1 liter of lactated Ringer's solution over 30 minutes.

ANS: D The priority for this patient is to treat the patient's hypovolemic shock with fluid infusion. The other actions should be implemented after starting the fluid infusion.

8. A patient with recurring heartburn receives a new prescription for esomeprazole (Nexium). In teaching the patient about this medication, the nurse explains that this drug a. neutralizes stomach acid and provides relief of symptoms in a few minutes. b. reduces the reflux of gastric acid by increasing the rate of gastric emptying. c. coats and protects the lining of the stomach and esophagus from gastric acid. d. treats gastroesophageal reflux disease by decreasing stomach acid production.

ANS: D The proton pump inhibitors decrease the rate of gastric acid secretion. Promotility drugs such as metoclopramide (Reglan) increase the rate of gastric emptying. Cryoprotective medications such as sucralfate (Carafate) protect the stomach. Antacids neutralize stomach acid and work rapidly.

8. The nurse explaining esomeprazole (Nexium) to a patient with recurring heartburn describes that the medication a. reduces gastroesophageal reflux by increasing the rate of gastric emptying. b. neutralizes stomach acid and provides relief of symptoms in a few minutes. c. coats and protects the lining of the stomach and esophagus from gastric acid. d. treats gastroesophageal reflux disease by decreasing stomach acid production.

ANS: D The proton pump inhibitors decrease the rate of gastric acid secretion. Promotility drugs such as metoclopramide (Reglan) increase the rate of gastric emptying. Cryoprotective medications such as sucralfate (Carafate) protect the stomach. Antacids neutralize stomach acid and work rapidly.

22. A 47-year-old female patient is transferred from the recovery room to a surgical unit after a transverse colostomy. The nurse observes the stoma to be deep pink with edema and a small amount of sanguineous drainage. The nurse should a. place ice packs around the stoma. b. notify the surgeon about the stoma. c. monitor the stoma every 30 minutes. d. document stoma assessment findings.

ANS: D The stoma appearance indicates good circulation to the stoma. There is no indication that surgical intervention is needed or that frequent stoma monitoring is required. Swelling of the stoma is normal for 2 to 3 weeks after surgery, and an ice pack is not needed.

A 47-year-old female patient is transferred from the recovery room to a surgical unit after a transverse colostomy. The nurse observes the stoma to be deep pink with edema and a small amount of sanguineous drainage. The nurse should a. place ice packs around the stoma. b. notify the surgeon about the stoma. c. monitor the stoma every 30 minutes. d. document stoma assessment findings.

ANS: D The stoma appearance indicates good circulation to the stoma. There is no indication that surgical intervention is needed or that frequent stoma monitoring is required. Swelling of the stoma is normal for 2 to 3 weeks after surgery, and an ice pack is not needed.

18. A patient comes to the diabetes clinic and confides to the nurse that she does not follow the diet exchange program that she was given. What is the best response by the nurse? a. "The exchange program is a carefully developed and very important program that allows you to take control of your disease." b. "A lot of people have trouble with that program. You aren't the first one to go off your diet." c. "We had better check your blood work to see what you've done to yourself." d. "Okay. Let's talk about what you do eat and drink and how you manage your diabetes."

ANS: D To evaluate the effectiveness of treatment, the nurse must first find out how the patient perceives the importance of diet, drugs, and exercise.

2. A patient newly diagnosed with type 2 diabetes mellitus asks the nurse why she has to take a pill instead of insulin. The nurse explains that in type 2 diabetes mellitus, the body still makes insulin. What other information is pertinent for the nurse to relay? a. Overweight and underactive people cannot simply use the insulin produced. b. Metabolism is slowed in some people, so they have to take a pill to speed up their metabolism. c. Sometimes the autoimmune system works against the action of the insulin. d. The cells become resistant to the action of insulin. Pills are given to increase the sensitivity.

ANS: D Type 2 diabetes mellitus is a disease in which the cells become resistant to the action of insulin and the blood glucose level rises. Oral hyperglycemic agents make the cells more sensitive.

15. A patient who is nauseated and vomiting up blood-streaked fluid is admitted to the hospital with acute gastritis. To determine possible risk factors for gastritis, the nurse will ask the patient about a. the amount of fat in the diet. b. history of recent weight gain or loss. c. any family history of gastric problems. d. use of nonsteroidal anti-inflammatory drugs (NSAIDs).

ANS: D Use of an NSAID is associated with damage to the gastric mucosa, which can result in acute gastritis. Family history, recent weight gain or loss, and fatty foods are not risk factors for acute gastritis.

The nurse determines that instruction regarding prevention of future urinary tract infections (UTIs) for a patient with cystitis has been effective when the patient states, a. "I can use vaginal sprays to reduce bacteria." b. "I will drink a quart of water or other fluids every day." c. "I will wash with soap and water before sexual intercourse." d. "I will empty my bladder every 3 to 4 hours during the day."

ANS: D Voiding every 3 to 4 hours is recommended to prevent UTIs. Use of vaginal sprays is dis- couraged. The bladder should be emptied before and after intercourse, but cleaning with soap and water is not necessary. A quart of fluids is insufficient to provide adequate urine output to decrease risk for UTI.

13. A nurse preparing to administer insulin to a patient who has type 1 diabetes. The physician has prescribed two types of insulin, 10 U of regular insulin and 35 U of neutral protamine Hagedorn (NPH) insulin. Which is the proper procedure for the nurse to follow when preparing these medications? a. Draw up the insulins in two separate syringes to avoid confusion. b. Draw up the regular insulin before drawing up the NPH insulin. c. Inject air into the NPH insulin, draw it up to 35 U, and then inject air into the clear regular insulin and withdraw to 45 U. d. Inject 35 U air into the NPH insulin, inject 10 U air into the regular insulin, withdraw 10 U of the regular insulin, and withdraw 35 U of the NPH insulin.

ANS: D When drawing up two insulins, the vials are injected with air, and the regular insulin is drawn first. This slow and time-consuming activity has been greatly reduced with the advent of premixed insulins.

Spironolactone (Aldactone), an aldosterone antagonist, is prescribed for a patient as a diuretic. Which statement by the patient indicates that the teaching about this medication has been effective? a. "I will try to drink at least 8 glasses of water every day." b. "I will use a salt substitute to decrease my sodium intake." c. "I will increase my intake of potassium-containing foods." d. "I will drink apple juice instead of orange juice for breakfast."

ANS: D "I will drink apple juice instead of orange juice for breakfast." Since spironolactone is a potassium-sparing diuretic, patients should be taught to choose low potassium foods such as apple juice rather than foods that have higher levels of potassium, such as citrus fruits. Because the patient is using spironolactone as a diuretic, the nurse would not encourage the patient to increase fluid intake. Teach patients to avoid salt substitutes, which are high in potassium.

A postoperative patient who is receiving nasogastric suction is complaining of anxiety and incisional pain. The patient's respiratory rate is 32 breaths/minute and the arterial blood gases (ABGs) indicate respiratory alkalosis. Which action should the nurse take first? a. Discontinue the nasogastric suctions for a few hours. b. Notify the health care provider about the ABG results. c. Teach the patient about the need to take slow, deep breaths. d. Give the patient the PRN morphine sulfate 4 mg intravenously.

ANS: D Give the patient the PRN morphine sulfate 4 mg intravenously. The patient's respiratory alkalosis is caused by the increased respiratory rate associated with pain and anxiety. The nurse's first action should be to medicate the patient for pain. Although the nasogastric suction may contribute to the alkalosis, it is not appropriate to discontinue the tube when the patient needs gastric suction. The health care provider may be notified about the ABGs but is likely to instruct the nurse to medicate for pain. The patient will not be able to take slow, deep breaths when experiencing pain.

The nurse has administered 3% saline to a patient with hyponatremia. Which one of these assessment data will require the most rapid response by the nurse? a. The patient's radial pulse is 105 beats/minute. b. There is sediment and blood in the patient's urine. c. The blood pressure increases from 120/80 to 142/94. d. There are crackles audible throughout both lung fields.

ANS: D There are crackles audible throughout both lung fields. Crackles throughout both lungs suggest that the patient may be experiencing pulmonary edema, a life-threatening adverse effect of hypertonic solutions. The increased pulse rate and blood pressure and the appearance of the urine also should be reported, but they are not as dangerous as the presence of fluid in the alveoli.

A patient is admitted with diabetes insipidus. Which action will be appropriate for the registered nurse (RN) to delegate to an experienced licensed practical/vocational nurse (LPN/LVN)? a. Titrate the infusion of 5% dextrose in water. b. Administer prescribed subcutaneous DDAVP. c. Assess the patient's overall hydration status every 8 hours. d. Teach the patient how to use desmopressin (DDAVP) nasal spray.

B Administration of medications is included in LPN/LVN education and scope of practice. Assessments, patient teaching, and titrating fluid infusions are more complex skills and should be done by the RN.

A patient with hypercalcemia is being cared for on the medical unit. Nursing actions included on the care plan will include a. maintaining the patient on bed rest. b. auscultating lung sounds every 4 hours. c. monitoring for Trousseau's and Chvostek's signs. d. encouraging fluid intake up to 4000 ml every day.

ANS: D encouraging fluid intake up to 4000 ml every day. To decrease the risk for renal calculi, the patient should have an intake of 3000 to 4000 ml daily. Ambulation helps decrease the loss of calcium from bone and is encouraged in patients with hypercalcemia. Trousseau's and Chvostek's signs are monitored when there is a possibility of hypocalcemia. There is no indication that the patient needs frequent assessment of lung sounds, although these would be assessed every shift.

When considering the following causes of acute abdomen, the nurse should know that surgery wound be indicated for

Acute ischemic bowel Ruptured ectopic pregnancy Foreign- body perforation Ruptured abdominal aneurysm

Which hormone is associated with fostering growth of the adrenal cortex?

Adrenocorticotropic hormone (ACTH)

C

After receiving change-of-shift report, which patient should the nurse assess first? a. Patient with serum potassium level of 5.0 mEq/L who is complaining of abdominal cramping b. Patient with serum sodium level of 145 mEq/L who has a dry mouth and is asking for a glass of water c. Patient with serum magnesium level of 1.1 mEq/L who has tremors and hyperactive deep tendon reflexes d. Patient with serum phosphorus level of 4.5 mg/dL who has multiple soft tissue calcium-phosphate precipitates

Which hormones are released by the adrenal cortex? Select all that apply.

Androgens Glucocorticoids Mineralocorticoids

An expected patient problem for a patient admitted to the hospital with symptoms of diabetes insipidus is a. excess fluid volume related to intake greater than output. b. impaired gas exchange related to fluid retention in lungs. c. sleep pattern disturbance related to frequent waking to void. d. risk for impaired skin integrity related to generalized edema.

C Nocturia occurs as a result of the polyuria caused by diabetes insipidus. Edema, excess fluid volume, and fluid retention are not expected.

A 29-yr-old woman with systemic lupus erythematosus has been prescribed 2 weeks of high-dose prednisone therapy. Which information about the prednisone is most important for the nurse to include? a. "Weigh yourself daily to monitor for weight gain." b. "The prednisone dose should be decreased gradually." c. "A weight-bearing exercise program will help minimize risk for osteoporosis." d. "Call the health care provider if you have mood changes with the prednisone."

B Acute adrenal insufficiency may occur if exogenous corticosteroids are suddenly stopped. Mood alterations and weight gain are possible adverse effects of corticosteroid use, but these are not life- threatening effects. Osteoporosis occurs when patients take corticosteroids for longer periods.

The nurse is planning postoperative care for a patient who is being admitted to the surgical unit from the recovery room after transsphenoidal resection of a pituitary tumor. Which nursing action should be included? a. Palpate extremities for edema. b. Measure urine volume every hour. c. Check hematocrit every 2 hours for 8 hours. d. Monitor continuous pulse oximetry for 24 hours.

B After pituitary surgery, the patient is at risk for diabetes insipidus caused by cerebral edema. Monitoring of urine output and urine specific gravity is essential. Hemorrhage is not a common problem. There is no need to check the hematocrit hourly. The patient is at risk for dehydration, not volume overload. The patient is not at high risk for problems with oxygenation, and continuous pulse oximetry is not needed.

The results of a patient's recent endoscopy indicate the presence of peptic ulcer disease (PUD). Which teaching point should the nurse provide to the patient based on this new diagnosis? a) "You'll need to drink at least two to three glasses of milk daily." b) "It would likely be beneficial for you to eliminate drinking alcohol." c) "Many people find that a minced or pureed diet eases their symptoms of PUD." d) "Your medications should allow you to maintain your present diet while minimizing symptoms."

B Alcohol increases the amount of stomach acid produced so it should be avoided. Although there is no specific recommended dietary modification for PUD, most patients find it necessary to make some sort of dietary modifications to minimize symptoms. Milk may exacerbate PUD.

The nurse is assessing a male patient diagnosed with a pituitary tumor causing panhypopituitarism. Assessment findings consistent with panhypopituitarism include a. high blood pressure. c. elevated blood glucose. b. decreased facial hair. d. tachycardia and palpitations.

B Changes in male secondary sex characteristics such as decreased facial hair, testicular atrophy, diminished spermatogenesis, loss of libido, impotence, and decreased muscle mass are associated with decreases in follicle-stimulating hormone (FSH) and luteinizing hormone (LH). Fasting hypoglycemia and hypotension occur in panhypopituitarism as a result of decreases in adrenocorticotropic hormone (ACTH) and cortisol. Bradycardia is likely due to the decrease in thyroid-stimulating hormone (TSH) and thyroid hormones associated with panhypopituitarism. DIF: Cognitive Level: Apply (application) REF: 1158

The nurse cares for a postoperative patient who has just vomited yellow green liquid and reports nausea. Which action would be an appropriate nursing intervention? a) Offer the patient a herbal supplement such as ginseng. b) Apply a cool washcloth to the forehead and provide mouth care. c) Take the patient for a walk in the hallway to promote peristalsis. d) Discontinue any medications that may cause nausea or vomiting.

B Cleansing the face and hands with a cool washcloth and providing mouth care are appropriate comfort interventions for nausea and vomiting. Ginseng is not used to treat postoperative nausea and vomiting. Unnecessary activity should be avoided. The patient should rest in a quiet environment. Medications may be temporarily withheld until the acute phase is over, but the *medications should not be discontinued without consultation with the health care provider.*

The nurse determines that demeclocycline is effective for a patient with syndrome of inappropriate antidiuretic hormone (SIADH) based on finding that the patient's a. weight has increased. b. urinary output is increased. c. peripheral edema is increased d. urine specific gravity is increased.

B Demeclocycline blocks the action of antidiuretic hormone (ADH) on the renal tubules and increases urine output. An increase in weight or an increase in urine specific gravity indicates that the SIADH is not corrected. Peripheral edema does not occur with SIADH. A sudden weight gain without edema is a common clinical manifestation of this disorder.

The cardiac telemetry unit charge nurse receives status reports from other nursing units about four patients who need cardiac monitoring. Which patient should be transferred to the cardiac unit first? a. Patient with Hashimoto's thyroiditis and a heart rate of 102 b. Patient with tetany who has a new order for IV calcium chloride c. Patient with Cushing syndrome and a blood glucose of 140 mg/dL d. Patient with Addison's disease who takes hydrocortisone twice daily

B Emergency treatment of tetany requires IV administration of calcium; electrocardiographic monitoring will be required because cardiac arrest may occur if high calcium levels result from too-rapid administration. The information about the other patients indicates that they are more stable than the patient with tetany.

After receiving change-of-shift report about the following four patients, which patient should the nurse assess first? a. A 31-yr-old female patient with Cushing syndrome and a blood glucose level of 244 mg/dL b. A 70-yr-old female patient taking levothyroxine (Synthroid) who has an irregular pulse of 134 c. A 53-yr-old male patient who has Addison's disease and is due for a prescribed dose of hydrocortisone (Solu-Cortef). d. A 22-yr-old male patient admitted with syndrome of inappropriate antidiuretic hormone (SIADH) who has a serum sodium level of 130 mEq/L

B Initiation of thyroid replacement in older adults may cause angina and cardiac dysrhythmias. The patient's high pulse rate needs rapid investigation by the nurse to assess for and intervene with any cardiac problems. The other patients also require nursing assessment and/or actions but are not at risk for life-threatening complications.

After receiving change-of-shift report about the following four patients, which patient should the nurse assess first? a. A 31-yr-old female patient with Cushing syndrome and a blood glucose level of 244 mg/dL b. A 70-yr-old female patient taking levothyroxine (Synthroid) who has an irregular pulse of 134 c. A 53-yr-old male patient who has Addison's disease and is due for a prescribed dose of hydrocortisone (Solu-Cortef). d. A 22-yr-old male patient admitted with syndrome of inappropriate antidiuretic hormone (SIADH) who has a serum sodium level of 130 mEq/L

B Initiation of thyroid replacement in older adults may cause angina and cardiac dysrhythmias. The patient's high pulse rate needs rapid investigation by the nurse to assess for and intervene with any cardiac problems. The other patients also require nursing assessment and/or actions but are not at risk for life-threatening complications.

Which question will the nurse in the endocrine clinic ask to help determine a patient's risk factors for goiter? a. "How much milk do you drink?" b. "What medications are you taking?" c. "Are your immunizations up to date?" d. "Have you had any recent neck injuries?"

B Medications that contain thyroid-inhibiting substances can cause goiter. Milk intake, neck injury, and immunization history are not risk factors for goiter.

A patient who had radical neck surgery to remove a malignant tumor developed hypoparathyroidism. The nurse should plan to teach the patient about a. bisphosphonates to reduce bone demineralization. b. calcium supplements to normalize serum calcium levels. c. increasing fluid intake to decrease risk for nephrolithiasis. d. including whole grains in the diet to prevent constipation.

B Oral calcium supplements are used to maintain the serum calcium in normal range and prevent the complications of hypocalcemia. Whole grain foods decrease calcium absorption and will not be recommended. Bisphosphonates will lower serum calcium levels further by preventing calcium from being reabsorbed from bone. Kidney stones are not a complication of hypoparathyroidism and low calcium levels.

Which information will the nurse include when teaching a 50-yr-old male patient about somatropin (Genotropin)? a. The medication will be needed for 3 to 6 months. b. Inject the medication subcutaneously every day. c. Blood glucose levels may decrease when taking the medication. d. Stop taking the medication if swelling of the hands or feet occurs.

B Somatropin is injected subcutaneously on a daily basis, preferably in the evening. The patient will need to continue on somatropin for life. If swelling or other common adverse effects occur, the health care provider should be notified. Growth hormone will increase blood glucose levels.

The nurse will plan to monitor a patient diagnosed with a pheochromocytoma for a. flushing. c. bradycardia. b. headache. d. hypoglycemia.

B The classic clinical manifestations of pheochromocytoma are hypertension, tachycardia, severe headache, diaphoresis, and abdominal or chest pain. Elevated blood glucose may also occur because of sympathetic nervous system stimulation. Bradycardia and flushing would not be expected.

A 37-yr-old patient has just arrived in the postanesthesia recovery unit (PACU) after a thyroidectomy. Which information about the patient is most important to communicate to the surgeon? a. Difficult to awaken. b. Increasing neck swelling. c. Reports 7/10 incisional pain. d. Cardiac rate 112 beats/minute.

B The neck swelling may lead to respiratory difficulty, and rapid intervention is needed to prevent airway obstruction. The incisional pain should be treated but is not unusual after surgery. A heart rate of 112 beats/min is not unusual in a patient who has been hyperthyroid and has just arrived in the PACU from surgery. Sleepiness in the immediate postoperative period is expected.

A 37-yr-old patient has just arrived in the postanesthesia recovery unit (PACU) after a thyroidectomy. Which information about the patient is most important to communicate to the surgeon? a. Difficult to awaken. b. Increasing neck swelling. c. Reports 7/10 incisional pain. d. Cardiac rate 112 beats/minute.

B The neck swelling may lead to respiratory difficulty, and rapid intervention is needed to prevent airway obstruction. The incisional pain should be treated but is not unusual after surgery. A heart rate of 112 beats/min is not unusual in a patient who has been hyperthyroid and has just arrived in the PACU from surgery. Sleepiness in the immediate postoperative period is expected.

The nurse admits a patient to the hospital in Addisonian crisis. Which patient statement supports the need to plan additional teaching? a. "I frequently eat at restaurants, and my food has a lot of added salt." b. "I had the flu earlier this week, so I couldn't take the hydrocortisone." c. "I always double my dose of hydrocortisone on the days that I go for a long run." d. "I take twice as much hydrocortisone in the morning dose as I do in the afternoon."

B The need for hydrocortisone replacement is increased with stressors such as illness, and the patient needs to be taught to call the health care provider because medication and IV fluids and electrolytes may need to be given. The other patient statements indicate appropriate management of the Addison's disease.

The nurse admits a patient to the hospital in Addisonian crisis. Which patient statement supports the need to plan additional teaching? a. "I frequently eat at restaurants, and my food has a lot of added salt." b. "I had the flu earlier this week, so I couldn't take the hydrocortisone." c. "I always double my dose of hydrocortisone on the days that I go for a long run." d. "I take twice as much hydrocortisone in the morning dose as I do in the afternoon."

B The need for hydrocortisone replacement is increased with stressors such as illness, and the patient needs to be taught to call the health care provider because medication and IV fluids and electrolytes may need to be given. The other patient statements indicate appropriate management of the Addison's disease.

Which nursing action will be included in the plan of care for a patient with Graves' disease who has exophthalmos? a. Place cold packs on the eyes to relieve pain and swelling. b. Elevate the head of the patient's bed to reduce periorbital fluid. c. Apply alternating eye patches to protect the corneas from irritation. d. Teach the patient to blink every few seconds to lubricate the corneas.

B The patient should sit upright as much as possible to promote fluid drainage from the periorbital area. With exophthalmos, the patient is unable to close the eyes completely to blink. Lubrication of the eyes, rather than eye patches, will protect the eyes from developing corneal scarring. The swelling of the eye is not caused by excessive blood flow to the eye, so cold packs will not be helpful.

Which nursing action will be included in the plan of care for a patient with Graves' disease who has exophthalmos? a. Place cold packs on the eyes to relieve pain and swelling. b. Elevate the head of the patient's bed to reduce periorbital fluid. c. Apply alternating eye patches to protect the corneas from irritation. d. Teach the patient to blink every few seconds to lubricate the corneas.

B The patient should sit upright as much as possible to promote fluid drainage from the periorbital area. With exophthalmos, the patient is unable to close the eyes completely to blink. Lubrication of the eyes, rather than eye patches, will protect the eyes from developing corneal scarring. The swelling of the eye is not caused by excessive blood flow to the eye, so cold packs will not be helpful.

A patient with primary hyperparathyroidism has a serum phosphorus level of 1.7 mg/dL (0.55 mmol/L) and calcium of 14 mg/dL (3.5 mmol/L). Which nursing action should be included in the plan of care? a. Restrict the patient to bed rest. b. Encourage 4000 mL of fluids daily. c. Institute routine seizure precautions. d. Assess for positive Chvostek's sign.

B The patient with hypercalcemia is at risk for kidney stones, which may be prevented by a high fluid intake. Seizure precautions and monitoring for Chvostek's or Trousseau's sign are appropriate for hypocalcemic patients. The patient should engage in weight-bearing exercise to decrease calcium loss from bone.

A patient with primary hyperparathyroidism has a serum phosphorus level of 1.7 mg/dL (0.55 mmol/L) and calcium of 14 mg/dL (3.5 mmol/L). Which nursing action should be included in the plan of care? a. Restrict the patient to bed rest. b. Encourage 4000 mL of fluids daily. c. Institute routine seizure precautions. d. Assess for positive Chvostek's sign.

B The patient with hypercalcemia is at risk for kidney stones, which may be prevented by a high fluid intake. Seizure precautions and monitoring for Chvostek's or Trousseau's sign are appropriate for hypocalcemic patients. The patient should engage in weight-bearing exercise to decrease calcium loss from bone.

A patient develops carpopedal spasms and tingling of the lips following a parathyroidectomy. Which action will provide the patient with rapid relief from the symptoms? a. Administer the prescribed muscle relaxant. b. Have the patient rebreathe from a paper bag. c. Start the PRN O2 at 2 L/min per cannula. d. Stretch the muscles with passive range of motion.

B The patient's symptoms suggest mild hypocalcemia. The symptoms of hypocalcemia will be temporarily reduced by having the patient breathe into a paper bag, which will raise the PaCO2 and create a more acidic pH. Applying as-needed O2 or range of motion will have no impact on the ionized calcium level. Calcium supplements will be given to normalize calcium levels quickly, but oral supplements will take time to be absorbed.

A 56-yr-old patient who is disoriented and reports a headache and muscle cramps is hospitalized with possible syndrome of inappropriate antidiuretic hormone (SIADH). The nurse would expect the initial laboratory results to include a(n) a. elevated hematocrit. b. decreased serum sodium. c. increased serum chloride d. low urine specific gravity.

B When water is retained, the serum sodium level will drop below normal, causing the clinical manifestations reported by the patient. The hematocrit will decrease because of the dilution caused by water retention. Urine will be more concentrated with a higher specific gravity. The serum chloride level will usually decrease along with the sodium level.

A patient who was admitted with myxedema coma and diagnosed with hypothyroidism is improving. Discharge is expected to occur in 2 days. Which teaching strategy is likely to result in effective patient self-management at home? a. Delay teaching until closer to discharge date. b. Provide written reminders of information taught. c. Offer multiple options for management of therapies. d. Ensure privacy for teaching by asking the family to leave.

B Written instructions will be helpful to the patient because initially the hypothyroid patient may be unable to remember to take medications and other aspects of self-care. Because the treatment regimen is somewhat complex, teaching should be initiated well before discharge. Family members or friends should be included in teaching because the hypothyroid patient is likely to forget some aspects of the treatment plan. A simpler regimen will be easier to understand until the patient is euthyroid.

A patient who was admitted with myxedema coma and diagnosed with hypothyroidism is improving. Discharge is expected to occur in 2 days. Which teaching strategy is likely to result in effective patient self-management at home? a. Delay teaching until closer to discharge date. b. Provide written reminders of information taught. c. Offer multiple options for management of therapies. d. Ensure privacy for teaching by asking the family to leave.

B Written instructions will be helpful to the patient because initially the hypothyroid patient may be unable to remember to take medications and other aspects of self-care. Because the treatment regimen is somewhat complex, teaching should be initiated well before discharge. Family members or friends should be included in teaching because the hypothyroid patient is likely to forget some aspects of the treatment plan. A simpler regimen will be easier to understand until the patient is euthyroid.

A 30-year-old patient seen in the emergency department for severe headache and acute confusion is found to have a serum sodium level of 118 mEq/L. The nurse will anticipate the need for which diagnostic test? A. Urinary 17-ketosteroids B. Antidiuretic hormone level C. Growth hormone stimulation test D. Adrenocorticotropic hormone level

B. Antidiuretic hormone level - Elevated levels of antidiuretic hormone will cause water retention and decrease serum sodium levels. The other tests would not be helpful in determining possible causes of the patient's hyponatremia.

During the physical examination of a 36-year-old female, the nurse finds that the patient's thyroid gland cannot be palpated. The most appropriate action by the nurse is to A. Palpate the patient's neck more deeply. B. Document that the thyroid was nonpalpable. C. Notify the health care provider immediately. D. Teach the patient about thyroid hormone testing.

B. Document that the thyroid was nonpalpable. - The thyroid is frequently nonpalpable. The nurse should simply document the finding. There is no need to notify the health care provider immediately about a normal finding. There is no indication for thyroid-stimulating hormone (TSH) testing unless there is evidence of thyroid dysfunction. Deep palpation of the neck is not appropriate.

A nurse will teach a patient who is scheduled to complete a 24-hour urine collection for 17-ketosteroids to A. Insert and maintain a retention catheter. B. Keep the specimen refrigerated or on ice. C. Drink at least 3 L of fluid during the 24 hours. D. Void and save that specimen to start the collection.

B. Keep the specimen refrigerated or on ice. - The specimen must be kept on ice or refrigerated until the collection is finished. Voided or catheterized specimens are acceptable for the test. The initial voided specimen is discarded. There is no fluid intake requirement for the 24-hour collection.

Corticosteroids can affect blood glucose results. The other information will be provided to the health care provider but will not affect the test results. A. The RN checks the blood pressure on both arms. B. The RN palpates the neck thoroughly to check thyroid size. C. The RN lowers the thermostat to decrease the temperature in the room. D. The RN orders nonmedicated eye drops to lubricate the patient's bulging eyes.

B. The RN palpates the neck thoroughly to check thyroid size. - Palpation can cause the release of thyroid hormones in a patient with an enlarged thyroid and should be avoided. The other actions by the new RN are appropriate when caring for a patient with an enlarged thyroid.

The nurse is caring for a 45-year-old male patient during a water deprivation test. Which finding is most important for the nurse to communicate to the health care provider? A. The patient complains of intense thirst. B. The patient has a 5-lb (2.3 kg) weight loss. C. The patient's urine osmolality does not increase. D. The patient feels dizzy when sitting on the edge of the bed.

B. The patient has a 5-lb (2.3 kg) weight loss. - A drop in the weight of more than 2 kg indicates severe dehydration, and the test should be discontinued. The other assessment data are not unusual with this test.

Which information about a 30-year-old patient who is scheduled for an oral glucose tolerance test should be reported to the health care provider before starting the test? A. The patient reports having occasional orthostatic dizziness. B. The patient takes oral corticosteroids for rheumatoid arthritis. C. The patient has had a 10-pound weight gain in the last month. D. The patient drank several glasses of water an hour previously.

B. The patient takes oral corticosteroids for rheumatoid arthritis. - Corticosteroids can affect blood glucose results. The other information will be provided to the health care provider but will not affect the test results.

To prevent recurrence of uric acid renal calculi, the nurse teaches the patient to avoid eating a. milk and cheese. b. sardines and liver. c. legumes and dried fruit. d. spinach, chocolate, and tea.

B. sardines and liver Organ meats and fish such as sardines increase purine levels and uric acid. Spinach, chocolate, and tomatoes should be avoided in patients who have oxalate stones. Milk, dairy products, legumes, and dried fruits may increase the incidence of calcium-containing stones.

A patient has just arrived on the unit after a thyroidectomy. Which action should the nurse take first? a. Observe the dressing for bleeding. b. Check the blood pressure and pulse. c. Assess the patient's respiratory effort. d. Support the patient's head with pillows.

C Airway obstruction is a possible complication after thyroidectomy because of swelling or bleeding at the site or tetany. The priority nursing action is to assess the airway. The other actions are also part of the standard nursing care postthyroidectomy but are not as high of a priority.

A patient has just arrived on the unit after a thyroidectomy. Which action should the nurse take first? a. Observe the dressing for bleeding. b. Check the blood pressure and pulse. c. Assess the patient's respiratory effort. d. Support the patient's head with pillows.

C Airway obstruction is a possible complication after thyroidectomy because of swelling or bleeding at the site or tetany. The priority nursing action is to assess the airway. The other actions are also part of the standard nursing care postthyroidectomy but are not as high of a priority.

A female patient has a sliding hiatal hernia. What nursing interventions will prevent the symptoms of heartburn and dyspepsia that she is experiencing? a) Keep the patient NPO. b) Put the bed in the Trendelenberg position. c) Have the patient eat 4 to 6 smaller meals each day. d) Give various antacids to determine which one works for the patient.

C Eating smaller meals during the day will decrease the gastric pressure and the symptoms of hiatal hernia. Keeping the patient NPO or in a Trendelenberg position are not safe or realistic for a long period of time for any patient. Varying antacids will only be done with the care provider's prescription, so this is not a nursing intervention.

Which information will the nurse teach a patient who has been newly diagnosed with Graves' disease? a. Exercise is contraindicated to avoid increasing metabolic rate. b. Restriction of iodine intake is needed to reduce thyroid activity. c. Antithyroid medications may take several months for full effect. d. Surgery will eventually be required to remove the thyroid gland.

C Medications used to block the synthesis of thyroid hormones may take 2 to 3 months before the full effect is seen. Large doses of iodine are used to inhibit the synthesis of thyroid hormones. Exercise using large muscle groups is encouraged to decrease the irritability and hyperactivity associated with high levels of thyroid hormones. Radioactive iodine is the most common treatment for Graves' disease, although surgery may be used.

Following administration of a dose of metoclopramide (Reglan) to the patient, the nurse determines that the medication has been effective when what is noted? a) Decreased blood pressure b) Absence of muscle tremors c) Relief of nausea and vomiting d) No further episodes of diarrhea

C Metoclopramide is classified as a prokinetic and antiemetic medication. If it is effective, the patient's nausea and vomiting should resolve. Metoclopramide does not affect blood pressure, muscle tremors, or diarrhea.

The patient receiving chemotherapy rings the call bell and reports the onset of nausea. The nurse should prepare an as-needed dose of which medication? a) Morphine sulfate b) Zolpidem (Ambien) c) Ondansetron (Zofran) d) Dexamethasone (Decadron)

C Ondansetron is a 5-HT3 receptor antagonist antiemetic that is especially effective in reducing cancer chemotherapy-induced nausea and vomiting. Morphine sulfate may cause nausea and vomiting. Zolpidem does not relieve nausea and vomiting. Dexamethasone is usually used in combination with ondansetron for acute and chemotherapy-induced emesis.

A patient is being admitted with a diagnosis of Cushing syndrome. Which findings will the nurse expect during the assessment? a. Chronically low blood pressure b. Bronzed appearance of the skin c. Purplish streaks on the abdomen d. Decreased axillary and pubic hair

C Purplish-red striae on the abdomen are a common clinical manifestation of Cushing syndrome. Hypotension and bronzed-appearing skin are manifestations of Addison's disease. Decreased axillary and pubic hair occur with androgen deficiency.

Which intervention will the nurse include in the plan of care for a patient with syndrome of inappropriate antidiuretic hormone (SIADH)? a. Encourage fluids to 2 to 3 L/day. b. Monitor for increasing peripheral edema. c. Offer the patient hard candies to suck on. d. Keep head of bed elevated to 30 degrees.

C Sucking on hard candies decreases thirst for a patient on fluid restriction. Patients with SIADH are on fluid restrictions of 800 to 1000 mL/day. Peripheral edema is not seen with SIADH. The head of the bed is elevated no more than 10 degrees to increase left atrial filling pressure and decrease antidiuretic hormone (ADH) release.

Which information obtained by the nurse in the endocrine clinic about a patient who has been taking prednisone 40 mg daily for 3 weeks is most important to report to the health care provider? a. Patient's blood pressure is 148/94 mm Hg. b. Patient has bilateral 2+ pitting ankle edema. c. Patient stopped taking the medication 2 days ago. d. Patient has not been taking the prescribed vitamin D.

C Sudden cessation of corticosteroids after taking the medication for a week or more can lead to adrenal insufficiency, with problems such as severe hypotension and hypoglycemia. The patient will need immediate evaluation by the health care provider to prevent or treat adrenal insufficiency. The other information will also be reported but does not require rapid treatment.

Which assessment finding for a 33-yr-old female patient admitted with Graves' disease requires the most rapid intervention by the nurse? a. Heart rate 136 beats/min b. Severe bilateral exophthalmos c. Temperature 103.8° F (40.4° C) d. Blood pressure 166/100 mm Hg

C The patient's temperature indicates that the patient may have thyrotoxic crisis and that interventions to lower the temperature are needed immediately. The other findings also require intervention but do not indicate potentially life-threatening complications.

Which assessment finding for a 33-yr-old female patient admitted with Graves'disease requires the most rapid intervention by the nurse? a. Heart rate 136 beats/min b. Severe bilateral exophthalmos c. Temperature 103.8° F (40.4° C) d. Blood pressure 166/100 mm Hg

C The patient's temperature indicates that the patient may have thyrotoxic crisis and that interventions to lower the temperature are needed immediately. The other findings also require intervention but do not indicate potentially life-threatening complications.

A 62-yr-old patient with hyperthyroidism is to be treated with radioactive iodine (RAI). The nurse instructs the patient a. about radioactive precautions to take with all body secretions. b. that symptoms of hyperthyroidism should be relieved in about a week. c. that symptoms of hypothyroidism may occur as the RAI therapy takes effect. d. to discontinue the antithyroid medications taken before the radioactive therapy.

C There is a high incidence of postradiation hypothyroidism after RAI, and the patient should be monitored for symptoms of hypothyroidism. RAI has a delayed response, with the maximum effect not seen for 2 to 3 months, and the patient will continue to take antithyroid medications during this time. The therapeutic dose of radioactive iodine is low enough that no radiation safety precautions are needed.

A 74-year-old female patient with gastroesophageal reflux disease (GERD) takes over-the-counter medications. For which medication, if taken long-term, should the nurse teach about an increased risk of fractures? a) Sucralfate (Carafate) b) Cimetidine (Tagamet) c) Omeprazole (Prilosec) d) Metoclopramide (Reglan)

C There is a potential link between proton pump inhibitors (PPIs) (e.g., omeprazole) use and bone metabolism. Long-term use or high doses of PPIs may increase the risk of fractures of the hip, wrist, and spine. Lower doses or shorter duration of therapy should be considered. REGLAN--> TARDIVE DYSKINESIA

The patient with chronic gastritis is being put on a combination of medications to eradicate H. pylori. Which drugs does the nurse know will probably be used for this patient? a) Antibiotic(s), antacid, and corticosteroid b) Antibiotic(s), aspirin, and antiulcer/protectant c) Antibiotic(s), proton pump inhibitor, and bismuth d) Antibiotic(s) and nonsteroidal antiinflammatory drugs (NSAIDs)

C To eradicate H. pylori, a combination of antibiotics, a proton pump inhibitor, and possibly bismuth (for quadruple therapy) will be used. Corticosteroids, aspirin, and NSAIDs are drugs that can cause gastritis and do not affect H. pylori.

The nurse receives an order for a parenteral dose of promethazine (Phenergan) and prepares to administer the medication to a 38-year-old male patient with nausea and repeated vomiting. Which action is most important for the nurse to take? a) Administer the medication subcutaneously for fast absorption. b) Administer the medication into an arterial line to prevent extravasation. c) Administer the medication deep into the muscle to prevent tissue damage. d) Administer the medication with 0.5 mL of lidocaine to decrease injection pain.

C Promethazine (Phenergan) is an antihistamine administered to relieve nausea and vomiting. Deep muscle injection is the preferred route of injection administration. This medication should not be administered into an artery or under the skin because of the risk of severe tissue injury, including gangrene. When administered IV, a risk factor is that it can leach out from the vein and cause serious damage to surrounding tissue.

Which question will provide the most useful information to a nurse who is interviewing a patient about a possible thyroid disorder? A. "What methods do you use to help cope with stress?" B. "Have you experienced any blurring or double vision?" C. "Have you had a recent unplanned weight gain or loss?" D. "Do you have to get up at night to empty your bladder?"

C. "Have you had a recent unplanned weight gain or loss?" - Because thyroid function affects metabolic rate, changes in weight may indicate hyperfunction or hypofunction of the thyroid gland. Nocturia, visual difficulty, and changes in stress level are associated with other endocrine disorders.

A 28-year-old male patient is diagnosed with polycystic kidney disease. Which information is most appropriate for the nurse to include in teaching at this time? a. Complications of renal transplantation b. Methods for treating severe chronic pain c. Discussion of options for genetic counseling d. Differences between hemodialysis and peritoneal dialysis

C. Discussion of options for genetic counseling Because a 28-year-old patient may be considering having children, the nurse should include information about genetic counseling when teaching the patient. The well-managed patient will not need to choose between hemodialysis and peritoneal dialysis or know about the effects of transplantation for many years. There is no indication that the patient has chronic pain.

A 60-year-old patient is taking spironolactone (Aldactone), a drug that blocks the action of aldosterone on the kidney, for hypertension. The nurse will monitor for A. Increased serum sodium. B. Decreased urinary output. C. Elevated serum potassium. D. Evidence of fluid overload.

C. Elevated serum potassium. - Because aldosterone increases the excretion of potassium, a medication that blocks aldosterone will tend to cause hyperkalemia. Aldosterone also promotes the reabsorption of sodium and water in the renal tubules, so spironolactone will tend to cause increased urine output, a decreased or normal serum sodium level, and signs of dehydration.

A 44-year-old patient is admitted with tetany. Which laboratory value should the nurse monitor? A. Total protein B. Blood glucose C. Ionized calcium D. Serum phosphate

C. Ionized calcium - Tetany is associated with hypocalcemia. The other values would not be useful for this patient.

Which additional information will the nurse need to consider when reviewing the laboratory results for a patient's total calcium level? A. The blood glucose is elevated. B. The phosphate level is normal. C. The serum albumin level is low. D. The magnesium level is normal.

C. The serum albumin level is low. - Part of the total calcium is bound to albumin so hypoalbuminemia can lead to misinterpretation of calcium levels. The other laboratory values will not affect total calcium interpretation.

Which laboratory value should the nurse review to determine whether a patient's hypothyroidism is caused by a problem with the anterior pituitary gland or with the thyroid gland? A. Thyroxine (T4) level B. Triiodothyronine (T3) level C. Thyroid-stimulating hormone (TSH) level D. Thyrotropin-releasing hormone (TRH) level

C. Thyroid-stimulating hormone (TSH) level - A low TSH level indicates that the patient's hypothyroidism is caused by decreased anterior pituitary secretion of TSH. Low T3 and T4 levels are not diagnostic of the primary cause of the hypothyroidism. TRH levels indicate the function of the hypothalamus.

Following rectal surgery, a patient voids about 50 mL of urine every 30 to 60 minutes for the first 4 hours. Which nursing action is most appropriate? a. Monitor the patient's intake and output over night. b. Have the patient drink small amounts of fluid frequently. c. Use an ultrasound scanner to check the postvoiding residual volume. d. Reassure the patient that this is normal after rectal surgery because of anesthesia.

C. Use an ultrasound scanner to check the post voiding residual volume An ultrasound scanner can be used to check for residual urine after the patient voids. Because the patient's history and clinical manifestations are consistent with overflow incontinence, it is not appropriate to have the patient drink small amounts. Although overflow incontinence is not unusual after surgery, the nurse should intervene to correct the physiologic problem, not just reassure the patient. The patient may develop reflux into the renal pelvis and discomfort from a full bladder if the nurse waits to address the problem for several hours.

The nurse identifies a risk factor for kidney and bladder cancer in a patient who relates a history of a. aspirin use. b. tobacco use. c. chronic alcohol abuse. d. use of artificial sweeteners.

Correct answer: b Rationale: Cigarette smoking is the most significant risk factor for renal cell carcinoma. An increased incidence has also been identified in first-degree relatives of patients with this condition. Other risk factors include obesity, hypertension, and exposure to asbestos, cadmium, and gasoline. Risk for renal cancer is also increased in individuals who have acquired cystic disease of the kidney in association with end-stage renal disease. Risk factors for bladder cancer include smoking, exposure to dyes used in the rubber and cable industries, and chronic abuse of phenacetin-containing analgesics.

The patient asks the nurse to explain what the physician meant when he said the patient had an anorectal abscess. Which description should the nurse use to explain this to the patient?

Collection of perianal pus

One of the nurse's most important roles in relation to acute poststreptococcal glomerulonephritis is to a. promote early diagnosis and treatment of sore throats and skin lesions. b. encourage patients to obtain antibiotic therapy for upper respiratory tract infections. c. teach patients with APSGN that long-term prophylactic antibiotic therapy is necessary to prevent recurrence. d. monitor patients for respiratory symptoms that indicate the disease is affecting the alveolar basement membrane.

Correct answer: a Rationale: Acute poststreptococcal glomerulonephritis (APSGN) develops 5 to 21 days after an infection of the tonsils, pharynx, or skin (e.g., streptococcal sore throat, impetigo) by nephrotoxic strains of group A β-hemolytic streptococci. The most important ways to prevent the development of APSGN are early diagnosis and treatment of sore throats and skin lesions.

A patient is admitted to the hospital with severe renal colic. The nurse's first priority in management of the patient is to a. administer opioids as prescribed. b. obtain supplies for straining all urine. c. encourage fluid intake of 3 to 4 L/day. d. keep the patient NPO in preparation for surgery.

Correct answer: a Rationale: Pain management and patient comfort are primary nursing responsibilities in managing an obstructing stone and renal colic.

In planning nursing interventions to increase bladder control in the patient with urinary incontinence, the nurse includes a. teaching the patient to use Kegel exercises. b. clamping and releasing a catheter to increase bladder tone. c. teaching the patient biofeedback mechanisms to suppress the urge to void. d. counseling the patient concerning choice of incontinence containment device.

Correct answer: a Rationale: Pelvic floor muscle training (i.e., Kegel exercises) is used to manage stress, urge, or mixed urinary incontinence.

A patient with a ureterolithotomy returns from surgery with a nephrostomy tube in place. Postoperative nursing care of the patient includes a. encouraging the patient to drink fruit juices and milk. b. encouraging fluids of at least 2 to 3 L/day after nausea has subsided. c. irrigating the nephrostomy tube with 10 mL of normal saline solution as needed. d. notifying the physician if nephrostomy tube drainage is more than 30 mL/hr.

Correct answer: b Rationale: The nephrostomy tube is inserted directly into the renal pelvis and attached to connecting tubing for closed drainage. The catheter should never be kinked, compressed, or clamped. If the patient complains of excessive pain in the area, or if drainage around the tube is excessive, check the catheter for patency. If irrigation is ordered, strict aseptic technique is required. To prevent overdistention of the renal pelvis and renal damage, no more than 5 mL of sterile saline solution is gently instilled at one time. Infection and secondary stone formation are complications associated with the insertion of a nephrostomy tube. Patients should drink 2 to 3 L of fluid per day to reduce risk of infection and stone formation

The nurse teaches the female patient who has frequent UTIs that she should a. take tub baths with bubble bath. b. urinate before and after sexual intercourse. c. take prophylactic sulfonamides for the rest of her life. d. restrict fluid intake to prevent the need for frequent voiding

Correct answer: b Rationale: When teaching a patient to prevent a recurrence of a urinary tract infection, the nurse should explain the importance of emptying the bladder before and after sexual intercourse.

A patient has had a cystectomy and ileal conduit diversion performed. Four days postoperatively, mucous shreds are seen in the drainage bag. The nurse should a. notify the physician. b. notify the charge nurse. c. irrigate the drainage tube. d. chart it as a normal observation.

Correct answer: d Rationale: Patients with an ileal conduit have mucus in the urine. The mucus is secreted by intestinal mucosa, which is used to create the ileal conduit, in response to the irritating effect of urine.

The edema that occurs in nephrotic syndrome is due to a. increased hydrostatic pressure caused by sodium retention. b. decreased aldosterone secretion from adrenal insufficiency. c. increased fluid retention caused by decreased glomerular filtration. d. decreased colloidal osmotic pressure caused by loss of serum albumin.

Correct answer: d Rationale: The increased permeability of the glomerular membrane found in nephrotic syndrome is responsible for the massive excretion of protein in the urine. This results in decreased serum protein levels and subsequent edema formation. Ascites and anasarca (i.e., massive generalized edema) develop if hypoalbuminemia is severe.

In teaching a patient with pyelonephritis about the disorder, the nurse informs the patient that the organisms that cause pyelonephritis most commonly reach the kidneys through a. the bloodstream. b. the lymphatic system. c. a descending infection. d. an ascending infection.

Correct answer: d Rationale: The organisms that usually cause urinary tract infections (UTIs) are introduced via the ascending route from the urethra, and the infections originate in the perineum.

The medications prescribed for the patient with IBD include cobalamin and iron injections. What is the rationale for using these drugs?

Correct malnutrition.

A 72-year-old patient was admitted with epigastric pain due to a gastric ulcer. Which patient assessment warrants an urgent change in the nursing plan of care? a) Chest pain relieved with eating or drinking water b) Back pain 3 or 4 hours after eating a meal c) Burning epigastric pain 90 minutes after breakfast d) Rigid abdomen and vomiting following indigestion

D A rigid abdomen with vomiting in a patient who has a gastric ulcer indicates a perforation of the ulcer, especially if the manifestations of perforation appear suddenly. Midepigastric pain is relieved by eating, drinking water, or antacids with duodenal ulcers, not gastric ulcers. Back pain 3-4 hours after a meal is more likely to occur with a duodenal ulcer. Burning epigastric pain 1-2 hours after a meal is an expected manifestation of a gastric ulcer related to increased gastric secretions and does not cause an urgent change in the nursing plan of care.

Which information is most important for the nurse to communicate rapidly to the health care provider about a patient admitted with possible syndrome of inappropriate antidiuretic hormone (SIADH)? a. The patient has a recent weight gain of 9 lb. b. The patient complains of dyspnea with activity. c. The patient has a urine specific gravity of 1.025. d. The patient has a serum sodium level of 118 mEq/L.

D A serum sodium of less than 120 mEq/L increases the risk for complications such as seizures and needs rapid correction. The other data are not unusual for a patient with SIADH and do not indicate the need for rapid action.

Which finding by the nurse when assessing a patient with a large pituitary adenoma is most important to report to the health care provider? a. Changes in visual field b. Milk leaking from breasts c. Blood glucose 150 mg/dL d. Nausea and projectile vomiting

D Nausea and projectile vomiting may indicate increased intracranial pressure, which will require rapid actions for diagnosis and treatment. Changes in the visual field, elevated blood glucose, and galactorrhea are common with pituitary adenoma, but these do not require rapid action to prevent life-threatening complications.

A 44-yr-old female patient with Cushing syndrome is admitted for adrenalectomy. Which intervention by the nurse will be most helpful for the patient problem of disturbed body image related to changes in appearance? a. Reassure the patient that the physical changes are very common in patients with Cushing syndrome. b. Discuss the use of diet and exercise in controlling the weight gain associated with Cushing syndrome. c. Teach the patient that the metabolic impact of Cushing syndrome is of more importance than appearance. d. Remind the patient that most of the physical changes caused by Cushing syndrome will resolve after surgery.

D The most reassuring and accurate communication to the patient is that the physical and emotional changes caused by the Cushing syndrome will resolve after hormone levels return to normal postoperatively. Reassurance that the physical changes are expected or that there are more serious physiologic problems associated with Cushing syndrome are not therapeutic responses. The patient's physiological changes are caused by the high hormone levels, not by the patient's diet or exercise choices.

A 61-year-old female patient admitted with pneumonia has a total serum calcium level of 13.3 mg/dL (3.3 mmol/L). The nurse will anticipate the need to teach the patient about testing for _____ levels. A. Calcitonin B. Catecholamine C. Thyroid hormone D. Parathyroid hormone

D. Parathyroid hormone - Parathyroid hormone is the major controller of blood calcium levels. Although calcitonin secretion is a countermechanism to parathyroid hormone, it does not play a major role in calcium balance. Catecholamine and thyroid hormone levels do not affect serum calcium level.

What is a nursing intervention that is indicated for a male patient following an inguinal herniorrhaphy

Elevating the scrotum with a scrotal support

The nurse plans teaching for the patient with a colostomy but the patient refuses to look at the nurse or the stoma, stating, "I just can't see myself with this thing." What is the best nursing intervention for this patient?

Encourage the patient to share concerns and ask questions.

A 22 year old patient calls the outpatient clinic complaining of nausea and vomiting and right lower abdominal pain. What should the nurse advise the patient to do?

Have the symptoms evaluated by a health care provider right away

Which organ in a patient secretes atrial natriuretic peptide (ANP) hormone?

Heart

C

The nurse assesses a patient who has been hospitalized for 2 days. The patient has been receiving normal saline IV at 100 mL/hr, has a nasogastric tube to low suction, and is NPO. Which assessment finding would be a priority for the nurse to report to the health care provider? a. Oral temperature of 100.1° F b. Serum sodium level of 138 mEq/L (138 mmol/L) c. Gradually decreasing level of consciousness (LOC) d. Weight gain of 2 pounds (1 kg) above the admission weight

B

M.J. calls to tell the nurse that her elderly mother, who is 85 years of age, has been nauseated all day and has vomited twice. Before the nurse hangs up and telephones the healthcare provider to communicate your assessment data, she should instruct M.J. to: A. administer antispasmodic drugs and observe skin turgor B. give her mother sips of water and elevate the head of her bed to prevent aspiration C. offer her mother a high-protein liquid supplement to drink to maintain her nutritional needs D. Offer her mother large quantities of gatorade to drink because elderly people are at risk for sodium depletion

The RN coordinating the care for a patient who is 2 days postop following an anterior- posterior resection with colostomy may delegate which interventions to the LPN?

Monitor and record the volume, color, and odor of the drainage Empty the ostomy bag and measure and record the amount of drainage

An 82 year old man is admitted with an acute attack of diverticulitis. What should the nurse include in his care?

Monitor for signs of peritonitis

For the patient hospitalized with IBD, which treatments would be used to rest the bowel?

NPO IV fluids NG suctioning Parenteral nutrition

B

When caring for a patient with renal failure on a low phosphate diet, the nurse will inform unlicensed assistive personnel (UAP) to remove which food from the patient's food tray? a. Grape juice b. Milk carton c. Mixed green salad d. Fried chicken breast

Priority Decision: Based on the assessment data presented, what are the priority nursing diagnoses? Are there any collaborative problems? Patient Profile: Male patient, married, 55-year-old mechanic who has been healthy all of his life until he passed some blood in his urine and saw a urologist at his wife's insistence. A urine specimen for cytology revealed atypical cells, and a diagnosis of bladder cancer was made following a cystoscopy with biopsy of bladder tissue. The tumor was removed with a transurethral resection and laser cauterization. Intravesical therapy with bacille Calmette-Guérin (BCG), a weak- ened strain of Mycobacterium bovis, is planned. Subjective Data • Has smoked a pack of cigarettes a day since he was a teenager • Says he dreads having the chemotherapy because he has heard cancer drugs cause severe side effects Objective Data • Cystoscopy and biopsy results: Moderately differentiated stage II tumor on the left lateral bladder wall, with T1N0M0 pathologic stage • Continues to have gross hematuria

Nursing diagnoses: • Anxiety related to unknown outcome • Impaired urinary elimination related to effects of treatment • Acute pain related to effects of treatment • Risk for infection related to effects of treatment • Risk for sexual dysfunction related to reactions to treatment Collaborative problems: Potential complication: bladder injury, impaired skin integrity

Post-op care for transurethral resection and laser cauterization?

Opioids and stool softeners may be used for a short time after the procedure. Patient will be taught to drink a large volume of fluid and monitor the color and consistency of the urine. The urine will be pink at first but should not be red or contain clots. Seven to 10 days after the tumor resection Patient may observe dark red or rust-colored flecks in the urine from the healing tumor resection sites.

Which hormone increases bone mineralization?

Parathyroid hormone

The nurse reviews the laboratory reports of several female patients. Which patient has abnormal follicle stimulating hormone (FSH) and luteinizing hormone (LH) levels?

Patient C

D

Spironolactone (Aldactone), an aldosterone antagonist, is prescribed for a patient. Which statement by the patient indicates that the teaching about this medication has been effective? a. "I will try to drink at least 8 glasses of water every day." b. "I will use a salt substitute to decrease my sodium intake." c. "I will increase my intake of potassium-containing foods." d. "I will drink apple juice instead of orange juice for breakfast."

Which function of prolactin should the nurse include in a teaching session for a patient who is newly pregnant?

Stimulation of milk production

A nurse is doing a nursing assessment on a patient with chronic constipation. What data obtained during the interview may be a factor contributing to the constipation?

Suppressing the urge to defecate while at work

A patient with a gunshot wound to the abdomen complains of increasing abdominal pain several hours after surgery to repair the bowel. What action should the nurse take first?

Take the patient's vital signs

Following a hemorrhoidectomy, what should the nurse advise the patient to do?

Take warm sitz baths several times a day to promote comfort and cleaning.

A

The home health nurse cares for an alert and oriented older adult patient with a history of dehydration. Which instructions should the nurse give to this patient related to fluid intake? a. "Increase fluids if your mouth feels dry. b. "More fluids are needed if you feel thirsty." c. "Drink more fluids in the late evening hours." d. "If you feel lethargic or confused, you need more to drink."

C

The long-term care nurse is evaluating the effectiveness of protein supplements for an older resident who has a low serum total protein level. Which assessment finding indicates that the patient's condition has improved? a. Hematocrit 28% b. Absence of skin tenting c. Decreased peripheral edema d. Blood pressure 110/72 mm Hg

D

The nurse explains to the patient with gastroesophageal reflux diease that this disorder: A. results in acid erosion and ulceration of the esophagus caused by frequent vomiting B. will require surgical wrapping or repair of the pyloric sphincter to control the symptoms C. is the protrusion of a portion of the stomach into the esophagus through an opening in the diaphragm D. often involves relaxation of the lower esophageal sphincter, allowing stomach contents to back up into the esophagus

D

The nurse is caring for a patient who has a calcium level of 12.1 mg/dL. Which nursing action should the nurse include on the care plan? a. Maintain the patient on bed rest. b. Auscultate lung sounds every 4 hours. c. Monitor for Trousseau's and Chvostek's signs. d. Encourage fluid intake up to 4000 mL every day.

If surgery is needed (for bladder cancer), what elements will be considered in deciding the type of surgery?

The patient's ability and readiness to learn must be considered. His anxiety and fear may affect the teaching. Psychosocial aspects of living with a stoma may affect the choice. Acceptance of the surgery and of alterations in body image is needed to ensure the patient's best adjustment. If a continent diversion is used, the patient must be able to catheterize the pouch every 4 to 6 hours and irrigate it daily. Orthotopic bladder reconstruction may be considered if the cancer did not involve the bladder neck or urethra and the patient has normal renal and liver function, a longer than 1- to 2-year life expectancy, adequate motor skills, and no history of inflammatory bowel disease or colon cancer. Obese patients are not good candidates for orthotopic bladder reconstruction.

C

The pernicious anemia that may accompany gastritis is due to which of the following: A. chronic autoimmune destructionof cobalamin stores in the body B. progressive gastric atrophy from chronic breakage in the mucosal barrier and blood loss C. a lack of intrinsic factor normally produced by acid-secreting cells of the gastric mucosa D. hyperchlorhydria resulting from an increase in acid-secreting parietal cells and degradation of RBCs.

B

Which action can the registered nurse (RN) who is caring for a critically ill patient with multiple IV lines delegate to an experienced licensed practical/vocational nurse (LPN/LVN)? a. Administer IV antibiotics through the implantable port. b. Monitor the IV sites for redness, swelling, or tenderness. c. Remove the patient's nontunneled subclavian central venous catheter. d. Adjust the flow rate of the 0.9% normal saline in the peripheral IV line.

C

Which action should the nurse take first when a patient complains of acute chest pain and dyspnea soon after insertion of a centrally inserted IV catheter? a. Notify the health care provider. b. Offer reassurance to the patient. c. Auscultate the patient's breath sounds. d. Give the prescribed PRN morphine sulfate IV.

The nurse performs a detailed assessment of the abdomen of a patient with a possible bowel obstruction, knowing that manifestations of a obstruction in the large intestine are (select all that apply): a. persistent abdominal pain b. marked abdominal distention c. diarrhea that is loose or liquid d. colicky, severe, intermittent pain e. profuse committing that relieves abdominal pain

a & b Rationale: With lower intestinal obstructions, abdominal distention is markedly increased and pain is persistent. Onset of a large intestine obstruction is gradual, vomiting is rare, and there is usually absolute constipation, not diarrhea.

Which statement by the patient with type 2 diabetes is accurate? a) "I am supposed to have a meal or snack if I drink alcohol." b) I am not allowed to eat any sweets because of my diabetes. c) "I do not need to watch what I eat because my diabetes is not the bad kind." d) "The amount of fat in my diet is not important. Only carbohydrates raise my blood sugar."

a) "I am supposed to have a meal or snack if I drink alcohol." Alcohol should be consumed with food to reduce the risk of hypoglycemia.

Analyze the following diagnostic findings for your patient with type 2 diabetes. Which result will need further assessment? a) A1C 9% b) BP 126/80 mmHg c) FBG 130 mg/dL (7.2 mmol/L) d) LDL cholesterol 100 mg/dL (2.6 mmol/L)

a) A1C 9% Lowering hemoglobin A1C (to less than 7%) reduces microvascular and neuropathic complications. Tighter glycemic control (normal hemoglobin A1C level, less than 6%) may further reduce complications but increases hypoglycemia risk.

A patient is admitted with diabetes mellitus, malnutrition, and cellulitis. The patient's potassium level is 5.6 mEq/L. The nurse understands that what could be contributing factors for this laboratory result (select all that apply)? a. The level may be increased as a result of dehydration that accompanies hyperglycemia. b. The patient may be excreting extra sodium and retaining potassium because of malnutrition. c. The level is consistent with renal insufficiency that can develop with renal nephropathy. d. The level may be raised as a result of metabolic ketoacidosis caused by hyperglycemia. e. This level demonstrates adequate treatment of the cellulitis and effective serum glucose control.

a, c, & d The additional stress of cellulitis may lead to an increase in the patient's serum glucose levels. Dehydration may cause hemoconcentration, resulting in elevated serum readings. Kidneys may have difficulty excreting potassium if renal insufficiency exists. Finally, the nurse must consider the potential for metabolic ketoacidosis since potassium will leave the cell when hydrogen enters in an attempt to compensate for a low pH. Malnutrition does not cause sodium excretion accompanied by potassium retention. Thus it is not a contributing factor to this patient's potassium level. The elevated potassium level does not demonstrate adequate treatment of cellulitis or effective serum glucose control.

Which are appropriate therapies for patients with diabetes mellitus (select all that apply)? a) Use of statins to treat dyslipidemia b) Use of diuretics to treat nephropathy c) Use of ACE inhibitors to treat nephropathy d) Use of serotonin agonists to decrease appetite e) Use of laser photocoagulation to treat retinopathy

a, c, & e In patients with diabetes who have microalbuminuria or macroalbuminuria, angiotensin-converting enzyme (ACE) inhibitors (e.g., lisinopril [Prinivil, Zestril]) or angiotensin II receptor antagonists (ARBs) (e.g., losartan [Cozaar]) should be used. Both classes of drugs are used to treat hypertension and have been found to delay the progression of nephropathy in patients with diabetes. The statin drugs are the most widely used lipid-lowering agents. Laser photocoagulation therapy is indicated to reduce the risk of vision loss in patients with proliferative retinopathy, in those with macular edema, and in some cases of nonproliferative retinopathy.

You are caring for a patient with newly diagnosed type 1 diabetes. What information is essential to include in your patient teaching before discharge from the hospital (select all that apply)? a) Insulin administration b) Elimination of sugar from diet c) Need to reduce physical activity d) Use of a portable blood glucose monitor e) Hypoglycemia prevention, symptoms, and treatment

a, d, & e The nurse ensures that the patient understands the proper use of insulin. The nurse teaches the patient how to use the portable blood glucose monitor and how to recognize and treat signs and symptoms of hypoglycemia and hyperglycemia.

Which question during the assessment of a diabetic patient will help the nurse identify autonomic neuropathy? a. "Do you feel bloated after eating?" b. "Have you seen any skin changes?" c. "Do you need to increase your insulin dosage when you are stressed?" d. "Have you noticed any painful new ulcerations or sores on your feet?"

a. "Do you feel bloated after eating?"

Which statement by the patient indicates a need for additional instruction in administering insulin? a. "I need to rotate injection sites among my arms, legs, and abdomen each day." b. "I can buy the 0.5 mL syringes because the line markings will be easier to see." c. "I should draw up the regular insulin first after injecting air into the NPH bottle." d. "I do not need to aspirate the plunger to check for blood before injecting insulin."

a. "I need to rotate injection sites among my arms, legs, and abdomen each day."

The nurse teaches a 21-year-old female patient who came to the clinic to discuss interventions to prevent a recurrence of urinary tract infections. Which statement, if made by the patient, indicates that teaching was effective? a. "I will urinate before and after having intercourse." Correct b. "I will use vinegar as a vaginal douche every week." c. "I should drink three 8-ounce glasses of water daily." d. "I can stop the antibiotics when symptoms disappear."

a. "I will urinate before and after having intercourse." The woman should empty her bladder before and after sexual intercourse. She should avoid vaginal douches and maintain adequate oral fluid intake (15 mL per pound of body weight). All of the antibiotics should be taken as prescribed even if symptoms are no longer present.

The nurse is preparing to insert a nasogastric (NG) tube into a 68-year-old female patient who is nauseated and vomiting. She has an abdominal mass and suspected small intestinal obstruction. The patient asks the nurse why this procedure is necessary. What response by the nurse is most appropriate? a. "The tube will help to drain the stomach contents and prevent further vomiting." b. "The tube will push past the area that is blocked and thus help to stop the vomiting." c. "The tube is just a standard procedure before many types of surgery to the abdomen." d. "The tube will let us measure your stomach contents so that we can plan what type of IV fluid replacement would be best."

a. "The tube will help to drain the stomach contents and prevent further vomiting." The NG tube is used to decompress the stomach by draining stomach contents and thereby prevent further vomiting. The NG tube will not push past the blocked area. Potential surgery is not currently indicated. The location of the obstruction will determine the type of fluid to use, not measure the amount of stomach contents.

A patient receives aspart (NovoLog) insulin at 8:00 AM. Which time will it be most important for the nurse to monitor for symptoms of hypoglycemia? a. 10:00 AM b. 12:00 AM c. 2:00 PM d. 4:00 PM

a. 10:00 AM

Number in sequence the following ascending pathologic changes that occur in the urinary tract in the presence of a bladder outlet obstruction. a. Hydronephrosis b. Reflux of urine into ureter c. Bladder detrusor muscle hypertrophy d. Ureteral dilation e. Renal atrophy f. Vesicoureteral reflux g. Large residual urine in bladder h. Chronic pyelonephritis

a. 6; b. 3; c. 1; d. 4; e. 8; f. 5; g. 2; h. 7

Which nursing diagnosis is a priority in the care of a patient with renal calculi? a. Acute pain b. Risk for constipation c. Deficient fluid volume d. Risk for powerlessness

a. Acute pain Urinary stones are associated with severe abdominal or flank pain. Deficient fluid volume is unlikely to result from urinary stones, whereas constipation is more likely to be an indirect consequence rather than a primary clinical manifestation of the problem. The presence of pain supersedes powerlessness as an immediate focus of nursing care.

Which nursing action is of highest priority for a 68-year-old patient with renal calculi who is being admitted to the hospital with gross hematuria and severe colicky left flank pain? a. Administer prescribed analgesics. b. Monitor temperature every 4 hours. c. Encourage increased oral fluid intake. d. Give antiemetics as needed for nausea.

a. Administer prescribed analgesics. Although all of the nursing actions may be used for patients with renal lithiasis, the patient's presentation indicates that management of pain is the highest priority action. If the patient has urinary obstruction, increasing oral fluids may increase the symptoms. There is no evidence of infection or nausea.

Which classification of urinary tract infection (UTI) is described as infection of the renal parenchyma, renal pelvis, and ureters? a. Upper UTI b. Lower UTI c. Complicated UTI d. Uncomplicated UTI

a. An upper urinary tract infection (UTI) affects the renal parenchyma, renal pelvis, and ureters. A lower UTI is an infection of the bladder and/or urethra. A complicated UTI exists in the presence of obstruction, stones, or preexisting diseases. An uncomplicated UTI occurs in an otherwise normal urinary tract.

Which classification of urinary tract infection (UTI) is described as infection of the renal parenchyma, renal pelvis, and ureters? a. Upper UTI b. Lower UTI c. Complicated UTI d. Uncomplicated UTI

a. An upper urinary tract infection (UTI) affects the renal parenchyma, renal pelvis, and ureters. A lower UTI is an infection of the bladder and/or urethra. A complicated UTI exists in the presence of obstruction, stones, or preexisting diseases. An uncomplicated UTI occurs in an otherwise normal urinary tract.

The nurse is assigned to the care of a 64-year-old patient diagnosed with type 2 diabetes. In formulating a teaching plan that encourages the patient to actively participate in management of the diabetes, what should be the nurse's initial intervention? a. Assess patient's perception of what it means to have diabetes. b. Ask the patient to write down current knowledge about diabetes. c. Set goals for the patient to actively participate in managing his diabetes. d. Assume responsibility for all of the patient's care to decrease stress level.

a. Assess patient's perception of what it means to have diabetes. In order for teaching to be effective, the first step is to assess the patient. Teaching can be individualized once the nurse is aware of what a diagnosis of diabetes means to the patient. After the initial assessment, current knowledge can be assessed, and goals should be set with the patient. Assuming responsibility for all of the patient's care will not facilitate the patient's health.

A 44-year-old patient is unable to void after having an open loop resection and fulguration of the bladder. Which nursing action should be implemented first? a. Assist the patient to soak in a 15-minute sitz bath. b. Insert a straight urethral catheter and drain the bladder. c. Encourage the patient to drink several glasses of water. d. Teach the patient how to do isometric perineal exercises.

a. Assist the patient to soak in a 15-minute sitz bath. Sitz baths will relax the perineal muscles and promote voiding. Although the patient should be encouraged to drink fluids and Kegel exercises are helpful in the prevention of incontinence, these activities would not be helpful for a patient experiencing retention. Catheter insertion increases the risk for urinary tract infection (UTI) and should be avoided when possible

When obtaining a nursing history from a patient with cancer of the urinary system, what does the nurse recognize as a risk factor associated with both kidney cancer and bladder cancer? a. Smoking b. Family history of cancer c. Chronic use of phenacetin d. Chronic, recurrent nephrolithiasis

a. Both cancer of the kidney and cancer of the bladder are associated with smoking. A family history of renal cancer is a risk factor for kidney cancer and cancer of the bladder has been associated with the use of phenacetin-containing analgesics and recurrent upper UTIs.

When obtaining a nursing history from a patient with cancer of the urinary system, what does the nurse recognize as a risk factor associated with both kidney cancer and bladder cancer? a. Smoking b. Family history of cancer c. Chronic use of phenacetin d. Chronic, recurrent nephrolithiasis

a. Both cancer of the kidney and cancer of the bladder are associated with smoking. A family history of renal cancer is a risk factor for kidney cancer and cancer of the bladder has been associated with the use of phenacetin-containing analgesics and recurrent upper UTIs.

Besides being mixed with struvite or oxalate stones, what characteristic is associated with calcium phosphate calculi? a. Associated with alkaline urine b. Genetic autosomal recessive defect c. Three times as common in women as in men d. Defective gastrointestinal (GI) and kidney absorption

a. Calcium phosphate calculi are typically mixed with struvite or oxalate stones and related to alkaline urine. Cystine calculi are associated with a genetic autosomal recessive defect and defective GI and kidney absorption of cystine. Struvite calculi are three to four times more common in women than in men.

Besides being mixed with struvite or oxalate stones, what characteristic is associated with calcium phosphate calculi? a. Associated with alkaline urine b. Genetic autosomal recessive defect c. Three times as common in women as in men d. Defective gastrointestinal (GI) and kidney absorption

a. Calcium phosphate calculi are typically mixed with struvite or oxalate stones and related to alkaline urine. Cystine calculi are associated with a genetic autosomal recessive defect and defective GI and kidney absorption of cystine. Struvite calculi are three to four times more common in women than in men.

The nurse is caring for a patient who has had an ileal conduit for several years. Which nursing action could be delegated to unlicensed assistive personnel (UAP)? a. Change the ostomy appliance. b. Choose the appropriate ostomy bag. c. Monitor the appearance of the stoma. d. Assess for possible urinary tract infection (UTI).

a. Change the ostomy appliance. Changing the ostomy appliance for a stable patient could be done by UAP. Assessments of the site, choosing the appropriate ostomy bag, and assessing for (UTI) symptoms require more education and scope of practice and should be done by the registered nurse (RN).

A patient is admitted to the emergency department with possible renal trauma after an automobile accident. Which prescribed intervention will the nurse implement first? a. Check blood pressure and heart rate. b. Administer morphine sulfate 4 mg IV. c. Transport to radiology for an intravenous pyelogram. d. Insert a urethral catheter and obtain a urine specimen.

a. Check blood pressure and heart rate. Because the kidney is very vascular, the initial action with renal trauma will be assessment for bleeding and shock. The other actions are also important once the patient's cardiovascular status has been determined and stabilized.

The nurse is assisting a patient with newly diagnosed type 2 diabetes to learn dietary planning as part of the initial management of diabetes. The nurse would encourage the patient to limit intake of which foods to help reduce the percent of fat in the diet? a. Cheese b. Broccoli c. Chicken d. Oranges

a. Cheese Cheese is a product derived from animal sources and is higher in fat and calories than vegetables, fruit, and poultry. Excess fat in the diet is limited to help avoid macrovascular changes.

The nurse has been teaching a patient with diabetes mellitus how to perform self-monitoring of blood glucose (SMBG). During evaluation of the patient's technique, the nurse identifies a need for additional teaching when the patient does what? a. Chooses a puncture site in the center of the finger pad. b. Washes hands with soap and water to cleanse the site to be used. c. Warms the finger before puncturing the finger to obtain a drop of blood. d. Tells the nurse that the result of 110 mg/dL indicates good control of diabetes.

a. Chooses a puncture site in the center of the finger pad. The patient should select a site on the sides of the fingertips, not on the center of the finger pad as this area contains many nerve endings and would be unnecessarily painful. Washing hands, warming the finger, and knowing the results that indicate good control all show understanding of the teaching.

In order to assist an older diabetic patient to engage in moderate daily exercise, which action is most important for the nurse to take? a. Determine what type of activities the patient enjoys. b. Remind the patient that exercise will improve self-esteem. c. Teach the patient about the effects of exercise on glucose level. d. Give the patient a list of activities that are moderate in intensity.

a. Determine what type of activities the patient enjoys.

A stroke patient who primarily uses a wheelchair for mobility has diarrhea with fecal incontinence. What should the nurse assess first? a. Fecal impaction b. Perineal hygiene c. Dietary fiber intake d. Antidiarrheal agent use

a. Fecal impaction Patients with limited mobility are at risk for fecal impactions due to constipation that may lead to liquid stool leaking around the hardened impacted feces, so assessing for fecal impaction is the priority. Perineal hygiene can be assessed at the same time. Assessing the dietary fiber and fluid intake and antidiarrheal agent use will be assessed and considered next.

The nurse has administered 4 oz of orange juice to an alert patient whose blood glucose was 62 mg/dL. Fifteen minutes later, the blood glucose is 67 mg/dL. Which action should the nurse take next? a. Give the patient 4 to 6 oz more orange juice. b. Administer the PRN glucagon (Glucagon) 1 mg IM. c. Have the patient eat some peanut butter with crackers. d. Notify the health care provider about the hypoglycemia.

a. Give the patient 4 to 6 oz more orange juice.

Laboratory results have been obtained for a 50-year-old patient with a 15-year history of type 2 diabetes. Which result reflects the expected pattern accompanying macrovascular disease as a complication of diabetes? a. Increased triglyceride levels b. Increased high-density lipoproteins (HDL) c. Decreased low-density lipoproteins (LDL) d. Decreased very-low-density lipoproteins (VLDL)

a. Increased triglyceride levels Macrovascular complications of diabetes include changes to large- and medium-sized blood vessels. They include cerebrovascular, cardiovascular, and peripheral vascular disease. Increased triglyceride levels are associated with these macrovascular changes. Increased HDL, decreased LDL, and decreased VLDL are positive in relation to atherosclerosis development.

A 76-year-old with benign prostatic hyperplasia (BPH) is agitated and confused, with a markedly distended bladder. Which intervention prescribed by the health care provider should the nurse implement first? a. Insert a urinary retention catheter. b. Schedule an intravenous pyelogram (IVP). c. Draw blood for a serum creatinine level. d. Administer lorazepam (Ativan) 0.5 mg PO.

a. Insert a urinary retention catheter. The patient's history and clinical manifestations are consistent with acute urinary retention, and the priority action is to relieve the retention by catheterization. The BUN and creatinine measurements can be obtained after the catheter is inserted. The patient's agitation may resolve once the bladder distention is corrected, and sedative drugs should be used cautiously in older patients. The IVP is an appropriate test but does not need to be done urgently.

What is the nurse's priority when changing the appliance of a patient with an ileal conduit? a. Keep the skin free of urine. b. Inspect the peristomal area. c. Cleanse and dry the area gently. d. Affix the appliance to the faceplate

a. Keep the skin free of urine. The nurse's priority is to keep the skin free of urine because the peristomal skin is at high risk for damage from the urine if it is alkaline. The peristomal area will be assessed; the area will be gently cleaned and dried, and the appliance will be affixed to the faceplate if one is being used, but these are not as much of a priority as keeping the skin free of urine to prevent skin damage.

Eight months after the delivery of her first child, a 31-year-old woman has sought care because of occasional incontinence that she experiences when sneezing or laughing. Which measure should the nurse first recommend in an attempt to resolve the woman's incontinence? a. Kegel exercises b. Use of adult incontinence pads c. Intermittent self-catheterization d. Dietary changes including fluid restriction

a. Kegel exercises Patients who experience stress incontinence frequently benefit from Kegel exercises (pelvic floor muscle exercises). The use of incontinence pads does not resolve the problem, and intermittent self-catheterization would be a premature recommendation. Dietary changes are not likely to influence the patient's urinary continence.

A 32-year-old patient with diabetes is starting on intensive insulin therapy. Which type of insulin will the nurse discuss using for mealtime coverage? a. Lispro (Humalog) b. Glargine (Lantus) c. Detemir (Levemir) d. NPH (Humulin N)

a. Lispro (Humalog)

The nurse is conducting discharge teaching for a patient with metastatic lung cancer who was admitted with a bowel impaction. Which instructions would be most helpful to prevent further episodes of constipation? a. Maintain a high intake of fluid and fiber in the diet. b. Reduce intake of medications causing constipation. c. Eat several small meals per day to maintain bowel motility. d. Sit upright during meals to increase bowel motility by gravity.

a. Maintain a high intake of fluid and fiber in the diet. Increased fluid intake and a high-fiber diet reduce the incidence of constipation caused by immobility, medications, and other factors. Fluid and fiber provide bulk that in turn increases peristalsis and bowel motility. Analgesics taken for lung cancer probably cannot be reduced. Other medications may decrease constipation, but it is best to avoid laxatives. Eating several small meals per day and position do not facilitate bowel motility. Defecation is easiest when the person sits on the commode with the knees higher than the hips.

Which nursing action can the nurse delegate to unlicensed assistive personnel (UAP) who are working in the diabetic clinic? a. Measure the ankle-brachial index. b. Check for changes in skin pigmentation. c. Assess for unilateral or bilateral foot drop. d. Ask the patient about symptoms of depression.

a. Measure the ankle-brachial index.

The nurse plans care for the patient with APSGN based on what knowledge? a. Most patients with APSGN recover completely or rapidly improve with conservative management. b. Chronic glomerulonephritis leading to renal failure is a common sequela to acute glomerulonephritis. c. Pulmonary hemorrhage may occur as a result of antibodies also attacking the alveolar basement membrane. d. A large percentage of patients with APSGN develop rapidly progressive glomerulonephritis, resulting in kidney failure.

a. Most patients recover completely from acute poststreptococcal glomerulonephritis (APSGN) with supportive treatment. Chronic glomerulonephritis that progresses insidiously over years and rapidly progressive glomerulonephritis that results in renal failure within weeks or months occur in only a few patients with APSGN. In Goodpasture syndrome, antibodies are present against both the GBM and the alveolar basement membrane of the lungs and dysfunction of both renal and pulmonary are present.

The nurse plans care for the patient with APSGN based on what knowledge? a. Most patients with APSGN recover completely or rapidly improve with conservative management. b. Chronic glomerulonephritis leading to renal failure is a common sequela to acute glomerulonephritis. c. Pulmonary hemorrhage may occur as a result of antibodies also attacking the alveolar basement membrane. d. A large percentage of patients with APSGN develop rapidly progressive glomerulonephritis, resulting in kidney failure.

a. Most patients recover completely from acute poststreptococcal glomerulonephritis (APSGN) with supportive treatment. Chronic glomerulonephritis that progresses insidiously over years and rapidly progressive glomerulonephritis that results in renal failure within weeks or months occur in only a few patients with APSGN. In Goodpasture syndrome, antibodies are present against both the GBM and the alveolar basement membrane of the lungs and dysfunction of both renal and pulmonary are present.

The urinalysis of a male patient reveals a high microorganism count. What data should the nurse use to determine the area of the urinary tract that is infected (select all that apply)? a. Pain location Correct b. Fever and chills Incorrect c. Mental confusion Incorrect d. Urinary hesitancy Incorrect e. Urethral discharge Correct f. Post-void dribbling

a. Pain location e. Urethral discharge Although all the manifestations are evident with urinary tract infections (UTIs), pain location is useful in differentiating among pyelonephritis, cystitis, and urethritis because flank pain is characteristic of pyelonephritis, but dysuria is characteristic of cystitis and urethritis. Urethral discharge is indicative of urethritis, not pyelonephritis or cystitis. Fever and chills and mental confusion are nonspecific indicators of UTIs. Urinary hesitancy and postvoid dribbling may occur with a UTI but may also occur with prostate enlargement in the male patient.

A 27-year-old patient admitted with diabetic ketoacidosis (DKA) has a serum glucose level of 732 mg/dL and serum potassium level of 3.1 mEq/L. Which action prescribed by the health care provider should the nurse take first? a. Place the patient on a cardiac monitor. b. Administer IV potassium supplements. c. Obtain urine glucose and ketone levels. d. Start an insulin infusion at 0.1 units/kg/hr.

a. Place the patient on a cardiac monitor.

The patient has a thoracic spinal cord lesion and incontinence that occurs equally during the day and night. What type of incontinence is this patient experiencing? a. Reflex incontinence b. Overflow incontinence c. Functional incontinence d. Incontinence after trauma

a. Reflex incontinence occurs with no warning, equally during the day and night, and with spinal cord lesions above S2. Overflow incontinence is when the pressure of urine in the overfull bladder overcomes sphincter control and is caused by bladder or urethral outlet obstruction. Functional incontinence is loss of urine resulting from cognitive, functional, or environmental factors. Incontinence after trauma or surgery occurs when fistulas have occurred or after a prostatectomy.

The patient has a thoracic spinal cord lesion and incontinence that occurs equally during the day and night. What type of incontinence is this patient experiencing? a. Reflex incontinence b. Overflow incontinence c. Functional incontinence d. Incontinence after trauma

a. Reflex incontinence occurs with no warning, equally during the day and night, and with spinal cord lesions above S2. Overflow incontinence is when the pressure of urine in the overfull bladder overcomes sphincter control and is caused by bladder or urethral outlet obstruction. Functional incontinence is loss of urine resulting from cognitive, functional, or environmental factors. Incontinence after trauma or surgery occurs when fistulas have occurred or after a prostatectomy.

"Which characteristic is more likely with acute pyelonephritis than with a lower UTI? a. Fever b. Dysuria c. Urgency d. Frequency

a. Systemic manifestations of fever and chills with leukocytosis and nausea and vomiting are more common in pyelonephritis than in a lower UTI. Dysuria, frequency, and urgency can be present with both.

Which characteristic is more likely with acute pyelonephritis than with a lower UTI? a. Fever b. Dysuria c. Urgency d. Frequency

a. Systemic manifestations of fever and chills with leukocytosis and nausea and vomiting are more common in pyelonephritis than in a lower UTI. Dysuria, frequency, and urgency can be present with both.

A patient in the urology clinic is diagnosed with monilial urethritis. Which action will the nurse include in the plan of care? a. Teach the patient about the use of antifungal medications. b. Tell the patient to avoid tub baths until the symptoms resolve. c. Instruct the patient to refer recent sexual partners for treatment. d. Teach the patient to avoid nonsteroidal antiinflammatory drugs (NSAIDs).

a. Teach the patient about the use of antifungal medications. Monilial urethritis is caused by a fungus and antifungal medications such as nystatin (Mycostatin) or fluconazole (Diflucan) are usually used as treatment. Because monilial urethritis is not sexually transmitted, there is no need to refer sexual partners. Warm baths and NSAIDS may be used to treat symptoms.

Which urinary diversion is a continent diversion created by formation of an ileal pouch with a stoma for catheterization? a. Kock pouch b. Ileal conduit c. Orthotopic neobladder d. Cutaneous ureterostomy

a. The Kock pouch is a continent diversion created by formation of an ileal pouch with an external stoma requiring catheterization. Ileal conduit is the most common incontinent diversion using a stoma of resected ileum with implanted ureters. Orthotopic neobladder is a new bladder from a reshaped segment of intestine in the anatomic position of the bladder with urine discharged through the urethra. A cutaneous ureterostomy diverts the ureter from the bladder to the abdominal skin but there is frequent scarring and strictures of the ureters, so ileal conduits are used more often.

Which urinary diversion is a continent diversion created by formation of an ileal pouch with a stoma for catheterization? a. Kock pouch b. Ileal conduit c. Orthotopic neobladder d. Cutaneous ureterostomy

a. The Kock pouch is a continent diversion created by formation of an ileal pouch with an external stoma requiring catheterization. Ileal conduit is the most common incontinent diversion using a stoma of resected ileum with implanted ureters. Orthotopic neobladder is a new bladder from a reshaped segment of intestine in the anatomic position of the bladder with urine discharged through the urethra. A cutaneous ureterostomy diverts the ureter from the bladder to the abdominal skin but there is frequent scarring and strictures of the ureters, so ileal conduits are used more often.

The wound, ostomy, and continence (WOC) nurse selects the site where the ostomy will be placed. What should be included in the consideration for the site? a. The patient must be able to see the site. b. Outside the rectus muscle area is the best site. c. It is easier to seal the drainage bag to a protruding area. d. The ostomy will need irrigation, so area should not be tender.

a. The patient must be able to see the site. In selection of the ostomy site, the WOC nurse will want a site visible to the patient so the patient can take care of it, within the rectus muscle to avoid hernias, and on a flat surface to more easily create a good seal with the drainage bag.

Which patient action indicates a good understanding of the nurse's teaching about the use of an insulin pump? a. The patient programs the pump for an insulin bolus after eating. b. The patient changes the location of the insertion site every week. c. The patient takes the pump off at bedtime and starts it again each morning. d. The patient plans for a diet that is less flexible when using the insulin pump.

a. The patient programs the pump for an insulin bolus after eating.

A female patient is scheduled for an oral glucose tolerance test. Which information from the patient's health history is most important for the nurse to communicate to the health care provider? a. The patient uses oral contraceptives. b. The patient runs several days a week. c. The patient has been pregnant three times. d. The patient has a family history of diabetes.

a. The patient uses oral contraceptives.

A 55-year-old female patient with type 2 diabetes has a nursing diagnosis of imbalanced nutrition: more than body requirements. Which goal is most important for this patient? a. The patient will reach a glycosylated hemoglobin level of less than 7%. b. The patient will follow a diet and exercise plan that results in weight loss. c. The patient will choose a diet that distributes calories throughout the day. d. The patient will state the reasons for eliminating simple sugars in the diet.

a. The patient will reach a glycosylated hemoglobin level of less than 7%.

The nurse counsels a 64-year-old man on dietary restrictions to prevent recurrent uric acid renal calculi. Which foods should the nurse teach the patient to avoid? a. Venison, crab, and liver b. Spinach, cabbage, and tea c. Milk, yogurt, and dried fruit Incorrect d. Asparagus, lentils, and chocolate

a. Venison, crab, and liver Foods high in purines (e.g., venison, crab, liver) should be avoided to prevent uric acid calculi formation. Foods high in calcium (e.g., milk, yogurt, dried fruit, lentils, chocolate) should be avoided to prevent calcium calculi formation. Foods high in oxalate (e.g., spinach, cabbage, tea, asparagus, chocolate) should be avoided to prevent oxalate calculi formation

The nurse explains to the patient undergoing ostomy surgery that the procedure that maintains the most normal functioning of the bowel is: a. a sigmoid colostomy b. a transverse colostomy c. a descending colostomy d. an ascending colostomy

a. a sigmoid colostomy Rationale: The more distal the ostomy is, the more the intestinal contents resemble feces eliminated from an intact colon and rectum. Output from a sigmoid colostomy resembles normally formed stool, and some patients are able to regulate emptying time so they do not need to wear a collection bag.

*6. A patient is admitted to the hospital with severe renal colic. The nurse's first priority in management of the patient is to* a. administer opioids as prescribed. b. obtain supplies for straining all urine c. encourage fluid intake of 3-4L/day d. keep the patient NPO in preparation for surgery

a. administer opioids as prescribed.

The nursing care for a patient with hyponatremia and fluid volume excess includes a. fluid restriction b. administration of hypotonic IV fluids c. administration of a cation-exchange resin d. placement of an indwelling urinary catheter

a. fluid restriction

Two days following a colectomy for an abdominal mass, a patient reports gas pains and abdominal distention. The nurse plans care for the patient based on the knowledge that the symptoms are occurring as a result of a. impaired peristalsis. b. irritation of the bowel. c. nasogastric suctioning. d. inflammation of the incision site.

a. impaired peristalsis. Until peristalsis returns to normal following anesthesia, the patient may experience slowed gastrointestinal motility leading to gas pains and abdominal distention. Irritation of the bowel, nasogastric suctioning, and inflammation of the surgical site do not cause gas pains or abdominal distention.

The appropriate collaborative therapy for the patient with acute diarrhea caused by a viral infection is to: a. increase fluid intake b. administer an antibiotic c. administer antimotality drugs d. quarantine the patient to prevent spread of the virus

a. increase fluid intake Rationale: Acute diarrhea resulting from infectious causes (e.g., virus) is usually self-limiting. The major concerns are transmission prevention, fluid and electrolyte replacement, and resolution of the diarrhea. Antidiarrheal agents are contraindicated in the treatment of infectious diarrhea because they potentially prolong exposure to the infectious organism. Antibiotics are rarely used to treat acute diarrhea. To prevent transmission of diarrhea caused by a virus, hands should be washed before and after contact with the patient and when body fluids of any kind are handled. Vomitus and stool should be flushed down the toilet, and contaminated clothing should be washed immediately with soap and hot water.

*4.One of the nruse's most important roles in relation to acute poststreptococcal golmerulonephritis is to* a. promote early diagnosis and treatment of sore throats and skin lesions b.encourage patients to obtain antibiotic therapy for upper respiratory tract infections c.teach patients with APSGN that long term prophylactic antibiotic therapy is necessary to prevent recurrence d.monitor patients for respiratory symptoms that indicate the disease is affecting the alveolar basement membrane

a. promote early diagnosis and treatment of sore throats and skin lesions

The nurse determines that the goals of dietary teaching have been met when the patient with celiac disease selects from the menu: a. scrambled eggs and sausage b. buckwheat pancakes with syrup c. oatmeal, skim milk, and orange juice d. yogurt, strawberries, and rye toast with butter

a. scrambled eggs and sausage Rationale: Celiac disease is treated with lifelong avoidance of dietary gluten. Wheat, barley, oats, and rye products must be avoided. Although pure oats do not contain gluten, oat products can become contaminated with wheat, rye, and barley during the milling process. Gluten is also found in some medications and in many food additives, preservatives, and stabilizers.

*9. In planning nursing interventions to increase bladder control in the patient with urinary incontinence, the nurse includes:* a. teaching the patient to use Kegel exercises b.clamping and releasing a catheter to increase bladder tone c.teaching the patient biofeedback mechanisms to suppress the urge to void d.counseling the patient concerning choices of incontinence containment devices

a. teaching the patient to use Kegel exercises

An older woman was admitted to the medical unit with GI bleeding and fluid volume deficit . Clinical manifestations of this problem are (select all that apply) a. weight loss b. dry oral mucosa c. full bounding pulse d. engorged neck veins e. decreased central venous pressure

a. weight loss b. dry oral mucosa e. decreased central venous pressure

Polydipsia and polyuria related to diabetes mellitus are primarily due to: a) the release of ketone bodies from cells during fat metabolism b) fluid shifts resulting from the osmotic effect of hyperglycemia c) damage to the kidneys from exposure to high levels of glucose d) changes in RBCs resulting from attachment of excessive glucose to hemoglobin.

b) fluid shifts resulting from the osmotic effect of hyperglycemia The osmotic effect of glucose produces the manifestations of polydipsia and polyuria.

The male patient is Jewish, has a history of gout, and has been diagnosed with renal calculi. Which treatment will be used with this patient (select all that apply)? a. Reduce dietary oxalate b. Administer allopurinol c. Administer α-penicillamine d. Administer thiazide diuretics e. Reduce animal protein intake f. Reduce intake of milk products

b, e. This patient is most likely to have uric acid calculi, which have a high incidence in Jewish men, and gout is a predisposing factor. The treatment will include allopurinol and reducing animal protein intake to reduce purine, as uric acid is a waste product from purine metabolism. Reducing oxalate and using thiazide diuretics help to treat calcium oxalate calculi. Administration of α-penicillamine and tiopronin prevent cystine crystallization for cystine calculi. Reducing intake of milk products to reduce calcium intake may be used with calcium calculi.

he male patient is Jewish, has a history of gout, and has been diagnosed with renal calculi. Which treatment will be used with this patient (select all that apply)? a. Reduce dietary oxalate b. Administer allopurinol c. Administer α-penicillamine d. Administer thiazide diuretics e. Reduce animal protein intake f. Reduce intake of milk products

b, e. This patient is most likely to have uric acid calculi, which have a high incidence in Jewish men, and gout is a predisposing factor. The treatment will include allopurinol and reducing animal protein intake to reduce purine, as uric acid is a waste product from purine metabolism. Reducing oxalate and using thiazide diuretics help to treat calcium oxalate calculi. Administration of α-penicillamine and tiopronin prevent cystine crystallization for cystine calculi. Reducing intake of milk products to reduce calcium intake may be used with calcium calculi.

The nurse gives instructions to a 62-year-old woman taking tolterodine (Detrol) to treat urinary urgency and incontinence. The nurse should include which statement? a. "Stop smoking for 2 to 3 weeks before starting to take this medication." b. "Suck on sugarless candy or chew sugarless gum if you develop a dry mouth." c. "Have your vision checked every 6 months because this drug can cause cataracts." d. "Ask your physician to prescribe an extended-release form if you have loose stools."

b. "Suck on sugarless candy or chew sugarless gum if you develop a dry mouth." Dry mouth is a common side effect of tolterodine. Patients can suck on hard candy or ice chips or chew gum if dry mouth occurs. Tobacco use does not affect the initiation of this medication. Visual changes (but not cataracts) can occur while taking this medication. Constipation may occur as a side effect of this medication.

A 54-year-old patient admitted with type 2 diabetes asks the nurse what "type 2" means. What is the most appropriate response by the nurse? a. "With type 2 diabetes, the body of the pancreas becomes inflamed." b. "With type 2 diabetes, insulin secretion is decreased, and insulin resistance is increased." c. "With type 2 diabetes, the patient is totally dependent on an outside source of insulin." d. "With type 2 diabetes, the body produces autoantibodies that destroy β-cells in the pancreas."

b. "With type 2 diabetes, insulin secretion is decreased, and insulin resistance is increased." In type 2 diabetes mellitus, the secretion of insulin by the pancreas is reduced, and/or the cells of the body become resistant to insulin. The pancreas becomes inflamed with pancreatitis. The patient is totally dependent on exogenous insulin and may have had autoantibodies destroy the β-cells in the pancreas with type 1 diabetes mellitus.

After change-of-shift report, which patient should the nurse assess first? a. 19-year-old with type 1 diabetes who has a hemoglobin A1C of 12% b. 23-year-old with type 1 diabetes who has a blood glucose of 40 mg/dL c. 40-year-old who is pregnant and whose oral glucose tolerance test is 202 mg/dL d. 50-year-old who uses exenatide (Byetta) and is complaining of acute abdominal pain

b. 23-year-old with type 1 diabetes who has a blood glucose of 40 mg/dL

A 61-year-old patient with suspected bowel obstruction had a nasogastric tube inserted at 4:00 AM. The nurse shares in the morning report that the day shift staff should check the tube for patency at what times? a. 7:00 AM, 10:00 AM, and 1:00 PM b. 8:00 AM, 12:00 PM, and 4:00 PM c. 9:00 AM and 3:00 PM d. 9:00 AM, 12:00 PM, and 3:00 PM

b. 8:00 AM, 12:00 PM, and 4:00 PM A nasogastric tube should be checked for patency routinely at 4-hour intervals. Thus if the tube were inserted at 4:00 AM, it would be due to be checked at 8:00 AM, 12:00 PM, and 4:00 PM.

Priority Decision: Following electrohydraulic lithotripsy for treatment of renal calculi, the patient has a nursing diagnosis of risk for infection related to the introduction of bacteria following manipulation of the urinary tract. What is the most appropriate nursing intervention for this patient? a. Monitor for hematuria. b. Encourage fluid intake of 3 L/day. c. Apply moist heat to the flank area. d. Strain all urine through gauze or a special strainer.

b. A high fluid intake maintains dilute urine, which decreases bacterial concentration in addition to washing stone fragments and expected blood through the urinary system following lithotripsy. High urine output also prevents supersaturation of minerals. Moist heat to the flank may be helpful to relieve muscle spasms during renal colic and all urine should be strained in patients with renal stones to collect and identify stone composition but these are not related to infection.

Priority Decision: Following electrohydraulic lithotripsy for treatment of renal calculi, the patient has a nursing diagnosis of risk for infection related to the introduction of bacteria following manipulation of the urinary tract. What is the most appropriate nursing intervention for this patient? a. Monitor for hematuria. b. Encourage fluid intake of 3 L/day. c. Apply moist heat to the flank area. d. Strain all urine through gauze or a special strainer.

b. A high fluid intake maintains dilute urine, which decreases bacterial concentration in addition to washing stone fragments and expected blood through the urinary system following lithotripsy. High urine output also prevents supersaturation of minerals. Moist heat to the flank may be helpful to relieve muscle spasms during renal colic and all urine should be strained in patients with renal stones to collect and identify stone composition but these are not related to infection.

During assessment of the patient who has a nephrectomy, what should the nurse expect to find? a. Shallow, slow respirations b. Clear breath sounds in all lung fields c. Decreased breath sounds in the lower left lobe d. Decreased breath sounds in the right and left lower lobes

b. A nephrectomy incision is usually in the flank, just below the diaphragm or in the abdominal area. Although the patient is reluctant to breathe deeply because of incisional pain, the lungs should be clear. Decreased sounds and shallow respirations are abnormal and would require intervention.

A patient who had a subtotal thyroidectomy earlier today develops laryngeal stridor and a cramp in the right hand upon returning to the surgical nursing unit. Which collaborative action will the nurse anticipate next? a. Suction the patient's airway. b. Administer IV calcium gluconate. c. Plan for emergency tracheostomy. d. Prepare for endotracheal intubation.

b. Administer IV calcium gluconate. The patient's clinical manifestations of stridor and cramping are consistent with tetany caused by hypocalcemia resulting from damage to the parathyroid glands during surgery. Endotracheal intubation or tracheostomy may be needed if the calcium does not resolve the stridor. Suctioning will not correct the stridor

A colectomy is scheduled for a 38-year-old woman with ulcerative colitis. The nurse should plan to include what prescribed measure in the preoperative preparation of this patient? a. Instruction on irrigating a colostomy b. Administration of a cleansing enema c. A high-fiber diet the day before surgery d. Administration of IV antibiotics for bowel preparation

b. Administration of a cleansing enema Preoperative preparation for bowel surgery typically includes bowel cleansing with antibiotics, such as oral neomycin and cleansing enemas, including Fleet enemas. Instructions to irrigate the colostomy will be done postoperatively. Oral antibiotics are given preoperatively, and an IV antibiotic may be used in the OR. A clear liquid diet will be used the day before surgery with the bowel cleansing.

In providing care for the patient with adult-onset polycystic kidney disease, what should the nurse do? a. Help the patient to cope with the rapid progression of the disease b. Suggest genetic counseling resources for the children of the patient c. Expect the patient to have polyuria and poor concentration ability of the kidneys d. Implement measures for the patient's deafness and blindness in addition to the renal problems

b. Adult-onset polycystic kidney disease is an inherited autosomal dominant disorder that often manifests after the patient has children but the children should receive genetic counseling regarding their life choices. The disease progresses slowly, eventually causing progressive renal failure. Hereditary medullary cystic disease causes poor concentration ability of the kidneys and classic Alport syndrome is a hereditary nephritis that is associated with deafness and deformities of the optic lens.

In providing care for the patient with adult-onset polycystic kidney disease, what should the nurse do? a. Help the patient to cope with the rapid progression of the disease b. Suggest genetic counseling resources for the children of the patient c. Expect the patient to have polyuria and poor concentration ability of the kidneys d. Implement measures for the patient's deafness and blindness in addition to the renal problems

b. Adult-onset polycystic kidney disease is an inherited autosomal dominant disorder that often manifests after the patient has children but the children should receive genetic counseling regarding their life choices. The disease progresses slowly, eventually causing progressive renal failure. Hereditary medullary cystic disease causes poor concentration ability of the kidneys and classic Alport syndrome is a hereditary nephritis that is associated with deafness and deformities of the optic lens.

Which nursing assessment of a 69-year-old patient is most important to make during initiation of thyroid replacement with levothyroxine (Synthroid)? a. Fluid balance b. Apical pulse rate c. Nutritional intake d. Orientation and alertness

b. Apical pulse rate In older patients, initiation of levothyroxine therapy can increase myocardial oxygen demand and cause angina or dysrhythmias. The medication also is expected to improve mental status and fluid balance and will increase metabolic rate and nutritional needs, but these changes will not result in potentially life-threatening complications

An active 28-year-old male with type 1 diabetes is being seen in the endocrine clinic. Which finding may indicate the need for a change in therapy? a. Hemoglobin A1C level 6.2% b. Blood pressure 146/88 mmHg c. Heart rate at rest 58 beats/minute d. High density lipoprotein (HDL) level 65 mg/dL

b. Blood pressure 146/88 mmHg

Which assessment finding for a patient who has just been admitted with acute pyelonephritis is most important for the nurse to report to the health care provider? a. Complaint of flank pain b. Blood pressure 90/48 mm Hg c. Cloudy and foul-smelling urine d. Temperature 100.1° F (57.8° C)

b. Blood pressure 90/48 mmHg The low blood pressure indicates that urosepsis and septic shock may be occurring and should be immediately reported. The other findings are typical of pyelonephritis.

Which nursing action will be included in the plan of care for a 55-year-old patient with Graves' disease who has exophthalmos? a. Place cold packs on the eyes to relieve pain and swelling. b. Elevate the head of the patient's bed to reduce periorbital fluid. c. Apply alternating eye patches to protect the corneas from irritation. d. Teach the patient to blink every few seconds to lubricate the corneas.

b. Elevate the head of the patient's bed to reduce periorbital fluid. The patient should sit upright as much as possible to promote fluid drainage from the periorbital area. With exophthalmos, the patient is unable to close the eyes completely to blink. Lubrication of the eyes, rather than eye patches, will protect the eyes from developing corneal scarring. The swelling of the eye is not caused by excessive blood flow to the eye, so cold packs will not be helpful

Which action should the nurse take after a 36-year-old patient treated with intramuscular glucagon for hypoglycemia regains consciousness? a. Assess the patient for symptoms of hyperglycemia. b. Give the patient a snack of peanut butter and crackers. c. Have the patient drink a glass of orange juice or nonfat milk. d. Administer a continuous infusion of 5% dextrose for 24 hours.

b. Give the patient a snack of peanut butter and crackers.

Which information will the nurse include when teaching a 50-year-old patient who has type 2 diabetes about glyburide (Micronase, DiaBeta, Glynase)? a. Glyburide decreases glucagon secretion from the pancreas. b. Glyburide stimulates insulin production and release from the pancreas. c. Glyburide should be taken even if the morning blood glucose level is low. d. Glyburide should not be used for 48 hours after receiving IV contrast media.

b. Glyburide stimulates insulin production and release from the pancreas.

What information would have the highest priority to be included in preoperative teaching for a 68-year-old patient scheduled for a colectomy? a. How to care for the wound b. How to deep breathe and cough c. The location and care of drains after surgery d. Which medications will be used during surgery

b. How to deep breathe and cough Because anesthesia, an abdominal incision, and pain can impair the patient's respiratory status in the postoperative period, it is of high priority to teach the patient to cough and deep breathe. Otherwise, the patient could develop atelectasis and pneumonia, which would delay early recovery from surgery and hospital discharge. Care for the wound and location and care of the drains will be briefly discussed preoperatively, but done again with higher priority after surgery. Knowing which drugs will be used during surgery may not be meaningful to the patient and should be reviewed with the patient by the anesthesiologist.

Which information will the nurse include when teaching a 50-year-old male patient about somatropin (Genotropin)? a. The medication will be needed for 3 to 6 months. b. Inject the medication subcutaneously every day. c. Blood glucose levels may decrease when taking the medication. d. Stop taking the medication if swelling of the hands or feet occurs.

b. Inject the medication subcutaneously every day. Somatropin is injected subcutaneously on a daily basis, preferably in the evening. The patient will need to continue on somatropin for life. If swelling or other common adverse effects occur, the health care provider should be notified. Growth hormone will increase blood glucose levels

A 22-year-old female patient seen in the clinic for a bladder infection describes the following symptoms. Which information is most important for the nurse to report to the health care provider? a. Urinary urgency b. Left-sided flank pain c. Intermittent hematuria d. Burning with urination

b. Left-sided flank pain Flank pain indicates that the patient may have developed pyelonephritis as a complication of the bladder infection. The other clinical manifestations are consistent with a lower urinary tract infection (UTI).

The nurse is planning postoperative care for a patient who is being admitted to the surgical unit form the recovery room after transsphenoidal resection of a pituitary tumor. Which nursing action should be included? a. Palpate extremities for edema. b. Measure urine volume every hour. c. Check hematocrit every 2 hours for 8 hours. d. Monitor continuous pulse oximetry for 24 hours.

b. Measure urine volume every hour. After pituitary surgery, the patient is at risk for diabetes insipidus caused by cerebral edema. Monitoring of urine output and urine specific gravity is essential. Hemorrhage is not a common problem. There is no need to check the hematocrit hourly. The patient is at risk for dehydration, not volume overload. The patient is not at high risk for problems with oxygenation, and continuous pulse oximetry is not needed

The nurse is beginning to teach a diabetic patient about vascular complications of diabetes. What information is appropriate for the nurse to include? a. Macroangiopathy does not occur in type 1 diabetes but rather in type 2 diabetics who have severe disease. b. Microangiopathy is specific to diabetes and most commonly affects the capillary membranes of the eyes, kidneys, and skin. c. Renal damage resulting from changes in large- and medium-sized blood vessels can be prevented by careful glucose control. d. Macroangiopathy causes slowed gastric emptying and the sexual impotency experienced by a majority of patients with diabetes.

b. Microangiopathy is specific to diabetes and most commonly affects the capillary membranes of the eyes, kidneys, and skin. Microangiopathy occurs in diabetes mellitus. When it affects the eyes, it is called diabetic retinopathy. When the kidneys are affected, the patient has nephropathy. When the skin is affected, it can lead to diabetic foot ulcers. Macroangiopathy can occur in either type 1 or type 2 diabetes and contributes to cerebrovascular, cardiovascular, and peripheral vascular disease. Sexual impotency and slowed gastric emptying result from microangiopathy and neuropathy.

A 51-year-old patient with diabetes mellitus is scheduled for a fasting blood glucose level at 8:00 AM. The nurse instructs the patient to only drink water after what time? a. 6:00 PM on the evening before the test b. Midnight before the test c. 4:00 AM on the day of the test d. 7:00 AM on the day of the test

b. Midnight before the test Typically, a patient is ordered to be NPO for 8 hours before a fasting blood glucose level. For this reason, the patient who has a lab draw at 8:00 AM should not have any food or beverages containing any calories after midnight.

The nurse should administer an as-needed dose of magnesium hydroxide (MOM) after noting what information while reviewing a patient's medical record? a. Abdominal pain and bloating b. No bowel movement for 3 days c. A decrease in appetite by 50% over 24 hours d. Muscle tremors and other signs of hypomagnesemia

b. No bowel movement for 3 days MOM is an osmotic laxative that produces a soft, semisolid stool usually within 15 minutes to 3 hours. This medication would benefit the patient who has not had a bowel movement for 3 days. MOM would not be given for abdominal pain and bloating, decreased appetite, or signs of hypomagnesemia.

The nurse provides nutritional counseling for a 45-year-old man with nephrotic syndrome. The nurse determines that teaching has been successful if the patient selects which breakfast menu? a. Scrambled eggs, milk, yogurt, and sliced ham b. Oatmeal, nondairy creamer, banana, and orange juice c. Cottage cheese, peanut butter, white bread, and coffee d. Waffle, bacon strips, tomato juice, and canned peaches

b. Oatmeal, nondairy creamer, banana, and orange juice Patients with nephrotic syndrome should follow a low-sodium (2 to 3 g/day), low- to moderate-protein (0.5 to 0.6 g/kg/day) diet. Ham, milk products, peanut butter, and bacon are high in sodium. Eggs, milk products, and peanut butter are high in protein.

A patient with diabetes mellitus who has multiple infections every year needs a mitral valve replacement. What is the most important preoperative teaching the nurse should provide to prevent a cardiac infection postoperatively? a. Avoid sick people and wash hands. b. Obtain comprehensive dental care. c. Maintain hemoglobin A1 c below 7%. d. Coughing and deep breathing with splinting

b. Obtain comprehensive dental care. A person with diabetes is at high risk for postoperative infections. The most important preoperative teaching to prevent a postoperative infection in the heart is to have the patient obtain comprehensive dental care because the risk of septicemia and infective endocarditis increases with poor dental health. Avoiding sick people, hand washing, maintaining hemoglobin A1c below 7%, and coughing and deep breathing with splinting would be important for any type of surgery, but not the priority with mitral valve replacement for this patient.

A patient has a right ureteral catheter placed following a lithotripsy for a stone in the ureter. In caring for the patient after the procedure, what is an appropriate nursing action? a. Milk or strip the catheter every 2 hours. b. Measure ureteral urinary drainage every 1 to 2 hours. c. Irrigate the catheter with 30-mL sterile saline every 4 hours. d. Encourage ambulation to promote urinary peristaltic action.

b. Output from ureteral catheters must be monitored every 1 to 2 hours because an obstruction will cause overdistention of the renal pelvis and renal damage. The renal pelvis has a capacity of only 3 to 5 mL and if irrigation is ordered, no more than 5 mL of sterile saline is used. The patient with a ureteral catheter is usually kept on bed rest until specific orders for ambulation are given. Suprapubic tubes may be milked to prevent obstruction of the catheter by sediment and clots.

A patient has a right ureteral catheter placed following a lithotripsy for a stone in the ureter. In caring for the patient after the procedure, what is an appropriate nursing action? a. Milk or strip the catheter every 2 hours. b. Measure ureteral urinary drainage every 1 to 2 hours. c. Irrigate the catheter with 30-mL sterile saline every 4 hours. d. Encourage ambulation to promote urinary peristaltic action.

b. Output from ureteral catheters must be monitored every 1 to 2 hours because an obstruction will cause overdistention of the renal pelvis and renal damage. The renal pelvis has a capacity of only 3 to 5 mL and if irrigation is ordered, no more than 5 mL of sterile saline is used. The patient with a ureteral catheter is usually kept on bed rest until specific orders for ambulation are given. Suprapubic tubes may be milked to prevent obstruction of the catheter by sediment and clots.

Which action will the nurse anticipate taking for an otherwise healthy 50-year-old who has just been diagnosed with Stage 1 renal cell carcinoma? a. Prepare patient for a renal biopsy. b. Provide preoperative teaching about nephrectomy. c. Teach the patient about chemotherapy medications. d. Schedule for a follow-up appointment in 3 months.

b. Provide preoperative teaching about nephrectomy. The treatment of choice in patients with localized renal tumors who have no co-morbid conditions is partial or total nephrectomy. A renal biopsy will not be needed in a patient who has already been diagnosed with renal cancer. Chemotherapy is used for metastatic renal cancer. Because renal cell cancer frequently metastasizes, treatment will be started as soon as possible after the diagnosis.

Which information will the nurse include when teaching the patient with a urinary tract infection (UTI) about the use of phenazopyridine (Pyridium)? a. Pyridium may cause photosensitivity b. Pyridium may change the urine color. c. Take the Pyridium for at least 7 days. d. Take Pyridium before sexual intercourse.

b. Pyridium may change the urine color. Patients should be taught that Pyridium will color the urine deep orange. Urinary analgesics should only be needed for a few days until the prescribed antibiotics decrease the bacterial count. Pyridium does not cause photosensitivity. Taking Pyridium before intercourse will not be helpful in reducing the risk for UTI.

A 56-year-old female patient is admitted to the hospital with new onset nephrotic syndrome. Which assessment data will the nurse expect? a. Poor skin turgor b. Recent weight gain c. Elevated urine ketones d. Decreased blood pressure

b. Recent weight gain The patient with a nephrotic syndrome will have weight gain associated with edema. Hypertension is a clinical manifestation of nephrotic syndrome. Skin turgor is normal because of the edema. Urine protein is high.

A 22-year-old patient's blood pressure at her physical done for her new job was 110/68. At the health fair two months later, her blood pressure is 154/96. What renal problem should the nurse be aware of that could contribute to this abrupt rise in blood pressure? a. Renal trauma b. Renal artery stenosis c. Renal vein thrombosis d. Benign nephrosclerosis

b. Renal artery stenosis Renal artery stenosis contributes to an abrupt rise in blood pressure, especially in people under 30 or over 50 years of age. Renal trauma usually has hematuria. Renal vein thrombosis causes flank pain, hematuria, fever, or nephrotic syndrome. Benign nephrosclerosis usually occurs in adults 30 to 50 years of age and is a result of vascular changes resulting from hypertension.

The nurse is taking a health history from a 29-year-old pregnant patient at the first prenatal visit. The patient reports no personal history of diabetes but has a parent who is diabetic. Which action will the nurse plan to take first? a. Teach the patient about administering regular insulin. b. Schedule the patient for a fasting blood glucose level. c. Discuss an oral glucose tolerance test for the twenty-fourth week of pregnancy. d. Provide teaching about an increased risk for fetal problems with gestational diabetes.

b. Schedule the patient for a fasting blood glucose level.

The female patient with a UTI also has renal calculi. The nurse knows that these are most likely which type of stone? a. Cystine b. Struvite c. Uric acid d. Calcium phosphate

b. Struvite calculi are most common in women and always occur with UTIs. They are also usually large staghorn type.

Which nursing intervention is most appropriate in providing care for an adult patient with newly diagnosed adult onset polycystic kidney disease (PKD)? a. Help the patient cope with the rapid progression of the disease. b. Suggest genetic counseling resources for the children of the patient. c. Expect the patient to have polyuria and poor concentration ability of the kidneys. d. Implement appropriate measures for the patient's deafness and blindness in addition to the renal problems.

b. Suggest genetic counseling resources for the children of the patient. PKD is one of the most common genetic diseases. The adult form of PKD may range from a relatively mild disease to one that progresses to chronic kidney disease. Polyuria, deafness, and blindness are not associated with PKD.

A female patient with a UTI has a nursing diagnosis of risk for infection related to lack of knowledge regarding prevention of recurrence. What should the nurse include in the teaching plan instructions for this patient? a. Empty the bladder at least 4 times a day. b. Drink at least 2 quarts of water every day. c. Wait to urinate until the urge is very intense. d. Clean the urinary meatus with an antiinfective agent after voiding.

b. The bladder should be emptied at least every 3 to 4 hours. Fluid intake should be increased to about 2000 mL/ day without caffeine, alcohol, citrus juices, and chocolate drinks, because they are potential bladder irritants. Cleaning the urinary meatus with an antiinfective agent after voiding will irritate the meatus but the perineal area should be wiped from front to back after urination and defecation to prevent fecal contamination of the meatus.

A female patient with a UTI has a nursing diagnosis of risk for infection related to lack of knowledge regarding prevention of recurrence. What should the nurse include in the teaching plan instructions for this patient? a. Empty the bladder at least 4 times a day. b. Drink at least 2 quarts of water every day. c. Wait to urinate until the urge is very intense. d. Clean the urinary meatus with an antiinfective agent after voiding

b. The bladder should be emptied at least every 3 to 4 hours. Fluid intake should be increased to about 2000 mL/ day without caffeine, alcohol, citrus juices, and chocolate drinks, because they are potential bladder irritants. Cleaning the urinary meatus with an antiinfective agent after voiding will irritate the meatus but the perineal area should be wiped from front to back after urination and defecation to prevent fecal contamination of the meatus.

Which infection is asymptomatic in the male patient at first and then progresses to cystitis, frequent urination, burning on voiding, and epididymitis? a. Urosepsis b. Renal tuberculosis c. Urethral diverticula d. Goodpasture syndrome

b. The manifestations of renal tuberculosis are described. Urosepsis is when the UTI has spread systemically. Urethral diverticula are localized outpouching of the urethra and occur more often in women. Goodpasture syndrome manifests with flu-like symptoms with pulmonary symptoms that include cough, shortness of breath, and pulmonary insufficiency and renal manifestations that include hematuria, weakness, pallor, anemia, and renal failure.

Which infection is asymptomatic in the male patient at first and then progresses to cystitis, frequent urination, burning on voiding, and epididymitis? a. Urosepsis b. Renal tuberculosis c. Urethral diverticula d. Goodpasture syndrome

b. The manifestations of renal tuberculosis are described. Urosepsis is when the UTI has spread systemically. Urethral diverticula are localized outpouching of the urethra and occur more often in women. Goodpasture syndrome manifests with flu-like symptoms with pulmonary symptoms that include cough, shortness of breath, and pulmonary insufficiency and renal manifestations that include hematuria, weakness, pallor, anemia, and renal failure.

A 58-year-old male patient who weighs 242 lb (110 kg) undergoes a nephrectomy for massive kidney trauma due to a motor vehicle crash. Which postoperative assessment finding is most important to communicate to the surgeon? a. Blood pressure is 102/58. b. Urine output is 20 mL/hr for 2 hours. c. Incisional pain level is reported as 9/10. d. Crackles are heard at bilateral lung bases.

b. Urine output is 20 mL/hr for 2 hours. Because the urine output should be at least 0.5 mL/kg/hr, a 40 mL output for 2 hours indicates that the patient may have decreased renal perfusion because of bleeding, inadequate fluid intake, or obstruction at the suture site. The blood pressure requires ongoing monitoring but does not indicate inadequate perfusion at this time. The patient should cough and deep breathe, but the crackles do not indicate a need for an immediate change in therapy. The incisional pain should be addressed, but this is not as potentially life threatening as decreased renal perfusion. In addition, the nurse can medicate the patient for pain.

The nurse is caring for a patient with a nephrostomy tube. The tube has stopped draining. After receiving orders, what should the nurse do? a. Keep the patient on bed rest. b. Use 5 mL of sterile saline to irrigate. c. Use 30 mL of water to gently irrigate. d. Have the patient turn from side to side.

b. Use 5 mL of sterile saline to irrigate With a nephrostomy tube, if the tube is occluded and irrigation is ordered, the nurse should use 5 mL or less of sterile saline to gently irrigate it. The patient with a ureteral catheter may be kept on bed rest after insertion, but this is unrelated to obstruction. Only sterile solutions are used to irrigate any type of urinary catheter. With a suprapubic catheter, the patient should be instructed to turn from side to side to ensure patency.

The nurse expects the long term treatment of a patient with hyperphosphatemia secondary to renal failure will include a. fluid restriction b. calcium supplements c. magnesium supplements d. increased intake of dairy products

b. calcium supplements

The nurse identifies a need for additional teaching when the patient who is self-monitoring blood glucose a. washes the puncture site using warm water and soap. b. chooses a puncture site in the center of the finger pad. c. hangs the arm down for a minute before puncturing the site. d. says the result of 120 mg indicates good blood sugar control.

b. chooses a puncture site in the center of the finger pad.

The nurse is assessing a 41-year-old African American male patient diagnosed with a pituitary tumor causing panhypopituitarism. Assessment findings consistent with panhypopituitarism include a. high blood pressure. b. decreased facial hair. c. elevated blood glucose. d. tachycardia and cardiac palpitations.

b. decreased facial hair. Changes in male secondary sex characteristics such as decreased facial hair, testicular atrophy, diminished spermatogenesis, loss of libido, impotence, and decreased muscle mass are associated with decreases in follicle stimulating hormone (FSH) and luteinizing hormone (LH). Fasting hypoglycemia and hypotension occur in panhypopituitarism as a result of decreases in adrenocorticotropic hormone (ACTH) and cortisol. Bradycardia is likely due to the decrease in thyroid stimulating hormone (TSH) and thyroid hormones associated with panhypopituitarism

*3. The immunologic mechanisms involved in acute poststreptococal glomerulonephritis include:* a. tubular blocking by precipitates of bacteria and antibody reactions b. deposition of immune complexes and complement along the GBM c. thickening of the GBM from autoimmune microangiopathic changes d. destruction of glomeruli by proteolytic enzymes contained in the GBM

b. deposition of immune complexes and complement along the GBM

A patient who had surgery for creation of an ileal conduit 3 days ago will not look at the stoma and requests that only the ostomy nurse specialist does the stoma care. The nurse identifies a nursing diagnosis of a. anxiety related to effects of procedure on lifestyle. b. disturbed body image related to change in function. c. readiness for enhanced coping related to need for information. d. self-care deficit, toileting, related to denial of altered body function.

b. disturbed body image related to change in function. The patient's unwillingness to look at the stoma or participate in care indicates that disturbed body image is the best diagnosis. No data suggest that the impact on lifestyle is a concern for the patient. The patient does not appear to be ready for enhanced coping. The patient's insistence that only the ostomy nurse care for the stoma indicates that denial is not present.

A 34-year-old male patient seen at the primary care clinic complains of feeling continued fullness after voiding and a split, spraying urine stream. The nurse will ask about a history of a. recent kidney trauma. b. gonococcal urethritis. c. recurrent bladder infection. d. benign prostatic hyperplasia.

b. gonococcal bladder infection The patient's clinical manifestations are consistent with urethral strictures, a possible complication of gonococcal urethritis. These symptoms are not consistent with benign prostatic hyperplasia, kidney trauma, or bladder infection.

An unresponsive patient with type 2 diabetes is brought to the emergency department and diagnosed with hyperosmolar hyperglycemic syndrome (HHS). The nurse will anticipate the need to a. give a bolus of 50% dextrose. b. insert a large-bore IV catheter. c. initiate oxygen by nasal cannula. d. administer glargine (Lantus) insulin.

b. insert a large-bore IV catheter.

In contrast to diverticulitis, the patient with diverticulosis: a. has rectal bleeding b. often has no symptoms c. has localized cramping pain d. frequently develops peritonitis

b. often has no symptoms Rationale: Many people with diverticulosis have no symptoms. Patients with diverticulitis have symptoms of inflammation. Diverticulitis can lead to obstruction or perforation.

It is most important that the nurse ask a patient admitted with acute glomerulonephritis about a. history of kidney stones. b. recent sore throat and fever. c. history of high blood pressure. d. frequency of bladder infections.

b. recent sore throat and fever. Acute glomerulonephritis frequently occurs after a streptococcal infection such as strep throat. It is not caused by kidney stones, hypertension, or urinary tract infection (UTI).

The nurse determines that demeclocycline (Declomycin) is effective for a patient with syndrome of inappropriate antidiuretic hormone (SIADH) based on finding that the patient's a. weight has increased. b. urinary output is increased. c. peripheral edema is decreased. d. urine specific gravity is increased.

b. urinary output is increased. Demeclocycline blocks the action of antidiuretic hormone (ADH) on the renal tubules and increases urine output. An increase in weight or an increase in urine specific gravity indicates that the SIADH is not corrected. Peripheral edema does not occur with SIADH. A sudden weight gain without edema is a common clinical manifestation of this disorder

*10. A patient with ureterolithotomy returns from surgery with a nephrostomy tube in place. Postoperative nursing care of the patient includes:* a.encourage the patient to drink fruit juices and milk b.encouraging fluids of at least 2-3 L/day after nausea has subsided c. irrigating the nephrostomy tube with 10ml of NS solution as needed d. notifying the physician if nephrostomy tube drainage is more than 30ml/hr

b.encouraging fluids of at least 2-3 L/day after nausea has subsided

*2. The nurse teaches the female paitent who has frequent UTIs that she should* a. take tub baths with bubble bath b.urinate before and after sexual intercourse c.take prophylactic sufonamides for the rest of her life d. restrict fluid intake to prevent the need for frequent voiding

b.urinate before and after sexual intercourse

After the nurse has finished teaching a patient who has a new prescription for exenatide (Byetta), which patient statement indicates that the teaching has been effective? a. "I may feel hungrier than usual when I take this medicine." b. "I will not need to worry about hypoglycemia with the Byetta." c. "I should take my daily aspirin at least an hour before the Byetta." d. "I will take the pill at the same time I eat breakfast in the morning."

c. "I should take my daily aspirin at least an hour before the Byetta."

The home health nurse teaches a patient with a neurogenic bladder how to use intermittent catheterization for bladder emptying. Which patient statement indicates that the teaching has been effective? a. "I will buy seven new catheters weekly and use a new one every day." b. "I will use a sterile catheter and gloves for each time I self-catheterize." c. "I will clean the catheter carefully before and after each catheterization." d. "I will need to take prophylactic antibiotics to prevent any urinary tract infections."

c. "I will clean the catheter carefully before and after each catheterization." Patients who are at home can use a clean technique for intermittent self-catheterization and change the catheter every 7 days. There is no need to use a new catheter every day, to use sterile catheters, or to take prophylactic antibiotics.

The nurse determines that further instruction is needed for a patient with interstitial cystitis when the patient says which of the following? a. "I should stop having coffee and orange juice for breakfast." b. "I will buy calcium glycerophosphate (Prelief) at the pharmacy." c. "I will start taking high potency multiple vitamins every morning." d. "I should call the doctor about increased bladder pain or odorous urine."

c. "I will start taking high potency multiple vitamins every morning." High-potency multiple vitamins may irritate the bladder and increase symptoms. The other patient statements indicate good understanding of the teaching.

The patient received regular insulin 10 units subcutaneously at 8:30 PM for a blood glucose level of 253 mg/dL. The nurse plans to monitor this patient for signs of hypoglycemia at which time related to the insulin's peak action? a. 8:40 PM to 9:00 PM b. 9:00 PM to 11:30 PM c. 10:30 PM to 1:30 AM d. 12:30 AM to 8:30 AM

c. 10:30 PM to 1:30 AM Regular insulin exerts peak action in 2 to 5 hours, making the patient most at risk for hypoglycemia between 10:30 PM and 1:30 AM. Rapid-acting insulin's onset is between 10-30 minutes with peak action and hypoglycemia most likely to occur between 9:00 PM and 11:30 PM. With intermediate acting insulin, hypoglycemia may occur from 12:30 AM to 8:30 AM.

After change-of-shift report, which patient will the nurse assess first? a. 19-year-old with type 1 diabetes who was admitted with possible dawn phenomenon b. 35-year-old with type 1 diabetes whose most recent blood glucose reading was 230 mg/dL c. 60-year-old with hyperosmolar hyperglycemic syndrome who has poor skin turgor and dry oral mucosa d. 68-year-old with type 2 diabetes who has severe peripheral neuropathy and complains of burning foot pain

c. 60-year-old with hyperosmolar hyperglycemic syndrome who has poor skin turgor and dry oral mucosa

On assessment of the patient with a renal calculus passing down the ureter, what should the nurse expect the patient to report? a. A history of chronic UTIs b. Dull, costovertebral flank pain c. Severe, colicky back pain radiating to the groin d. A feeling of bladder fullness with urgency and frequency

c. A classic sign of the passage of a calculus down the ureter is intense, colicky back pain that may radiate into the testicles, labia, or groin and may be accompanied by mild shock with cool, moist skin. Many patients with renal stones do not have a history of chronic UTIs. Stones obstructing a calyx or at the ureteropelvic junction may produce dull costovertebral flank pain and large bladder stones may cause bladder fullness and lower obstructive symptoms.

On assessment of the patient with a renal calculus passing down the ureter, what should the nurse expect the patient to report? a. A history of chronic UTIs b. Dull, costovertebral flank pain c. Severe, colicky back pain radiating to the groin d. A feeling of bladder fullness with urgency and frequency

c. A classic sign of the passage of a calculus down the ureter is intense, colicky back pain that may radiate into the testicles, labia, or groin and may be accompanied by mild shock with cool, moist skin. Many patients with renal stones do not have a history of chronic UTIs. Stones obstructing a calyx or at the ureteropelvic junction may produce dull costovertebral flank pain and large bladder stones may cause bladder fullness and lower obstructive symptoms.

What is included in nursing care that applies to the management of all urinary catheters in hospitalized patients? a. Measuring urine output every 1 to 2 hours to ensure patency b. Turning the patient frequently from side to side to promote drainage c. Using strict sterile technique during irrigation and obtaining culture specimens d. Daily cleaning of the catheter insertion site with soap and water and application of lotion

c. All urinary catheters in hospitalized patients pose a very high risk for infection, especially antibiotic-resistant, health care-associated infections, and scrupulous aseptic technique is essential in the insertion and maintenance of all catheters. Routine irrigations are not performed. Turning the patient to promote drainage is recommended only for suprapubic catheters. Cleaning the insertion site with soap and water should be performed for urethral and suprapubic catheters but lotion or powder should be avoided and site care for other catheters may require special interventions.

What is included in nursing care that applies to the management of all urinary catheters in hospitalized patients? a. Measuring urine output every 1 to 2 hours to ensure patency b. Turning the patient frequently from side to side to promote drainage c. Using strict sterile technique during irrigation and obtaining culture specimens d. Daily cleaning of the catheter insertion site with soap and water and application of lotion

c. All urinary catheters in hospitalized patients pose a very high risk for infection, especially antibiotic-resistant, health care-associated infections, and scrupulous aseptic technique is essential in the insertion and maintenance of all catheters. Routine irrigations are not performed. Turning the patient to promote drainage is recommended only for suprapubic catheters. Cleaning the insertion site with soap and water should be performed for urethral and suprapubic catheters but lotion or powder should be avoided and site care for other catheters may require special interventions.

Which information will the nurse teach a 48-year-old patient who has been newly diagnosed with Graves' disease? a. Exercise is contraindicated to avoid increasing metabolic rate. b. Restriction of iodine intake is needed to reduce thyroid activity. c. Antithyroid medications may take several months for full effect. d. Surgery will eventually be required to remove the thyroid gland.

c. Antithyroid medications may take several months for full effect. Medications used to block the synthesis of thyroid hormones may take 2 to 3 months before the full effect is seen. Large doses of iodine are used to inhibit the synthesis of thyroid hormones. Exercise using large muscle groups is encouraged to decrease the irritability and hyperactivity associated with high levels of thyroid hormones. Radioactive iodine is the most common treatment for Graves' disease although surgery may be used

What is the most common cause of acute pyelonephritis resulting from an ascending infection from the lower urinary tract? a. The kidney is scarred and fibrotic. b. The organism is resistant to antibiotics. c. There is a preexisting abnormality of the urinary tract. d. The patient does not take all of the antibiotics for treatment of a UTI.

c. Ascending infections from the bladder to the kidney are prevented by the normal anatomy and physiology of the urinary tract unless a preexisting condition, such as vesicoureteral reflux or lower urinary tract dysfunction (bladder tumors, prostatic hyperplasia, strictures, or stones), is present. Resistance to antibiotics and failure to take a full prescription of antibiotics for a UTI usually result in relapse or reinfection of the lower urinary tract.

What is the most common cause of acute pyelonephritis resulting from an ascending infection from the lower urinary tract? a. The kidney is scarred and fibrotic. b. The organism is resistant to antibiotics. c. There is a preexisting abnormality of the urinary tract. d. The patient does not take all of the antibiotics for treatment of a UTI

c. Ascending infections from the bladder to the kidney are prevented by the normal anatomy and physiology of the urinary tract unless a preexisting condition, such as vesicoureteral reflux or lower urinary tract dysfunction (bladder tumors, prostatic hyperplasia, strictures, or stones), is present. Resistance to antibiotics and failure to take a full prescription of antibiotics for a UTI usually result in relapse or reinfection of the lower urinary tract.

Nursing staff on a hospital unit are reviewing rates of hospital-acquired infections (HAI) of the urinary tract. Which nursing action will be most helpful in decreasing the risk for HAI in patients admitted to the hospital? a. Encouraging adequate oral fluid intake b. Testing urine with a dipstick daily for nitrites c. Avoiding unnecessary urinary catheterizations d. Providing frequent perineal hygiene to patients

c. Avoiding unnecessary urinary catheterizations Because catheterization bypasses many of the protective mechanisms that prevent urinary tract infection (UTI), avoidance of catheterization is the most effective means of reducing HAI. The other actions will also be helpful, but are not as useful as decreasing urinary catheter use.

The patient has scleroderma and is experiencing hypertension. The nurse should know that this could be related to which renal problem? a. Obstructive uropathy b. Goodpasture syndrome c. Chronic glomerulonephritis d. Calcium oxalate urinary calculi

c. Chronic glomerulonephritis Hypertension occurs with chronic glomerulonephritis that may be found in patients with scleroderma. Obstructive uropathy, Goodpasture syndrome, and calcium oxalate urinary calculi are not related to scleroderma and do not cause hypertension.

What can patients at risk for renal lithiasis do to prevent the stones in many cases? a. Lead an active lifestyle b. Limit protein and acidic foods in the diet c. Drink enough fluids to produce dilute urine d. Take prophylactic antibiotics to control UTIs

c. Because crystallization of stone constituents can precipitate and unite to form a stone when in supersaturated concentrations, one of the best ways to prevent stones of any type is by drinking adequate fluids to keep the urine dilute and flowing (e.g., an output of about 2 L of urine a day). Sedentary lifestyle is a risk factor for renal stones but exercise also causes fluid loss and a need for additional fluids. Protein foods high in purine should be restricted only for the small percentage of patients with uric acid stones and although UTIs contribute to stone formation, prophylactic antibiotics are not indicated.

"What can patients at risk for renal lithiasis do to prevent the stones in many cases? a. Lead an active lifestyle b. Limit protein and acidic foods in the diet c. Drink enough fluids to produce dilute urine d. Take prophylactic antibiotics to control UTIs

c. Because crystallization of stone constituents can precipitate and unite to form a stone when in supersaturated concentrations, one of the best ways to prevent stones of any type is by drinking adequate fluids to keep the urine dilute and flowing (e.g., an output of about 2 L of urine a day). Sedentary lifestyle is a risk factor for renal stones but exercise also causes fluid loss and a need for additional fluids. Protein foods high in purine should be restricted only for the small percentage of patients with uric acid stones and although UTIs contribute to stone formation, prophylactic antibiotics are not indicated.

An older male patient visits his primary care provider because of burning on urination and production of urine that he describes as "foul smelling." The health care provider should assess the patient for what factor that may put him at risk for a urinary tract infection (UTI)? a. High-purine diet b. Sedentary lifestyle c. Benign prostatic hyperplasia (BPH) d. Recent use of broad-spectrum antibiotics

c. Benign prostatic hyperplasia (BPH) BPH causes urinary stasis, which is a predisposing factor for UTIs. A sedentary lifestyle and recent antibiotic use are unlikely to contribute to UTIs, whereas a diet high in purines is associated with renal calculi.

Which type of urinary tract calculi are the most common and frequently obstruct the ureter? a. Cystine b. Uric acid c. Calcium oxalate d. Calcium phosphate

c. Calcium oxalate calculi are most common and small enough to get trapped in the ureter.

Which type of urinary tract calculi are the most common and frequently obstruct the ureter? a. Cystine b. Uric acid c. Calcium oxalate d. Calcium phosphate

c. Calcium oxalate calculi are most common and small enough to get trapped in the ureter.

After a ureterolithotomy, a female patient has a left ureteral catheter and a urethral catheter in place. Which action will the nurse include in the plan of care? a. Provide teaching about home care for both catheters. b. Apply continuous steady tension to the ureteral catheter. c. Call the health care provider if the ureteral catheter output drops suddenly. d. Clamp the ureteral catheter off when output from the urethral catheter stops.

c. Call the health care provider if the ureteral catheter output drops suddenly. The health care provider should be notified if the ureteral catheter output decreases because obstruction of this catheter may result in an increase in pressure in the renal pelvis. Tension on the ureteral catheter should be avoided in order to prevent catheter displacement. To avoid pressure in the renal pelvis, the catheter is not clamped. Because the patient is not usually discharged with a ureteral catheter in place, patient teaching about both catheters is not needed.

A 65-year-old patient with type 2 diabetes has a urinary tract infection (UTI). The unlicensed assistive personnel (UAP) reported to the nurse that the patient's blood glucose is 642 mg/dL and the patient is hard to arouse. When the nurse assesses the urine, there are no ketones present. What collaborative care should the nurse expect for this patient? a. Routine insulin therapy and exercise b. Administer a different antibiotic for the UTI. c. Cardiac monitoring to detect potassium changes d. Administer IV fluids rapidly to correct dehydration.

c. Cardiac monitoring to detect potassium changes This patient has manifestations of hyperosmolar hyperglycemic syndrome (HHS). Cardiac monitoring will be needed because of the changes in the potassium level related to fluid and insulin therapy and the osmotic diuresis from the elevated serum glucose level. Routine insulin would not be enough, and exercise could be dangerous for this patient. Extra insulin will be needed. The type of antibiotic will not affect HHS. There will be a large amount of IV fluid administered, but it will be given slowly because this patient is older and may have cardiac or renal compromise requiring hemodynamic monitoring to avoid fluid overload during fluid replacement.

A 68-year-old male patient who has bladder cancer had a cystectomy with creation of an Indiana pouch. Which topic will be included in patient teaching? a. Application of ostomy appliances b. Barrier products for skin protection c. Catheterization technique and schedule d. Analgesic use before emptying the pouch

c. Catheterization technique and schedule The Indiana pouch enables the patient to self-catheterize every 4 to 6 hours. There is no need for an ostomy device or barrier products. Catheterization of the pouch is not painful.

Which statement by a nurse to a patient newly diagnosed with type 2 diabetes is correct? a. Insulin is not used to control blood glucose in patients with type 2 diabetes. b. Complications of type 2 diabetes are less serious than those of type 1 diabetes. c. Changes in diet and exercise may control blood glucose levels in type 2 diabetes. d. Type 2 diabetes is usually diagnosed when the patient is admitted with a hyperglycemic coma.

c. Changes in diet and exercise may control blood glucose levels in type 2 diabetes. For some patients with type 2 diabetes, changes in lifestyle are sufficient to achieve blood glucose control. Insulin is frequently used for type 2 diabetes, complications are equally severe as for type 1 diabetes, and type 2 diabetes is usually diagnosed with routine laboratory testing or after a patient develops complications such as frequent yeast infections.

The health care provider suspects the Somogyi effect in a 50-year-old patient whose 6:00 AM blood glucose is 230 mg/dL. Which action will the nurse teach the patient to take? a. Avoid snacking at bedtime. b. Increase the rapid-acting insulin dose. c. Check the blood glucose during the night d. Administer a larger dose of long-acting insulin.

c. Check the blood glucose during the night

Which instruction should the nurse provide when teaching a patient to exercise the pelvic floor? a. Tighten both buttocks together. b. Squeeze thighs together tightly. c. Contract muscles around rectum. d. Lie on back and lift legs together.

c. Contract muscles around rectum. To teach pelvic floor exercises, or Kegel exercise, the nurse should instruct the patient (without contracting the legs, buttocks, or abdomen) to contract the muscles around the rectum (pelvic floor muscles) as if stopping a stool, which should result in a pelvic lifting sensation.

The nurse observes unlicensed assistive personnel (UAP) taking the following actions when caring for a female patient with a urethral catheter. Which action requires that the nurse intervene? a. Taping the catheter to the skin on the patient's upper inner thigh b. Cleaning around the patient's urinary meatus with soap and water c. Disconnecting the catheter from the drainage tube to obtain a specimen d. Using an alcohol-based gel hand cleaner before performing catheter care

c. Disconnecting the catheter from the drainage tube to obtain a specimen The catheter should not be disconnected from the drainage tube because this increases the risk for urinary tract infection (UTI). The other actions are appropriate and do not require any intervention.

A college student is newly diagnosed with type 1 diabetes. She now has a headache, changes in her vision, and is anxious, but does not have her portable blood glucose monitor with her. Which action should the campus nurse advise her to take? a. Eat a piece of pizza. b. Drink some diet pop. c. Eat 15 g of simple carbohydrates. d. Take an extra dose of rapid-acting insulin.

c. Eat 15 g of simple carbohydrates. When the patient with type 1 diabetes is unsure about the meaning of the symptoms she is experiencing, she should treat herself for hypoglycemia to prevent seizures and coma from occurring. She should also be advised to check her blood glucose as soon as possible. The fat in the pizza and the diet pop would not allow the blood glucose to increase to eliminate the symptoms. The extra dose of rapid-acting insulin would further decrease her blood glucose.

A 32-year-old patient with a history of polycystic kidney disease is admitted to the surgical unit after having shoulder surgery. Which of the routine postoperative orders is most important for the nurse to discuss with the health care provider? a. Infuse 5% dextrose in normal saline at 75 mL/hr. b. Order regular diet after patient is awake and alert. c. Give ketorolac (Toradol) 10 mg PO PRN for pain. d. Draw blood urea nitrogen (BUN) and creatinine in 2 hours.

c. Give ketorolac (Toradol) 10 mg PO PRN for pain. The nonsteroidal antiinflammatory drugs (NSAIDs) should be avoided in patients with decreased renal function because nephrotoxicity is a potential adverse effect. The other orders do not need any clarification or change.

Glomerulonephritis is characterized by glomerular damage caused by a. growth of microorganisms in the glomeruli. b. release of bacterial substances toxic to the glomeruli. c. accumulation of immune complexes in the glomeruli. d. hemolysis of red blood cells circulating in the glomeruli.

c. Glomerulonephritis is not an infection but rather an antibody-induced injury to the glomerulus, where either autoantibodies against the glomerular basement membrane (GBM) directly damage the tissue or antibodies reacting with nonglomerular antigens are randomly deposited as immune complexes along the GBM. Prior infection by bacteria or viruses may stimulate the antibody production but is not present or active at the time of glomerular damage.

Glomerulonephritis is characterized by glomerular damage caused by a. growth of microorganisms in the glomeruli. b. release of bacterial substances toxic to the glomeruli. c. accumulation of immune complexes in the glomeruli. d. hemolysis of red blood cells circulating in the glomeruli.

c. Glomerulonephritis is not an infection but rather an antibody-induced injury to the glomerulus, where either autoantibodies against the glomerular basement membrane (GBM) directly damage the tissue or antibodies reacting with nonglomerular antigens are randomly deposited as immune complexes along the GBM. Prior infection by bacteria or viruses may stimulate the antibody production but is not present or active at the time of glomerular damage.

A 63-year-old male patient had a cystectomy with an ileal conduit yesterday. Which new assessment data is most important for the nurse to communicate to the physician? a. Cloudy appearing urine b. Hypotonic bowel sounds c. Heart rate 102 beats/minute d. Continuous stoma drainage

c. Heart rate 102 beats/minute Tachycardia may indicate infection, hemorrhage, or hypovolemia, which are all serious complications of this surgery. The urine from an ileal conduit normally contains mucus and is cloudy. Hypotonic bowel sounds are expected after bowel surgery. Continuous drainage of urine from the stoma is normal.

A patient, who is admitted with diabetes mellitus, has a glucose level of 380 mg/dL and a moderate level of ketones in the urine. As the nurse assesses for signs of ketoacidosis, which respiratory pattern would the nurse expect to find? a. Central apnea b. Hypoventilation c. Kussmaul respirations d. Cheyne-Stokes respirations

c. Kussmaul respirations In diabetic ketoacidosis, the lungs try to compensate for the acidosis by blowing off volatile acids and carbon dioxide. This leads to a pattern of Kussmaul respirations, which are deep and nonlabored. Central apnea occurs because the brain temporarily stops sending signals to the muscles that control breathing, which is unrelated to ketoacidosis. Hypoventilation and Cheyne-Stokes respirations do not occur with ketoacidosis.

A patient who was admitted with diabetic ketoacidosis secondary to a urinary tract infection has been weaned off an insulin drip 30 minutes ago. The patient reports feeling lightheaded and sweaty. Which action should the nurse take first? a. Infuse dextrose 50% by slow IV push. b. Administer 1 mg glucagon subcutaneously. c. Obtain a glucose reading using a finger stick. d. Have the patient drink 4 ounces of orange juice.

c. Obtain a glucose reading using a finger stick.

Which information is most important for the nurse to report to the health care provider before a patient with type 2 diabetes is prepared for a coronary angiogram? a. The patient's most recent HbA1C was 6.5%. b. The patient's admission blood glucose is 128 mg/dL. c. The patient took the prescribed metformin (Glucophage) today. d. The patient took the prescribed captopril (Capoten) this morning.

c. The patient took the prescribed metformin (Glucophage) today.

A 46-year-old female patient returns to the clinic with recurrent dysuria after being treated with trimethoprim and sulfamethoxazole (Bactrim) for 3 days. Which action will the nurse plan to take? a. Teach the patient to take the prescribed Bactrim for 3 more days. b. Remind the patient about the need to drink 1000 mL of fluids daily. c. Obtain a midstream urine specimen for culture and sensitivity testing. d. Suggest that the patient use acetaminophen (Tylenol) to treat the symptoms.

c. Obtain a midstream urine specimen for culture and sensitivity testing. Because uncomplicated urinary tract infections (UTIs) are usually successfully treated with 3 days of antibiotic therapy, this patient will need a urine culture and sensitivity to determine appropriate antibiotic therapy. Acetaminophen would not be as effective as other over-the-counter (OTC) medications such as phenazopyridine (Pyridium) in treating dysuria. The fluid intake should be increased to at least 1800 mL/day. Because the UTI has persisted after treatment with Bactrim, the patient is likely to need a different antibiotic.

To assist the patient with stress incontinence, what is the best thing the nurse should teach the patient to do? a. Void every 2 hours to prevent leakage. b. Use absorptive perineal pads to contain urine. c. Perform pelvic floor muscle exercises 40 to 50 times per day. d. Increase intraabdominal pressure during voiding to empty the bladder completely.

c. Pelvic floor exercises (Kegel exercises) increase the tone of the urethral sphincters and should be done in sets of 10 or more contractions four to five times a day (total of 40 to 50 per day). Frequent bladder emptying is recommended for patients with urge incontinence and an increase in pressure on the bladder is recommended for patients with overflow incontinence. Absorptive perineal pads should be only a temporary measure because long- term use discourages continence and can lead to skin problems.

To assist the patient with stress incontinence, what is the best thing the nurse should teach the patient to do? a. Void every 2 hours to prevent leakage. b. Use absorptive perineal pads to contain urine. c. Perform pelvic floor muscle exercises 40 to 50 times per day. d. Increase intraabdominal pressure during voiding to empty the bladder completely.

c. Pelvic floor exercises (Kegel exercises) increase the tone of the urethral sphincters and should be done in sets of 10 or more contractions four to five times a day (total of 40 to 50 per day). Frequent bladder emptying is recommended for patients with urge incontinence and an increase in pressure on the bladder is recommended for patients with overflow incontinence. Absorptive perineal pads should be only a temporary measure because long- term use discourages continence and can lead to skin problems.

A patient admitted to the hospital with pneumonia has a history of functional urinary incontinence. Which nursing action will be included in the plan of care? a. Demonstrate the use of the Credé maneuver. b. Teach exercises to strengthen the pelvic floor. c. Place a bedside commode close to the patient's bed. d. Use an ultrasound scanner to check postvoiding residuals.

c. Place a bedside commode close to the patient's bed Modifications in the environment make it easier to avoid functional incontinence. Checking for residual urine and performing the Credé maneuver are interventions for overflow incontinence. Kegel exercises are useful for stress incontinence.

Following bowel resection, a patient has a nasogastric (NG) tube to suction, but complains of nausea and abdominal distention. The nurse irrigates the tube as necessary as ordered, but the irrigating fluid does not return. What should be the priority action by the nurse? a. Notify the physician. b. Auscultate for bowel sounds. c. Reposition the tube and check for placement. d. Remove the tube and replace it with a new one.

c. Reposition the tube and check for placement. The tube may be resting against the stomach wall. The first action by the nurse (since this is intestinal surgery and not gastric surgery) is to reposition the tube and check it again for placement. The physician does not need to be notified unless the tube function cannot be restored by the nurse. The patient does not have bowel sounds, which is why the NG tube is in place. The NG tube would not be removed and replaced unless it was no longer in the stomach or the obstruction of the tube could not be relieved.

When caring for a patient with nephrotic syndrome, the nurse should know the patient understands dietary teaching when the patient selects which food item? a. Peanut butter and crackers b. One small grilled pork chop c. Salad made of fresh vegetables d. Spaghetti with canned spaghetti sauce

c. Salad made of fresh vegetables Of the options listed, only salad made with fresh vegetables would be acceptable for the diet that limits sodium and protein as well as saturated fat if hyperlipidemia is present. Peanut butter and crackers are processed so they contain significant sodium, and peanut butter contains some protein. A pork chop is a high-protein food with saturated fat. Canned spaghetti sauce is also high in sodium.

What results in the edema associated with nephrotic syndrome? a. Hypercoagulability b. Hyperalbuminemia c. Decreased plasma oncotic pressure d. Decreased glomerular filtration rate

c. The massive proteinuria that results from increased glomerular membrane permeability in nephrotic syndrome leaves the blood without adequate proteins (hypoalbuminemia) to create an oncotic colloidal pressure to hold fluid in the vessels. Without oncotic pressure, fluid moves into the interstitium, causing severe edema. Hypercoagulability occurs in nephrotic syndrome but is not a factor in edema formation and glomerular filtration rate (GFR) is not necessarily affected in nephrotic syndrome.

What results in the edema associated with nephrotic syndrome? a. Hypercoagulability b. Hyperalbuminemia c. Decreased plasma oncotic pressure d. Decreased glomerular filtration rate

c. The massive proteinuria that results from increased glomerular membrane permeability in nephrotic syndrome leaves the blood without adequate proteins (hypoalbuminemia) to create an oncotic colloidal pressure to hold fluid in the vessels. Without oncotic pressure, fluid moves into the interstitium, causing severe edema. Hypercoagulability occurs in nephrotic syndrome but is not a factor in edema formation and glomerular filtration rate (GFR) is not necessarily affected in nephrotic syndrome.

Which information from a patient who had a transurethral resection with fulguration for bladder cancer 3 days ago is most important to report to the health care provider? a. The patient is voiding every 4 hours. b. The patient is using opioids for pain. c. The patient has seen clots in the urine. d. The patient is anxious about the cancer.

c. The patient has seen clots in the urine. Clots in the urine are not expected and require further follow-up. Voiding every 4 hours, use of opioids for pain, and anxiety are typical after this procedure.

While caring for a 77-year-old woman who has a urinary catheter, the nurse monitors the patient for the development of a UTI. What clinical manifestations is the patient most likely to experience? a. Cloudy urine and fever b. Urethral burning and bloody urine c. Vague abdominal discomfort and disorientation d. Suprapubic pain and slight decline in body temperature

c. The usual classic manifestations of UTI are often absent in older adults, who tend to experience nonlocalized abdominal discomfort and cognitive impairment characterized by confusion or decreased level of consciousness rather than dysuria and suprapubic pain.

While caring for a 77-year-old woman who has a urinary catheter, the nurse monitors the patient for the development of a UTI. What clinical manifestations is the patient most likely to experience? a. Cloudy urine and fever b. Urethral burning and bloody urine c. Vague abdominal discomfort and disorientation d. Suprapubic pain and slight decline in body temperature

c. The usual classic manifestations of UTI are often absent in older adults, who tend to experience nonlocalized abdominal discomfort and cognitive impairment characterized by confusion or decreased level of consciousness rather than dysuria and suprapubic pain.

Thirty percent of patients with kidney cancer have metastasis at the time of diagnosis. Why does this occur? a. The only treatment modalities for the disease are palliative. b. Diagnostic tests are not available to detect tumors before they metastasize. c. Classic symptoms of hematuria and palpable mass do not occur until the disease is advanced. d. Early metastasis to the brain impairs the patient's ability to recognize the seriousness of symptoms.

c. There are no early characteristic symptoms of cancer of the kidney and gross hematuria, flank pain, and a palpable mass do not occur until the disease is advanced. The treatment of choice is a partial or radical nephrectomy, which can be successful in early disease. Many kidney cancers are diagnosed as incidental imaging findings. Targeted therapy is the preferred treatment for metastatic kidney cancer. Radiation is palliative. The most common sites of metastases are the lungs, liver, and long bones.

Thirty percent of patients with kidney cancer have metastasis at the time of diagnosis. Why does this occur? a. The only treatment modalities for the disease are palliative. b. Diagnostic tests are not available to detect tumors before they metastasize. c. Classic symptoms of hematuria and palpable mass do not occur until the disease is advanced. d. Early metastasis to the brain impairs the patient's ability to recognize the seriousness of symptoms.

c. There are no early characteristic symptoms of cancer of the kidney and gross hematuria, flank pain, and a palpable mass do not occur until the disease is advanced. The treatment of choice is a partial or radical nephrectomy, which can be successful in early disease. Many kidney cancers are diagnosed as incidental imaging findings. Targeted therapy is the preferred treatment for metastatic kidney cancer. Radiation is palliative. The most common sites of metastases are the lungs, liver, and long bones.

A woman with no history of UTIs who is experiencing urgency, frequency, and dysuria comes to the clinic, where a dipstick and microscopic urinalysis indicate bacteriuria. What should the nurse anticipate for this patient? a. Obtaining a clean-catch midstream urine specimen for culture and sensitivity b. No treatment with medication unless she develops fever, chills, and flank pain c. Empirical treatment with trimethoprim-sulfamethoxazole (TMP-SMX, Bactrim) for 3 days d. Need to have a blood specimen drawn for a complete blood count (CBC) and kidney function tests

c. Unless a patient has a history of recurrent UTIs or a complicated UTI, trimethoprim-sulfamethoxazole (TMP- SMX) or nitrofurantoin (Macrodantin) is usually used to empirically treat an initial UTI without a culture and sensitivity or other testing. Asymptomatic bacteriuria does not justify treatment but symptomatic UTIs should always be treated.

A woman with no history of UTIs who is experiencing urgency, frequency, and dysuria comes to the clinic, where a dipstick and microscopic urinalysis indicate bacteriuria. What should the nurse anticipate for this patient? a. Obtaining a clean-catch midstream urine specimen for culture and sensitivity b. No treatment with medication unless she develops fever, chills, and flank pain c. Empirical treatment with trimethoprim-sulfamethoxazole (TMP-SMX, Bactrim) for 3 days d. Need to have a blood specimen drawn for a complete blood count (CBC) and kidney function tests

c. Unless a patient has a history of recurrent UTIs or a complicated UTI, trimethoprim-sulfamethoxazole (TMP- SMX) or nitrofurantoin (Macrodantin) is usually used to empirically treat an initial UTI without a culture and sensitivity or other testing. Asymptomatic bacteriuria does not justify treatment but symptomatic UTIs should always be treated.

The nurse will anticipate teaching a patient with nephrotic syndrome who develops flank pain about treatment with a. antibiotics. b. antifungals. c. anticoagulants. d. antihypertensives.

c. anticoagulants. Flank pain in a patient with nephrotic syndrome suggests a renal vein thrombosis, and anticoagulation is needed. Antibiotics are used to treat a patient with flank pain caused by pyelonephritis. Fungal pyelonephritis is uncommon and is treated with antifungals. Antihypertensives are used if the patient has high blood pressure.

A 42-year-old female patient is scheduled for transsphenoidal hypophysectomy to treat a pituitary adenoma. During preoperative teaching, the nurse instructs the patient about the need to a. cough and deep breathe every 2 hours postoperatively. b. remain on bed rest for the first 48 hours after the surgery. c. avoid brushing teeth for at least 10 days after the surgery. d. be positioned flat with sandbags at the head postoperatively.

c. avoid brushing teeth for at least 10 days after the surgery. To avoid disruption of the suture line, the patient should avoid brushing the teeth for 10 days after surgery. It is not necessary to remain on bed rest after this surgery. Coughing is discouraged because it may cause leakage of cerebrospinal fluid (CSF) from the suture line. The head of the bed should be elevated 30 degrees to reduce pressure on the sella turcica and decrease the risk for headaches

When preparing a female patient with bladder cancer for intravesical chemotherapy, the nurse will teach about a. premedicating to prevent nausea. b. obtaining wigs and scarves to wear. c. emptying the bladder before the medication. d. maintaining oral care during the treatments.

c. emptying the bladder before the medication. The patient will be asked to empty the bladder before instillation of the chemotherapy. Systemic side effects are not usually experienced with intravesical chemotherapy.

A patient with stage I colorectal cancer is scheduled for surgery. Patient teaching for this patient would include an explanation that: a. chemotherapy will begin after the patient recovers from the surgery b. both chemotherapy and radiation can be used as palliative treatments c. follow-up colonoscopies will be needed to ensure that the cancer does not occur d. a wound, ostomy, and continence nurse will visit the patient to identify an abdominal site for the ostomy

c. follow-up colonoscopies will be needed to ensure that the cancer does not occur Rationale: Stage 1 colorectal cancer is treated with surgical removal of the tumor and reanastomosis, and so there is no ostomy. Chemotherapy is not recommended for stage I tumors. Follow-up colonoscopy is recommended because colorectal cancer can recur.

After obtaining the info below regarding a pt with Addison's disease, which prescribed action will the nurse take first? complaints of fatigue, bronze skin, poor turgor, BP 76/40, HR 126, RR 24, O2 Sats 94%, Na 123, K+ 5.1, Glucose 62 a. Give 4 oz fruit juice orally b. recheck blood glucose lvl c. infuse 5% dextrose and 0.9% saline d. Admin O2 therapy PRN

c. infuse 5% dextrose & 0.9% saline The patient's poor skin turgor, hypotension, and hyponatremia indicate an Addisonian crisis. Immediate correction of the hypovolemia and hyponatremia is needed. The other actions may also be needed but are not the initial action for the patient.

A 26-year-old female with type 1 diabetes develops a sore throat and runny nose after caring for her sick toddler. The patient calls the clinic for advice about her symptoms and a blood glucose level of 210 mg/dL despite taking her usual glargine (Lantus) and lispro (Humalog) insulin. The nurse advises the patient to a. use only the lispro insulin until the symptoms are resolved. b. limit intake of calories until the glucose is less than 120 mg/dL. c. monitor blood glucose every 4 hours and notify the clinic if it continues to rise. d. decrease intake of carbohydrates until glycosylated hemoglobin is less than 7%.

c. monitor blood glucose every 4 hours and notify the clinic if it continues to rise.

In planning care for the patient with Crohn's disease, the nurse recognizes that a major difference between ulcerative colitis and Crohn's disease is that Crohn's disease: a. frequently results in toxic megacolon b. causes fever nutritional deficiencies than ulcerative colitis c. often recurs after surgery, whereas ulcerative colitis is curable with a colectomy d. is manifested by rectal bleeding and anemia more frequently than is ulcerative colitis

c. often recurs after surgery, whereas ulcerative colitis is curable with a colectomy Rationale: Ulcerative colitis affects only the colon and rectum; it can cause megacolon and rectal bleeding, but not nutrient malabsorption. Surgical removal of the colon and rectum cures it. Crohn's disease usually involves the ileum, where bile salts and vitamin cobalamin are absorbed. After surgical treatment, disease recurrence at the site is common.

When a patient who takes metformin (Glucophage) to manage type 2 diabetes develops an allergic rash from an unknown cause, the health care provider prescribes prednisone (Deltasone). The nurse will anticipate that the patient may a. need a diet higher in calories while receiving prednisone. b. develop acute hypoglycemia while taking the prednisone. c. require administration of insulin while taking prednisone. d. have rashes caused by metformin-prednisone interactions.

c. require administration of insulin while taking prednisone.

An expected nursing diagnosis for a 30-year-old patient admitted to the hospital with symptoms of diabetes insipidus is a. excess fluid volume related to intake greater than output. b. impaired gas exchange related to fluid retention in lungs. c. sleep pattern disturbance related to frequent waking to void. d. risk for impaired skin integrity related to generalized edema.

c. sleep pattern disturbance related to frequent waking to void. Nocturia occurs as a result of the polyuria caused by diabetes insipidus. Edema, excess fluid volume, and fluid retention are not expected

A 62-year-old patient with hyperthyroidism is to be treated with radioactive iodine (RAI). The nurse instructs the patient a. about radioactive precautions to take with all body secretions. b. that symptoms of hyperthyroidism should be relieved in about a week. c. that symptoms of hypothyroidism may occur as the RAI therapy takes effect. d. to discontinue the antithyroid medications taken before the radioactive therapy.

c. that symptoms of hypothyroidism may occur as the RAI therapy takes effect. There is a high incidence of postradiation hypothyroidism after RAI, and the patient should be monitored for symptoms of hypothyroidism. RAI has a delayed response, with the maximum effect not seen for 2 to 3 months, and the patient will continue to take antithyroid medications during this time. The therapeutic dose of radioactive iodine is low enough that no radiation safety precautions are needed

Which statement would be correct for a patient with type 2 diabetes who was admitted to the hospital with pneumonia? a) The patient must receive insulin therapy to prevent ketoacidosis. b) The patient has islet cell antibodies that have destroyed the pancreas's ability to produce insulin. c) The patient has minimal or absent endogenous insulin secretion and requires daily insulin injections d) The patient may have sufficient endogenous insulin to prevent ketosis but is at risk for hyperosmolar hyperglycemic syndrome

d) The patient may have sufficient endogenous insulin to prevent ketosis but is at risk for hyperosmolar hyperglycemic syndrome Hyperosmolar hyperglycemic syndrome (HHS) is a life-threatening syndrome that can occur in a patient with diabetes who is able to produce enough insulin to prevent diabetic ketoacidosis (DKA) but not enough to prevent severe hyperglycemia, osmotic diuresis, and extracellular fluid depletion.

Which characteristics are associated with urge incontinence (select all that apply)? a. Treated with Kegel exercises b. Found following prostatectomy c. Common in postmenopausal women d. Involuntary urination preceded by urgency e. Caused by overactivity of the detrusor muscle f. Bladder contracts by reflex, overriding central inhibition

d, e, f. Urge incontinence is involuntary urination preceded by urgency caused by overactivity of the detrusor muscle when the bladder contracts by reflex, which overrides central inhibition. Treatment includes treating the underlying cause and retraining the bladder with urge suppression, anticholinergic drugs, or containment devices. The other options are characteristic of stress incontinence. Patients may have a combination of urge and stress incontinence.

Which characteristics are associated with urge incontinence (select all that apply)? a. Treated with Kegel exercises b. Found following prostatectomy c. Common in postmenopausal women d. Involuntary urination preceded by urgency e. Caused by overactivity of the detrusor muscle f. Bladder contracts by reflex, overriding central inhibition

d, e, f. Urge incontinence is involuntary urination preceded by urgency caused by overactivity of the detrusor muscle when the bladder contracts by reflex, which overrides central inhibition. Treatment includes treating the underlying cause and retraining the bladder with urge suppression, anticholinergic drugs, or containment devices. The other options are characteristic of stress incontinence. Patients may have a combination of urge and stress incontinence.

The nurse determines that instruction regarding prevention of future urinary tract infections (UTIs) has been effective for a 22-year-old female patient with cystitis when the patient states which of the following? a. "I can use vaginal antiseptic sprays to reduce bacteria." b. "I will drink a quart of water or other fluids every day." c. "I will wash with soap and water before sexual intercourse." d. "I will empty my bladder every 3 to 4 hours during the day."

d. "I will empty my bladder every 3 to 4 hours during the day." Voiding every 3 to 4 hours is recommended to prevent UTIs. Use of vaginal sprays is discouraged. The bladder should be emptied before and after intercourse, but cleaning with soap and water is not necessary. A quart of fluids is insufficient to provide adequate urine output to decrease risk for UTI.

The nurse has been teaching a patient with type 2 diabetes about managing blood glucose levels and taking glipizide (Glucotrol). Which patient statement indicates a need for additional teaching? a. "If I overeat at a meal, I will still take the usual dose of medication." b. "Other medications besides the Glucotrol may affect my blood sugar." c. "When I am ill, I may have to take insulin to control my blood sugar." d. "My diabetes won't cause complications because I don't need insulin."

d. "My diabetes won't cause complications because I don't need insulin."

The nurse is caring for a postoperative patient with a colostomy. The nurse is preparing to administer a dose of famotidine (Pepcid) when the patient asks why the medication was ordered since the patient does not have a history of heartburn or gastroesophageal reflux disease (GERD). What response by the nurse would be the most appropriate? a. "This will prevent air from accumulating in the stomach, causing gas pains." b. "This will prevent the heartburn that occurs as a side effect of general anesthesia." c. "The stress of surgery is likely to cause stomach bleeding if you do not receive it." d. "This will reduce the amount of HCl in the stomach until the nasogastric tube is removed and you can eat a regular diet again."

d. "This will reduce the amount of HCl in the stomach until the nasogastric tube is removed and you can eat a regular diet again." Famotidine is an H2-receptor antagonist that inhibits gastric HCl secretion and thus minimizes damage to gastric mucosa while the patient is not eating a regular diet after surgery. Famotidine does not prevent air from accumulating in the stomach or stop the stomach from bleeding. Heartburn is not a side effect of general anesthesia.

"Which test is required for a diagnosis of pyelonephritis? a. Renal biopsy b. Blood culture c. Intravenous pyelogram (IVP) d. Urine for culture and sensitivity

d. A urine specimen specifically obtained for culture and sensitivity is required to diagnose pyelonephritis because it will show pyuria, the specific bacteriuria, and what drug the bacteria is sensitive to for treatment. The renal biopsy is used to diagnose chronic pyelonephritis or cancer. Blood cultures would be done if bacteremia is suspected. Intravenous pyelogram (IVP) would increase renal irritation, but CT urograms may be used to assess for signs of infection in the kidney and complications of pyelonephritis.

Which test is required for a diagnosis of pyelonephritis? a. Renal biopsy b. Blood culture c. Intravenous pyelogram (IVP) d. Urine for culture and sensitivity

d. A urine specimen specifically obtained for culture and sensitivity is required to diagnose pyelonephritis because it will show pyuria, the specific bacteriuria, and what drug the bacteria is sensitive to for treatment. The renal biopsy is used to diagnose chronic pyelonephritis or cancer. Blood cultures would be done if bacteremia is suspected. Intravenous pyelogram (IVP) would increase renal irritation, but CT urograms may be used to assess for signs of infection in the kidney and complications of pyelonephritis.

What manifestation in the patient will indicate the need for restriction of dietary protein in management of acute poststreptococcal glomerulonephritis (APSGN)? a. Hematuria b. Proteinuria c. Hypertension d. Elevated blood urea nitrogen (BUN)

d. An elevated blood urea nitrogen (BUN) indicates that the kidneys are not clearing nitrogenous wastes from the blood and protein may be restricted until the kidney recovers. Proteinuria indicates loss of protein from the blood and possibly a need for increased protein intake. Hypertension is treated with sodium and fluid restriction, diuretics, and antihypertensive drugs. The hematuria is not specifically treated.

What manifestation in the patient will indicate the need for restriction of dietary protein in management of acute poststreptococcal glomerulonephritis (APSGN)? a. Hematuria b. Proteinuria c. Hypertension d. Elevated blood urea nitrogen (BUN)

d. An elevated blood urea nitrogen (BUN) indicates that the kidneys are not clearing nitrogenous wastes from the blood and protein may be restricted until the kidney recovers. Proteinuria indicates loss of protein from the blood and possibly a need for increased protein intake. Hypertension is treated with sodium and fluid restriction, diuretics, and antihypertensive drugs. The hematuria is not specifically treated.

When caring for the patient with interstitial cystitis, what can the nurse teach the patient to do? a. Avoid foods that make the urine more alkaline. b. Use high-potency vitamin therapy to decrease the autoimmune effects of the disorder. c. Always keep a voiding diary to document pain, voiding frequency, and patterns of nocturia. d. Use the dietary supplement calcium glycerophosphate (Prelief) to decrease bladder irritation.

d. Calcium glycerophosphate (Prelief) alkalinizes the urine and can help to relieve the irritation from acidic foods. A diet low in acidic foods is recommended and if a multivitamin is used, high-potency vitamins should be avoided because these products may irritate the bladder. A voiding diary is useful in diagnosis but does not need to be kept indefinitely.

When caring for the patient with interstitial cystitis, what can the nurse teach the patient to do? a. Avoid foods that make the urine more alkaline. b. Use high-potency vitamin therapy to decrease the autoimmune effects of the disorder. c. Always keep a voiding diary to document pain, voiding frequency, and patterns of nocturia. d. Use the dietary supplement calcium glycerophosphate (Prelief) to decrease bladder irritation.

d. Calcium glycerophosphate (Prelief) alkalinizes the urine and can help to relieve the irritation from acidic foods. A diet low in acidic foods is recommended and if a multivitamin is used, high-potency vitamins should be avoided because these products may irritate the bladder. A voiding diary is useful in diagnosis but does not need to be kept indefinitely.

The nurse is preparing to administer a scheduled dose of docusate sodium (Colace) when the patient reports an episode of loose stool and does not want to take the medication. What is the appropriate action by the nurse? a. Write an incident report about this untoward event. b. Attempt to have the family convince the patient to take the ordered dose. c. Withhold the medication at this time and try to administer it later in the day. d. Chart the dose as not given on the medical record and explain in the nursing progress notes.

d. Chart the dose as not given on the medical record and explain in the nursing progress notes. Whenever a patient refuses medication, the dose should be charted as not given with an explanation of the reason documented in the nursing progress notes. In this instance, the refusal indicates good judgment by the patient, and the patient should not be encouraged to take it today.

The nurse is providing care for a patient who has been admitted to the hospital for the treatment of nephrotic syndrome. What are priority nursing assessments in the care of this patient? a. Assessment of pain and level of consciousness b. Assessment of serum calcium and phosphorus levels c. Blood pressure and assessment for orthostatic hypotension d. Daily weights and measurement of the patient's abdominal girth

d. Daily weights and measurement of the patient's abdominal girth Peripheral edema is characteristic of nephrotic syndrome, and a key nursing responsibility in the care of patients with the disease is close monitoring of abdominal girth, weights, and extremity size. Pain, level of consciousness, and orthostatic blood pressure are less important in the care of patients with nephrotic syndrome. Abnormal calcium and phosphorus levels are not commonly associated with the diagnosis of nephrotic syndrome.

Which assessment finding is most important to report to the health care provider regarding a patient who has had left-sided extracorporeal shock wave lithotripsy? a. Blood in urine b. Left flank bruising c. Left flank discomfort d. Decreased urine output

d. Decreased urine output Because lithotripsy breaks the stone into small sand, which could cause obstruction, it is important to report a drop in urine output. Left flank pain, bruising, and hematuria are common after lithotripsy.

When teaching the patient about the diet for diverticular disease, which foods should the nurse recommend? a. White bread, cheese, and green beans b. Fresh tomatoes, pears, and corn flakes c. Oranges, baked potatoes, and raw carrots d. Dried beans, All Bran (100%) cereal, and raspberries

d. Dried beans, All Bran (100%) cereal, and raspberries A high fiber diet is recommended for diverticular disease. Dried beans, All Bran (100%) cereal, and raspberries all have higher amounts of fiber than white bread, cheese, green beans, fresh tomatoes, pears, corn flakes, oranges, baked potatoes, and raw carrots.

A patient with type 2 diabetes is scheduled for a follow-up visit in the clinic several months from now. Which test will the nurse schedule to evaluate the effectiveness of treatment for the patient? a. Urine dipstick for glucose b. Oral glucose tolerance test c. Fasting blood glucose level d. Glycosylated hemoglobin level

d. Glycosylated hemoglobin level

The nurse caring for a patient hospitalized with diabetes mellitus would look for which laboratory test result to obtain information on the patient's past glucose control? a. Prealbumin level b. Urine ketone level c. Fasting glucose level d. Glycosylated hemoglobin level

d. Glycosylated hemoglobin level A glycosylated hemoglobin level detects the amount of glucose that is bound to red blood cells (RBCs). When circulating glucose levels are high, glucose attaches to the RBCs and remains there for the life of the blood cell, which is approximately 120 days. Thus the test can give an indication of glycemic control over approximately 2 to 3 months. The prealbumin level is used to establish nutritional status and is unrelated to past glucose control. The urine ketone level will only show that hyperglycemia or starvation is probably currently occurring. The fasting glucose level only indicates current glucose control.

A patient with suprapubic pain and symptoms of urinary frequency and urgency has two negative urine cultures. What is one assessment finding that would indicate interstitial cystitis? a. Residual urine greater than 200 mL b. A large, atonic bladder on urodynamic testing c. A voiding pattern that indicates psychogenic urinary retention d. Pain with bladder filling that is transiently relieved by urination

d. The symptoms of interstitial cystitis (IC) imitate those of an infection of the bladder but the urine is free of infectious agents. Unlike a bladder infection, the pain with IC increases as urine collects in the bladder and is temporarily relieved by urination. Acidic urine is very irritating to the bladder in IC and the bladder is small but urinary retention is not common.

The nurse is caring for a 68-year-old patient admitted with abdominal pain, nausea, and vomiting. The patient has an abdominal mass, and a bowel obstruction is suspected. The nurse auscultating the abdomen listens for which type of bowel sounds that are consistent with the patient's clinical picture? a. Low-pitched and rumbling above the area of obstruction b. High-pitched and hypoactive below the area of obstruction c. Low-pitched and hyperactive below the area of obstruction d. High-pitched and hyperactive above the area of obstruction

d. High-pitched and hyperactive above the area of obstruction Early in intestinal obstruction, the patient's bowel sounds are hyperactive and high-pitched, sometimes referred to as "tinkling" above the level of the obstruction. This occurs because peristaltic action increases to "push past" the area of obstruction. As the obstruction becomes complete, bowel sounds decrease and finally become absent.

The nurse would question the use of which cathartic agent in a patient with renal insufficiency? a. Bisacodyl (Dulcolax) b. Lubiprostone (Amitiza) c. Cascara sagrada (Senekot) d. Magnesium hydroxide (Milk of Magnesia)

d. Magnesium hydroxide (Milk of Magnesia) Milk of Magnesia may cause hypermagnesemia in patients with renal insufficiency. The nurse should question this order with the health care provider. Bisacodyl, lubiprostone, and cascara sagrada are safe to use in patients with renal insufficiency as long as the patient is not currently dehydrated.

The nurse is caring for a 73-year-old man patient with a history of benign prostatic hyperplasia and symptoms of a possible urinary tract infection. Which diagnostic finding would support this diagnosis? a. White blood cell count is 7500 cells/µL. b. Antistreptolysin-O (ASO) titer is 106 Todd units/mL. c. Glucose, protein, and ketones are present in the urine. d. Nitrites and leukocyte esterase are present in the urine.

d. Nitrites and leukocyte esterase are present in the urine. A diagnosis of urinary tract infection is suspected if there are nitrites (indicating bacteriuria), white blood cells (WBCs), and leukocyte esterase (an enzyme present in WBCs indicating pyuria). Presence of glucose and ketones indicate uncontrolled diabetes mellitus. An elevated WBC count (>11,000 cells/µL) indicates a bacterial infection. Antistreptolysin-O (ASO) titer is a blood test to measure antibodies against streptolysin O, a substance produced by group A Streptococcus bacteria.

Prevention of calcium oxalate stones would include dietary restriction of which foods or drinks? a. Milk and milk products b. Dried beans and dried fruits c. Liver, kidney, and sweetbreads d. Spinach, cabbage, and tomatoes

d. Oxalate-rich foods should be limited to reduce oxalate excretion. Foods high in oxalate include spinach, rhubarb, asparagus, cabbage, and tomatoes, in addition to chocolate, coffee, and cocoa. Currently, it is believed that high dietary calcium intake may actually lower the risk for renal stones by reducing the intestinal oxalate absorption and therefore the urinary excretion of oxalate. Milk, milk products, dried beans, and dried fruits are high sources of calcium. Organ meats are high in purine, which contributes to uric acid lithiasis.

Prevention of calcium oxalate stones would include dietary restriction of which foods or drinks? a. Milk and milk products b. Dried beans and dried fruits c. Liver, kidney, and sweetbreads d. Spinach, cabbage, and tomatoes

d. Oxalate-rich foods should be limited to reduce oxalate excretion. Foods high in oxalate include spinach, rhubarb, asparagus, cabbage, and tomatoes, in addition to chocolate, coffee, and cocoa. Currently, it is believed that high dietary calcium intake may actually lower the risk for renal stones by reducing the intestinal oxalate absorption and therefore the urinary excretion of oxalate. Milk, milk products, dried beans, and dried fruits are high sources of calcium. Organ meats are high in purine, which contributes to uric acid lithiasis.

Which disease causes connective tissue changes that cause glomerulonephritis? a. Gout b. Amyloidosis c. Diabetes mellitus d. Systemic lupus erythematosus

d. Systemic lupus erythematosus causes connective tissue damage that affects the glomerulus. Gout deposits uric acid crystals in the kidney. Amyloidosis deposits hyaline bodies in the kidney. Diabetes mellitus causes microvascular damage affecting the kidney.

A 28-year-old male patient with type 1 diabetes reports how he manages his exercise and glucose control. Which behavior indicates that the nurse should implement additional teaching? a. The patient always carries hard candies when engaging in exercise. b. The patient goes for a vigorous walk when his glucose is 200 mg/dL. c. The patient has a peanut butter sandwich before going for a bicycle ride. d. The patient increases daily exercise when ketones are present in the urine.

d. The patient increases daily exercise when ketones are present in the urine.

Which finding indicates a need to contact the health care provider before the nurse administers metformin (Glucophage)? a. The patient's blood glucose level is 174 mg/dL. b. The patient has gained 2 lb (0.9 kg) since yesterday. c. The patient is scheduled for a chest x-ray in an hour. d. The patient's blood urea nitrogen (BUN) level is 52 mg/dL.

d. The patient's blood urea nitrogen (BUN) level is 52 mg/dL.

Which finding for a patient admitted with glomerulonephritis indicates to the nurse that treatment has been effective? a. The patient denies pain with voiding. b. The urine dipstick is negative for nitrites. c. The antistreptolysin-O (ASO) titer is decreased. d. The periorbital and peripheral edema is resolved.

d. The periorbital and peripheral edema is resolved. Because edema is a common clinical manifestation of glomerulonephritis, resolution of the edema indicates that the prescribed therapies have been effective. Nitrites will be negative and the patient will not experience dysuria because the patient does not have a urinary tract infection. Antibodies to streptococcus will persist after a streptococcal infection.

A patient with suprapubic pain and symptoms of urinary frequency and urgency has two negative urine cultures. What is one assessment finding that would indicate interstitial cystitis? a. Residual urine greater than 200 mL b. A large, atonic bladder on urodynamic testing c. A voiding pattern that indicates psychogenic urinary retention d. Pain with bladder filling that is transiently relieved by urination

d. The symptoms of interstitial cystitis (IC) imitate those of an infection of the bladder but the urine is free of infectious agents. Unlike a bladder infection, the pain with IC increases as urine collects in the bladder and is temporarily relieved by urination. Acidic urine is very irritating to the bladder in IC and the bladder is small but urinary retention is not common.

"A patient with bladder cancer undergoes cystectomy with formation of an ileal conduit. During the patient's first postoperative day, what should the nurse plan to do? a. Measure and fit the stoma for a permanent appliance. b. Encourage high oral intake to flush mucus from the conduit. c. Teach the patient to self-catheterize the stoma every 4 to 6 hours. d. Empty the drainage bag every 2 to 3 hours and measure the urinary output.

d. Urine drains continuously from an ileal conduit and the drainage bag must be emptied every 2 to 3 hours and measured to ensure adequate urinary output. Fitting for a permanent appliance is not done until the stoma shrinks to its normal size in a few weeks. With an ileal conduit, mucus is present in the urine because it is secreted by the ileal segment as a result of the irritating effect of the urine but the surgery causes paralytic ileus and the patient will be NPO for several days postoperatively. Self-catheterization is performed when patients have formation of a continent Kock pouch.

What should a patient be taught after a hemorrhoidectomy? a. Take a mineral oil before bedtime b. Eat a low-fiber diet to rest the colon c. Administer oil-retention enema to empty the colon d. Use prescribed pain medication before a bowel movement

d. Use prescribed pain medication before a bowel movement Rationale: After a hemorrhoidectomy, the patient usually dreads the first bowel movement and often resists the urge to defecate. Pain medication may be given before the bowel movement to reduce discomfort. The patient should avoid constipation and straining. A high-fiber diet can reduce constipation. A stool softener such as docusate (Colace) is usually ordered for the first few postoperative days. If the patient does not have a bowel movement within 2 to 3 days, an oil-retention enema is administered.

A nursing intervention that is most appropriate to decrease post-operative edema and pain after an inguinal herniorrhaphy is: a. applying a truss to the hernia site b. allowing the patient to stand to void c. supporting the incision during coughing d. applying a scrotal support with ice bag

d. applying a scrotal support with ice bag Rationale: Scrotal edema is a painful complication after an inguinal hernia repair. Scrotal support with application of an ice bag may help relieve pain and edema.

Which assessment data reported by a 28-year-old male patient is consistent with a lower urinary tract infection (UTI)? a. Poor urine output b. Bilateral flank pain c. Nausea and vomiting d. Burning on urination

d. burning on urination Pain with urination is a common symptom of a lower UTI. Urine output does not decrease, but frequency may be experienced. Flank pain and nausea are associated with an upper UTI.

*11. A patient has has a cystectomy and ileal conduit diversion performed. Four days postoperatively, mucous shred are seen in the drainage bag. The nurse should.* a. notify the physician b.notify the charge nurse c. irrigate the drainage tube d. chart it as a normal observation

d. chart it as a normal observation

The patient is wondering why anesthesia is needed when the lithotripsy being done is noninvasive. The nurse explains that the anesthesia is required to ensure the patient's position is maintained during the procedure. The nurse knows that this type of lithotripsy is called a. laser lithotripsy. b. electrohydraulic lithotripsy. c. percutaneous ultrasonic lithotripsy. d. extracorporeal shock-wave lithotripsy (ESWL).

d. extracorporeal shock-wave lithotripsy (ESWL). ESWL is noninvasive, but anesthesia is used to maintain the patient's position. The other types of lithotripsy are invasive. Laser lithotripsy uses an ureteroscope and small fiber to reach the stone. Electrohydraulic lithotripsy positions a probe directly on the stone; then continuous saline irrigation flushes are used to rinse the stone out. Percutaneous ultrasonic lithotripsy places an ultrasonic probe in the renal pelvis via a percutaneous nephroscope inserted through an incision in the flank.

A hospitalized diabetic patient received 38 U of NPH insulin at 7:00 AM. At 1:00 PM, the patient has been away from the nursing unit for 2 hours, missing the lunch delivery while awaiting a chest x-ray. To prevent hypoglycemia, the best action by the nurse is to a. save the lunch tray for the patient's later return to the unit. b. ask that diagnostic testing area staff to start a 5% dextrose IV. c. send a glass of milk or orange juice to the patient in the diagnostic testing area. d. request that if testing is further delayed, the patient be returned to the unit to eat.

d. request that if testing is further delayed, the patient be returned to the unit to eat.

A patient has the following arterial blood gas results: pH 7.52 PaCO2 30 mmHg HCO3- 24 mEq/L. The nurse determines these results indicate a. metabolic acidosis b. metabolic alkalosis c. respiratory acidosis d. respiratory alkalosis

d. respiratory alkalosis

During the postoperative care of a 76 yo patient, the nurse monitors the patient's intake and output carefully, knowing that the patient is at risk for fluid and electrolyte imbalances primarily because: a. older adults have an impaired thirst mechanism and need reminding to drink fluids b. water accounts for a greater percentage of body weight in the older adult than in younger adults c. older adults are more likely than younger adults to lose extra cellular fluid during surgical procedures d. small losses of fluid are significant because body fluids account for 45-50% of body weight in older adults

d. small losses of fluid are significant because body fluids account for 45-50% of body weight in older adults

A 55-year-old woman admitted for shoulder surgery asks the nurse for a perineal pad, stating that laughing or coughing causes leakage of urine. Which intervention is most appropriate to include in the care plan? a. Assist the patient to the bathroom q3hr. b. Place a commode at the patient's bedside. c. Demonstrate how to perform the Credé maneuver. d. Teach the patient how to perform Kegel exercises.

d. teaching the patient how to perform kegel exercises Kegel exercises to strengthen the pelvic floor muscles will help reduce stress incontinence. The Credé maneuver is used to help empty the bladder for patients with overflow incontinence. Placing the commode close to the bedside and assisting the patient to the bathroom are helpful for functional incontinence.

*7. The nurse recommends genetic counseling for the children of a patient with* a.nephrotic syndrome b.chronic pyelonephritis c. malignant nephrosclerosis d.adult onset polycystic kidney disease

d.adult onset polycystic kidney disease

*5.The edema that occurs in nephrotic syndrome is due to* a. increased hydrostatic pressure caused by sodium retention. b. decreased aldosterone secretion from adrenal insufficiency. c. increased fluid retention caused by decreased glomerular filtration d.decreased colloidal osmotic pressure caused by loss of serum albumin

d.decreased colloidal osmotic pressure caused by loss of serum albumin

Delegation Decision: Which nursing interventions could be delegated to unlicensed assistive personnel (UAP) (select all that apply)? a. Assess the need for catheterization. b. Use bladder scanner to estimate residual urine. c. Teach patient pelvic floor muscle (Kegel) exercises. d. Insert indwelling catheter for uncomplicated patient. e. Assist incontinent patient to commode at regular intervals. f. Provide perineal care with soap and water around a urinary catheter.

e, f. The unlicensed assistive personnel (UAP) may assist the incontinent patient to void at regular intervals and provide perineal care. An RN should perform the assessments and teaching. In long-term care and rehabilitation facilities, UAP may use bladder scanners after they are trained.

Delegation Decision: Which nursing interventions could be delegated to unlicensed assistive personnel (UAP) (select all that apply)? a. Assess the need for catheterization. b. Use bladder scanner to estimate residual urine. c. Teach patient pelvic floor muscle (Kegel) exercises. d. Insert indwelling catheter for uncomplicated patient. e. Assist incontinent patient to commode at regular intervals. f. Provide perineal care with soap and water around a urinary catheter.

e, f. The unlicensed assistive personnel (UAP) may assist the incontinent patient to void at regular intervals and provide perineal care. An RN should perform the assessments and teaching. In long-term care and rehabilitation facilities, UAP may use bladder scanners after they are trained.


Kaugnay na mga set ng pag-aaral

Chapter 6: Microbial Metabolism: Fueling Cell Growth

View Set

Mastering A&P Chapter 5 - Integumentary System

View Set

Chapter Two: Difference Between Private and Government

View Set

conditioning and learning: ch 4 pavlovian textbook readings

View Set

Bergmen/Clem Acct 284 Exam 2 Review

View Set